WBCHSE Class 12 Physics Notes For Magnetic Properties Of Materials

Magnetic Effect Of Current And Magnetism

Magnetic Properties Of Materials Introduction

Electric dipole end electric dipole moment:

From the chapter ‘Electric Field’ we know that, two equal but opposite charges +q and -q kept close to each other form an electric dipole.

Electric dipole moment \(\vec{p}\) of this di[o;e is defined as:

⇒ \(\vec{p}=q \vec{r}\)….(1)

= magnitude of any charge x position vector of charge +q with respect to charge -q

So, the magnitude of \(\vec{p} \text { is } p=|\vec{p}|=q r\) and the direction is from -q to +q.

Electromagnetism electric dipole and electric dipole moment

Electric field on the axis of an electric dipole: Let p be a point on the axis of an electric dipole. If the distance

Electromagnetism Electric field on the axis of an electric dipole

X of the point P from the mid-point of the dipole be much greater than the length r of the dipole, the glottic field at the point P due to that dipole,

⇒ \(\vec{E}=\frac{1}{4 \pi \epsilon_0} \cdot \frac{2 \vec{p}}{x^3}\)

Here, ∈0 = permittivity of air or Vacuum

Read and Learn More Class 12 Physics Notes

= 8.854 x 10-12 C2.N-1.m-2

Torque on an electric dipole in a uniform electric field: \(\vec{E}\) is a uniform electric field (-q, +q) is an electric dipole whose dipole moment = \(\vec{p}\).

Now, the torque acting on the dipole due to the electric field \(\vec{E}\) is,

⇒ \(\vec{\tau}=\vec{p} \times \vec{E}\)

In the direction of \(\vec{\tau}\) is particularly downward concerning the page of the tired book, which Is denoted by the symbol\(\otimes\).

Electromagnetism Torque on an electric dipole in a uniform electric field

Magnetic field on the axis of a current loop:

r = radius of a circle conductor of a single turn, i.e., the radius of the current loop,

I = current in that loop.

P is any point on the axis of the current loop which is at a distance x from the center of the loop.

Electromagnetism Magnetic field on the axis of a current loop

In the chapter ‘Electromagnetism’ we know that the magnetic field produced at the point P due to the current loop is,

⇒ \(B=\frac{\mu_0 I}{2} \cdot \frac{r^2}{\left(r^2+x^2\right)^{3 / 2}}\) [μ0 = magnetic permeability of air or vacuum = 4π x 10-7 Hm-1]

⇒ \(\text { If } r \ll x \text {, then } B \approx \frac{\mu_0 I}{2} \cdot \frac{r^2}{x^3}=\frac{\mu_0 I}{2 \pi} \cdot \frac{\pi r^2}{x^3}=\frac{\mu_0 I}{2 \pi} \cdot \frac{A}{x^3}\)

where, A = πr2 = area of the current loop.

∴ \(B=\frac{\mu_0}{4 \pi} \cdot \frac{2 I A}{x^3}\)

From the corkscrew rule we get, that the direction of \(\vec{B}\) is along the axis of the loop; the direction of \(\vec{B}\) at the point P is outward along the axis. Again, taking that direction as the direction of the area A, it can be expressed as \(\vec{A}\).

Therefore, \(\vec{B}=\frac{\mu_0}{4 \pi} \cdot \frac{2(\overrightarrow{I A})}{x^3}\)….(4)

WBCHSE Class 12 Physics Notes For Magnetic Properties Of Materials

Torque on a current loop in a uniform magnetic field:

In the chapter ‘Electromagnetism’ we know that, if a current loop of single turn is kept in a uniform magnetic field B, the torque acting on the loop is,

⇒ \(\vec{\tau}=\overrightarrow{I A} \times \vec{B}\)….(5)

The direction of this torque is perpendicularly downward concerning the page of the book, which is denoted by the symbol.

Electromagnetism Torque on a current loop in a uniform magnetic field

Magnetic Effect Of Current And Magnetism

Magnetic Properties Of Materials Magnetic Dipole And Magnetic Dipole Moment Or Magnetic Moment

Comparing equations (2) and (4) and at the same time equations (3) and (5) as described, we get,

1. Electric field \(\vec{E}\) in electrostatics plays the same role as that of magnetic field \(\vec{B}\) in magnetism.

2. The role of the quantity \(\frac{1}{4 \pi \epsilon_0}\) in electrostatics is the same as that of the quantity \(\frac{\mu_0}{4 \pi}\) in magnetism.

3. The role of electric dipole moment \(\vec{p}\) in electrostatics is the same as the quantity I\(\vec{A}\) related to a current loop in magnetism.

Inference: Any current loop behaves as a magnetic dipole. The dipole moment of this dipole is,

⇒ \(\vec{p}_m=I \vec{A}\)

where, I = current through the loop, \(\vec{A}\) = area vector of the loop.

The magnitude of \(\vec{A}\) is the same as the magnitude of the area of the loop; the direction of \(\vec{A}\) is in the direction of advancement of the screw-head when a right-handed screw is rotated in the direction of current I through the loop.

Naturally, if the current loop contains N turns, it’s the magnetic moment, becoming \(\vec{p}_m=N I \vec{A}\).

Unit of magnetic moment:

Unit of pm = unit of I x unit of A

= ampere.metrer2 (A.m2)

In the CGS or Gaussian system:

The magnetic dipole moment of a current loop, \(\vec{p}_m \equiv I \vec{A}\); if the current loop contains N turns instead of a single turn, \(\vec{p}_m=N I \vec{A}\). Here, the units of

A, I and pm are cm2, emu of current, and emu.m2, respectively.

∴ 1 emu.cm2 = 10A x 10-4 m2

= 10-3 A.m2

Significance: Any current loop behaves as a magnetic dipole-it means that a current loop and a magnet having a north and a south pole are qualitatively equivalent. This similarity is discussed with the help of the following two examples.

Similarities between a circular conductor and a magnet:

Magnetic lines of force near the center of a circular conductor are almost parallel to each other and they remain perpendicular to the plane of the circle.

So, almost a uniform magnetic field is generated at that region, which acts normally to the plane of the circular conductor.

Electromagnetism Similarities between a circular conductor and a magnet

From the properties of magnetic lines of force of a permanent magnet, we know that, if the circular conductor is replaced by a small permanent magnet in that region, similar lines of force will be obtained.

So, we can conclude that a circular current-carrying coil behaves as a permanent magnet.

We can also get the rule for the determination of the polarity of the circular conductor.

Electromagnetism Similarities between a circular conductor and a magnet.

1. The face of a circular conductor on which the current appears to flow clockwise, develops a magnetic south pole.

2. The face of a circular conductor, on which the current appears to flow anticlockwise, develops a magnetic north pole.

With the help of the following experiment, the magnetic property of a circular conductor can be shown.

De la Rives floating battery: In a wide test tube some mercury is taken so that it can float upright in water

Electromagnetism de la Rives floating battery

Some dilute sulphuric acid (H2SO4) is poured into the test tube above mercury, and zinc and copper plates are dipped in the acid so that a voltaic cell is formed. This is known as a floating battery.

Two conducting wires from the two plates are brought outside the test tube through a cork fitted at the top.

These two wires act as two poles of the battery. Now the two ends of a circular coil are joined with those two poles. Usually, the coil contains several number of turns instead of a single turn.

When the coil is connected to the battery, the whole system begins to oscillate about a vertical axis.

At last, when the battery comes to rest, the axis of the circular coil sets itself in the north-south direction.

From this directive property, it is understood that the current-carrying coil behaves as a magnet.

With the help of a bar magnet, it can also be observed that like poles repel and unlike poles attract each other.

Similarities between a current-carrying solenoid and a magnet:

The arrangement of the lines of force in the chapter ‘Electromagnetism’ is similar to the arrangement of lines of force of a bar magnet.

On the face, on which the direction of current flow appears clockwise, a magnetic south pole develops, and on the other face of the solenoid, a magnetic north pole develops.

So, we can conclude that a current-carrying solenoid behaves as a permanent bar magnet.

Experimental demonstration:

The magnetic properties of a solenoid can be shown by replacing the circular coil attached to the de la Rive’s floating battery with a solenoid.

In this case, also, the directive and the attractive or repulsive properties of the solenoid with another magnet are observed.

Actually, from the similarities of a magnet and a current loop, it can be concluded that magnetism is not a separate branch of physics, rather it is a part of electricity and this specific subject is known as electromagnetism.

Pule-sought of a Magnet:

In modem theory of magnetism, the concept of the pole-strength of a magnet is not essential; but for the comparison with electrostatics and also to get an idea about the old theory of magnetism, we may discuss here about the pole-strength of a magnet.

Let a bar magnet NS of effective length r be kept at an angle θ with the direction of a uniform magnetic f field \(\vec{B}\).

The effect on the N and S poles are equal and opposite; hence due to the magnetic field \(\vec{B}\), two equal but opposite forces will act on the two poles.

Electromagnetism Pule-strength of a Magnet

The distance between the line of action of the two forces, NC = rsinθ. So, the torque acting on the bar magnet due to these two forces,

\(\tau\) = magnitude of any one force x perpendicular distance between the two forces

= Frsinθ….(1)

substituting \(\vec{p}_m=I \vec{A}\) we get,

⇒ \(\vec{\tau}=\vec{p}_m \times \vec{B}\)

We get the value of the torque \(\vec{\tau}\),

⇒ \(\tau=p_m B \sin \theta\)

Since a current loop and a magnet are identical, from equations (1) and (2) we get,

⇒ \(F r \sin \theta=p_m B \sin \theta \quad \text { or, } F=\frac{p_m}{r} B=q_m B\)….(3)

The term qm is known as the pole strength of a magnet. The value of the pole-strength of each of the N and S poles is qm; conventionally the pole-strength of the N pole is taken as + qm and that of the S pole is taken as -qm.

Definition:

The ratio of the magnetic moment of a magnet to its effective length is called the pole strength of that magnet.

The strength of the two poles of the magnet is equal but opposite; the strength of the north pole is taken as positive and that of the south pole as negative.

Using the vector symbol, equation (3) can be written as

⇒ \(\vec{p}_m=q_m \vec{r} \text { and } \vec{F}=q_m \vec{B}\)

These two equations are identical to the equations \(\vec{p}=q \vec{r} \text { and } \vec{F}=q \vec{E}\) in electrostatics.

Due to the similarities between the electric field \(\vec{E}\) and magnetic field \(\vec{B}\), we can say that the role of positive arid negative charges (±q) in electrostatics is the same as that of the north and south poles (±qm) in electromagnetism.

Unit of pole-strength:

In SI: \(\text { Unit of } q_m=\frac{\text { unit of } p_m}{\text { unit of } r}=\frac{\mathrm{A} \cdot \mathrm{m}^2}{\mathrm{~m}}=\mathrm{A} \cdot \mathrm{m}\)

In CGS: Unit of qm is emu of current cm;

⇒ \(1 \mathrm{emu} \text { of current } \cdot \mathrm{cm}=10 \mathrm{~A} \times 10^{-2} \mathrm{~m}=\frac{1}{10} \mathrm{~A} \cdot \mathrm{m}\)

The magnetic moment of a magnet: The magnetic moment of a magnet can also be defined from the concept of pole strength.

Definition: The product of the pole-strength of any pole of a magnet and its effective, length is called the magnetic moment of that magnet.

If the distance vector from the south pole to the north pole of a bar magnet is 2\(\vec{l}\) and its pole strength is qm, then the magnetic moment,

⇒ \(\vec{p}_m=2 q_m \vec{l}\)

Mutual force between two magnetic poles: We know that, for a charge q, the electric field at any point at a distance r from the charge = force acting on unit positive charge placed at that point,

⇒ \(\text { i.e., } E=\frac{1}{4 \pi \epsilon_0} \cdot \frac{q}{r^2}\) [∈0 = electrical permittivity of vacuum]

From analogy, we can say that the magnetic field at any point at a distance r from a magnetic pole of pole-strength qm = force acting on a unit north pole placed at that point,

⇒ \(\text { i.e., } B=\frac{\mu_0}{4 \pi} \cdot \frac{q_m}{r^2}\) [μ0 = magnetic permeability of vacuum]

If another magnetic pole of pole-strength ‘m is placed at a distance r from qm, from the equation F = qmB we can say that, force acting on that pole

⇒ \(F=\frac{\mu_0}{4 \pi} \cdot \frac{q_m \cdot q_m^{\prime}}{r^2}\)…(4)

According to Newton’s third law of motion, it is the mutual force acting between the poles having pole strengths qm and I’m. Equation (4) is called Coulomb’s law in magnetism.

If both the poles are north poles or both the poles are south poles, the product qmq’m becomes positive, and hence F is also positive. It means that the direction of F is the same as r, i.e., the force F is repulsive.

If the two poles are unlike, the product qmq am becomes negative. It means that F is in the direction opposite to r, i.e., the force F is attractive. From equation (4), we can say that the force acting between two magnetic poles is,

1. Directly proportional to the product of the pole strengths of the two poles, and

2. Inversely proportional to the square of the distance between the two poles.

In electrostatics, the similar law is the Coulomb’s law:

⇒ \(F=\frac{1}{4 \pi \epsilon_0} \cdot \frac{q q^{\prime}}{r^2}\)

In CGS or Gaussian system:

The corresponding relations of electric field E, magnetic intelligence H, and magnetic force Fin vacuum or in air are respectively,

⇒ \(E=\frac{q}{r^2}, H=\frac{q_m}{r^2} \text { and } F=\frac{q_m \dot{q}_m^{\prime}}{r^2}\)

Magnetic Field due to a Bar Magnet:

End-on or axial position:

A bar magnet SN is kept in the air, Its pole strength is qm, and its magnetic length = SN = 21. A point P is taken on the axis of the magnet at a distance d from its mid-point O’.

The position of point P is called the end-on or axial position: We have to determine the magnetic field at point F, due to the magnet

Electromagnetism End on or axial position

Calculation: Magnetic field at the point P due to N-pole,

⇒ \(B_1=\frac{\mu_0}{4 \pi} \cdot \frac{q_m}{(d-l)^2} ; \text { the direction of } \vec{B}_1 \text { is along } \overrightarrow{O P}\)

Again, the magnetic field at the point P due to S-the pole,

⇒ \(B_2=\frac{\mu_0}{4 \pi} \cdot \frac{q_m}{(d+l)^2} ; \text { the direction of } \vec{B}_2 \text { is along } \overrightarrow{P O}\)

∵ The N-pole is nearer to P than the S-pole, and the value of B1 is greater than that of B2.

Therefore, the resultant magnetic field at the point P,

⇒ \(B=B_1-B_2=\frac{\mu_0}{4 \pi}\left[\frac{q_m}{(d-l)^2}-\frac{q_m}{(d+l)^2}\right]\)

⇒ \(\frac{\mu_0}{4 \pi} \cdot \frac{q_m \cdot 4 d l}{\left(d^2-l^2\right)^2}\)

= \(\frac{\mu_0}{4 \pi} \cdot \frac{2 p_m d}{\left(d^2-l^2\right)^2}\) [pm = qm.21 = magnetic moment of the bar magnet]

The direction of \(\vec{B}\) is along \(\vec{OP}\).

The vector representation of the equation is,

⇒ \(\vec{B}=\frac{\mu_0}{4 \pi} \cdot \frac{2 \vec{p}_m d}{\left(d^2-l^2\right)^2}\)

The resultant magnetic intensity in the CGS system

⇒ \(H=\frac{2 p_m d}{\left(d^2-l^2\right)^2}\)

If the length of the bar magnet is very small and the point p is taken at a very- large distance from the bar magnet, l.e., d>>l, then (d²- l²)² ≈ d4.

∴ \(B=\frac{\mu_0}{4 \pi} \cdot \frac{2 p_m}{d^3}\)

Broadside on or equatorial position:

A bar magnet SN is kept in the air, its pole strength is qm, and magnetic length = SN = 2l.

A point P is taken on the perpendicular bisector of the magnetic length and at a distance d from the mid-point O of the magnet.

The position of the point p is called the broadside-oil or equatorial position. We have to determine the magnetic field at the point P due to the magnet.

Electromagnetism Broadside on or equatorial position

Calculation: Magnetic field at the point P due to N-pole,

⇒ \(B_1=\frac{\mu_0}{4 \pi} \cdot \frac{q_m}{N P^2} \text {; the direction of } \vec{B}_1 \text { is along } \overrightarrow{P Q}\)

Again, the magnetic field at the point P due to the S-pole,

⇒ \(B_2=\frac{\mu_0}{4 \pi} \cdot \frac{q_m}{S P^2} ; \text { the direction of } \vec{B}_2 \text { is along } \overrightarrow{P S}\)

The components of B1 and B2, i.e., B1sinθ and B2sinθ, along with PT and PO respectively, cancel each other.

On the other hand, the components B1cosθ and B2cosθ along PR are added together.

So, the resultant magnetic field at the point P,

B = 2B1cosθ [∵ magnitudes of B1 , B2 are equal]

or, \(B=\frac{2 \mu_0}{4 \pi} \cdot \frac{q_m}{N P^2} \cos \theta=\frac{2 \mu_0}{4 \pi} \cdot \frac{q_m}{\left(d^2+l^2\right)} \cdot \frac{l}{\sqrt{d^2+l^2}}\)

⇒ \(\frac{\mu_0}{4 \pi} \cdot \frac{p_m}{\left(d^2+l^2\right)^{3 / 2}} \text {; the direction of } \vec{B} \text { is along } \overrightarrow{P R}\)

The vector representation of the equation is,

⇒ \(\vec{B}=-\frac{\mu_0}{4 \pi} \cdot \frac{\vec{p}_m}{\left(d^2+l^2\right)^{3 / 2}}\)

The resultant magnetic intensity in the CGS system,

⇒ \(H=\frac{p_m}{\left(d^2+l^2\right)^{3 / 2}}\)

If the length of the bar magnet is very small and the point P is
taken at a very large distance from the bar magnet, i.e., l<<d

then, \(B=\frac{\mu_0}{4 \pi} \cdot \frac{p_m}{d^3}\)

So, magnetic field at the end-on or axial position = 2 x magnetic field at the broadside-on or equatorial position

Any position:

Concerning the bar magnet of magnetic length 2l, the point P under consideration is at the position (r, θ), i.e., the point P is at a distance r from the mid-point of the magnet and lies at an angle θ with the magnetic axis.

The component of pm along r is pmcosθ; concerning this component, the point P is at the end of the position, and hence the magnetic field at the point P,

Electromagnetism bar magnet of magnetic length

⇒ \(B_r=\frac{\mu_0}{4 \pi} \cdot \frac{2 p_m \cos \theta}{r^3}(∵ r \gg l)\)

Again, the component of pm normal to r is pmsinθ; concerning this component point P is at the equatorial position, and hence the magnetic field at point P,

⇒ \(B_\theta=\frac{\mu_0}{4 \pi} \cdot \frac{p_m \sin \theta}{r^3}\)

So, according to the resultant magnetic field at the point P,

⇒ \(B=\sqrt{B_r^2+B_\theta^2}=\sqrt{\left(\frac{\mu_0}{4 \pi} \cdot \frac{p_m}{r^3}\right)^2\left(4 \cos ^2 \theta+\sin ^2 \theta\right)}\)

⇒ \(\frac{\mu_0}{4 \pi} \cdot \frac{p_m}{r^3} \sqrt{3 \cos ^2 \theta+1}\)

The direction of \(\vec{B} \text { is along } \overrightarrow{P R} \text {; where } \angle T P R=\phi \text { (say) }\)

∴ \(\tan \phi=\frac{B_\theta}{B_r}=\frac{1}{2} \tan \theta\)

The resultant magnetic intensity in the CGS system,

⇒ \(H=\frac{p_m}{r^3} \sqrt{3 \cos ^2 \theta+1}\)

Equivalence of a solenoid and a bar magnet: Let us assume a solenoid of radius = a, length = 2l, number of turns per unit length = n. Current through it = I.

So, the total number of turns of the solenoid = 2ln, and hence the effective net current around its axis =2lnI. As the cross-sectional area of the solenoid = πa², so, the magnetic dipole moment of the current-carrying solenoid,

⇒ \(p_m=(2 \ln I) \cdot\left(\pi a^2\right)=2 \pi \ln a^2 I\)

Electromagnetism Equivalence of a solenoid and a bar magnet

Now the center (O) of the solenoid is taken as the origin and its axis as the x-axis. Considering a small length dx at a distance x from 0, we get a circular current-carrying coil with several turns dx. For this coil, the magnetic field is thus produced at a point P on the axis (where, OP = r and r>>a, r>>x),

⇒ \(d B=\frac{\mu_0(n d x) I}{2} \cdot \frac{a^2}{\left\{a^2+(r-x)^2\right\}^{3 / 2}}=\frac{\mu_0 n I a^2}{2 r^3} d x\) [As a and x are negligible with respect to r, \(\left\{a^2+(r-x)^2\right\}^{3 / 2} \approx r^3\)]

So, the magnetic field at P for the entire solenoid,

⇒ \(B=\frac{\mu_0 n I a^2}{2 r^3} \int_{-l}^l d x=\frac{\mu_0 n I a^2}{2 r^3} \cdot 2 l=\frac{\mu_0}{4 \pi} \cdot \frac{2\left(2 \pi \ln a^2 I\right)}{r^3}\)

i.e., \(B=\frac{\mu_0}{4 \pi} \frac{2 p_m}{r^3}\)

In this very section, we have already found the same expression due to a bar magnet. Thus we conclude that a bar magnet and a current-carrying solenoid are equivalent to each other.

Comparing their magnetic moments, we get,

⇒ \(q_m \cdot 2 l=2 \pi \ln a^2 I \quad \text { or, } q_m=\pi n a^2 I\)

Magnetic Moment of Charged Particle Moving in a Circle:

We know that a revolving charged particle is equivalent to an electric current. So, the orbit of that particle is equivalent to a current loop and as a result, it behaves as a magnetic dipole which must have a magnetic moment.

Suppose a particle having a charge q is revolving in a plane circular path; r = radius of the orbit of that particle and v = velocity of the particle.

Electromagnetism Magnetic Moment of Charged Particle moving in a circle

The period of revolution of the particle, \(T=\frac{2 \pi r}{v}\)

∴ Equivalent current, \(I=\frac{q}{T}=\frac{q v}{2 \pi r}\)

So, the magnetic moment of the particle,

⇒ \(p_m=\frac{q \nu}{2 \pi r} \cdot A=\frac{q \nu}{2 \pi r} \cdot \pi r^2=\frac{q v r}{2}\)

The direction of this magnetic moment pm can be obtained by applying a corkscrew rule. If the mass of the particle is m then along the axis of the circular path, Le., along the direction of pm, the angular momentum of the particle,

L = mv [L = moment of momentum = momentum x radius of circular path]

∴ \(p_m=\frac{q v r}{2}=\frac{q}{2 m} \cdot m v r=\frac{q}{2 m} L\)

Since the directions of pm and L are the same, using vector notation we can write,

⇒ \(\vec{p}_m=\frac{q}{2 m} \vec{L}\)…(1)

Magnetic dipole moment of an electron:

Electrons inside an atom revolve around the nucleus continuously. As a revolving charged particle, each electron behaves as a magnetic dipole. Substituting the charge of an electron -e in place of q in equation (1), we can write,

⇒ \(\vec{p}_m=-\frac{e}{2 m} \cdot \vec{L}\)…(2)

The negative sign on the right-hand side indicates that the charge of an electron is negative, its angular momentum and magnetic moment are oppositely directed.

From Bohr’s theory related to atomic structure: Atoms, it is known that electrons revolving in different orbits have discrete angular momenta, defined as

⇒ \(L=n \frac{h}{2 \pi}=n \hbar \quad[n=1,2,3, \cdots]\)

Here, \(h=\text { Planck’s constant }=6.626 \times 10^{-34} \mathrm{~J} \cdot \mathrm{s} \text { and } \dot{\hbar}=\frac{h}{2 \pi}\)

= 1.05 x 10-34 J.s = reduced Planck’s constant

So, the magnitude of the magnetic moment of an electron [from equation (2)],

⇒ \(p_m=\frac{e}{2 m} n \hbar=n \frac{e \hbar}{2 m}\)….(3)

In the case of the innermost orbit of an atom (K-orbit), n = 1; then \(p_m=\frac{e \hbar}{2 m}\) – this quantity is called Bohr magneton. It is denoted by μB and it is the atomic unit of the magnetic moment

⇒ \(I \mu_B=\frac{e h}{2 m}=\frac{\left.\left(1.6 \times 10^{-19} \mathrm{C}\right) \times\left(1.05 \times 10^{-34}\right) \cdot s\right)}{2 \times\left(9.1 \times 10^{-31} \mathrm{~kg}\right)}\)

= 9.23 x 10-24 A m2

= 9.23 x 10-24 J. T-1

The spin of an electron: Besides the revolution of each electron in its orbit, it also rotates about its axis (like the diurnal motion of the earth). This is known as the spin of an electron. An electron may have any of two oppositely directed spins.

One kind of spin (say clockwise) is called up spin and is denoted by ( symbol) The opposite kind of spin is then known as down spin and is denoted by the symbol.

Due to the spin of any electron. a magnetic moment generates. By theoretical analysis, its Value is found to be l Bohr magneton. It is allied tire intrinsic spin magnetic moment.

The concept of rotation of an electron about its axis is oversimplified and requires quantum physics for a proper explanation.

The magnetism of an atom: If the resultant moment i.e., the vector sum of tire magnetic moments of the electrons rotating, around the nucleus inside an atom is zero, we can say that the atom as a whole does not show the airy magnetic property.

On the other hand, if the resultant is not zero, the tire atom behaves like a magnetic dipole. Due to this property, each atom or molecule of iron, nickel eta behaves like a small magnet and is called an atomic magnet.

Magnetic Effect Of Current And Magnetism

Magnetic Properties Of Materials Numerical Examples

Example 1. A torque of 8 units is applied on a magnet when it is kept at 30° with the direction of a uniform magnetic field of intensity 0.32 units. Determine the magnetic moment of the magnet.
Solution:

If a magnet having magnetic moment pm is placed inclined at an angle θ with a uniform magnetic field B, the torque acting on the magnet,

⇒ \(\tau=p_m B \sin \theta\)

Here, B = 0.32 units, θ = 30° and \(\tau\) = 8 units.

∴ \(p_m=\frac{\tau}{B \sin \theta}=\frac{8}{0.32 \times \sin 30^{\circ}}\)

⇒ \(\frac{8}{0.32 \times \frac{1}{2}}=50 \text { units }\)

Example 2. lf the distance between two north poles of equal strength is 2 cm, the mutual force of repulsion between them becomes 2.5 dyn. What should be the distance of separation between them for which the repulsive force becomes 3.6 dyn?
Solution:

Magnetic force between two magnetic poles,

⇒ \(F=\frac{\mu_0}{4 \pi} \cdot \frac{q_m \cdot q_m^{\prime}}{r^2}\)

So, if the pole strength remains unchanged then, \(F \propto \frac{1}{r^2}\)

Hence, for two separate distances,

⇒ \(\frac{F_1}{F_2}=\left(\frac{r_2}{r_1}\right)^2 \quad \text { or, } \frac{r_2}{r_1}=\sqrt{\frac{F_1}{F_2}}\)

∴ \(r_2=r_1 \sqrt{\frac{F_1}{F_2}}=2 \times \sqrt{\frac{2.5}{3.6}}=2 \times \frac{5}{6}\)

= 1.67cm

Example 3. The length of a bar magnet is 20 cm and its magnetic moment is 0.6 Am2. Determine the magnetic field at a point 30 cm away from either end.
Solution:

PN = PS

= 30 cm

= 0.3 m

= \(\sqrt{d^2+l^2}\)

The magnetic field at the point P due to the magnet,

⇒ \(B=\frac{\mu_0}{4 \pi} \cdot \frac{p_m}{\left(d^2+l^2\right)^{3 / 2}}\)

⇒ \(\frac{4 \pi \times 10^{-7}}{4 \pi} \times \frac{0.6}{(0.3)^3}\)

= 2.22 x 10-6 Wb.m-2

Electromagnetism Example 3 The length of a bar magne

Example 4. The length of a bar magnet is 20 cm and its magnetic moment is 0.6 A.m2. Determine the magnetic field at a point on the axis of the magnet and 30 cm away from the north pole.
Solution:

Length of the magnet. 2l = 20 cm

= 0.2 m

So, l = 0.1 m

Distance of the given point from the center of the magnet,

d = (0.3 + 0.1)m

= 0.4 m

∴ The magnetic field at the given point due to the magnet,

⇒ \(B=\frac{\mu_0}{4 \pi} \cdot \frac{2 p_m d}{\left(d^2-l^2\right)^2}=\frac{4 \pi \times 10^{-7}}{4 \pi} \times \frac{2 \times 0.6 \times 0.4}{\left\{(0.4)^2-(0.1)^2\right\}^2}\)

⇒ \(10^{-7} \times \frac{0.48}{0.0225}=2.13 \times 10^{-6} \mathrm{~Wb} \cdot \mathrm{m}^{-2}\)

Example 5. The radius of a circular conducting coil of 100 turns Is 10 cm. If 2A current passes through the coil, what will be the magnetic moment generated?
Solution:

Magnetic moment,

pm = NIA = NIπr2

= 100 x 2 x (π x 0.1 x 0.1) [∵ 10 cm = 0.1 m]

= 6.28 A.m2

Example 6. If the magnetic moment of a straight magnetized wire is pm, what will be Its magnetic moment when the wire is bent in the form of a semicircle?
Solution:

Let the length of the magnetized wire be l.

∴ Pole-strength of the wire, \(m=\frac{p_m}{l}\)

When the wire is bent in the form of a semicircle, its effective length = diameter of the semicircle = 2r.

According to the problem, \(\pi r=l \quad \text { or, } r=\frac{l}{\pi}\)

∴ The changed magnetic moment of the wire = effective length x pole-strength

⇒ \(2 r \times \frac{p_m}{l}=\frac{2 l}{\pi} \cdot \frac{p_m}{l}=\frac{2 p_m}{\pi}\)

Example 7. The magnetic moments of two small magnets are pm and p’m; they are kept on a table. What will be the magnitude and direction of the magnetic field produced by the gents at point P? [pm = 2.7 A m2, pm = 3,2 A m2; d1 = 30 cm, d2 = 40 cm]

Electromagnetism Example 7 Magnetic moments of two small magnets

Solution:

According to the point, P lies on the perpendicular bisectors of both magnets.

The magnetic field at the point P due to the magnet having magnetic moment pm,

⇒ \(B_1=\frac{\mu_0}{4 \pi} \cdot \frac{p_m}{d_1^3}=10^{-7} \times \frac{2.7}{(0.3)^3}=10^{-5} \mathrm{~T}\)

At point P, the magnetic field due to the magnet having magnetic moment I’m is,

⇒ \(B_2=\frac{\mu_0}{4 \pi} \cdot \frac{p_m^{\prime}}{d_2^3}=10^{-7} \times \frac{3.2}{(0.4)^3}=5 \times 10^{-6} \mathrm{~T}\)

Since these two magnetic fields are perpendicular to each other, the resultant magnetic field at the point P,

⇒ \(B=\sqrt{B_1^2+B_2^2}\)

⇒ \(\sqrt{\left(10^{-5}\right)^2+\left(5 \times 10^{-6}\right)^2}\)

= 1.12 x 10-5T

If the resultant magnetic field makes an angle θ with B1, then,

⇒ \(\tan \theta=\frac{B_2}{B_1}=\frac{5 \times 10^{-6}}{10^{-5}}=0.5\)

∴ \(\theta=\tan ^{-1}(0.5)=26.57^{\circ}\)

Example 8. Two bar magnets A and B, each having a magnetic length of 4 cm are placed along a straight line with their north poles 8 cm apart and facing each other. The neutral point lies on the axis, 2 cm from the north pole of the magnet A. Calculate the ratio of the magnetic moments of A and B.
Solution:

O is die neutral point.

Given, NO = 2 cm

N’O = (8-2)

= 6 cm

⇒ \(X N=X^{\prime} N^{\prime}=l=\frac{4}{2} \mathrm{~cm}=2 \times 10^{-2} \mathrm{~m}\)

d1 = XO

= (2 + 2) cm

= 4 x 10-3 m

d2 = X’O

= (6 + 2) cm

= 8 x 10-3 m

Electromagnetism Example 8 Two bar magnets

∴ The magnetic field at point O due to the magnet A,

⇒ \(B_1=\frac{\mu_0}{4 \pi} \frac{2 p_{m_1} d_1}{\left(d_1^2-l^2\right)^2}\)

or, \(B_1=\frac{\mu_0}{4 \pi} \frac{2 p_{m_1} \times 4 \times 10^{-2}}{\left[\left(4 \times 10^{-2}\right)^2-\left(2 \times 10^{-2}\right)^2\right]^2}\)

⇒ \(=\frac{\mu_0}{4 \pi} \cdot \frac{2 p_{m_1} \times 4 \times 10^{-2}}{(16-4)^2 \times 10^{-8}}\)

∴ The magnetic field at O due to the magnet B,

⇒ \(B_2=\frac{\mu_0}{4 \pi} \frac{2 p_{m_2} \times 8 \times 10^{-2}}{\left[\left(8 \times 10^{-2}\right)^2-\left(2 \times 10^{-2}\right)^2\right]^2}\)

⇒ \(\frac{\mu_0}{4 \pi} \cdot \frac{2 p_{m_2} \times 8 \times 10^{-2}}{(64-4)^2 \times 10^{-8}}\)

According to the question, B1 = B2

∴ \(\frac{\mu_0}{4 \pi} \cdot \frac{2 p_{m_1} \times 4 \times 10^{-2}}{144 \times 10^{-8}}=\frac{\mu_0}{4 \pi} \cdot \frac{2 p_{m_2} \times 8 \times 10^{-2}}{3600 \times 10^{-8}}\)

or, \(\frac{p_{m_1}}{p_{m_2}}=\frac{2 \times 144}{3600}=\frac{2}{25}\)

∴ Ratio of the magnetic moments \(p_{m_1}: p_{m_2}=2: 25\)

Magnetic Effect Of Current And Magnetism

Magnetic Properties Of Materials Magnetic Lines Of Induction

The magnetic lines of force in a uniform magnetic field are equidistant parallel straight lines.

If a piece of any magnetic material (i.e., iron, steel, nickel, etc.) is placed in this kind of external magnetic field, a magnetic field is induced in the specimen which is converted into a temporary magnet. The external magnetic field is referred to as the inducing magnetic field.

Electromagnetism Magnetic lines of induction

Let an iron bar PQ be placed parallel to a uniform magnetic field. At the end of PQ through which the magnetic lines of force enter, an S-pole is developed and at the other end, an AT-pole is developed.

So, due to induction, the magnetic lines of force are rearranged. This rearrangement inside and outside the magnetic material is discussed below.

The lines inside the magnetic material are generated due to the superposition of two kinds of lines of force:

  1. The lines of force are due to the inducing magnetic field.
  2. The lines of magnetization (the magnetic material is temporarily converted into a magnet due to induction and the lines of the magnetic field of this temporary magnet are called lines of magnetization).

Electromagnetism magnetic material

The lines of force outside the magnetic material are generated due to the superposition of two types of lines of force:

  1. The lines of force due to inducing magnetic field
  2. The lines of force are due to the temporary magnetic field of the magnetized specimen.

Outside the iron bar, the lines of force suffer bending and inside the bar are densely crowded. In the region adjacent to points A and B outside the iron bar, the lines of force are less dense.

Note that, inside the bar PQ, the lines of force of the inducing field and the lines of magnetization are unidirectional. It means that inside the bar PQ, the resultant of the two magnetic fields increases, and hence in that region and the adjacent regions C and D, the lines of force are densely crowded.

In the regions adjacent to points A and B, the inducing magnetic field and the temporary magnetic field are oppositely directed and hence the resultant magnetic field decreases there. So the lines of force become less dense in that region

Definition:

The lines of force generated inside a magnetic material due to the superposition of the lines of force of the inducing magnetic field and the lines of magnetization are called the lines of induction.

Outside a magnetic material lines of magnetic induction follow magnetic lines of force. In some materials, called diamagnetic materials, the scenario would be the reverse of the above picture

Magnetic Effect Of Current And Magnetism

Magnetic Properties Of Materials Some Magnetic Quantities And Their Relations

Magnetic permeability of a material:

Let the current through a long straight solenoid = I; the number of turns per unit length of the solenoid = n.

If air or vacuum is taken as the core of the solenoid, the magnetic field produced along the axis of the solenoid,

B0 = μ0nI…(1)

where, μ0 = magnetic permeability of vacuum (or air)

= 4π x 10-7 H.m-1

If a rod of any material is introduced inside the solenoid co-axially, the magnetic field along the axis of the solenoid will change; this magnetic field can be expressed as,

B = μnI….(2)

This quantity ju is called the magnetic permeability of the material used.

Relative magnetic permeability: If air or vacuum is replaced by a material, the fractional change of magnetic field in that material is called the relative magnetic permeability of that material.

So, relative magnetic permeability,

⇒ \(\mu_r=\frac{B}{B_0}=\frac{\mu n I}{\mu_o n I}=\frac{\mu}{\mu_0}\)….(3)

Unit: Equations (1) and (2) show that the units of μ0 are the same, i.e., H m-1.

Again, equation (3) shows that being the ratio of two identical quantities μ and μ0, μr has no unit.

Classification of materials: Depending on the value of relative magnetic permeability μr, different materials can be divided into three groups.

1. \(\mu_r<1 \text {, i.e., } \mu<\mu_0\): Diamagnetic material;

2. \(\mu_r>1 \text {, i.e., } \mu>\mu_0\): Paramagnetic material;

3. \(\mu_r \gg 1 \text {, i.e., } \mu \gg \mu_0\): Ferromagnetic material.

For Example, in the case of aluminum, μr = 1.00002, hence it is a paramagnetic material; in the case of copper, μr = 0.9999904, hence it is a diamagnetic material; in the case of iron, μr = 1000 to 5000 (approx.), hence it is a ferromagnetic material. These materials are discussed elaborately later

Magnetization: If the magnetizing field at any point in vacuum or air is \(\vec{B}\), the magnetic field at that point, \(\vec{B}_0=\mu_0 \vec{H}\).

If vacuum or air at a point is replaced by any other material, magnetism is induced in that material and hence the magnetic field acting will also change from \(\vec{B}_0 \text { to } \vec{B}\) (say).

Naturally, \(\vec{B} \neq \mu_0 \vec{H}\). If the material is paramagnetic or ferromagnetic, \(\vec{B}>\mu_0 \vec{H}\). Here, it is assumed that,

⇒ \(\vec{B}=\mu_0 \vec{H}+\mu_0 \vec{M}=\mu_0(\vec{H}+\vec{M})\)….(4)

The first term on the right-hand side \(\left(\mu_0 \vec{H}\right)\) comes from the magnetizing field \(\vec{H}\) produced because of electric current or some other external cause, and tire second term \(\left(\mu_0 \vec{M}\right)\) comes due to induced magnetism inside the material.

So, the term \(\mu_0 \vec{M}\) indicates the additional magnetic field produced due to magnetic induction.

The quantity \(\vec{M}\) is called the intensity of magnetization of the material or simply the magnetization vector, the magnitude of which at a point is given as the net dipole moment per unit volume around that point.

By calculation, we can show that (this calculation is omitted here), the magnetic moment per unit volume of a material is \(\vec{M}\).

Definition: If the unit volume is considered around any point in a material, the magnetic moment of that volume is called the intensity of magnetization at that point. So, relative magnetic permeability

Unit: From equation (4) it is clear that the unit of \(\vec{M}\) is the same as the unit of \(\vec{H}\).

This unit is A.m-1. \(\vec{B}\), \(\vec{H}\) and \(\vec{M}\) are called the three magnetic vectors,

Magnetic susceptibility of material: In most materials, the intensity of magnetization at a point is directly proportional to the magnetic intensity at that point, i.e.,

⇒ \(M \propto H \quad \text { or, } M=k H\)

This constant fc is called the magnetic susceptibility of the material. The property by which a material can be magnetized is its magnetic susceptibility.

In vector form, \(\vec{M}\) = k\(\vec{H}\)….(5)

Now, if H = 1, k = M; from this we can define k.

Definition: The magnetic moment induced per unit volume of a material due to unit magnetic intensity is called the magnetic susceptibility of that material.

Unit: Since the units of M and H are the same, k has no unit.

Mass (magnetic) susceptibility: The ratio of the magnetic susceptibility of a material and its density is called the mass susceptibility of that material. It Is denoted by the symbol \(\chi\).

∴ Mass susceptibility,

⇒ \(x=\frac{k}{\rho}\) [where, ρ = density of the material]

Rotation between magnetic susceptibility and relative magnetic permeability:

We know that,

⇒ \(\vec{B}=\mu \vec{H}\)

⇒ \(\vec{B}=\mu_0(\vec{H}+\vec{M}) \text { and } \vec{M}=k \vec{H}\) [from equations (4) and (5)]

Combining them we can write,

⇒ \(\mu \vec{H}=\mu_0(\vec{H}+\vec{M})=\mu_0(\vec{H}+k \vec{H})=\mu_0(1+k) \vec{H}\)

∴ \(\frac{\mu}{\mu_0}=1+k \quad \text { or, } \mu_r=1+k\) [\(\mu_r=\frac{\mu}{\mu_0}\) = relative magnetic permeability]

or, k = μr – 1….(6)

For vacuum or air, μr = 1 , hence, k = 0

The values of μr and k of some materials:

Electromagnetism The values of μr and k of some materials

Hence, for paramagnetic material, k > 0 (positive); for diamagnetic material, k<0 (negative); for ferromagnetic material, k >> 0 (large positive number).

Significance:

1. In the case of paramagnetic and diamagnetic materials magnetic susceptibilities are very low and are respectively positive and negative numbers.

This means that magnetic induction in these materials is negligible. Thus, these are called non-magnetic materials.

2. On the other hand, in the case of ferromagnetic materials magnetic susceptibilities are large positive numbers.

Hence, in this kind of material, strong magnetic induction takes place. Thus, these are known as magnetic materials (e.g., iron, nickel, cobalt).

Since the value of k is very large, from the relation, \(\vec{B}=\mu_0(\vec{H}+\vec{M})=\mu_0(\vec{H}+k \vec{H})\), we come to know that the term pgM is much greater than the term p0H.

So, the magnitude of the magnetic field \(\vec{B}\) is mainly determined by induced magnetism.

In CGS or Gaussian system:

Equations (l) and (2) respectively, can be replaced by the relations, H0 = 4π ni [magnetic permeability of vacuum or air =1 ] and H = 4πμni [magnetic permeability of the material = μ] So, relative magnetic permeability, \(\mu_r=\frac{H}{H_0}=\mu\); so in this system, magnetic permeability and relative magnetic permeability of a material are the same.

Again, in this system equation (4) is written as,

⇒ \(\vec{B}=\vec{H}+4 \pi \vec{M}\)

But we know that, \(\vec{B}=\mu \vec{H}\)

So, \(\mu \vec{H}=\vec{H}+4 \pi k \vec{H} \quad[∵ \vec{M}=k \vec{H}]\)

⇒ \(\mu=1+4 \pi k \quad \text { or, } k=\frac{\mu-1}{4 \pi}\)

Magnetic retentivity and coercivity:

When a magnetic material is placed in a magnetic field, the material acquires magnetism due to induction. This magnetism does not vanish completely even after the withdrawal of the magnetic field; some amount of magnetism is left behind in the material.

Magnetic retentivity:

The property by which a magnetic material retains some magnetism in it even after the withdrawal of the magnetizing field is called the retentivity of that material.

The magnetism, retained in a magnetic material even after the removal of the magnetic field applied to it, is called residual magnetism.

There are some magnetic materials for which the residual magnetism is almost zero, i.e., they are almost completely demagnetized.

Magnetic coercivity: The property by which magnetic material can retain induced magnetism even if used roughly, i.e., subjected to demagnetizing forces, is called the coercivity of that material.

Differences between wrought iron and steel on magnetic properties:

If two identical rods one of soft iron and the other of steel-are placed in the same magnetic field, both of them acquire approximately equal amounts of magnetism.

If the rods are then removed from the magnetic field, both soft iron and steel retain almost the entire magnetism.

Soft iron can hold slightly more magnetism than steel. But if this soft iron magnet is handled roughly, its magnetism dies out easily compared to that of steel.

So, it can be said that the magnetic retentivity of soft iron is slightly greater, but the coercivity of soft iron is much less than that of steel.

Electromagnetism differences between wrought iron and steel on magnetic properties

A ferromagnetic material can be converted into a strong magnet easily. In the case of a ferromagnetic material like soft iron or steel, if a graph of intensity of magnetization (M) vs magnetic field intensity (H) is drawn, we will get a closed loop.

It is known as the magnetization cycle. In the case of steel, the loop is OABCDA and in the case of soft iron, the loop is OA’B’C’D’A’. From the graph, it is evident that the intensity of magnetization (M) is not zero ( OB or OB’) even if the magnetic field intensity (H) is reduced to zero from its maximum value, i.e., M lags behind H.

This lagging of the intensity of magnetization is called hysteresis. However, M falls to zero if H is given a certain value (OC or OC’) in the opposite direction. According to the diagram, OB is the retentivity of steel, OB’ is the retentivity of soft iron, OC is the coercivity of steel, and OC’ is the coercivity of soft iron.

Selection of material to construct a permanent magnet:

The magnetic material chosen to construct a permanent magnet should have the following properties m The material should have high retentivity so that it can retain sufficient magnetism even after the withdrawal of the magnetizing field.

If The saturation magnetization of the material should be high enough it can make a strong polarity.

The material should also have high coercivity so that it can retain induced magnetization, even if used roughly. The magnetic susceptibility of the material should be of high magnitude.

Though all of the above properties do not match properly, steel, rather than soft iron is used to construct permanent magnets.

Besides steel, there are some metallic alloys like alnico (Fe 51%, Cu 3%, A1 8%, Ni 14%, Co 24% ); national (Fe 47%, Cu 3%, A1 8%, Ti 2%, Ni 15%, Co 25% ), etc. possess the above-mentioned qualities and can be used to construct permanent magnets.

Selection of material to construct an electromagnet: To construct an electromagnet, a material that possesses the following properties should be chosen.

1. The material should have low retentivity so that it can lose almost all of its magnetism as soon as the applied magnetizing field is withdrawn.

2. The saturation magnetisation of the material should be high enough which can make a strong polarity.

3. The material must have low coercivity so that it can be easily demagnetized.

4. Hysteresis loss for the material should be low so that during magnetization and demagnetization the temperature of the material should remain more or less constant.

Soft iron or alloy (an alloy of 5% silicon and 95% iron) possesses these qualities and hence can be used as the core of an electromagnet.

Selection of material as the core of a transformer or dynamo:

To prepare the core of a transformer or a dynamo, a material of high magnetic permeability should be chosen.

Soft iron possesses such qualities, and hence it is used as the core. Moreover, metallic alloys, like permalloy (50% iron and 50% nickel) and transformer steel (96% iron and 4% silicon), are used nowadays for the same purpose.

Magnetic Effect Of Current And Magnetism

Magnetic Properties Of Materials Numerical Examples

Example 1. The number of turns of a solenoid of length 10 cm is 1000. If the air inside it is replaced by a magnetic material and 1 A current is passed through the coil, the magnitude of the magnetic field at any point on its axis becomes 20 T. Determine the magnetic Intensity at that point and relative magnetic permeability of the magnetic material.
Solution:

Number of turns per unit length of the coil,

⇒ \(n=\frac{1000}{10}=100 \mathrm{~cm}^{-1}=10000 \mathrm{~m}^{-1}\)

∴ Magnetic intensity on the axis

H = nI = 10000 x 1

= 10000 A.m-1

If the interior of the solenoid is a vacuum or contains air then the magnetic field on the axis,

B0 = μ0nI = (4π x 10-7) x 10000 = 12.56 x 10-3T

Due to the presence of the magnetic material,

B = μnl = 20 T

∴ Relative magnetic permeability,

⇒ \(\mu_r=\frac{\mu}{\mu_0}=\frac{B}{B_0}=\frac{20}{12.56 \times 10^{-3}}=1592\)

Example 2. The relative magnetic permeability of a magnetic medium is 1000. If the magnetic field at any point in the medium is 0.1 Wb.m-2, what will be the values of magnetic intensity and intensity of magnetization at that point?
Solution:

Here, B = 0.1 Wb.m-2 and μr – 1000.

So, μ = μ0 x μr

= 4π x 10-7 x 1000

= 4π x 10-4 H.m-1

Now, magnetic intensity, \(H=\frac{B}{\mu}=\frac{0.1}{4 \pi \times 10^{-4}}=79.6 \mathrm{~A} \cdot \mathrm{m}^{-1}\)

Again, B = μ0(H + M)

∴ The intensity of magnetization,

⇒ \(M=\frac{B}{\mu_0}-H=\frac{0.1}{4 \pi \times 10^{-7}}-79.6\)

= 79577.5 – 79.6

= 79497.9 A.m-1

Example 3. An iron-cored toroid has a ring radius of 7 cm and several turns of 500. If 2 A current is passed through the wire, what will be the value of a magnetic field on the axis of the toroid? Given, the relative magnetic permeability of iron = 1500.
Solution:

Length of the circular axis of the ring

⇒ \(2 \pi r=2 \times \frac{22}{7} \times 7=44 \mathrm{~cm}=0.44 \mathrm{~m}\)

∴ The number of turns per unit length of the toroid,

⇒ \(n=\frac{500}{0.44} \mathrm{~m}^{-1}\)

∴ Magnetic field on the axis of the toroid,

B = μ0nI

= μ0μrnI

⇒ \(\left(4 \pi \times 10^{-7}\right) \times 1500 \times \frac{500}{0.44} \times 2=4.28 \mathrm{~Wb} \cdot \mathrm{m}^{-2}\)

Magnetic Effect Of Current And Magnetism

Magnetic Properties Of Materials Paramagnetic Diamagnetic And Ferromagnetic Materials

Depending on the behavior of a material placed in an external magnetic field.

Michael Faraday first classified all materials into three groups:

  1. Paramagnetic material,
  2. Diamagnetic material
  3. Ferromagnetic material.

Paramagnetic Material:

The materials attracted feebly by a strong magnet are known as paramagnetic materials. Paramagnetic materials may be solid, liquid, or gaseous.

Examples: Solid elements like aluminum, potassium, platinum, sodium, tin, manganese, etc., copper sulfate, ferric chloride salts and their aqueous solutions, and gaseous materials like oxygen, air, etc.

The relative magnetic permeability of a paramagnetic material varies from 1 to 1.001 and its magnetic susceptibility has a very small positive value (10-4 or less).

Properties:

1. When a paramagnetic material is placed in a non-uniform magnetic field, it moves gradually from the weaker part to the stronger part of that magnetic field, i.e., the material is attracted feebly by the magnet

2. If a paramagnetic material is placed in a uniform magnetic field, the lines of force close up a little and pass through the material. As a result, the lines inside that material get crowded slightly.

So, the magnetic field (B), increases slightly inside a paramagnetic material.

Electromagnetism Paramagnetic Material

3. A small number of molecules of a paramagnetic material are magnetic dipoles. When heated, the random thermal motion of these dipoles increases, and hence their magnetic alignment is disturbed. As a result, both magnetic susceptibility and magnetic permeability decrease.

Magnetic susceptibility per unit mass of a paramagnetic material, \(\chi=\frac{k}{\rho}(\rho=\text { density })\) is inversely proportional to the absolute temperature; i.e., \(\chi=\frac{C}{T}\) (for a particular material C is constant).

This is known as Curie law. Experimental observations show that Curie’s law applies only to gases. In the case of paramagnetic solids, Curie- Weiss law is applicable.

According to this law,

Magnetic susceptibility \(\chi=\frac{C}{T-\theta}\)

where θ is a specific temperature known as the Curie temperature of that paramagnetic material. The values of Curie temperature are very small for almost all paramagnetic materials.

4. If a liquid is poured through one of the limbs of a vertical U-tube then the liquid rises to the same level in both limbs.

If the liquid is paramagnetic and if one of the limbs of the U-tube is placed between the two poles of a strong electromagnet, it is observed that the liquid rises in that limb. This is due to the attraction of paramagnetic liquid by a magnet.

Electromagnetism vertical U-tube

Diamagnetic Material:

The material repelled feebly by strong magnets are known as diamagnetic materials. Diamagnetic materials may be solid, liquid, or gaseous.

Examples: Antimony, bismuth, zinc, copper, silver, gold, lead, mercury, water, hydrogen, etc.

The relative permeability of a diamagnetic material is slightly less than 1 and its magnetic susceptibility is slightly negative (≥ -10-4).

Properties:

1. If a diamagnetic material is placed in a non-uniform magnetic field, it tries to move from the stronger region to the weaker region of the magnetic field. So, the material is fully repelled by a magnet.

2. If a diamagnetic, material is kept in a uniform magnetic field, the lines of force move away a little. As a result, the lines are more sparse inside the material than outside. For a sphere.

So, the magnetic field (B) decreases slightly inside a diamagnetic material.

Electromagnetism Diamagnetic Material

3. In a diamagnetic material, almost none of the molecules are magnetic dipoles. So the random molecular motion due to increasing temperature has a negligible effect on the magnetic properties.

An increase in the applied magnetic field also does not affect the magnetic alignment of molecules.

As a result, the magnetic susceptibility of a diamagnetic material does not depend on the applied magnetic field and temperature.

4. Theoretically it is proved that diamagnetism is present in every material. The diamagnetic property of a material is very much weaker than its paramagnetic and ferromagnetic properties.

As a result, despite the magnetic properties present in paramagnetic and ferromagnetic materials, diamagnetism remains dormant in them. Hence, it can be said that diamagnetism is the most fundamental magnetic property.

5. If some diamagnetic liquid is poured into a U-tube and if one of the limbs is now placed between the pole pieces of a strong magnet, the level of the liquid in the limb falls. This is due to the repulsion of diamagnetic liquid by the magnet.

Ferromagnetic Material:

Five metals iron (Fe), nickel (Ni), cobalt (Co), gadolinium (Gd), dysprosium (Dy), and alloys like steel, etc. are attracted strongly by any magnet. Even if the inducing magnetic field is removed, these materials can retain some induced magnetism. These are known as ferromagnetic materials.

The relative magnetic permeability of a ferromagnetic material is usually very high (102 to 106 ) and its magnetic susceptibility is also very high and positive (200 to 2000 approximately).

Though the magnetic properties of ferromagnetic materials are just like that of paramagnetic materials they show stronger paramagnetism and hence they are placed in a separate group.

Heusler alloy is a special kind of ferromagnetic material because none of its components (Cu, 64%, Mn 24%, A1 12% ) are ferromagnetic.

Properties:

1. Generally, ferromagnetic materials are solid and crystalline.

2. If a ferromagnetic material is placed in a non-uniform field, it moves very fast from the weaker to the stronger region of the field and sets itself parallel to the direction of the magnetic field. So, ferromagnetic materials are very strongly attracted by a magnet.

If a ferromagnetic material is placed in a uniform magnetic field, the lines of force crowd too much within the material than in the air.

3. So, the magnetic field (B) is very high inside a ferromagnetic material. The value of μ of a ferromagnetic material is also very high.

For Example, the relative magnetic permeability of iron is approximately 2000, for nickel it is 300, and for cobalt, it is 250. Magnetic susceptibility (k) of ferromagnetic material has a high positive value.

4. Magnetic permeability and magnetic susceptibility of a ferromagnetic material are not constants. They change with the change in the magnitude of the applied magnetic field.

5. Magnetic susceptibility of ferromagnetic materials changes with the temperature change.

If the magnetic field is weak, magnetic susceptibility increases with an increase in temperature. In a stronger field, the magnetic susceptibility decreases with an increase in temperature.

6. If a ferromagnetic material is heated gradually, at a particular temperature the material loses its ferromagnetic property completely and is converted into a paramagnetic material.

The transition temperature at which a ferromagnetic material is converted into a paramagnetic one is called the Curie point or Curie temperature for that material.

Above Curie point, magnetic susceptibility obeys Curie-Weiss law. Curie points of some materials are mentioned in Table.

Electromagnetism Curie-Weiss law

7. Even if the inducing magnetic field is removed, a ferromagnetic material can retain some induced magnetism, i.e., a ferromagnetic material possesses magnetic retentivity.

It should be mentioned here that, though a ferromagnetic material can be converted into a paramagnetic one by heating above Curie temperature, ferromagnetism, and paramagnetism are two separate phenomena.

A paramagnetic material cannot be converted into a ferromagnetic material by cooling. The change is irreversible.

Magnetic screen:

A magnetic screen is an arrangement that works on the property of ferromagnetism and with this, a region can be kept free from the influence of any external magnetic field.

If a magnetic needle is kept in front of any pole of a bar magnet, the needle gets deflected. But if a plate of soft iron is placed in between them, the magnetic needle suffers no deflection.

This is because the magnetic permeability of the plate of soft iron is much greater than that of air.

Hence, the magnetic lines of force emerging from the bar magnet try to traverse the maximum distance through the plate of soft iron, and hence the lines of force cannot cross over to the other side of the plate.

Therefore, the region beyond the plate remains free from the influence of the magnetic field.

Electromagnetism Magnetic screen

Similarly, if a soft-iron ring is placed in front of any pole of a bar magnet, most of the lines of force pass through the iron ring just avoiding the air gap inside the ring.

Since no line of force enters this region A inside the ring, it remains free from the influence of all external magnetic fields. If a magnetic needle is placed in that region, it sets itself at rest in any direction.

So, a magnetic material used to make a particular region free from the influence of any external magnetic field is called a magnetic screen.

Magnetic screens made of soft iron are used to protect delicate measuring instruments like galvanometers, ammeters, etc., from external magnetic fields. To make a costly wristwatch antimagnetic, a soft iron ring is fitted around it.

Comparison of Ferromagnetic, Paramagnetic and Diamagnetic Materials:

Electromagnetism comparison of ferromagnetic paramagnetic and diamagnetic meterials

Magnetic Effect Of Current And Magnetism

Magnetic Properties Of Materials Geomagnetism

A freely suspended magnet always sets itself at rest along the north-south direction. If disturbed, it will again come to the previous position after a few oscillations.

This phenomenon is observed anywhere on the earth. Since only a magnetic field can influence a magnet, we can infer that a magnetic field exists throughout the earth, i.e., the earth behaves as a magnet.

The following two phenomena can be mentioned in support of this concept:

1. If a soft iron rod is kept at a place on the surface of the earth facing north-south direction, after a long period (say, six months), a feeble magnetism is found to have developed in the rod.

2. If a closed conductor is moved in a magnetic field an electric current is induced in that conductor. Similarly, if a closed conductor is moved at any place on the earth’s surface, a feeble electric current is seen to be induced in that conductor.

The Earth is a huge magnet: To explain the cause of the above-mentioned phenomena, scientists concluded that the Earth behaves as a huge magnet In 1600, physicist William Gilbert suggested this theory for the first time.

Later on, Gilbert performed a simple experiment. He shaped a lodestone into a sphere and demonstrated that small magnets placed at different positions on the sphere behave exactly as they do on Earth.

Locations of the Earth’s magnetic poles: Like an ordinary magnet, the Earth’s magnet also has two poles. The north pole of Earth’s magnet is situated at Ellef Ringnes Island in Canada.

The latitude and longitude of this region are 78.3°N and 104° W. This region is at an approximate distance of 1300 km from the geographical north pole.

The south pole of the earth’s magnet is located in the sea near the sea-shore of Wilkes Land belonging to Antarctica.

The latitude and longitude of this region are 65°S and 139°E. This region is at a distance of about 2550 km from the geographical south pole.

Electromagnetism Locations of the earth's magnetic poles

These poles are not stationary. The north pole shifts towards the northwest by 15 km every year; the shifting of the south pole is nearly the same. At present times, the magnetic axis of the earth is inclined at an angle of 11.5° with its geographical axis and the distance of the earth’s magnetic axis from the center of the earth is about 1120 km.

Nature of Earth’s magnetic poles:

The pole of a magnetic needle that points towards the north is called the north-seeking pole or simply the north pole, and the pole that points towards the south is called the south-seeking pole or simply the south pole.

The magnetic poles of Earth at the north and the south are called the magnetic north pole and magnetic south pole. Now, the north and south poles of a magnetic needle direct themselves towards the magnetic north and south poles of the earth.

It means that the north pole of the earth behaves as the south pole of an ordinary magnet and the south pole of the earth as the north pole of an ordinary magnet Sometimes the north pole of the earth’s magnet is called the blue pole and its south pole is called a red pole.

Geomagnetic Field:

The magnetic field of the earth extends up to a great height above its surface. Practically, the influence of this field is found up to a height of 105 kilometers (approx.) above the earth’s surface.

There was no clear concept about the source of the geomagnetic field for a long time. If we imagine a bar magnet kept inclined at an angle of 11.5° with the geographical axis at the center of the earth, it is possible to describe the alignment of the geomagnetic field more or less correctly. For a hypothetical bar magnet, the nature of the lines of force.

Electromagnetism Geomagnetic Field

Sources of geomagnetic field: The geomagnetic field has three different sources. From these three sources, the following three magnetic fields are produced.

1. Main field: The source of this magnetic field is the outer core of Earth which is made of liquid iron. The electric current in this part produces the main field. The main field develops 97% to 99% of the geomagnetic field.

2. Crustal field: The source of this magnetic field is the earth’s crust Some parts of the earth’s crust made of hard rock become magnetized due to the presence of the main field. This magnetized part creates the crustal field. 1% to 2% of the geomagnetic field is due to this crustal field.

3. External field: The source of this magnetic field is the ionosphere present in the atmosphere of Earth. This part made up of ions, produces the external field under the influence of solar wind. 1% to 2% of the geomagnetic field is due to this external field.

Direction of the geomagnetic field: Magnetic lines of force of the geomagnetic field are naturally from the N-pole to the S-pole. But the geomagnetic N-pole is in the south and the S-pole is in the north, As a result, to show the direction of the geomagnetic field at a place, the arrow sign on each line of force should be from the south to the north direction.

Elements of Earth’s Magnetism:

To know the magnitude and direction of the geomagnetic field at any place on Earth, the quantities required are called the elements of Earth’s magnetism. There are three elements of earth’s magnetism.

These are:

  1. Angle of dip
  2. Angle of declination
  3. Horizontal component of geomagnetic intensity.

Dip or angle Of dip: The angle made by the intensity of the earth’s magnetic field with the horizontal at any place on the earth is called dip or angle of dip at that place. If a bar magnet is suspended from its center of gravity by a thread, the magnet at rest, does not remain horizontal but inclines a little.

This means that the magnetic axis sets’ itself along the direction of the intensity of the geomagnetic field at that place. The angle (θ) made by the magnetic axis with the horizontal straight line drawn on the magnetic meridian is called dip or the angle of dip at that, place.

So, if the angle of a place is known,’ We can determine the direction of the intensity of the magnetic field at that place.

Electromagnetism Dip or angle Of dip

Positive and negative dips: The inclination of a magnet is different at different places on Earth, i.e., the values of the dip are different at different places. If the north pole of a magnet leans downwards at a place, the value of the dip is taken as positive; but if the south pole of the magnet leans downwards, the dip at that place is taken as negative.

Positive and negative dips are denoted by the symbols N and S, respectively. For any place in the northern hemisphere of the earth, the angle of dip becomes positive, but for places in the southern hemisphere, this angle of dip is negative.

‘The angle of dip at Kolkata is 31° N’, which means that if a magnet is suspended from its center of gravity at Kolkata, the north pole of the magnet leans downwards and the magnetic axis of the magnet inclines at an angle of 31° with the horizontal plane.

At the two magnetic poles of the earth, angles of dip are 90° each, i.e., at these two places, a freely suspended magnet remains vertical. At the magnetic equator, the angle of dip is 0°, i.e., a freely suspended magnet remains horizontal at the magnetic equator.

Declination or angle of declination: The geographical poles and the magnetic poles of the earth are not located in the same places. So if a magnetic needle, capable of rotating in the horizontal plane freely, is kept at a place, we will observe that, at rest, the magnetic axis of that needle makes a definite angle with the geographical north-south horizontal line at that place.

At different places on the earth’s surface, the value of this angle is different The vertical plane passing through the magnetic axis of a freely suspended magnetic die at any place is called the magnetic meridian at that place. Again, the vertical plane at a place containing the geographical north and south poles of Earth is called the geographical meridian at that place.

Electromagnetism Declination or angle of declination

Definition: The angle between the magnetic meridian and the geographical meridian at a place is called the angle of declination at that place.

It is usually denoted by the symbol δ. If the north pole of the magnetic needle turns towards the east or the west concerning the geographical meridian, the angle of declination is called \(\delta^{\circ} E \text { or- } \delta^{\circ} W\), respectively.

For Example, the angle of declination of Delhi is 2°E means that at Delhi, the north pole of a magnetic needle capable of rotating freely on a horizontal plane moves away from the geographic meridian towards east through 2°.

Naturally, the angles of declination at different places on the earth are different. At a place, where the magnetic meridian and geographical meridian coincide, the angle of declination becomes zero.

Horizontal component of geomagnetic intensity: The direction of the geomagnetic intensity at a place does not lie on the horizontal plane usually, but it inclines at a definite angle with the horizontal plane.

This definite angle is called the angle of dip. Since magnetic intensity is a vector quantity, the geomagnetic intensity can be resolved into a horizontal component and a vertical component It should be clear that these two components lie on the magnetic meridian.

ABCD is the geographical meridian and GBCJ is the magnetic meridian. At point B, the magnitude and direction of the geomagnetic intensity I can be represented by BR. The magnitudes and directions of the vertical component V and the horizontal component H of I can be expressed by BM and BN, respectively.

Electromagnetism Horizontal component of geomagnetic intensity

Let the angle of dip be 8 and the angle of declination be δ.

∴ V = Isinθ and H = Icosθ

or, \(\frac{V}{H}=\frac{I \sin \theta}{I \cos \theta}=\tan \theta\)

∴ V = Htanθ…(1)

Again, V² + H² = I²sin²θ + I²cos²θ = I²

∴ \(I=\sqrt{V^2+H^2}\)….(2)

The value of the horizontal component H of geomagnetic intensity is not the same throughout the earth’s surface face. At the magnetic equator, θ = .0° and hence H = I; this is the maximum value of H. Again, at the magnetic poles, θ = 90° and hence H = 0; this is the minimum v value of H. Note that, at a place where the angle of dip is 45°, the values of horizontal and vertical components are equal.

Electromagnetism Horizontal component of geomagnetic intensity.

The elements of earth’s magnetism for the northern hemisphere. In the case of the southern hemisphere, the elements.

‘Horizontal component of earth’s magnetic field at Kolkata is 0.37 Oe’ means that the magnitude of the horizontal component of the geomagnetic intensity, i.e., the force acting on a unit pole along the magnetic meridian at that place is 0.37 Dyn.

In most cases, we require the horizontal component of the geomagnetic Intensity, the vertical component Is of less importance. This is because the magnetic needles we use in the laboratory, can rotate in the horizontal plane but not In the vertical plane.

As a result,’ only the horizontal component acts on the needle to create deflection in it, but the vertical component does not affect it,

Values of the geomagnetic elements at some places:

Electromagnetism value of the geomagnetic elements at some places

Mariner’s Compass:

Mariner’s compass, as the name suggests, is used to ascertain the direction in the sea when the sun, the pole star, or other stars are not visible.

The construction of a compass is based on the directive property of a magnet.

Working principle: Noting the position of the crown mark on the compass disc, navigators determine the direction. The crown mark on the compass disc indicates the magnetic north.

To determine the geographical north at a place, the angle of declination at the place should be collected from the magnetic maps. If the value of that declination is δ°W, it should be understood that the crown mark lies at an angle of δ°, west of the geographical north.

In this way from the position of the crown mark on the compass disc, navigators can determine directions.

Electromagnetism Mariner’s Compass

Magnetic Maps: F

Values of the elements of Earth’s magnetism are different at different places on Earth. But the value of any one of the elements at different places on earth’s surface may be the same.

The places on the earth’s surface possessing the same value of a particular element are along a line. Thus different lines are drawn for different values of that element.

The geographical map of Earth containing such lines is called a magnetic map. For three different elements of Earth’s magnetism, three different magnetic maps are obtained.

The values of Earth’s magnetic elements at a particular place change gradually with time. Hence new magnetic maps should be drawn at different times. Maps of this kind are essential to navigators and also for searching minerals under the earth’s crust Three different kinds of magnetic maps.

Isogonic lines:

Places on earth’s surface, having equal declination are joined by lines, called isogonic lines. The lines shown are isogonic lines drawn in 2000 AD. The lines of zero declination are called agonic lines.

Isoclinic lines:

The lines obtained by joining the places on the earth’s surface having equal dips are called isoclinic lines. The lines shown are isoclinic lines drawn in -2000 AD. The line of zero angle of dip corresponds to the magnetic equator, and it is called an aclinic line.

Isodynamic lines:

Places on the earth’s surface having equal horizontal components of geomagnetic field intensity are joined by lines, called isodynamic lines. The lines are isodynamic lines drawn in 2000 AD.

Electromagnetism Magnetic Maps..

Electromagnetism Magnetic Maps.

Electromagnetism Magnetic Maps

Variations of the Elements of Geomagnetism:

The values of the elements of geomagnetism are different at different places on the earth. Again, at a particular place, the values of these elements do not remain the same always; they vary with time. This change is periodic, i.e., after a definite period, the elements return to their initial values. This kind of variation is known as periodic or regular variation.

Besides this, the elements of geomagnetism suffer another kind of variation. This kind of variation is known as a magnetic storm.

Regular variation: Regular variations are of three kinds

  1. Daily variation,
  2. Annual variation
  3. Secular variation.

Daily variation: The elements of geomagnetism undergo a kind of slow variation daily. At two different specific times on a day, any element of geomagnetism attains maximum and minimum values.

Annual variation: The elements of geomagnetism also undergo a kind of very slow variation. On two specific months in a year, the value of any element becomes maximum and minimum.

For Example, in London, the declination is found to have a maximum value in February and a minimum value in August every year. Variations of declination in the northern and southern hemispheres are just the opposite.

Secular variation: A kind of secular variation is observed in the elements of geomagnetism. The rate of this kind of variation is very slow, and its period is approximately 960 years. Perhaps this variation is due to the rotation of the earth’s magnetic north and south poles around its geographical south and north poles, respectively.

Magnetic storm: Sometimes sudden huge changes in the elements of geomagnetism are seen throughout a large region on the earth’s surface. This phenomenon is known as a magnetic storm. This kind of variation cannot be predicted.

However, after a while, the values of the elements return to their normal state. Magnetic storms usually occur due to earthquakes, volcanic eruptions, aurora borealis, the appearance of large sunspots, etc. During a magnetic storm, radio communication, television, telephone systems, etc. are disturbed greatly.

Lines of Force of a Bar Magnet in Earth’s Magnetic Field Neutral Points:

In the discussions of the magnetic lines of force of a bar magnet, the influence of the earth’s magnetic field has not been taken into account so far. The lines of force for the geomagnetic field at a place remain parallel to the magnetic meridian at that place and the direction of the lines of force is from south to north. The pattern of the lines of force near a bar magnet gets distorted under the influence of the geomagnetic field. A few cases are shown below.

N-pola pointing north: Let a bar magnet be placed along the magnetic meridian in such a manner that its 4-pole points north. In this case, the pattern of the magnetic lines of force due to the combined effect of the geomagnetic field and the magnetic field due to the bar magnet is shown.

Electromagnetism N-pola pointing north

In the vicinity of the magnet, the lines of force are curved. In this region, the influence 6f the .bar magnet is more effective. The greater the distance from the magnet, the lesser its influence but the greater the influence of the geomagnetic field.

At a sufficient distance from the magnet,” its influence almost vanishes. In that region, ‘the lines of force due to geomagnetic field only. Hence, those lines of force are straight, parallel, and directed from south to north

At different points on the axis of the bar magnet, as the lines of force due to the bar magnet and geomagnetic field are; in the same direction, the value of the magnetic intensity increases.

But at different points on the perpendicular bisector of the magnetic axis, as the lines of force due to the geomagnetic field and the bar magnet are opposite in direction, the value of the magnetic intensity decreases.

On this perpendicular bisector, there are two points X and, X where the intensities due to the geomagnetic field and the magnetic- field of the magnet become equal and opposite.

As a result, these two intensities cancel each other; i.e., at these two points, the resultant magnetic intensity becomes zero. These two points are known as neutral points.

They lie at equal distances on either side of the magnet. A magnetic needle placed at any of these two natural points does not show any ‘directive property. Naturally, ho line of force passes through the neutral points.

Neutral point: A point in a magnetic field where the resultant magnetic intensity due to the superposition of two or more magnetic fields becomes zero is called a neutral point.

When the N-pole of the bar magnet points north, at the neutral point,

F = H

or, \(\frac{p_m}{\left(d^2+l^2\right)^{3 / 2}}=H\) [in CGS system]

[where, F-resultant intensity due to the bar magnet, H = horizontal component of geomagnetic intensity, pm = magnetic moment of the bar magnet, d distance, of the neutral point from the center of the bar magnet, and 2l = magnetic length of the bar magnet.]

For a tiny bar magnet, \(\frac{p_m}{d^3}=H\). Usually, if the value of d is more than ten times l, then l2 can be neglected compared to d2.

S-pola pointing north: Let a bar magnet be placed along the magnetic meridian in such a way that its S-pole points north. In this case, the pattern of the magnetic lines of force is due to the combined effect of the geomagnetic field and magnetic field due to the bar magnet.

Electromagnetism S-pola pointing north

Here the neutral points (X, X) lie on the axis of the bar magnet at equal distances from its two ends.

When the S-pole of the bar magnet points north, at neutral points,

⇒ \(F=H \quad \text { or, } \frac{2 p_m d}{\left(d^2-l^2\right)^2}=H \text { [in CGS system] }\)

[where, F = intensity of the bar magnet, H = horizontal component of the geomagnetic field, pm = magnetic moment of the bar magnet, d = distance of the neutral point from the center of the magnet, and 21 = magnetic length of the bar magnet]

For a tiny bar magnet, \(\frac{2 p_m}{d^3}=H\). If the value of d is more than

10 times of l, then l2 can be neglected compared to d2.

N – pole pointing east: The north pole of a bar magnet points east, the pattern of magnetic lines of force due to the combined effect of the geomagnetic field and the magnetic field of the bar magnet. The two points X, X at the northwest and southeast of the bar magnet are the neutral points. These are at equal distances from the center of the magnet.

Electromagnetism N-po!e pointing east

N-pole pointing west: If the north pole of a bar magnet points west, the pattern of the magnetic lines of force is due to the combined effect of the geomagnetic field and magnetic field due to the bar magnet. The two points X, X at the northeast and southwest of the magnet are the neutral points. Their distances from the center of the magnet are equal.

Electromagnetism N-pole pointing west

Magnetic Effect Of Current And Magnetism

Magnetic Properties Of Materials Numerical Examples

Example 1. The angle of dip at a place is 30° and the horizontal component of the earth’s magnetic field at that place is 0.39 CGS units. Determine the vertical component of the earth’s magnetic field at that place.
Solution:

Here, θ = 30° and H = 0.39 CGS units.

If the intensity of the earth’s magnetic field is I, then

⇒ \(H=I \cos \theta \quad \text { or, } I=\frac{H}{\cos \theta}\)

Vertical component,

⇒ \(V=I \sin \theta=\frac{H}{\cos \theta} \cdot \sin \theta=H \tan \theta\)

⇒ \(0.39 \times \tan 30^{\circ}=0.39 \times \frac{1}{\sqrt{3}}=0.225 \text { CGS units }\)

Example 2. At two places, the angles of dip are 30° 1V and 30° S and the intensity of the earth’s magnetic field is 0.42 Oe. Determine the horizontal and the vertical components of the earth’s magnetic field at these two places and also indicate their directions with the help of a diagram.
Solution:

At the first place, θ = 30° N, and at the second place, θ = 30°S.

The intensity of the earth’s magnetic field at both places, I = 0.42 Oe

Horizontal component,

⇒ \(H=I \cos 30^{\circ}=0.42 \times \frac{\sqrt{3}}{2}=0.364 \mathrm{Oe}\)

and vertical component,

⇒ \(V=I \sin 30^{\circ}=0.42 \times \frac{1}{2}=0.210 \mathrm{e}\)

Electromagnetism Example 2 At two places the angles of dip

Example 3. At a place, the horizontal and vertical components of the earth’s magnetic field are 0.3 Oe and 0.2 Oe, respectively. Determine the resultant intensity and angle of dip there.
Solution:

Here, H = 0.3 Oe, V = 0.2 Oe.

If the resultant intensity is I then

⇒ \(I=\sqrt{H^2+V^2}=\sqrt{(0.3)^2+(0.2)^2}=\sqrt{0.13}=0.3605 \mathrm{Oe}\)

If the angle of dip is θ then,

⇒ \(\tan \theta=\frac{V}{H}=\frac{0.2}{0.3}=0.6667\)

∴ \(\theta=\tan ^{-1}(0.6667)=33.69^{\circ}\)

Example 4. The angle of dip and the horizontal component of the earth’s magnetic field at a place is 30° S and 0.36 Oe. Determine the magnitude and direction of the vertical component of the earth’s magnetic field at that place.
Solution:

Here, θ = 30° S. So, in equilibrium, the south pole of the magnetic needle leans downwards.

We know that, tanθ = \(\frac{V}{H}\)

∴ V = vertical component of the earth’s magnetic field,

= Htanθ

= 0.36tan30° [∵ H = 0.36 Oe]

⇒ \(0.36 \times \frac{1}{\sqrt{3}}\)

= 0.208 Oe

Since the north pole of the magnetic needle lies above the horizontal line, the direction of the vertical component (V) will be vertically upwards.

Electromagnetism Example 4 The angle of dip and the horizontal

Example 5. At a place, the angle of declination is 30°E and the angle of dip is 45°N. Determine the horizontal and vertical components of the geomagnetic intensity In the geographical meridian at that place. Given, the horizontal component of earth’s magnetic held at that place = 0.3 Oe.
Solution:

Here, 8 = 30°E, 0 = 45°A’ and H = 0.3 Oe.

If the horizontal component of the earth’s magnetic field in the geographical meridian is H’

Electromagnetism Example 5 At a place the angle of declination

⇒ \(H^{\prime}=H \cos \delta=0.3 \cos 30^{\circ}=0.3 \times \frac{\sqrt{3}}{2}=0.2598 \mathrm{Oe}\)

The vertical component remains the same in the geographical and the magnetic meridian planes, hence

V = Htanθ

= 0.3 tan45°

= 0.3 x 1

= 0.3 Oe

Example 6. The mass of a magnetic needle is -7.5 g and its magnetic moment is 98 units. To keep the magnetic needle horizontal in the northern hemisphere, what should be the position of its fulcrum concerning its center of gravity? The vertical component of the earth’s magnetic field = 0.25 Oe.
Solution:

Let the fulcrum be kept at a distance x (towards the north pole) from the center of gravity to keep the magnetic needle horizontal.

If the length of the magnetic needle = 2l and the strength of each pole = m, magnetic moment, pm = m.21.

∴ In equilibrium,

mV.2l = W.x

or, V.pm = W.x

or, \(x=\frac{V^{:} \dot{p}_m}{W}=\frac{0.25 \times 98}{7.5 \times 980}\)

= 0.0033cm

Electromagnetism Example 6 The mass of a magnetic needle

Example 7. The magnetic moment of a magnetic needle of mass 3.2 g is 980 CGS units. From which point should the needle be hung so that it will remain horizontal in the magnetic meridian? The horizontal component of the earth’s magnetic field at that place is 0.32 Oe and the angle of dip = 45°N. [g = 980 cm.s-2]
Solution:

Let the magnetic needle be hung from a point at a distance x from its center of gravity (towards the north N pole)

Electromagnetism Example 7 The magnetic moment of a magnetic needle

In equilibrium,

mV x 2l = W.x

or, V.pm = W.x [∵ Pm = m.2l]

or, \(x=\frac{V \cdot p_m}{W}=\frac{H \tan \theta \cdot p_m}{W}\) [∵ V = Htanθ]

⇒ \(=\frac{0.32 \times \tan 45^{\circ} \times 980}{3.2 \times 980}\)

= 0.1cm

Example 8. The angle of dip at a place = θ; if the angle of dip in a vertical plane making angle δ with the magnetic meridian be θ’, show that tanθ’: tanθ = Secδ: 1
Solution:

If the true dip angle in the magnetic meridian is θ, the vertical and horizontal components of earth’s magnetic field are V and H, respectively then,

⇒ \(\tan \theta=\frac{\dot{V}}{H}\)

In a vertical plane inclined at an angle δ with the magnetic meridian, the horizontal component of intensity,

H’ = Hcosδ, apparent dip at that plane = θ’ and vertical component = V.

∴ \(\tan \theta^{\prime}=\frac{V}{H^{\prime}}=\frac{V}{H \cos \delta}=\tan \theta \cdot \sec \delta\)

or, \(\frac{\tan \theta^{\prime}}{\tan \theta}=\sec \delta\)

∴ \(\tan \theta^{\prime}: \tan \theta=\sec \delta: 1\)

Example 9. At a place, the apparent geomagnetic dip in a vertical plane is 40°, and in another plane, perpendicular to it is 30°. What is the real dip at the place? Similar problem: If θ1 is the angle of dip of the magnetic axis of a magnetic needle with horizontal at any vertical plane and θ2 is that in another vertical plane at right angles to the former, prove that the real angle of dip, θ is given by \(\cot ^2 \theta=\cot ^2 \theta_1+\cot ^2 \theta_2\).
Solution:

If the horizontal and vertical components of the earth’s magnetic field are H and V, respectively and the true dip angle at that place is θ then,

⇒ \(\tan \theta=\frac{V}{H}\)…..(1)

M is the magnetic meridian and X, and Y are two vertical planes inclined at a right angle to each other.

The angle between the planes X and M is δ. If the apparent dip in the plane X is θ1 then,

Electromagnetism Example 9 At a place the apparent geomagnetic dip in a vertical

⇒ \(\tan \theta_1=\frac{V}{H \cos \delta}\)….(2)

If the apparent dip in plane Y is θ2 then,

⇒ \(\tan \theta_2=\frac{V}{H \sin \delta}\)…..(3)

From equations (2) and (3) we get,

⇒ \(\cot ^2 \theta_1+\cot ^2 \theta_2=\frac{H^2}{V^2}\left(\cos ^2 \delta+\sin ^2 \delta\right)=\frac{H^2}{V^2}\)

= cot²θ [from equation (1)]

Here, θ1 = 40° and θ2 = 30°.

∴ \(\cot ^2 \theta=\cot ^2 40^{\circ}+\cot ^2 30^{\circ}\)

or, \(\cot \theta=\sqrt{1.42+3}=\sqrt{4.42}=2.1\)

or, θ = cot-1(2.1)

= 25.46°

Example 10. A bar magnet of length 6 cm is kept vertically with its north pole on the ground. If the distance of the neutral point on the ground is 8 cm from the north pole, what will be the magnetic moment of that magnet? [If = 0.36 CGS units]
Solution:

Let O be the neutral point

Here, NS = 6 cm, NO = 8 cm

So, \(S O=\sqrt{6^2+8^2}=10 \mathrm{~cm}\)

If the pole strength of the magnet NS is qm, magnetic intensity at the point O due to the north pole,

⇒ \(H_1=\frac{q_m}{O N^2} \text {, in the direction of } O A \text { [in CGS] }\)

Electromagnetism Example 10 A bar magnet of length

Again, due to the south pole magnetic intensity at the point O,

⇒ \(H_2=\frac{q_m}{O S^2} \text {, in the direction of } O S\)

∴ Component of H2 in the direction ON,

⇒ \(H_2 \cos \theta=\frac{q_m}{O S^2} \cdot \frac{O N}{O S}\)

So, the horizontal magnetic intensity at the point O due to the entire magnet,

⇒ \(H_1-H_2 \cos \theta=\frac{q_m}{O N^2}-\frac{q_m}{O S^2}: \frac{O N}{O S}=q_m\left(\frac{1}{8^2}-\frac{1}{10^2} \times \frac{8}{10}\right)\)

= qm x 0.007625 Oe

Since O is the neutral point, the magnetic intensity at that point due to the magnet will be equal but opposite to the horizontal component H of Earth’s magnetic field.

∴ \(q_m \times 0.007625=0.36 \quad \text { or, } q_m=\frac{0.36}{0.007625} \mathrm{emu} \cdot \mathrm{cm}\)

∴ Magnetic moment of the magnet

⇒ \(q_m \cdot N S=\frac{0.36}{0.007625} \times 6=283.3 \mathrm{emu} \cdot \mathrm{cm}^2\)

Example 11. A bar magnet of length 8 cm is placed on a horizontal plane in the magnetic meridian with its north pole pointing north. If the magnetic moment of the magnet is 90 CGS units and the horizontal, component of the earth’s magnetic field is 0.35 Oe, determine the positions of the neutral points.
Solution:

Let each of the two neutral points be at a distance d from the center of the magnet along its perpendicular bisector. If the magnetic moment of the magnet is pm and its magnetic length is 2l, the magnetic intensity at the neutral points due to the magnet (in the CGS system) is,

⇒ \(H_m=\frac{p_m}{\left(d^2+l^2\right)^{3 / 2}}\)

At the neutral points, the horizontal component H of the earth’s magnetic field will be equal and opposite.

∴ \(H=H_m=\frac{p_m}{\left(d^2+l^2\right)^{3 / 2}} \quad \text { or, }\left(d^2+l^2\right)^{3 / 2}=\frac{p_m}{H}\)

or, \(d^2+l^2=\left(\frac{p_m}{H}\right)^{2 / 3} \quad\)

or, \(d=\sqrt{\left(\frac{p_m}{H}\right)^{2 / 3}-l^2}\)

Here, pm = 90 CGS units, H = 0.35 Oe, 2l = 8 cm i.e.,
I = 4 cm.

∴ \(d=\sqrt{\left(\frac{90}{0.35}\right)^{2 / 3}-4^2}=4.94 \mathrm{~cm}\)

Example 12. At a place, the vertical component of the earth’s magnetic field is V3 times its horizontal component. What will be the angle of drop at that place?
Solution:

If the intensity of the earth’s magnetic field is I and the angle of dip is θ, then the horizontal component of the earth’s magnetic field, H = Icosθ, and a vertical component, V = Isinθ

According to the problem,

⇒ \(V=\sqrt{3} H \quad\)

or, \(\frac{V}{H}=\sqrt{3} \quad\)

or,\(\frac{I \sin \theta}{I \cos \theta}=\sqrt{3}\)

or, \(\tan \theta=\sqrt{3}=\tan 60^{\circ}\)

∴ θ = 60°

Magnetic Effect Of Current And Magnetism

Magnetic Properties Of Materials Very Short Questions and Answers

Question 1. The magnetic moment of a small bar magnet is 1 A.m2. What will be the magnitude of the magnetic field at a point 1 m away from the center of the magnet along its length?
Answer: 2 x 10-7T

Question 2. If the magnetic moment of a bar magnet of length 5 cm is 1 A.m2, what is the strength of each pole?
Answer: 20 A.m

Question 3. Pole-strengths of two magnetic poles are 1 A.m and 2 A.m. If they are kept 1 m apart in the air, then what is the magnitude of the force acting between them?
Answer: 2 x 10-7N

Question 4. A short bar magnet, placed with its axis at an angle 6 with a uniform external magnetic field B, experiences a torque r. What is the moment of the magnet?
Answer: \(\frac{\tau}{B \sin \theta}\)

Question 5. What is the unit of magnetic susceptibility?
Answer: No unit

Question 6. What is the unit of intensity of magnetization \(/vec{M}\)?
Answer: A.m-1

Question 7. If the relative magnetic permeability of a material is 1,00004, what will be its magnetic susceptibility?
Answer: 0.00004

Question 8. If the magnetic intensity at a point in a material is 100 A.m-1 and the magnetic susceptibility is 1000, what will be the value of the magnetic field at that point?
Answer: 4π x 10-2T

Question 9. If the magnetic susceptibility of a material is -0.0002, what is its relative magnetic permeability?
Answer: 0.99998

Question 10. The area of the hysteresis loop of a magnetic material A is larger than that of another magnetic material B. Which of the two materials is better suited for use as the core of an electromagnet?
Answer: Material B

Question 11. The relative permeability of iron is 5500. What is its magnetic susceptibility?
Answer: 5499

Question 12. The Curie temperature of nickel is 360°C. In which group of magnetic materials will you place nickel at 500°C?
Answer: Paramagnetic

Question 13. Name a non-magnetic alloy of iron.
Answer: Stainless Steel

Question 14. Name a material that is used as the core of an electromagnet.
Answer: Soft Iron

Question 15. For which type of material the magnetic susceptibility is negative?
Answer: Diamagnetic

Question 16. For which type of material, the magnetic susceptibility is independent of temperature?
Answer: Diamagnetic

Question 17. How does the magnetic susceptibility per unit mass (\(\chi\)) of a paramagnetic gas depend on absolute temperature T?
Answer: \(\chi \propto \frac{1}{T}\)

Question 18. If the intensity of the geomagnetic field at a place is 60 A.m-1 and the horizontal component of that intensity is 30 A.m-1, determine the angle of dip.
Answer: 60°

Question 19. Where on the surface of the earth, the value of the angle of dip is zero?
Answer: Geomagnetic equator

Question 20. Where on the earth’s surface, the value of the angle of dip is 90°?
Answer: Geomagnetic poles

Question 21. The vertical component of the earth’s magnetic field at a place is √3 times the horizontal component. What is the angle of dip at this place?
Answer: 60°

Question 22. The horizontal component of the earth’s magnetic field at a place is V3 times the vertical component. What is the angle of dip at this place?
Answer: 30°

Magnetic Effect Of Current And Magnetism

Magnetic Properties Of Materials Fill in The Blanks

1. The ratio of the magnetic field \(\vec{B}\) at a point in a material to the magnetic permeability of that material is called the intensity of the magnetic field \(\vec{H}\) at that point.

2. The magnetic moment per unit volume of a material due to unit magnetic intensity is called the magnetic susceptibility of that material.

3. The horizontal component of the earth’s magnetic field exists in all regions except at the geomagnetic poles

Magnetic Effect Of Current And Magnetism

Magnetic Properties Of Materials Assertion Reason Type

Direction: These questions have Statement 1 and Statement 2. Of the four choices given below, choose the one that best describes the two statements.

  1. Statement 1 is true, Statement 2 is true; Statement 2 is a correct explanation for Statement 1.
  2. Statement 1 is true, Statement 2 is true; Statement 2 is not a correct explanation for Statement 1.
  3. Statement 1 is true, Statement 2 is false.
  4. Statement 1 is false, and Statement 2 is true.

Question 1.

Statement 1: The magnetic moment of an electron rotating in an atom is proportional to its angular momentum.

Statement 2: The electrons in an atom can rotate only in those orbits for which the angular momentum of the electron is an integral multiple of \(\frac{h}{2 \pi}\)(h= Planck’s constant).

Answer: 2. Statement 1 is true, Statement 2 is true; Statement 2 is not a correct explanation for Statement 1.

Question 2.

Statement 1: Soft iron is preferred for making electromagnets.

Statement 2: Both the permeability and retentivity of soft iron are high.

Answer: 3. Statement 1 is true, Statement 2 is false.

Question 3.

Statement 1: The magnetic lines of force inside a piece of copper placed in a uniform magnetic field move away from each other.

Statement 2: The permeability of diamagnetic material is less than one.

Answer: 1. Statement 1 is true, Statement 2 is true; Statement 2 is a correct explanation for Statement 1.

Question 4.

Statement 1: The intensity of Earth’s magnetic field may be different even if the horizontal component of Earth’s magnetic field is the same at two different places.

Statement 2: The horizontal component of the earth’s magnetic field is Isinθ where I am the intensity of the earth’s magnetic field and θ is the angle of dip at a place.

Answer: 3. Statement 1 is true, Statement 2 is false.

Question 5.

Statement 1: The B-H curve of a ferromagnetic material is not linear which means that these materials do not obey \(\vec{B}=\mu \vec{H}\) rule.

Statement 2: The permeability of a ferromagnetic material is not constant, it can even have a negative value

Answer: 4. Statement 1 is false, Statement 2 is true.

Magnetic Effect Of Current And Magnetism

Magnetic Properties Of Materials Conclusion

  • A current loop behaves as a magnetic dipole.
  • The product of the effective area of a current-carrying loop and the current flowing through that loop is called the magnetic moment of the loop.
  • The behavior of a current-carrying circular coil is similar to that of a permanent magnet.
  • The behavior of a current-carrying solenoid is similar to that of a permanent bar magnet.
  • The ratio of the magnetic moment to the effective length of a magnet is the pole strength of that magnet. The strengths of the two poles of a magnet are equal but opposite.
  • Inside a magnetic material, the lines of force produced as a result of the superposition of the lines of force due to ‘the induced magnetic field and the lines of magnetization are called magnetic lines of induction.
  • If air or vacuum is replaced by any other material the fractional change in the magnetic field in that material is called the relative magnetic permeability (r) of that material.

Electromagnetism relative magnetic permeability

  • Tire intensity of magnetization (M) at a point, is defined as the magnetic moment of a unit volume surrounding the point.
  • The amount of induced magnetic moment per unit volume of a material due to unit magnetic intensity is called the magnetic susceptibility of that material.
  • The ratio of magnetic susceptibility and its density is called the mass (magnetic) susceptibility of the material.
  • The property by which a magnetic material retains some magnetism even after the withdrawal of the external magnetic field is called the magnetic retentivity of the material.
  • The property by which a magnetic material can retain induced magnetism even if it is handled roughly is called the coercivity of the material.
  • Magnetic susceptibility per unit mass of a paramagnetic material, \(\dot{\chi}=\frac{k}{\rho} \cdot(\rho=\text { density })\) varies inversely with the absolute temperature. So, \(\chi=\frac{C}{T}\) (for a particular-material C is constant). This is known as Curie law.
  • Diamagnetism is the most fundamental magnetic property of all materials.
  • The critical temperature at which a ferromagnetic material is converted into a paramagnetic material is called the Curie point dr Curie temperature of that material.
  • To make a particular place free from any external magnetic influence, a magnetic material having high magnetic permeability is used as a magnetic screen.
  • The earth behaves as a huge magnet.
  • Earth’s magnetic north pole is situated near its geographic south pole and the magnetic south pole is situated near its geographic north pole,

The elements of earthly magnetism are:

  1. The angle of dip,
  2. Angle of declination
  3. Horizontal component of the earth’s magnetic intensity.

The angle made by the tire intensity of the earth’s magnetic field with the horizontal at a place on the earth is called, the angle of dip at that place.

  • The angle between the magnetic meridian and geographical meridian at a place is called the angle of declination at that place.
  • Due to the superposition of two or more magnetic fields at a point, if the resultant, magnetic field becomes zero, then that point is called a neutral point.
  • The magnetic moment of a current loop (it is a magnetic dipole), \(\vec{p}_m=N I \vec{A}\)
  • where, I = current through the loop, \(\vec{A}\) = vector indicating the area of the loop and N = number of turns.
  • The torque acting on a bar magnet in a magnetic field,

⇒ \(\vec{\tau}=\vec{p}_m \times \vec{B}\)

  • The magnetic moment of a bar magnet of effective length 21 is,

⇒ \(\vec{p}_m=2 q_m \vec{l} \text { [where } q_m=\text { pole-strength] }\)

  • The mutual force between two magnetic poles of pole strengths strengths \(q_m \text { and } q_m^{\prime}\) is,

⇒ \(F=\frac{\mu_0}{4 \pi} \cdot \frac{q_m \cdot q_m^{\prime}}{r^2}\)

  • This is known as Coulomb’s law of magnetism.

Magnitudes of the magnetic field at different points due to a bar magnet:

Electromagnetism magnitudes of magnetic field at different points due to a bar magnet

Here, d = distance of the point from the mid-point of the tire magnet, 2l = effective length of the magnet, and pm = magnetic content of the bar magnet.

The magnetic moment of a revolving charged particle (q),

⇒ \(p_m=\frac{q v r}{2}=\frac{q}{2 m} L\)

[where, r = radius of tire circle, v = speed of the particle, m = mass of the particle, L = angular momentum]

A few relations:

  1. \(\vec{B}_0=\mu_0 \vec{H}\)
  2. \(\vec{B}=\mu_0(\vec{H}+\vec{M})\)
  3. \(\vec{M}=k \vec{H}\)
  4. \(\mu_r=\frac{\mu}{\mu_0}=1+k\)
  5. \(\chi=\frac{k}{\rho}\)
  6. \(k=\mu_r-1\)

[where, B0 = magnetic field in air (or vacuum),

  • B = magnetic field in airy medium,
  • H = magnetic intensity in air (or vacuum),
  • M = intensity of magnetization,
  • μ0 = magnetic permeability of air (or vacuum),
  • μ = permeability of a medium,
  • μr = relative permeability of a medium,
  • k = magnetic susceptibility,
  • ρ = density of the medium,
  • \(\chi\) = mass (magnetic) susceptibility]

If the angle of dip at a place is θ and the magnitude of tire Intensity of earth’s magnetic field is I, then the vertical component of earth’s magnetic field, V = Isinθ, and Its horizontal component H = Icosθ.

In this case, \(I=\sqrt{V^2+H^2}\)

Some physical quantities related to the magnetic property of material:

  • The magnetic moment for a straight current-carrying wire is zero.
  • The magnetic moment for a toroid is zero.
  • A magnetized wire having length L and magnetic moment M is bent.
  • Now the magnetic moment becomes \(M^{\prime}=\frac{M}{\sqrt{2}}\)

Electromagnetism a magnetised wire

  • A magnetized wire having length L and magnetic moment M is bent.
  • Now the magnetic moment becomes \({M}^{\prime}=\frac{M}{2}\)

Electromagnetism a magnetised wire.

  • If the magnetic latitude of a place is λ and the angle of dip is θ, then tanθ = 2tanλ.

Unit 3 Magnetic Effect Of Current And Magnetism Chapter 2 Magnetic Properties Of Materialsm Match The Columns

Question 1. Some quantities are given in column A and corresponding units are in column B.

Electromagnetism match the columns 1

Answer: 1-C, 2-B, 3-D, 4-A

Question 2. The magnetic moment of a magnetized steel wire is p. It is bent to form different shapes, given in column A. The magnetic moments are given in column B.

Electromagnetism match the columns 2

Answer: 1-D, 2-A, 3-B, 4-C

WBCHSE Class 12 Physics Notes For Electromagnetism

Magnetic Effect Of Current And Magnetism

Electromagnetism Magnetism

Magnetite, a black stone found in nature is called a natural magnet. It has two specific properties

  1. Attractive property
  2. Directive property.

1. Attractive property: Magnetite is found to attract pieces of iron.

2. Directive property:

  • If a piece of magnetite is suspended freely with the help of a thread, it aligns itself in the north-south direction.
  • Navigators used magnetite as a compass for guiding their ships and hence it was called leading stone or lodestone.
  • The two above-mentioned properties are called magnetic properties and the phenomenon is known as magnetism. Bodies showing these properties are called magnets.
  • Magnetism is a physical property of matter because when a body is magnetized, no chemical change occurs.

Artificial Magnet:

Magnets found in nature have no definite shape. So the directive property cannot be understood clearly. Moreover, the attractive power of this magnet is weak and hence is not so useful.

Read and Learn More Class 12 Physics Notes

Later artificial magnets were invented for practical use. Using some special processes magnetic properties can be built up in iron, steel, nickel, and some alloys.

This process is known as magnetization and the magnets thus made are known as artificial magnets.

Electromagnetism Artificial Magnet

There are different shapes and sizes of artificial magnets:

  1. Bar magnet
  2. Magnetic needle
  3. Horseshoe magnet
  4. Ball-ended magnet etc.

North and South Poles of a Magnet:

If a bar magnet is dipped into some iron filings and then withdrawn, a good amount of filings clings at the two ends of the magnet but almost none at its middle

Electromagnetism North and South Poles of a Magnet

So, attractive power is maximum at the two ends of the magnet, and these two regions are called the poles of a magnet. The middle portion, where no attraction is observed, is called the neutral region.

If a bar magnet is suspended freely by a thread, it sets itself at rest in the north-south direction. The pole of the magnet which always faces north is called the north pole (N-pole) or positive pole. Similarly, the pole facing south is called the south pole (S-pole) or negative pole of the magnet.

The line joining the two poles of a magnet is called the magnetic axis. If a magnet is suspended freely from its mid-point, it comes to rest after some time. The imaginary vertical plane through the magnetic axis of the magnet at this position is called the magnetic meridian of that place.

Usually, poles are considered as points and these two points lie very close to the ends of the magnet. The effective length of the magnet is about 80- 85% of its geometrical length.

Electromagnetism North and South Poles of a Magnet.

If different magnets are dipped into iron filings and withdrawn, the amounts of iron filings collected are not the same in all cases. So, the attractive power of different magnets is different, but there is no difference between powerful and weak magnets with respect to the directive property.

Mutual Action Between Two Magnetic Poles:

If the N-pole of a bar magnet is brought near the Af-pole of a magnetic needle, repulsion occurs between them, i.e., the Npole of the magnetic needle moves away from the bar magnet.

Again, if the N-pole of the bar magnet is brought near the S-poIe of the magnetic needle, they attract each other, i.e., the S-pole of the magnetic needle comes closer to the bar magnet.

If the experiment is performed with the S-pole of the bar magnet, the opposite action is observed, i.e., the S-pole of the bar magnet attracts the AT-pole of the magnetic needle but repels its S-pole.

Electromagnetism Mutual Action between Two Magnetic poles

Inference: Like poles repel each other but unlike poles attract each other.

Repulsion Is the conclusive tost of magnetization:

If a body is repelled by a magnet, it is sure that the body is a magnet

If one end of a body Is brought near the Af-poles of a powerful magnet and if an attraction is observed between them there are two probabilities:

  1. The body may be an ordinary piece of iron
  2. The body is a magnet and the end under investigation is the S-pole of that magnet. So, attraction cannot Identify whether a body is a magnet or not.

But if repulsion instead of attraction Is observed In the above experiment, it is definite that the body is a magnet and the end under investigation is the iV-pole of that magnet.

Hence, repulsion is the conclusive test of magnetization.

WBCHSE Class 12 Physics Notes For Electromagnetism.

Magnetic Effect Of Current And Magnetism

Electromagnetism Magnetic Field

A magnet attracts iron, nickel, cobalt, and some metallic alloys. This force of attraction even penetrates the wood, glass, paper, or other obstructions. It is always observed that the influence of a magnet is felt in its surrounding region. The greater the distance from the magnet, the lesser is its influence. Again if a weak magnet is replaced by a stronger one, the range of this influence increases.

Magnetic Field Definition: The region surrounding a magnet in which the Influence of that magnet is felt, is called the magnetic field of that magnet.

Theoretically, the magnetic field of a magnet extends up to infinity; but In practice, the field Is assumed to be extended up to a limited region due to,

Limitation of the experimental arrangement used for identification

Presence of other magnetic fields (like Earth’s magnetic field) in the environment.

Magnetic Lines of Force or Field lines:

Experiment: A small but powerful bar magnet is kept on a fairly large piece of cork and is let to float on water kept in a large vessel. In the floating condition, the magnet ultimately sets Itself at rest along the Nordic-south direction.

So, its N-pole faces the north and the S-pole faces the sound. With the help of a small cork, a long but comparatively weak magnetic needle is set to float vertically on the water in such a manner that the n-pole of the needle is just above the water’s surface but its s-pole remains deep inside the water.

In this situation, the effect of the bar magnet on the s-pole of the needle becomes negligible due to its depth. Hence, the n-pole of the needle can be treated as an isolated free n-pole with respect to the bar magnet.

The N-pole of the needle is brought in contact with the N-pole of the bar magnet at point A and then released. It is seen that this n-pole moves over the surface of water and follows a curved path ABCD to reach the S-pole of the bar magnet

Electromagnetism Magnetic Lines of Force or Field lines

Explanation of the experiment: The N-pole of the bar magnet exerts a force of repulsion on the isolated n-pole of the needle but the S-pole exerts a force of attraction on it.

The isolated free n-pole then starts moving along the resultant of the above two forces. At different points of the magnetic field of the bar magnet, the direction of this resultant force is different.

Then, the direction of motion of the n-pole will also change., So, when an isolated and free n-pole is allowed to travel in the magnetic field of a magnet, the pole describes a curved path and this path extends from the north pole to the south pole of the magnet.

A magnetic line of force is an imaginary curved line of a magnetic field; the direction of this field at any point is given by the tangent drawn at that point on the line of force passing through that point.

Properties of magnetic Hass of force:

1. A magnetic line of force emerges from the north pole of a magnet and terminates at the south pole.

2. For different initial points adjacent to the north pole of a magnet, different lines of force are obtained in the magnetic field. Different lines of force at one side of a bar magnet are shown. A number of such lines of force indicate a magnetic field.

Electromagnetism properties of magnetic lines of force

3. Two lines of force never intersect each other. If they intersect, through that point of intersection two tangents can be drawn on the two lines of force and each tangent will be the direction of the magnetic field at the point of inter the section. But two directions of the magnetic field at a single point are meaningless.

4. The concept of lines of force is totally imaginary, no such line exists in a magnetic field.

5. At any point in a magnetic field, if a unit area normal to the direction of the lines of force is imagined, the number of lines of force passing through that area is called the number density of the lines of force or magnetic flux at that point.

6. The greater this number density, the greater will be the strength of the magnetic field at that point For Example, the number density at point B is less than that at point A. So the magnetic field at point B is weaker than that at point A.

7. In general, the strength of a magnetic field is different at different points in that field and their directions are also different. Hence, the magnetic lines of force are usually curved lines at different distances.

8. But if the magnetic field is uniform, i.e., its magnitude and direction are the same up to a certain region, then it can be represented by equispaced parallel straight lines. Earth’s magnetic field, very close to the surface of the earth, is such a uniform magnetic field.

Electromagnetism uniform magnetic field

Magnetic Effect Of Current And Magnetism

Electromagnetism Action Of Current On Magnets

Oersted’s Experiment:

In 1820, scientist Hans Christian Oersted discovered that a magnetic field is generated around a current-carrying conductor. He inferred this through the following experiment.

A magnetic needle is kept just below a conducting wire stretched along a north-south direction. The magnetic needle should be free to rotate about its vertical axis. Now if current is passed through the straight conductor with the help of an external source, the magnetic needle gets deflected, i.e., the needle undergoes an angular deflection θ

Electromagnetism Oersteds Experiment

Observations related to deflection:

1. As soon as the current stops flowing through the wire, the magnetic needle rotates back to its initial position.

2. With the increase in current through the conducting wire, the angular deflection of the needle increases.

3. If the direction of the current in the conductor is reversed, the magnetic needle deflects in the opposite direction.

4. If the conducting wire is rotated slowly from its north-south direction while carrying current, the deflection of the magnetic needle gradually decreases. Ultimately when the direction of the current is along east-west, there will be no deflection of the magnetic needle.

5. So, when the conducting wire is placed normally to the axis of the magnetic needle and current is passed through the wire, no deflection of the magnetic needle is observed.

The direction of deflection of the magnetic needle due to the flow of electric current through the conductor can be determined with the help of any one of the following two rules:

1. Ampere’s swimming rule: If a man is imagined to be swimming along the direction of the current facing the magnetic needle with his arms outstretched, the north pole of the needle will be deflected towards his left hand

Electromagnetism Amperes swimming rule

2. Right-hand thumb rule: The right hand, with its thumb sticking out, is held in. Such a way that the conducting wire is in between the palm and the magnetic needle. If the other fingers point the direction of the current then the thumb will indicate the direction of deflection of the north pole of the needle.

The direction of deflection of the magnetic needle is shown in the following table:

Electromagnetism The direction of deflection of the magnetic needle..

Electromagnetism The direction of deflection of the magnetic needle

Electromagnetism The direction of deflection of the magnetic needle.

Discussions:

1. Dependence of magnetic field: At any adjacent point of a current-carrying conductor, the magnitude of the magnetic field depends on the magnitude of current and the direction of the magnetic field depends on the direction of current and on the position of the point with respect to the current-carrying conductor.

2. Presence of insulating material: The magnetic field is not affected if the current-carrying conductor is covered with an insulating material.

3. Current-carrying material: The current-carrying conductor itself is not magnetized. If some iron filings are brought in contact with the conductor, no attraction is observed.

4. Magnetic field due to a moving charged particle: The motion of charged particles is the cause of electric current. Hence, a moving charge can produce a magnetic field around it. Obviously, when a charged particle is at rest, it cannot produce a magnetic field.

Mapping of Magnetic Lines of Force due to an Electric Current:

Any magnetic field can be represented by magnetic lines of force. The direction of the magnetic field at any point is denoted by the direction of the magnetic line of force at that point. To determine the direction of the magnetic field at any point around a current-carrying conductor, either of the following two rules can be used.

1. Maxwell’s corkscrew rule: If we imagine a right-handed corkscrew to be driven along the direction of current in a conductor, then the direction in which it rotates, gives the direction of the magnetic field.

Electromagnetism Maxwells corkscrew rule

2. Right-hand grip rule: If a current-carrying conductor is imagined to be held within the grip of the right hand and if the direction of current through the conductor is indicated by the thumb, direction then the other fingers will curl in the direction of the magnetic field.

Electromagnetism Right hand grip rule

Long straight conductor: A long straight conducting wire carrying current is passed through the center of a cardboard and the cardboard is held normally to the length of the wire.

Some light iron filings are scattered over the cardboard. Now if the cardboard is slightly tapped, the iron filings arrange themselves in some concentric circles around the conducting wire. These concentric circles indicate the magnetic lines of force on a plane perpendicular to the current carrying a long straight conductor.

With the help of the corkscrew rule, the direction of the lines of force can also be determined. For an upward current, the directions of the magnetic lines of force. If the direction of current flow is reversed, i.e., for a downward current, the direction of the lines of force will also be reversed.

In the laboratory, generally, a magnetic needle is used instead of iron filings for plotting magnetic lines of force.

Electromagnetism Long straight conductor

Circular conductor: A circular current-carrying conductor is shown in penetrating a cardboard plate kept perpendicular to the plane of the circular conductor.

With the help of iron filings or a magnetic needle, if the lines of force of the magnetic field are drawn on the cardboard.

The directions of the magnetic lines of force may be determined by the corkscrew rule. If the direction of current in the circular conductor is reversed, the directions of the lines of force will also get reversed.

It is to be noted here that, at the centre of the circular conductor the lines of force are almost parallel to the axis going through the center.

Electromagnetism Circular conductor

Magnetic Effect Of Current And Magnetism

Electromagnetism Biot-Savart Law Or Laplace’s Law

A diagrammatic representation of a magnetic field is its representation using magnetic lines of force:

1. The direction of the tangent drawn at any point on a magnetic line of force is the direction of the magnetic field at the point;

2. Comparing the number density of magnetic lines of forces at different points in a magnetic field, the field strengths at those points can be compared. But to define magnetic field precisely as a definite physical quantity, at every point its magnitude should be represented by a number and an associated unit. This is not possible from the concept of magnetic lines of force only.

As a physical quantity, the usual symbol of the magnetic field is \(\vec{B}\) (as it is a vector). This vector B is named magnetic field magnetic induction or magnetic flux density. In the region surrounding a current-carrying conductor

  1. The direction of \(\vec{B}\) is determined by Maxwell’s corkscrew rule
  2. The magnitude of \(\vec{B}\) (i.e., B) is determined by Biot-Savart law.

Statement of Biot-Savart law:

Let δl= a small elementary part of a conducting wire

I = current through the wire

r = distance of any external point from the element δl

θ = angle between the element \(\delta \vec{l}\) and the position vector -r of the external point

δB = magnitude of the magnetic field at that external point.

Electromagnetism Statement of Biot-Savart law

Then the Biot-Savart law states that,

  1. \(\delta B \propto \delta l\)
  2. \(\delta B \propto I\)
  3. \(\delta B \propto \frac{1}{r^2}\)
  4. \(\delta B \propto \sin \theta\)

This means, \(\delta B \propto \frac{I \delta l \sin \theta}{r^2} \text { or, } \delta B=k \frac{I \delta l \sin \theta}{r^2}\)….(1)

This law is also called Laplace’s law. The value of the constant k in equation (1) depends on

1. The nature of the medium between the conductor and the point under consideration and

2. The system of units used for different physical quantities. In this chapter, we shall consider only vacuum as the medium.

SI unit: SI units of different physical quantities used in equation (1) are:

δl and r: meter (m); I: ampere (A); δB: weber/metre2 (Wb.m-2) or tesla (T).

For the definition of the unit of magnetic field (Wb.m-2)

If these units are used in equation (1) then another constant is traditionally used instead of k. This constant is μ0 = 4πk,

⇒ \(k=\frac{\mu_0}{4 \pi}\)

So, the usual form of Biot-Savart law in vacuum,

⇒ \(\delta B=\frac{\mu_0}{4 \pi} \cdot \frac{I \delta l \sin \theta}{r^2}\)….(2)

The constant μ0 is called the permeability of free space.

Unit of μ0: From equation (2),

⇒ \(\mu_0=\frac{4 \pi r^2 \delta B}{I \delta l \sin \theta}\)

∴ The unit of \(\mu_0=\frac{\mathrm{m}^2 \times \mathrm{Wb} \cdot \mathrm{m}^{-2}}{\mathrm{~A} \times \mathrm{m}}=\mathrm{Wb} \cdot \mathrm{A}^{-1} \cdot \mathrm{m}^{-1}\)

Here, Wb.A-1 is also called Henry (H).

Hence, the unit of μ0 is henry/metre (H.m-1)

Value of μ0: = 4π x 10-7 H m-1.

It is to be noted that,

⇒ \(\frac{\mu_0}{4 \pi}=10^{-7} \mathrm{H} \cdot \mathrm{m}^{-1}\)

Biot-Savart law for extended conductors: An extended conductor is assumed to be composed of a number of smaller parts \(\delta l_1, \delta l_2, \cdots\), etc. and the Biot-Savart law is applied for each part. At any point, the magnetic field due to the whole conductor will be,

⇒ \(B=\sum \delta B=\frac{\mu_0}{4 \pi} \sum \frac{I \delta l \sin \theta}{r^2}\)

The \(\) (summation) sign indicates the sum of a number of terms. \(\vec{B}\) is a vector; so to determine the resultant, the rule of vector addition is applied. Usually, this vector addition is very complicated. But in the case of symmetrical conductors like straight conducting wire, circular coil, etc., determination of the resultant magnetic field is not so troublesome.

Vector form of Wot-Savort low: If the unit vector towards the point P with respect to \(\delta l \text { be } \hat{r} \text {, then } \vec{r}=r \hat{r}\).
Hence, the magnitude of the vector product \((\delta \vec{l} \times \hat{r})\) is,

⇒ \(|\delta \vec{l} \times \hat{r}|=\delta l \cdot 1 \cdot \sin \theta=\delta l \sin \theta \quad[∵|\hat{r}|=1]\)

Again, the direction of \((\delta \vec{l} \times \hat{r})\) is downward with respect, which is actually the direction of the magnetic field as per the corkscrew rule. So, the vector form of the equation (2) is,

⇒ \(\delta \vec{B}=\frac{\mu_0}{4 \pi} \cdot \frac{I \delta \vec{l} \times \hat{r}}{r^2}\)….(3)

From equation (3) we get,

⇒ \(\vec{B}=\frac{\mu_0}{4 \pi} \sum \frac{I \delta \vec{l} \times \hat{r}}{r^2}=\frac{\mu_0}{4 \pi} \sum \frac{I \delta \vec{l} \times \vec{r}}{r^3}\)….(4)

Usually with respect to the plane of the paper, an upward vector is denoted by \(\odot\) sign to mean the tip of an arrow and a downward vector by \(\otimes\) sign to mean the tail of an arrow. Thus, the direction of the magnetic field at point P is denoted by the \(\otimes\) sign, and at point Q, \(\odot\) sign is used.

The magnetic intensity or magnetic field strength or magnetizing field:

In SI: The magnetic permeability of vacuum is μ0 this value of μ0 is also used for air because, in the presence of air, no remarkable change in the magnetic field is observed.

But for any other medium, the permeability of that medium is denoted by n. For different media the magnitude of μ is different. In the case of any medium, the general form of Biot-Savart law is,

⇒ \(\delta B=\frac{\mu}{4 \pi} \cdot \frac{I \delta l \sin \theta}{r^2}\)….(5)

From equations (2) and (5) we see that the quantity \(\frac{1}{4 \pi} \cdot \frac{I \delta l \sin \theta}{r^2}\) can be treated as the cause of the magnetic field in a medium. In the absence of the multiplier μ0 or μ in this expression, this quantity does not depend on the medium. To determine the magnetic field in a medium, this quantity is just multiplied by the magnetic permeability of that medium. This quantity is called magnetic intensity or magnetizing field. It is expressed by H.

From the equation (5) we can write,

⇒ \(\delta B=\frac{\mu}{4 \pi} \cdot \frac{I \delta l \sin \theta}{r^2}=\mu \delta H\)

where, \(\delta H=\frac{1}{4 \pi} \cdot \frac{I \delta l \sin \theta}{r^2}\)…(6)

Generally, \(B=\mu H \quad \text { or, } \quad H=\frac{1}{\mu} B\)

Like \(\vec{B}\), H is also a vector quantity whose direction is the same
as that of \(\vec{B}\).

So, \(\vec{B}=\mu \vec{H} \quad \text { or, } \quad \vec{H}=\frac{1}{\mu} \vec{B}\)…(7)

With the help of this equation (7), \(\vec{H}\) can be defined.

Definition: At any point in a medium the ratio of the acting magnetic, field and the magnetic permeability of the medium is called the magnetic intensity or magnetizing field at that point.

Unit: Unit of \(H=\frac{\text { unit of, } B}{\text { unit of } \mu}=\frac{\mathrm{Wb} \cdot \mathrm{m}^{-2}}{\mathrm{~Wb} \cdot \mathrm{A}^{-1} \cdot \mathrm{m}^{-1}}\)

= A.m-1 (ampere/metre)

CGS or Gaussian system:

In this system \(\vec{H}\), i.e., the magnetic intensification (instead of \(\vec{B}\)) is considered as the primary vector in magnetism. In this case, the unit of magnetic intensity H is d (Oe) arid the unit of magnetic field B is gauss (G). It may be of interest to note that 1 oersted is the same as 1 dyn per unit pole, and 1 gauss is the same as 1 maxwell/cm2 according to the old definition of magnetic intensity.

Relation between SI and CGS units:

1 A.m-1 = 4π x 10-3 Oe, 1 Wb.m-2 = 10-4 G

In the CGS system, the electromagnetic, unit (abbreviated as emu) of current is used as the unit of current and is expressed by the symbol i. This electromagnetic unit is so defined that if the other quantities in equation (6) are expressed in CGS units, thÿf-,ÿ constant \(\frac{1}{4 \pi}\) can be replaced by 1. In that case, the CGS form of the equation will be,

⇒ \(\delta H=\frac{i \delta l \sin \theta}{r^2}\)…(8)

So, the different units used in this equation are

δl and r: cm; i: emu of current; δH: Oe.

This equation (δ) indicates the Biot-Savart law or Laplace’s law in the CGS or Gaussian system.

Emu current: The current which, when flowing through a conducting wire of length in the form of an arc of a circle of radius 1 cm, produces a magnetic Intensity of 1 Oe at the center of the arc, is called 1 electromagnetic unit (emu) of current

The two parts of the wire except the circular are kept along the radius of the circle; thus no magnetic field is produced at the center of the circle due to the current flowing through these two parts.

The definition of the electromagnetic unit of current can be explained from the discussion of the magnetic field produced due to current in a circular conductor

Relation between ampere and emu of current:

1 emu of current = 10 A

Rules of conversion from SI to CGS:

The following replacements convert SI expressions into the corresponding ones. CGS expressions;

1. Magnetic intensity \(\vec{H}\), in place of magnetic field \(\vec{B}\);

2. Electric current in the emu, in place of I in amperes;

3. The constant 1 in place of \(\frac{\mu_0}{4 \pi}\), i.e., 4π in place of μ0

Application of Biot-Savart Law Long Straight Conductor:

Let I = strength of current in an electrical circuit. AB is a straight conductor and OP = r = perpendicular distance of the point P from the current carrying wire.

Electromagnetism Application of Biot-Savart Law and Long straight conductor

The two endpoints of the conducting wire make angles θ1 and θ2 at point P with respect to OP. In the direction of the current, if the angle θ2 is taken as positive then in the opposite direction, θ1 will be negative, i.e., it will be -θ1.

The conducting wire and its adjacent region in a magnified form. From a very small part dl of the wire, the distance of the point P is x and the angle between the direction of current and x is a.

So, according to Iliot-Savnrt law, the magnitude of the magnetic field at die point P due to dl is,

⇒ \(d B=\frac{\mu_0}{4 \pi} \cdot \frac{I d l \sin \alpha}{x^2}\)…(1)

From the diagram,

⇒ \(\alpha=180^{\circ}-\beta=180^{\circ}-\left(90^{\circ}-\theta\right)=90^{\circ}+\theta\)

So, sinaaa = sin(90° + θ) = cosθ ….(2)

Again, l = rtanθ

Since, r = constant, differentiating the equation we get,

dl = rsec²θdθ….(3)

Again, cosθ = \(\frac{r}{x}\)

or, \(x^2=\frac{r^2}{\cos ^2 \theta}=r^2 \sec ^2 \theta\)…..(4)

Now putting the values of sinaaa, dl, and x2 from equations (2), (3), and (4) into equation (1) we get,

⇒ \(d B=\frac{\mu_0}{4 \pi} \cdot \frac{I \cdot r \sec ^2 \theta d \theta \cdot \cos \theta}{r^2 \sec ^2 \theta}=\frac{\mu_0}{4 \pi} \cdot \frac{I}{r} \cdot \cos \theta d \theta\)….(5)

So, for the entire wire AB, the magnetic field at the point P is,

⇒ \(B=\int d B=\frac{\mu_0}{4 \pi} \cdot \frac{I}{r} \int_{-\theta_1}^{\theta_2} \cos \theta d \theta\)

⇒ \(\frac{\mu_0}{4 \pi} \cdot \frac{I}{r}[\sin \theta]_{-\theta_1}^{\theta_2}=\frac{\mu_0}{4 \pi} \cdot \frac{I}{r}\left[\sin \theta_2-\sin \left(-\theta_1\right)\right]\)

⇒ \(\frac{\mu_0}{4 \pi} \cdot \frac{I}{r}\left(\sin \theta_2+\sin \theta_1\right)\)

So, \(B=\frac{\mu_0}{4 \pi} \cdot \frac{I}{r}\left(\sin \theta_1+\sin \theta_2\right)\)…(6)

This is the expression for the magnitude of the magnetic field at point P due to the current carrying wire.

Special cases:

For infinitely long conducting wire: Let the length of the wire below the point O be Ll and above this point, I². If these two lengths are much greater than r, i.e., and r<<I², the wire can be treated as of infinite length with respect to the point P. In that case, for the lowermost end of the wire,θ1 ≈ 90° and for the uppermost end of the wire θ2 ≈ 90°.

So from the equation (6), we can write,

⇒ \(B=\frac{\mu_0}{4 \pi} \cdot \frac{I}{r}\left(\sin 90^{\circ}+\sin 90^{\circ}\right)\)

∴ \(B=\frac{\mu_0}{4 \pi} \cdot \frac{2 I}{r}\)…(7)

For semi-infinitely long conducting wire: Let the lowermost end of the wire be O and the length of the wire = I. If r<<L, the wire can be called semi-infinitely long with respect to the point P. In that case, θ1 = 0° and θ2 = 90°.

So, from the equation (6), we can write

⇒ \(B=\frac{\mu_0}{4 \pi} \cdot \frac{I}{r}\left(\sin 0^{\circ}+\sin 90^{\circ}\right)\)

∴ \(B=\frac{\mu_0}{4 \pi} \cdot \frac{I}{r}\)…(8)

For a point on the extended part of the wire: If point P1 is located according to, both θ1 and θ2 will be negative with respect to r which is the perpendicular distance between point P1 and the extension of the wire AB.

Electromagnetism point on the extended part of the wire

So, the magnetic field at P1,

⇒ \(B_1=\frac{\mu_0}{4 \pi} \cdot \frac{I}{r}\left[\sin \theta_1+\sin \left(-\theta_2\right)\right]\) [taking θ2 negative in equation (6)]

⇒ \(\frac{\mu_0}{4 \pi} \cdot \frac{I}{r}\left(\sin \theta_1-\sin \theta_2\right)\)…..(9)

If we consider the point P2 to lie on the extension of the wire AB, then θ1 = 90° and θ2 = 90°. Putting these in equation (9) we get, the magnetic field at the point P2,

⇒ \(B_2=\frac{\mu_0}{4 \pi} \cdot \frac{I}{r}\left[\sin 90^{\circ}-\sin 90^{\circ}\right]=\frac{\mu_0}{4 \pi} \cdot \frac{I}{r}[1-1]=0\)

So, at any point lying along the length of a current-carrying conductor, the magnetic field due to that conductor will be zero.

Expressions in CGS or Gaussian system: In the above-mentioned equations (6), (7), and (8), if we substitute B→H, I→I, and μ0→4π then we get

Magnetic intensity due to a straight conductor,

⇒ \(H=\frac{i}{r}\left(\sin \theta_1+\sin \theta_2\right)\)

Magnetic intensity due to an infinitely long conductor,

⇒ \(H=\frac{2 i}{r}\)

Magnetic intensity due to a semi-infinitely long conductor,

⇒ \(H=\frac{i}{r}\)

Magnetic Effect Of Current And Magnetism

Electromagnetism Numerical Examples

Example 1. The distance between two long straight conductors is 5m. Currents 2.5 A and 5 A are flowing through in the same direction. What will be the magnetic field at a themed point between them?
Solution:

According to the corkscrew rule, the magnetic fields at the mid-point due to die two conductors will be opposite in directions.

So, the relation, \(B=\frac{\mu_0}{4 \pi} \cdot \frac{2 I}{r}, \text { gives }\)

magnetic field at the mid-point due to the first conductor,

⇒ \(B_1=\frac{\mu_0}{4 \pi} \cdot \frac{2 I_1}{\frac{r}{2}}=\frac{\mu_0}{4 \pi} \cdot \frac{4 I_1}{r}\)

and magnetic field at the mid-point due to the second conductor,

⇒ \(B_2=\frac{\mu_0}{4 \pi} \cdot \frac{2 I_2}{\frac{r}{2}}=\frac{\mu_0}{4 \pi} \cdot \frac{4 I_2}{r}\)

⇒ \(I_2>I_1 \text { and hence } B_2>B_1\)

∴ The resultant magnetic field,

⇒ \(B=B_2-B_1=\frac{\mu_0}{4 \pi} \cdot \frac{4}{r}\left(I_2-I_1\right)\)

⇒ \(\frac{\mu_0}{4 \pi} \cdot \frac{4}{5}(5-2.5)=\frac{4 \pi \times 10^{-7}}{4 \pi} \times \frac{4}{5} \times 2.5\) [∵ r = 5m, I1 = 2.5 A, I2 = 5 A]

= 2 x 10-7 T

Example 2. 5 A current Is flowing in two opposite directions through each of two parallel straight conducting wires kept 0.2 m apart. find who second wins) Determine the magnitudes and directions of the magnetic field at the points P, Q, and R lying on the plane containing the two wires.
Solution:

The magnetic field at P due to the first wire,

⇒ \(B_1=\frac{\mu_0}{4 \pi} \cdot \frac{2 \times 5}{0.1} \text { (upward) }\)

The magnetic field at P due to the second wire,

⇒ \(B_2=\frac{\mu_0}{4 \pi} \cdot \frac{2 \times 5}{0.3} \text { (downward) }\)

Since, B1 > B2, the resultant magnetic field will be upward from the plane.

∴ Bp = B1 – B2

⇒ \(\frac{\mu_0}{4 \pi}(2 \times 5)\left(\frac{1}{0.1}-\frac{1}{0.3}\right)\)

⇒ \(10^{-7} \times 10 \times 10 \times \frac{2}{3}\)

⇒ \(6.67 \times 10^{-6} \mathrm{~Wb} \cdot \mathrm{m}^{-2}\)

Similarly, the upward magnetic field at the point Q,

BQ = 6.67 x 10-6 Wb.m-2

At point R, the magnetic field due to the first wire as well as for the second wire is downward. Hence, the resultant magnetic field will be the sum of these two fields.

So, \(B_R=2 B_1=2 \times \frac{\mu_0}{4 \pi} \cdot \frac{2 \times 5}{0.1}\)

= 2 x 10-7 x 10 x 10

= 2 x 10-5 Wb.m-2

Example 3. An infinitely long conducting wire POQ is bent through right angles at O. If a current I . 0 is sent through this bent wire, what will be the magnitude of the magnetic field at point A at a distance r from each part of the wire?

Electromagnetism Example 3 long conducting wire

Solution:

AM = AN = r

So, ∠OAM = ∠OAN = 45°

Now, the magnetic field at A due to PO is equal to that due to OQ both in magnitude and direction (downwards). Again, the points P and Q make angles at A relative to AM and AN. For infinitely long wires, each of these angles is close to 90°.

So the resultant magnetic field at A is

⇒ \(B=2 \times \frac{\mu_0}{4 \pi} \cdot \frac{I}{r}\left(\sin 90^{\circ}+\sin 45^{\circ}\right)\)

⇒ \(\frac{\mu_0}{4 \pi}: \frac{I}{r}(2+\sqrt{2}) \text { (directed downwards) }\)

Example 4. Through each of two wires POQ and P’O’Q’ an electric current I is passing. P The points Q, O, O’, and Q’ are collinear. Determine the magnetic field at the midpoint A of OO’.

Electromagnetism Example 4 magnetic field

Solution:

Point A lies along the length of the two parts OQ and O’Q’. Hence, no magnetic field exists at A due to these two parts.

The two parts PO and P’O’ are semi-infinite conducting wires with respect to point A. Hence, for each part, the magnetic field at \(A=\frac{\mu_0}{4 \pi} \cdot \frac{I}{r}\) and these two fields are downwards.

Hence, the downward magnetic field at the point A,

⇒ \(B=\frac{\mu_0}{4 \pi} \cdot \frac{I}{r}+\frac{\mu_0}{4 \pi} \cdot \frac{I}{r}=\frac{\mu_0}{4 \pi} \cdot \frac{2 I}{r}\)

Example 5. 5 A current is flowing through a long straight conducting wire. What is the magnitude of the magnetic field at a distance of 10 cm from the wire?
Solution:

We know, \(B=\cdot \frac{\mu_0}{4 \pi} \cdot \frac{2 I}{r}\) [for a straight infinite wire]

Here,I = 5 A, r = 10 cm = 0.1m, μ0 = 4π x 10-7 H.m-1

∴ \(B=\frac{4 \pi \times 10^{-7}}{4 \pi} \times \frac{2 \times 5}{0.1}=10^{-5} \mathrm{~Wb} \cdot \mathrm{m}^{-2}\)

Application of Biot-Savart Law Circular Conductor:

1. Magnetic field at the center of a circular conductor: Let the current through a circular conductor of radius r be I.

Due to an element of length δ1 of the conductor, the magnetic field at the center of a circle,

⇒ \(\delta B=\frac{\mu_0}{4 \pi} \cdot \frac{I \delta l \sin 90^{\circ}}{r^2}=\frac{\mu_0}{4 \pi} \cdot \frac{I \delta l}{r^2}\)

If the circumference of the circle is divided into a large number of such elements then, for each element, I and r remain the same and the angle between that element and the radius is 90°. Again sum of these elements = circumference of the circle = 2πr.

Electromagnetism Application of Biot-Savart Law Circular conductor

So, die magnetic field at the center of the circular conductor,

⇒ \(B=\frac{\mu_0}{4 \pi} \frac{I}{r^2} \sum \delta l=\frac{\mu_0}{4 \pi} \cdot \frac{I}{r^2} \cdot 2 \pi r=\frac{\mu_0 I}{2 r}\)…(1)

If the circular conductor of a single turn is replaced by a circular coil of N turns then,

⇒ \(B=\frac{\mu_0 N I}{2 r}\)….(2)

The direction magnetic field at the center of the circular conductor can be determined by the corkscrew rule. This direction is normally upward from the plane of the paper. If the conductor is in die form of a circular arc and if that arc makes an angle θ (rad) at the center, the magnetic field at the center of the circle

Electromagnetism The direction of magnetic field

⇒ \(B=\frac{\mu_0 I}{2 r} \cdot \frac{\theta}{2 \pi}=\frac{\mu_0 I}{4 \pi r} \cdot \theta\)….(3)

The magnetic field on the axis of a circular conductor:

Let r = radius of a circular conductor, I = current through the conductor, and O is the center of the circular conductor and hence. P is any point on the axis (OP – x). An element of length δl is considered at the topmost point C of the conductor. The line segment CP is perpendicular to this element So, for the element of length 8/ at C, the genetic field at the point P is,

⇒ \(\left.\delta B=\frac{\mu_0}{4 \pi} \cdot \frac{I \delta l \sin 90^{\circ}}{u^2}=\frac{\mu_0}{4 \pi} \cdot \frac{I \delta l}{u^2} \text { [here } C P=u\right]\)

Electromagnetism Magnetic field on the axis of a circular conductor

From the corkscrew rule, we see that the direction of δB is along PQ. The component of 8B along the axis is δBsinθ and perpendicular to the axis is δBcosθ. If an element of length δl is now considered at point D diametrically opposite to C on the circumference, the magnetic field at point P will be SB and its direction will be along PR. Naturally, its downward component δBcosθ neutralizes the previous component δBcosθ, but two components

δBsinθ each will be added together along the axis. In this way, if the whole circular conductor is considered, the algebraic sum of the components SBsinfl along the axis will be the resultant magnetic field due to the circular conductor at die point P.

∴ \(B=\sum \delta B \sin \theta=\frac{\mu_0}{4 \pi} \sum \frac{I \delta l}{u^2} \cdot \frac{r}{u}=\frac{\mu_0}{4 \pi} \sum \frac{I \delta l}{u^3} \cdot r\)

Now, due to the symmetry of the circular conductor, the magnitudes of I, r and u will be die same at every point on its circumference. Hence,

⇒ \(B=\frac{\mu_0}{4 \pi} \cdot \frac{I r}{u^3} \sum \delta l=\frac{\mu_0}{4 \pi} \cdot \frac{I r}{u^3} \cdot 2 \pi r=\frac{\mu_0 I}{2} \cdot \frac{r^2}{\left(u^2\right)^{3 / 2}}\)

So, \(B=\frac{\mu_0 I}{2} \cdot \frac{r^2}{\left(r^2+x^2\right)^{3 / 2}}\)….(4)

If a circular coil having N turns is taken instead of the circular conductor of single turn then,

⇒ \(B=\frac{\mu_0 N I}{2} \cdot \frac{r^2}{\left(r^2+x^2\right)^{3 / 2}}\)…..(5)

Now, at the center of the circle, i.e., at the point O, x = 0, and hence,

⇒ \(B=\frac{\mu_0 N l}{2 r}\), which is identical to the equation (2).

Expressions In COS or Gaussian system: In equations (1) to (5) above, substituting B→H, I→I, and μ0 → 4π we get,

magnetic intensity at the center of a circular conductor,.

⇒ \(H=\frac{2 \pi i}{r}\)

and magnetic intensity at the center of a circular coil having N-tums,

⇒ \(H=\frac{2 \pi N i}{r}\)

In the case of a conductor in the form of an arc of a circle, the magnetic intensity at the center of the conductor, H = \(\frac{i}{r}\)θ where θ is the angle made by the arc at the center.

Magnetic intensity at any point on the axis of a circular conductor,

⇒ \(H=2 \pi i \cdot \frac{r^2}{\left(r^2+x^2\right)^{3 / 2}}\)

In the case of a circular coil having N turns, the magnetic intensity at any point on its axis,

⇒ \(H=2 \pi N i \cdot \frac{r^2}{\left(r^2+x^2\right)^{3 / 2}}\)

Angle (θ) subtended by a current-carrying conductor of length 1 cm bent in the form of an arc of a circle of 1 cm radius is 1 rad. If the magnetic intensity at the center of the arc is 1 Oe, then substituting r = 1, θ = 1, H = 1 in the relation \(H=\frac{i \theta}{r}\), we get, i = 1.

Characteristics of the magnetic field of a circular conductor:

1. Direction of magnetic field: At any point on the axis of the conductor the direction of the resultant magnetic field is always along the axis. For the direction of current the magnetic field at the point P is along the axis and directed outward. But on the opposite side of the coil, at the point magnetic field is along the axis and directed inward, i.e., along Pf O. If the direction of current in the coil is reversed, at the point P magnetic field will be inward while at the point P’, it will be outward.

Electromagnetism characteristics of magnetic field of a circular conductor

2. Magnitude of the magnetic field: From equation (5) we see that the magnitude of the field becomes maximum at the center of the circle, and decreases along the axis, on either side.

As a result, no uniform magnetic field is obtained at any point on the axis and thus a problem arises while constructing electrical instruments with a circular coil. This problem can be removed by using a Helmholtz double coil.

Electromagnetism Magnitude of the magnetic field

Helmholtz double coll: Two circular coils having the same radius (= r) are placed coaxially at a distance equal to their radius. If a direct current is passed through them in the same direction, the magnetic fields generated between them will have the same direction. Thus the resultant magnetic field remains almost uniform in between the coils.

Electromagnetism Helmholtz double coil

Magnetic Effect Of Current And Magnetism

Electromagnetism Numerical Examples

Example 1. The radii of two concentric circular colls are 8 cm and 10 cm and the number of turns In them are 40 and 10, respectively. A 5A current Is passing through each of them in the same direction. Determine the magnetic field produced at the center of the two colls.
Solution:

Since current flows in the same direction through the two coils, the directions of magnetic fields at the center due to the coils will be the same. Hence, the resultant magnetic field will be obtained by adding these magnetic fields.

For the first coil, \(B_1=\frac{\mu_0 N_1 I}{2 r_1}\)

For the second coil, \(B_2=\frac{\mu_0 N_2 I}{2 r_2}\)

∴ The resultant magnetic field,

⇒ \(B=B_1+B_2=\frac{\mu_0 I}{2}\left(\frac{N_1}{r_1}+\frac{N_2}{r_2}\right)\)

⇒ \(\frac{4 \pi \times 10^{-7} \times 5}{2}\left(\frac{40}{8}+\frac{10}{10}\right)
\)

= 2π x 10-7 x 5 x 6

= 1.885 x 10-5 Wb.m-2

Example 2. A current I is flowing through an Infinitely long wire PQRS. The wire is bent a J S at right angles so that the part QR becomes one-fourth of the circumference of a circle of radius r whose center is at O. Determine the magnetic field at O.

Electromagnetism Example 2 Circumference of a circle of radius

Solution:

With respect to point O, both the parts PQ and

RS are semi-infinite wires and hence, for each part magnetic field at point O will be,

⇒ \(B_1=B_2=\frac{\mu_0}{4 \pi} \cdot \frac{I}{r}\)

Again for a complete circular conductor, the magnetic field at its center = \(\frac{\mu_0 I}{2 r}\)

So. for the one-fourth part QR of the circle, the magnetic field
at O,

⇒ \(B_3=\frac{1}{4} \times \frac{\mu_0 I}{2 r}=\frac{\mu_0 I}{8 r}\)

Hence, the resultant magnetic field at O,

B = B1 + B2 + B3

⇒ \(\frac{\mu_0 I}{4 \pi r}+\frac{\mu_0 I}{4 \pi r}+\frac{\mu_0 I}{8 r}\)

⇒ \(\frac{\mu_0 I}{4 r}\left(\frac{1}{\pi}+\frac{1}{\pi}+\frac{1}{2}\right)=\frac{\mu_0(4+\pi) I}{8 \pi r}\)

Example 3. Determine the magnetic field at point O due to the circuit.

Electromagnetism Example 3 the magnetic field

Solution:

Point O lies on the same straight line with the two linear parts of the circuit Hence, no magnetic field acts at O due to those two parts.

For a complete circular conductor, the magnetic field at the tire center of the circle = \(\frac{\mu_0 I}{2 r}\)

Hence, for a semicircular conductor, the magnetic field at the center = \(\frac{\mu_0 I}{4 r}\)

The magnetic field at O due to the semicircular conductor of radius r is \(B_1=\frac{\mu_0 I}{4 r}\) and that due to the semicircular conductor of radius \(R \text { is } B_2=\frac{\mu_0 I}{4 R}\).

The corkscrew rule shows that Bj and B2 are oppositely directed. Since Bj > Bz, the resultant magnetic field at the point O (upward with respect to the page of the book),

⇒ \(B=B_1-B_2=\frac{\mu_0 I}{4}\left(\frac{1}{r}-\frac{1}{R}\right)=\frac{\mu_0 I(R-r)}{4 r R}\)

Example 4. What is the magnetic field produced at the center of a hydrogen atom due to the revolution of its electron in the first orbit (K-orbit)? The radius of the first orbit = 0.53 x 10-10 m, the velocity of the electron in that orbit = 2.19 x 10s m s-1
Solution:

Time period of revolution, \(T=\frac{2 \pi r}{v}\).

So, the circular loop formed due to the revolution of the electron carries an effective current

⇒ \(I=\frac{\text { charge }}{\text { time period }}=\frac{e}{T}=\frac{e v}{2 \pi r}\)

So, the magnetic field at the center of the atom,

⇒ \(B=\frac{\mu_0 I}{2 r}=\frac{\mu_0}{4 \pi} \cdot \frac{e v}{r^2}\)

⇒ \(10^{-7} \times \frac{\left(1.6 \times 10^{-19}\right) \times\left(2.19 \times 10^6\right)}{\left(0.53 \times 10^{-10}\right)^2}\)

= 12.47 T

Example 5. Calculate the magnetic induction at point O (center of the partial circular conductor).

Electromagnetism Example 5 partial circular conductor

Solution:

Magnetic induction at die center due to whole conductor, B = Magnetic induction due to (straight part AB + curved part BCD + straight part DE)

∴ \(B_{\text {total }}=B_{A B}+B_{B C D}+B_{D E}\)

BAB = 0 [∵ thePoint O is along AB]

⇒ \(B_{B C D}=\frac{3}{4}\) of the ma8netic due to the whole circle

⇒ \(\frac{3}{4} \times \frac{\mu_0}{4 \pi} \cdot \frac{2 \pi I}{r}\) [directed inwards and perpendicular to the plane of the conductor]

⇒ \(B_{D E}=\frac{\mu_0}{4 \pi} \cdot \frac{I}{r}\) [directed outwards and perpendicular to the plane of the conductor]

∴ \(B_{\text {total }}=\frac{3}{4} \frac{\mu_0}{4 \pi} \frac{2 \pi l}{r}-\frac{\mu_0}{4 \pi} \frac{l}{r}\)

⇒ \(\frac{\mu_0}{4 \pi}, \frac{I}{r}\left[\frac{3}{2} \pi-1\right]\) [directed inwards and perpendicular to the plane of the conductor]

Example 6. Two concentric but mutually perpendicular conducting coils are carrying current 3 A and 4 A, respectively. If the radius of each coil is 2x cm, what will be the magnetic induction at the center of the coils? (μ0 = 4π x 10-7 Wb A-1.m-1)
Solution:

⇒ \(r=2 \pi \mathrm{cm}=\frac{\pi}{50} \mathrm{~m}\)

Magnetic field at the center of a circular coil, \(B=\frac{\mu_0 I}{2 r}\)

∴ For the first coil,

⇒ \(B_1=\frac{\left(4 \pi \times 10^{-7}\right) \times 3}{2 \times \frac{\pi}{50}}=3 \times 10^{-5} \mathrm{~Wb} \cdot \mathrm{m}^{-2}\)

and for the second coil,

⇒ \(B_2=\frac{\left(4 \pi \times 10^{-7}\right) \times 4}{\ddots 2 \times \frac{\pi}{50}}=4 \times 10^{-5} \mathrm{~Wb} \cdot \mathrm{m}^{-2}\)

Since the coils are mutually perpendicular, By and B2 are also at right angles to each other. Hence, the resultant magnetic field,

⇒ \(B=\sqrt{B_1^2+B_2^2}=\sqrt{(3)^2+(4)^2} \times 10^{-5}\)

= 5 x 10-5 Wb.m-2

Example 7. Each of two long straight wires, passing through points A and B, carries a current directed vertically upwards with respect to the plane of the paper. The separation between them is r. Find out the magnetic field at a point P on that plane, which is at a distance r from each of the wires.

Electromagnetism Example 7 two long straight wires

Solution:

Here PA = PB = r

Due to the current-carrying conductor, which passes through point A, the magnetic field intensity at P is,

⇒ \(B_1=\frac{\mu_0}{4 \pi} \frac{2 I}{P A}=\frac{\mu_0}{4 \pi} \frac{2 I}{r}\)

Similarly, due to the current-carrying conductor which passes through point B, the magnetic field intensity at point P is,

Electromagnetism Example 7 two long straight wires.

⇒ \(B_2=\frac{\mu_0}{4 \pi} \cdot \frac{2 I}{P B}=\frac{\mu_0}{4 \pi} \cdot \frac{2 I}{r} \text { (along } \overrightarrow{P B_2} \text { ) }\)

The resultant magnetic field intensity at point P is given by,

⇒ \(B=\sqrt{B_1^2+B_2^2+2 B_1 B_2 \cos 60^{\circ}}\)

⇒ \(\sqrt{\left(\frac{\mu_0}{4 \pi} \frac{2 I}{r}\right)^2\left[1^2+1^2+2 \cdot 1 \cdot 1 \cdot \frac{1}{2}\right]}\)

⇒ \(\frac{\mu_0}{4 \pi} \frac{2 I}{r} \sqrt{3} \text { (directed parallel to } \overrightarrow{B A} \text { ) }\)

Example 8. Two small identical circular loops marked (1) and (2) carrying equal currents are placed with their geometrical axes perpendicular to each other. Find the magnitude and direction of the net magnetic field produced at O. Also determine the field when the radius of the loop is very large as compared to the distance of the point from the center.

Electromagnetism Example 8 two small identical circular loops

Solution:

Let R be the radius of each loop. Magnetic field at O due to loop 1,

⇒ \(B_1=\frac{\mu_0 I R^2}{2\left(R^2+x^2\right)^{3 / 2}} \text { (it acts along } \overrightarrow{O X} \text { ) }\)

Magnetic field at O due to loop 2,

⇒ \(B_2=\frac{\mu_0 I R^2}{2\left(R^2+x^2\right)^{3 / 2}} \text { (it acts along } \overrightarrow{O Y} \text { ) }\)

Electromagnetism Example 8 two small identical circular loops.

∴ The net magnetic field at O due to both the loops,

⇒ \(B=\sqrt{B_1^2+B_2^2}=\sqrt{2 B_1^2}=\sqrt{2} B_1 \cdot\left[∵ B_1=B_2\right]\)

⇒ \(=\sqrt{2} \frac{\mu_0 I R^2}{2\left(R^2+x^2\right)^{3 / 2}}=\frac{\mu_0 I R^2}{\sqrt{2}\left(R^2+x^2\right)^{3 / 2}}\)

This field acts at an angle of 45° with \(\vec{OX}\).

Now if R>>x, neglecting x2/R2 we get

⇒ \(B=\frac{\mu_0 I R^2}{\sqrt{2}\left[R^2\left(1+x^2 / R^2\right)\right]^{3 / 2}} \approx \frac{\mu_0 I R^2}{\sqrt{2} R^3}\)

∴ \(B \approx \frac{\mu_0 I}{\sqrt{2} R}\)

Example 9. Two circular coils of radii a and 2a having a common center, carry identical currents, but opposite directions. The number of turns of the second Conductor is 8. Show that magnetic field intensity at the center 3 times that glue to the smaller one. Also, find out the ChangeinWe previous result when current flows in the same direction throughout the coils
Solution:

Magnetic field intensity at the center O due to the smaller loop is,

⇒ \(B_1=\frac{\mu_0 i}{2 a} \text { (upwards) }\)

Similarly magnetic field intensity at the centre due to the bigger loop,

⇒ \(B_2=8 \times \frac{\mu_0 i}{2(2 a)}=\frac{2 \mu_0 i}{a}\) (downwards)

∴ The net magnetic field at O,

B = B2-B1

⇒ \(\frac{\mu_0 i}{a}\left(2-\frac{1}{2}\right)\)

⇒ \(\frac{3 \mu_0 i}{2 a}=3 B_1 \quad\left[∵ B_1=\frac{\mu_0 i}{2 a}\right]\)

Hence resultant field is 3 times that due to the smaller loop.

Now if the direction of the current is the same for both loops, the resultant field will be,

⇒ \(B=B_1+B_2=\frac{\mu_0 i}{2 a}+\frac{2 \mu_0 i}{a}=5 \frac{\mu_0 i}{2 a}=5 B_1\)

Hence the resultant field will be 5 times that due to the smaller loop if current flows in the same direction.

Electromagnetism Example 9 Two circular coils of radii

Example 10. A wire loop is formed by joining two semicircular wires of radii r1 and r2. If the loop carries a current I, find the magnetic field at the center O.

Electromagnetism Example 10 two semicircular wires

Solution:

The magnetic field at the point O due to the semicircular part MNP is,

⇒ \(\dot{B}_{M N P}=\frac{\mu_0}{4} \cdot \frac{I}{r_1} \text { (upwards) }\)

Similarly magnetic field at O due to the semicircular part GFE is,

⇒ \(B_{G F E}=\frac{\mu_0}{4} \cdot \frac{I}{r_2} \text { (upwards) }\)

As the point, O lies along the straight parts ME and PG of the loop, the magnetic field due to them at O is zero.

The so-net magnetic field at O due to the whole loop,

B = B1 + B2

⇒ \(\frac{\mu_0}{4} \cdot \frac{I}{r_1}+\frac{\mu_0}{4} \cdot \frac{I}{r_2}\)

∴ \(B=\frac{\mu_0}{4} I\left[\frac{1}{r_1}+\frac{1}{r_2}\right] \text { (upwards) }\)

Example 11. The radius and number of turns of a circular coil are 10 cm and 25 respectively. What should be the current through the coil that will produce a magnetic field of 6.28 x 10-5Wb.m-2 at its center?
Solution:

Radius of the coil, r = \(\frac{10}{2}\) = 5cm = 0.05m;

number of turns, N = 25, and magnetic field at the center of the coil

B = 6.28 x 10-5Wb.m-2.

Now, \(B=\frac{\mu_0 N I}{2 r}\)

∴ \(I=\frac{2 r B}{\mu_0 r}=\frac{2 \times 0.05 \times\left(6.28 \times 10^{-5}\right)}{\left(4 \times \pi \times 10^{-7}\right) \times 25}\)

= 0.2A

Example 12. The magnetic field due to a current carrying a circular loop of radius 3 cm at a point on the axis at a distance of 4 cm from the center is 54μT. What will be Its value at the center of the loop?
Solution:

Magnetic field on the axis of a circular conductor; \(B=\frac{\mu_0 I}{2} \frac{r^2}{\left(r^2+x^2\right)^{3 / 2}}\) and magnetic field at the centre (x = 0) is

⇒ \(B^{\prime}=\frac{\mu_0 I}{2 r}\)

∴ \(\frac{B^{\prime}}{B}=\frac{\left(r^2+x^2\right)^{3 / 2}}{r^3}\)

or, \(B^{\prime}=\frac{\left(r^2+x^2\right)^{3 / 2}}{r^3} B\)

⇒ \(\frac{\left(3^2+4^2\right)^{3 / 2}}{3^3} \times 54\)

∴ \(B^{\prime}=250 \mu \mathrm{T}\)

Magnetic Effect Of Current And Magnetism

Electromagnetism Ampere’s Circuital Law

Line integral or path integral of a vector: Let \(\vec{A}\) be a vector and 81 a very small line segment. This segment can be treated as a vector \(\delta \vec{l}\), where the magnitude of \(\delta \vec{l}\) is equal to the length of the segment and its direction is along the tangent to that segment. (\(\delta \vec{l}\) is shown in a magnified form),

Electromagnetism Line Integral or path integral of a vector

Now, \(\vec{A} \cdot \delta \vec{l}=A \delta l \cos \theta\)

The sum of the above dot products along a finite line segment PQ can be expressed as an integral (using the symbol dl, instead of δl).

⇒ \(\lim _{n \rightarrow \infty} \sum_{i=1}^n \vec{A} \cdot \delta \vec{l}_i=\int_P^Q \vec{A} \cdot d \vec{l}=\int_P^Q A \cos \theta d l\)

It is called the line integral or path integral of vector \(\vec{A}\) along the path PQ. It is to be noted that the magnitude of vector \(\vec{A}\) may vary between point A to point B and if the direction of \(\vec{A}\) changes, θ will also change. The determination of the magnitude of a line integral is in general very complicated. However, due to different types of symmetry, the integral can often be determined easily.

Closed line integral:

A path that closes on itself is a closed path. To express the line integral of a vector along a closed path the symbol \(\oint\) is used.

For Example, the line integral of the vector \(\vec{A}\), called the circulation of \(\vec{A}\), is given by

⇒ \(\oint \vec{A} \cdot d \vec{l}=\oint A \cos \theta d l\)

Example:

The line integral of force vector work done:

If the force vector \(\vec{F}\) is taken as a special Example of vector \(\vec{A}\), according to the definition of work done we can write, work done by the force \(\vec{F}\) for displacement \(\overline{\delta l}\)

⇒ \(d W=\vec{F} \cdot \delta \vec{l}=F \delta l \cos \theta\)

So, tire total work done along the segment PQ.

⇒ \(W=\int_P^Q d W=\int_P^Q \vec{F} \cdot d \vec{l}=\int_P^Q F \cos \theta d l\)

Similarly, total work done by the force \(\vec{F}\) along a closed path,

⇒ \(W=\oint \vec{F} \cdot d \vec{l}=\oint F \cos \theta d l\)

If the force is conservative, the work done is zero. Naturally, the physical significance of the line integral of the force vector is this integral always indicates tire work done along a line.

Similarly, line integrals of different vectors in physics have different physical significances. For Example, the liter integral of tire electrostatic field \(\vec{E}\) around a closed path is zero because the electrostatic field is a conservation force field.

Statement of Ampere’s circuital law: The line integral of the magnetic field vector along a closed path in any magnetic field is equal to the product of the net current enclosed by the closed path and the permeability of vacuum, i.e.,

⇒ \(\oint \vec{B} \cdot d \vec{l}=\mu_0 I\)….(1)

Here, I = net current enclosed by the closed path.

Explanation:

If Ω is the closed path, it encloses current I, and as a result,

⇒ \(\oint_{X_1} \vec{B} \cdot d \vec{l}=\mu_0 I\)

On the other hand, if we consider the closed path X2, it encloses no current and henceI = 0.

⇒ \(\oint_{X_2} \vec{B} \cdot d \vec{l}\)

Electromagnetism Statement of Ampere's circuital law

2. If any closed path encloses a number of conductors, carrying currents in different directions, the algebraic sum of the enclosed currents is to be taken.

Electromagnetism Statement of Ampere's circuital law.

Currents in the parts AB, BC, and CD may be taken as I, -I, and I, respectively, and for the closed path X we can write,

⇒ \(\oint_X \vec{B} \cdot d \vec{l}=\mu_0(I-l+l)=\mu_0 l\)

Again, since an equal current, I flow through each of Tiro’s turns,

⇒ \(\oint_X \vec{B} \cdot d \vec{l}=\mu_0 N I \quad[N=\text { number of turns }]\)

CGS form of the law: Substituting B→H, I→i and μ0 → 4π we can write,

for a closed path enclosing a single turn

⇒ \(\oint \vec{H} \cdot d \vec{l}=4 \pi i\)

and for a closed path enclosing N turns,

⇒ \(\oint \vec{H} \cdot d \vec{l}=4 \pi N i\)

Application of Ampere’s Circuital Law:

The magnetic field of a long straight wire:

Let a current I flow through a straight long conductor. We have to determine the magnetic field at the point P at a perpendicular distance r from the wire.

Electromagnetism magnetic field of a long straight wire

Taking the wire as an axis, a circular path is drawn through the point P having radius r in such a manner that the tire path lies in a plane perpendicular to the wire.

It is convenient to consider this circular path as Ampere’s closed path. For an element of length \(\delta \vec{l}\) on this closed path, the corkscrew rule shows that the magnetic field \(\vec{B}\) is parallel to \(\delta \vec{l}\) at every place, i.e., the angle between them is 0°.

Again, due to symmetry, the magnitude of \(\vec{B} \text { (i.e., }|\vec{B}|=B \text { ) }\) is the same at every point on the closed path.

So, \(\oint \vec{B} \cdot d \vec{l}=\oint_{B d l \cos \theta}\)

Since the current enclosedÿ by the closed path is I, from Ampere’s circuital law,

B – 2πr = μ0I

or, \(B=\frac{\mu_0 I}{2 \pi r}=\frac{\mu_0}{4 \pi} \cdot \frac{2 I}{r}\)

Solenoid: If a long insulated conducting wire is wound tightly over the surface of a cylinder so that every circular turn is perpendicular to the axis of the cylinder, then this coll is called a solenoid.

The axis of the cylinder is the axis of the solenoid. Usually after making a solenoid the inner cylinder is removed. (A conductor with a coating of insulating material is called an insulated conductor.)

Electromagnetism Solenoid

Magnetic lines of force: The solenoid is placed on a cardboard with its axis lying on the cardboard plane. Now some light iron filings are scattered over the cardboard and the cardboard is slightly tapped.

The iron filings arrange themselves along the magnetic lines of force. The magnetic lines of force.

Electromagnetism Magnetic lines of force

Characteristics of the lines of force: The number density of magnetic lines of force inside the solenoid is very high, i.e., the magnetic field in that part is very strong.

The magnetic field outside the solenoid can be neglected in comparison. Moreover, the magnetic lines of force inside the solenoid are parallel to its axis. So, a strong and nearly uniform axial magnetic field is generated inside the solenoid

Magnetic field inside a long straight solenoid: Let the length of a long straight solenoid = L and its number of turns =N.

So, the number of turns per unit length of the solenoid, n = \(\frac{N}{L}\)A few number of turns of the solenoid is shown. Current through the solenoid = I.

Here, the rectangle abed is taken as the Ampere’s closed path whose side ab=L lies along the axis of the solenoid.

Electromagnetism Magnetic field inside a long straight solenoid

Let N be the number of turns enclosed by the rectangular path, \(\overrightarrow{d l}\) is a very small segment on this path, and \(\vec{B}\) is the magnetic field produced due to this part of the solenoid. According to Ampere’s circuital law,

⇒ \(\begin{array}{r} \oint_{a b c d} \vec{B} \cdot d \vec{l}=\int_a^b \vec{B} \cdot d \vec{l}+\int_b^c \vec{B} \cdot \overrightarrow{d l}+\int_c^d \vec{B} \cdot \overrightarrow{d l} \\ +\int_d^a \vec{B} \cdot d \vec{l}=\mu_0 N I \end{array}\) …(1)

As \(d \vec{l} \text { and } \vec{B}\) are in the same direction along ab so,

⇒ \(\int_a^b \vec{B} \cdot d \vec{l}=\int_a^b B d l \cos 0^{\circ}=B \int_a^b d l=B L\)….(2)

On the other hand, the magnetic field \(\vec{B}\) is perpendicular to the small segments \(d \vec{l}\) on the parts of me and da inside the solenoid. As the solenoid is an ideal one, \(\vec{B}\) = 0 for the side cd and also \(\vec{B}\) = 0 for the parts of me and da which are outside the solenoid.

Hence from equations (1) and (2) we get,

⇒ \(\oint_{a b c d} \vec{B} \cdot d \vec{l}=B L=\mu_0 N I\)…(3)

or, \(B=\mu_0 \frac{N}{L} \cdot I \quad \text { or, } B=\mu_0 n I\)….(4)

It is to be noted here that the magnitude of the magnetic field depends on the number of turns per unit length n but not on the total number of turns N of the solenoid. Hence, to increase the magnetic field it is not sufficient to increase the number of turns but it is also necessary to make the turns very close to each other so that the value of n increases.

CGS expression: Substituting B→H, I→I, and μ0 → 4π in equation (4), we get, H = 4πni.

Toroid: A toroid is a long insulated conducting wire, wound on a donut-shaped core having a circular axis and uniform cross-section so that each turn is normal to the axis.

A toroid is nothing but a solenoid bent to close on itself. A long straight solenoid has two definite ends but a toroid is an endless solenoid.

Electromagnetism Toroid

The magnetic field of a toroid: Let the radius of the ring of a toroid = r and the total number of turns in it = N. Sd,’ the circumference of the circular axis of the toroid = 2nr, arid the number of turns per unit length of it, \(n=\frac{N}{2 \pi r}\). A few turns of a toroid are shown. Current through the toroid = I.

Here, the axis of the toroid is considered as the Ampere’s closed path. If any small part \(8 \vec{l}\) is taken on the axis, according to the corkscrew rule, the direction of the magnetic field \(\vec{B}\) is parallel to \(8 \vec{l}\) at every point, i.e., the angle between them is 0°. Now, due to symmetry, the magnitude of \(\vec{B}\) is the same at all points on the axis.

Electromagnetism Magnetic field of a toroid

Hence,

⇒ \(\oint \vec{B} \cdot d \vec{l}=\oint B d l \cos 0^{\circ}=B \oint d l=B \cdot 2 \pi r\)

Again, the net current enclosed by the axis = current I through each of N turns = NI

So, according to Ampere’s circuital law,

⇒ \(B \cdot 2 \pi r=\mu_0 N I \quad \text { or, } B=\mu_0 \cdot \frac{N}{2 \pi r} I\)

or, B = μ0nI….(5)

It should be noted that equation (5) is identical to equation (4). From this, it is concluded that if a solenoid is too long, whatever may be the shape, the magnitude of the magnetic field at any point on its axis will be B = μ0nI.

CGS expression: Substituting B→H, I→i, and μ0 → 4π in equation (5), we get, H = 4πni.

The core of a solenoid: In the above discussion, air is considered the core of a solenoid or a toroid. Thus, magnetic permeability is taken as μ0. For any other core (like iron, nickel, etc.), the value of the magnetic permeability changes notably. In that case, Ampere’s circuital law takes the form

⇒ \(\oint \vec{B} \cdot d \vec{l}=\mu n I, \text { where } \mu\), is the permeability of the core.

Limitation of Ampere’s circuital law:

Maxwell proved that Ampere’s circuital law is valid only for steady current. If the enclosed current varies with time, on the right-hand side of equation (1), an additional term should be added.

By this correction, Maxwell arrived at his famous electromagnetic field equations. An elaborate discussion about it has been done in the chapter on Electromagnetic waves. We should remember that, Ampere’s law is not incorrect, it can only be called incomplete.

In our discussion, we consider the cases where electric current remains steady with time, and hence, the equations obtained from Ampere’s law are accurate.

Magnetic Effect Of Current And Magnetism

Electromagnetism Numerical Examples

Example 1. A solenoid with 7 turns per unit length Is carrying a current of 2.5 A. What is the magnetic Intensity Inside the solenoid?
Solution:

Turns per unit length, n = 7 cm-1 =700 m-1; if μ is the permeability of the medium inside the solenoid then, magnetic field B = μnl.

∴ Magnetic intensity,

H = \(\frac{B}{\mu}\) = nI

=700 x 2.5

= 1750 A m-1

Example 2. The length of a solenoid is GO cm and Its total number of turns is 1250. If 2 A current is passed through It, what will be the magnetic field at any point on Its axis?
Solution:

Number of turns = 1250, length = 60 cm = 0.6 m

∴ Number of turns per unit length, n = \(\frac{1250}{0.6}\) m-1

So, the magnetic field at any point on its axis,

B = μ0nI

= 4π x 10-7 x \(\frac{1250}{0.6}\) x 2 [μ0 = 47T x 10-7Wb.A-1 m-1]

= 5.23 x 10-3 Wb/m2

Example 3. Two solenoids made of insulated conducting wires and of equal lengths are such that one is wound over another. The resistance of each of them is R and the number of turns per unit length is n. The solenoids are now connected in

  1. Series
  2. Parallel and the combination Is then connected with a battery of emf E.
  3. If current flows through them in the same direction in both cases, determine the magnetic field along the axis of the solenoids in each case

Solution:

1. In case of series combination, equivalent resistance =2R and hence current through each solenoid, \(I_s=\frac{E}{2 R}\)

Hence, the resultant magnetic field along the axis,

⇒ \(B=B_1+B_2=\mu_0 n I_s+\mu_0 n I_s\)

⇒ \(2 \mu_0 n \cdot \frac{E}{2 R}=\frac{\mu_0 n E}{R}\)

in the case of parallel combination, the terminal potential difference across each solenoid = E.

So, current through each solenoid, Ip = \(\frac{E}{R}\).

⇒ \(B=B_1+B_2=\mu_0 n I_p+\mu_0 n I_p\)

⇒ \(2 \mu_0 n \cdot \frac{E}{R}=\frac{2 \mu_0 n E}{R}\)

Example 4. A long straight solid conductor of radius 5 cm carries a current of 2 A, which is uniformly distributed over its circular cross-section. Find the magnetic field at a distance of 3 cm from the axis of the conductor.
Solution:

Let us consider an internal point P at a distance r(= 3 cm) from the axis of the conductor. Imagine a circular path of radius r around the conductor, such that P lies on it. If R is the radius of the solid conductor then the current enclosed by the circular path,

⇒ \(I_1=\frac{I}{\pi R^2} \times \pi r^2=\frac{I r^2}{R^2}\)

Let B be the magnetic field at point P due to the current-carrying conductor. B acts tangentially to the circular path. So according to Ampere’s Circuital law,

⇒ \(\oint \vec{B} \cdot d \vec{l}=\mu_0 I_1\)

or, \(B \times 2 \pi r=\frac{\mu_0 I r^2}{R^2}\)

Here, I = 2 A, r = 3 cm = 0.03 m, R = 5 cm = 0.05 m

∴ \(B=\frac{10^{-7} \times 2 \times 2 \times 0.03}{(0.05)^2}=4.8 \times 10^{-6} \mathrm{~T}\)

Force on A Moving Charge In A Magnetic Field

Let \(\vec{B}\) = magnetic field at a point, q= electric charge of a particle, \(\vec{v}\) = velocity of the particle at the given point.

Magnetic force on the charged particle due to the magnetic field at that point,

⇒ \(\vec{F}=q \vec{v} \times \vec{B}\) ….(1)

If the angle between \(\vec{v} \text { and } \vec{B} \text { be } \theta\), from the cross product of two vectors,

⇒ \(F=|\vec{F}|=q \nu B \sin \theta\)…(2)

Naturally, if v = 0, F = 0, i.e., if a charged particle is at rest then no magnetic force acts on it.

Electromagnetism Example 4 A long straight solid conductor of radius

Definition of magnetic field \(\vec{B}{/latex]:

Direction of [latex]\vec{B}{/latex]: If θ = 0° or 0 = 180°, F = 0. Thus no magnetic force acts on a charged particle that is moving parallel or antiparallel to the magnetic field. Hence, in a magnetic field, the direction (or its opposite direction) of a moving charged particle for which no magnetic force acts on it, defines the direction of [latex]\vec{B}{/latex].

Magnitude of [latex]\vec{B}{/latex]: If v and B remain perpendicular to each other, d = 90°. Thus, the magnitude of the magnetic force [latex]\vec{F}{/latex] becomes maximum. Expressing this maximum force by Fm, equation (2) can be written as,

⇒ [latex]F_m=q v B\)…(3)

If q = 1 and v – 1, then B = Fm. The maximum possible force exerted by a magnetic field on a particle of unit charge moving with unit velocity through the field defines this magnitude of the magnetic field.

Unit of B: From equation (3), \([B]=\frac{[F]}{[q][v]}\)

Hence the unit of B is,

⇒ \(\frac{\mathrm{N}}{\mathrm{A} \cdot \mathrm{s} \cdot\left(\mathrm{m} \cdot \mathrm{s}^{-1}\right)} \quad \text { or } \mathrm{N} \cdot \mathrm{A}^{-1} \cdot \mathrm{m}^{-1}\)

This unit is known as tesla (T) or Weber/metre2 (Wb.m”2).

Significance of the cross product:

Applying the rule of the cross product of two vectors in equation (1) we can conclude that \(\vec{F}\) is always perpendicular to the plane containing \(\vec{v} \text { and } \vec{B}\).

If a right-handed corkscrew rotated from the direction of \(\vec{v} \text { to } \vec{B}\), the direction of advancement of the screw-head indicates the direction of \(\vec{F}\).

Electromagnetism Significance of the cross product

Work done by the magnetic force is zero: \(\vec{F} \text { and } \vec{v}\) is always perpendicular to each other. Since the displacement of a particle is taken along the direction of its velocity, at any point in the magnetic field, the force \(\vec{F}\) and a small displacement \(\vec{ds}\) of the particle are perpendicular to each other. Then, for the magnetic force F and its displacement \(\vec{ds}\), work done,

⇒ \(d W=\vec{F} \cdot d \vec{s}=F d s \cos 90^{\circ}=0\)

Hence, the work done by the magnetic force on a moving charged particle in a magnetic field is zero. In other words, the magnetic force is a no-work force.

Again, we know that, work done on a free particle = change in the kinetic energy of the particle. Since magnetic force is a no-work force, the kinetic energy of a charged particle is not affected.

∴ AK = W =0

or, \(\frac{1}{2}\) mv2 = constant (m = mass of the particle)

or, v = constant

So, when a magnetic force acts on a charged particle moving in a magnetic field, its speed as well as its kinetic energy remains unaffected.

Magnetic force In CGS system: in this system, charge q is expressed in esu [see the chapter: ‘Electric Fields’]. But to indicate the magnetic force, the current as well as the charge must be measured in emu.

1 emu of charge = 3 x 1010 esu of charge = c esu [ c = velocity of light in vacuum = 3 x 1010 cm.s-1 ]

So, if the magnitude of an electric charge is q esu, in the electromagnetic unit, it will be \(\vec{q}{c}\) emu.

Hence, the above-mentioned SI equation (1) can be expressed in

CGS system as,

⇒ \(\vec{F}=\frac{q}{c} \vec{v} \times \vec{B}\)

In this equation, magnetic field \(\vec{B}\) is used but not the magnetic intensity \(\vec{H}\). Here c = 3 x 1010 cm.s-1. F, q, v, and B are measured in dyn, esu, cm.s-1, and gauss or G, respectively.

Note that, 1 C = 0.1 emu of charge = 3 x 109 esu of charge.

Fleming’s left-hand rule: If a charged particle moves at right angles to the magnetic field, is if the angle between \(\vec{v}\) and \(\vec{B}\) be θ = 90°, then from it can be concluded that the three vectors \(\vec{v}, \vec{B} \text { and } \vec{F}\) are mutually perpendicular. This special case can be easily explained by Fleming’s left-hand rule.

Electromagnetism Fleming's left hand rule

Path of a Moving Charge in a Uniform Magnetic Field:

Uniform magnetic field: A magnetic field is said to be uniform if its magnitude and direction remain constant in a region. We know that a magnetic field is represented by magnetic lines of force. For a uniform magnetic field

Magnetic lines of force are parallel to each other because the direction of the magnetic field remains constant;

The magnitude of the magnetic field also remains unchanged and hence the number density of the lines of force at different points are equal, i.e., the lines of force are equispaced.

Moreover, to represent a uniform normal and upward magnetic field with respect to the plane of the page; equispaced marked points are used; on the other hand, to denote a downward uniform magnetic field similar marked points are used

Electromagnetism Uniform magnetic field

In the region adjacent to the surface, the terrestrial magnetic field is assumed to be formed. Only in the vicinity of magnets or magnetic materials, are these lines of force distorted a little.

A uniform magnetic field is also generated between two strong opposite magnetic poles kept very close to each other.

Determination of the path of a charged partake: The charged particle is at rest: Since \(\vec{F}=q \vec{v} \times \vec{B}\) if \(\vec{v}\) = 0, \(\vec{F}\) = 0. Hence, in this case, no magnetic force acts on the particle and the charged particle remains at rest.

The charged particle enters with a velocity \(\vec{v}\) parallel to the magnetic field: If the velocity of the particle and the magnetic field are parallel to each other, then \(\vec{v} \times \vec{B}=0\). So, the magnetic force, \(\vec{F}=q \vec{v} \times \vec{B}=0\). Since no magnetic force acts in this case, the particle continues to move along a straight line.

The charged particle enters with a velocity \(\vec{v}\) perpendicular to the magnetic field:

Let a uniform magnetic field B be acting upward, perpendicular to the plane of the paper. A particle having charge +q enters that magnetic field at P with a velocity \(\vec{v}\) parallel to the plane of the paper.

Applying cross product rule, we see that the magnetic force \(\vec{F}\) ing on the particle will be normal to \(\vec{v}\) and along the direction \(\vec{PO}\).

As a result, the particle will be accelerated towards \(\vec{PO}\), and hence it will tend to deviate from its path.

When the particle reaches another point Q, the magnetic force \(\vec{F}\) will still act normally to \(\vec{v}\) and along \(\vec{QO}\).

In this way, magnetic force acting on the charged particle at every point of its path is always directed towards point O.

This force acts as a centripetal force on the particle, which therefore keeps revolving along a circular path centered at O and of radius r (r = PO = QO). Since the magnitude of velocity does not change under the influence of the magnetic field, the charged particle will have a uniform circular motion.

Electromagnetism The charged particle enters with a velocity

The radius of the circular path: Magnetic force, \(\vec{F}=q \vec{v} \times \vec{B}\).

Since the angle between v and B is 90°, the magnitude of the magnetic force,

⇒ \(F=|\vec{F}|=q v B \sin 90^{\circ}=q v B\)

If the mass of the charged particle is m,

⇒ \(\text { centripetal force }=\frac{m v^2}{r}\)

So, \(q v B=\frac{m v^2}{r} \quad

or, [latex]r=\frac{m v}{q B}\)…(1)

We see from the equation (1) that:

For a given charged particle (q = constant) and for a definite magnetic field (B = constant), the radius of the circular path is directly proportional to the momentum (mv) of the particle. This property is utilized in the measurement of the mass of a charged particle in a mass spectroscope.

If a given charged particle (q = constant) enters a magnetic field with a definite momentum (mv= constant) then the radius of the circular path is inversely proportional to the applied magnetic field (B).

The radius of the circular path is inversely proportional to its specific charge \(\frac{q}{m}\). For Example, the charge of a proton and electron is the same but the mass of a proton is 1836 times that of an electron.

Hence, the value of \(\frac{q}{m}\) for an electron is 1836 times higher. It means that if a proton and an electron enter a magnetic field with equal velocity, the electron revolves in a circular path of a much smaller radius.

It is clear that if the charge is -q instead of +q, the direction of uniform circular motion will be reversed.

Period of resolution and cyclotron frequency: Circumference of the circular path =2πr.

Since it is a uniform circular Otiort, time period,

⇒ \(T=\frac{2 \pi r}{\nu}=2 \pi \frac{\dot{m}}{q B}\)…(2)

The number of complete revolutions made in unit time, i.e., frequency of the circular motion,

⇒ \(n=\frac{1}{T}=\frac{1}{2 \pi}\left(\frac{q}{m}\right) B\) ….(3)

This frequency n is called cyclotron frequency.

Evidently, both T and n are independent of the radius of the path as well as of the velocity. From equation (1), it is clear that \(frac{r}{v}\) = constant. This property for the motion of a charged particle in a magnetic field is utilized in particle accelerators like cyclotrons.

The charged particle enters the magnetic field at an inclined path: Let the z-axis be chosen along the direction of a uniform magnetic field acting at a place. A particle of charge +q and mass m enters that magnetic field with velocity \(\vec{v}\) at the point P on the xy -plane.

If this velocity \(\vec{v}\) is inclined at an angle θ with the magnetic field \(\vec{B}\), the component of velocity along \(\vec{B}\), i.e., along z-axis = vcosθ and the component of velocity perpendicular to B, i.e., on the xy -plane = vsinθ.

Electromagnetism The charged particle enters the magnetic field at an inclined path

Naturally, no magnetic force acts on the particle the component cost, and hence this corÿonent remains unchanged. So, the charged particle performs a uniform linear motion along the magnetic field.

Again, the component vsinθ produces a uniform circular motion. The radius o|this uniform circular motion can be obtained by using for v in equation (1),

⇒ \(r=\frac{m v \sin \theta}{q B}\)….(4)

The time period and frequency of the particle in uniform circular motion do not depend on the velocity of the particle; hence just like equations. (2) and (3), it can be written as

⇒ \(T=\frac{2 \pi m}{q B} \text { and } n=\frac{1}{T}=\frac{q B}{2 \pi m}\)….(5)

Due to the combination of linear motion parallel to the z-axis and uniform circular motion on the xy-plane, the charged particle follows a spiral or helical path.

The axis of this helical path is the z-axis. For each complete revolution of the particle, the distance covered along the z-axis, i.e., along the direction of the magnetic field is called the pitch of this helical motion.

So, pitch = time period x linear velocity

⇒ \(\frac{2 \pi m}{q B} \cdot v \cos \theta\)

⇒ \(\frac{2 \pi m}{q B} \cdot \frac{q B r}{m \sin \theta} \cdot \cos \theta\)

= 2πrcotθ

= circumference of the circular path x cotθ …(6)

The property of the helical motion of a charged particle in a magnetic field is utilized in the magnetic focussing of different equipment.

Cyclotron:

The cyclotron was developed in 1932 by Professor E O Lawrence at the BerkelyInstitute, taliformia is a powerful particle accelerator for accelerating positively charged particles such as protons, aaa particles, etc., to very high energies so that they can be used in disintegration experiments.

Electromagnetism Cyclotron

Description: It consists of two cylindrical shells of copper having semicircular cross sections with the same diameter and height. They are open towards their diameter and all other sides are closed.

The diameters of the two shells are much larger than their heights. They are arranged side by side in such a way that a small gap exists between their diameters. Each is called ‘dee’ on account of its shape like the letter D.

Two pole pieces of a strong electromagnet are placed above and below the dees in such a way that a uniform magnetic field -defects perpendicular to the plane of the dees.

An alternating potential of the order of 105 V and of high frequency (about 106 Hz) is applied between the dees. So the dees act as the two electrodes -of the source of potential.

An ion source S is located near the center of the dees and it supplies the positive ions to be accelerated. At the periphery of the dees, an auxiliary negative electrode deflects the accelerated ions onto the target to be bombarded.

The whole space inside the dee is evacuated to a pressure of about 10-6 mm of mercury. If the ‘whole arrangement is seen horizontally.

Electromagnetism Cyclotron.

Principle of action: Suppose that a positive ion (e.g., hydrogen ion H+ or helium ion He++ ) emerges from the ion sources S. Generally these ions are produced by bombarding gas molecules with high-velocity electrons.

The positive ion produced at S will be attracted to whichever dee happens to be negative at that moment. Due to this attractive force-velocity of the ion increases and it enters the dee. Inside the dee, there is no electric field.

Owing to the magnetic field which is perpendicular to the plane of the dee the ion describes a semicircular path. The radius of the path is obtained from equation (1) of the section.

The ion moves in this path at a constant speed. At the end of the semicircular path when the ion arrives at the gap between the dees, then the other dee should become negative. Then the ion is again attracted, its velocity increases and it enters the other dee. From equation

It is understood that due, to the increase of velocity v, the radius r of the semicircular path will also increase. But from the equation

Of section, it is seen that in spite of the increase in velocity and radius, the time taken to describe each of the semicircular paths remains the same.

In this way, then at last reaches the outer edge of the dee and comes out with a high velocity. To increase the velocity of the ion, the magnetic field (\(\vec{B}\)) may be increased

Resonance condition: It is obvious that the condition of proper operation of a cyclotron is that the time taken by the ion to traverse a semicircular path will be equal to half of the time period of the alternating potential. So the frequency of the applied alternating potential difference n0 must be equal to the frequency of revolution of the ion, i.e., cyclotron frequency n.

⇒ \(\frac{1}{2 \pi}\left(\frac{q}{m}\right) B=n_0\)….(1)

This equation is called the resonance condition of the cyclotron. In practice, the value of n0 is kept fixed and the magnetic field B is varied until the above condition is satisfied.

The kinetic energy of the accelerated particle: If v is the velocity of a charged particle of mass m and charge q and if it moves perpendicular to a magnetic field B, the radius of its circular path according to the equation (1) of the section is,

⇒ \(r=\frac{m v}{q B} \quad\)

or, \(v=\frac{q B r}{m}\)

If R is the radius of a dee of the cyclotron, then the velocity of the charged particle ejected from the outlet is given by,

⇒ \(v_0=\frac{q B R}{m}\)

So, the kinetic energy of, the particle,

⇒ \(E=\frac{1}{2} m \nu_0^2=\frac{q^2 B^2 R^2}{2 m}\)….(2)

The kinetic energy of the accelerated particle Disadvantages:

1. If the velocity of the charged particle is high enough to be displacement comparable to the velocity of light, the mass of the particle does not remain constant.

According to the theory of relativity, m increases with the increase of velocity. So the resonance condition according to equation (1) is violated and the cyclotron does not function.

For an electron, the relativistic increase of mass is much greater. So the electrons very quickly get out of step. Hence a cyclotron is not used for accelerating electrons.

2. To make the charged particle sufficiently fast, dees of very large diameter are to be taken. This diameter may even exceed 100m. The construction of an electromagnet of so large diameter is prohibitively expensive and technically complicated.

Remedies:

1. With the increase of mass due to relativity the frequency n0 of the alternating source may also be diminished in such a way that the product mn0 always remains constant. In that case, the resonance condition is never violated. The machine with this arrangement is called a synchro-cyclotron.

2. Both B and n0 can be changed simultaneously in such a way that

  1. The resonance condition is always satisfied
  2. In spite of the increase in velocity of the charged particle, the radius r of its circular path remains unchanged. So a thin annular electromagnet of that radius will serve the purpose. Hence technical complications and expenses may be reduced sufficiently. This machine is called a synchrotron.

Path of a Charged Particle in a Uniform Electric Field:

If a particle of charge +q experiences a force \(\vec{F}\) in a uniform electric field, from definition,

the electric field, \(\vec{E}=\frac{\vec{F}}{q} \quad \text { i.e., } \vec{F}=q \vec{E}\)…(1)

If the mass of the particle is m, then acceleration,

⇒ \(\vec{a}=\frac{\vec{F}}{m}=\frac{q \vec{E}}{m}\)….(2)

Naturally, for negative charges, the acceleration (\(\vec{a}\)) will be opposite to \(\vec{E}\). Due to this acceleration, the charged particle acquires different kinds of motion in different cases.

1. The charged particle is initially at rest: Initial velocity u = o. Hence, after time t,

the velocity of the particle, \(v=a t=\frac{q E}{m} t\)

and displacement, \(s=\frac{1}{2} a t^2=\frac{q E}{2 m} t^2\)

2. The charged particle enters with a velocity \(\vec{u}\) along the electric held: Since the direction, of acceleration, is in the direction of \(\vec{E}\) after time t,

velocity of the particle, \(v=u+\frac{q E}{m} t\)

and the displacement, \(s=u t+\frac{1}{2} \frac{q E}{m} t^2\)

3. The charged particle enters with a velocity \(\vec{u}\) perpendicular to the electric field: Let the uniform electric field act parallel to the y-axis and the charged particle enters this electric field with a velocity \(\vec{u}\) at the point P along the x-axis

Electromagnetism the charged particle enters with a velocity.

There is no component of E along the x-axis and hence the particle has no acceleration in this direction. So, along the x-axis, the particle possesses a uniform velocity u.

If the distance covered by the particle in time t is x, then t = \(\frac{x}{u}\). Again, along y-axis, initial velocity of the particle = 0, but acceleration, a = \(\frac{qE}{m}\).

So, in time t, displacement of the particle along the y-axis is,

⇒ \(y=\frac{1}{2} a t^2=\frac{1}{2} \frac{q E}{m}\left(\frac{x}{u}\right)^2=\frac{q E}{2 m u^2} \cdot x^2\)

or, \(x^2=\frac{2 m u^2}{q E} y\)….(3)

This is an equation of a parabola (in the form of x² = 4ay)

So, the path of the particle will be parabolic.

Path of Charged Particle in Crossed Electric Field and Magnetic Field:

Let a particle of charge +q and mass m enter an electromagnetic field at the point O along the z-axis with a velocity \(\vec{v}\),

Electric field \(\vec{E}\) at that point O is along the x-axis and magnetic field \(\vec{B}\) is along the y-axis, i.e.,\(\vec{v}\) \(\vec{E}\) and \(\vec{B}\) are mutually perpendicular. Now, the electric force acting on the particle along the x-axis = qE. The magnetic force acting on the particle = qvB, and the cross-product rule shows that this magnetic force acts along the negative direction of the x-axis.

Electromagnetism Path of Charged Particle in Crossed Electric field and magnetic field

So, here the electric and the magnetic fields are oppositely directed. Keeping the directions of i, and B unchanged, if these two forces are made equal in magnitude, the net force acting on the charged particle becomes zero. In this case, the charged particle continues its motion along the z-axis with its initial Velocity v without suffering any deviation. The condition for this situation is,

⇒ \(q E=q v B \quad \text { or, } \quad \nu=\frac{E}{B}\)

Suppose the electric field E and magnetic field \(\vec{B}\) are acting simultaneously perpendicular to each other.

Let a stream of charged particles enter the space in a direction perpendicular to both \(\vec{E}\) and \(\vec{B}\). If the speed of the charged particles is different, then only those particles whose speed is equal to the ratio \(\frac{E}{B}\) will pass through the hole S on the screen without any deflection.

All other particles will be deflected from, their path and will not reach point S. This arrangement is known as a velocity filter. In the figure only the green particle would pass through the hole S as its velocity is equal to \(\frac{E}{B}\).

Electromagnetism particle continues its motion along

This principle, known as the velocity selector, Is utilized In the determination of Q specific charge \(\frac{q}{m}\) of an electron by J J Thomson’s experiment and θ mass of the nucleus with the help of mass-spectrometer.

Lorentz Force:

In an electromagnetic field, if a charge q enters with velocity \(\vec{v}\), the forces acting on it are

Electric force, \(\vec{F}_e=q \vec{E} \quad[\vec{E}=\text { electric field }]\)

Magnetic force, \(\vec{F}_m=q \vec{v} \times \vec{B} \quad[\vec{B}=\text { magnetic field }]\)

These forces, \(\vec{F}_e \text { and } \vec{F}_m\) are called Lorentz electric force and Lorentz magnetic force, respectively. The resultant Lorentz
force acting on the particle (charge),

⇒ \(\vec{F}=\vec{F}_e+\vec{F}_m=q \vec{E}+q \vec{v} \times \vec{B}\) – q (\(\vec{F}\) x \(\vec{v}\) x \(\vec{B}\))….(1)

In CGS or Gaussian system:

⇒ \(\vec{F}_{e q}=q \vec{E}, \vec{F}_m=\frac{q}{c}(\vec{v} \times \vec{B})\)

So, the resultant Lorentz force, \(\vec{F}=q\left[\vec{E}+\frac{1}{c}(\vec{v} \times \vec{B})\right]\)

Remarks:

1. If the charged particle is at rest in an electromagnetic field, an electric force still acts on it, but as [lavec]\vec{v}[/latex] = 0 no magnetic, force acts.

2. For negatively charged particles (e.g., electron or negative ion) q is replaced by -q, and hence the direct|on of each force \(\left(\vec{F}_e \vec{F}_m \text { and } \vec{F}\right)\) will be reversed.

Magnetic Effect Of Current And Magnetism

Electromagnetism Numerical Examples

Example 1. An amagnetic field of 0.4 T is applied on a proton moving with a velocity of 5 x 10-6 m s-1. The magnetic field acts at an angle of 30º with the direction of velocity of the proton. What will be the acceleration of the proton? (mass ofa proton = 1.6.x 10-27 kg)
Solution:

Force acting on the proton, F= qvBsinθ.

∴ Acceleration antiproton = \(\frac{\text { force acting }}{\text { mass of proton }}=\frac{q v B \sin \theta}{m}\)

Here, q = 1.6 x 10-19 C, v = 5 x 106 m.s-1,

B = 0.40 T, θ = 30° and m = 1.6 x 10-27 kg.

∴ Acceleration of the proton

⇒ \(\frac{1.6 \times 10^{-19} \times 5 \times 10^6 \times 0.40 \times \sin 30^{\circ}}{1.6 \times 10^{-27}}\)

= 1014 m.s-2

Example 2. An electron (mass = 9 x 10-31g, charge = 1.6 x 10-19 C) enters a magnetic field with a velocity of 106 m si and starts rotating in a circular path of radius 10 cm. What is the value of the magnetic field?
Solution:

The electron is rotating in a circular path i.e. the direction Of the velocity of the electron is perpendicular to the direction of the magnetic field. So the magnetic force = qvB, which provides the necessary centripetal force for the circular motion.

∴ \(q v B=\frac{m v^2}{r}\) [r = radius of the circular path = 10cm = 0.1 m]

or, \(B=\frac{m v}{q r}=\frac{\left(9 \times 10^{-31}\right) \times 10^6}{\left(2.6 \times 10^{-19}\right) \times 0.1}\)

= 5.6 x 10-5 T

Example 3. Two particles of equal charge are accelerated by applying the same potential difference and then allowed to enter a uniform magnetic field normally. If the particles keep revolving along circular paths of radii R1 and R2, determine the ratio of their masses
Solution:

If the charge of each particle is q and the potential difference applied is V, kinetic energy acquired = qV

So, \(q V=\frac{1}{2} m_1 v_1^2=\frac{1}{2} m_2 v_2^2\)

or, \(\frac{v_1}{v_2}=\sqrt{\frac{m_2}{m_1}}\)

Again, the magnetic force required to revolve along a circular path = centripetal force.

∴ For the first particle,

⇒ \(q v_1 B=\frac{m_1 v_1^2}{R_1} \quad\)

or, \(q B=\frac{m_1 v_1}{R_1}\)

For the second particle, \(q B=\frac{m_2 v_2}{R_2}\)

∴ \(\frac{m_1 v_1}{R_1}=\frac{m_2 v_2}{R_2} \quad\)

or, \(\frac{m_1}{m_2}, \frac{v_1}{v_2}=\frac{R_1}{R_2}\)

or, \(\frac{m_1}{m_2} \cdot \sqrt{\frac{m_2}{m_1}}=\frac{R_1}{R_2} \quad\)

or, \(\sqrt{\frac{m_1}{m_2}}=\frac{R_1}{R_2}\)

or, \(\frac{m_1}{m_2}=\left(\frac{R_1}{R_2}\right)^2\)

Example 4. In a cyclotron, the frequency of alternating current is 12 MHz and the radius of its dee is 0.53 m.

  1. What should be the operating magnetic field to accelerate protons?
  2. What is the kinetic energy of the proton beam produced by the cyclotron? Given, the mass of the proton = 1.67 x 10-27 kg and charge = +1.6 X 10-19 C.

Solution:

1. Condition of resonance,

⇒ \(\frac{1}{2 \pi}\left(\frac{q}{m}\right) B=n_0 \quad \text { or, } B=\frac{2 \pi n_0 m}{q}\)

Here, n0 = 12 MHz = 12 x 106 Hz = 12 x 106 s-1

So, \(B=\frac{2 \times 3.14 \times\left(12 \times 10^6\right) \times\left(1.67 \times 10^{-27}\right)}{1.6 \times 10^{-19}}\)

= 0.79 Wb.m-2

2. Kinetic energy of proton,

⇒ \(E^{\prime}=\frac{q^2 B^2 R^2}{2 m}\)

= \(\frac{\left(1.6 \times 10^{-19}\right)^2 \times(0.79)^2 \times(0.53)^2}{2 \times\left(1.67 \times 10^{-27}\right)}\)

= \(1.34 \times 10^{-12} \mathrm{~J}\)

= \(\frac{1.34 \times 10^{-12}}{1.6 \times 10^{-19}} \mathrm{eV}\)

= 8375 x 10-6 eV

= 8.375 MeV

Example 5. A beam of protons with velocity 4 x 10 sm.s-1 enters a uniform magnetic field of 0.4 T at an angle of 60° to the magnetic field. Find the radius of the helical path of the proton beam and the time period of revolution. Also, find the pitch of the helix. Mass of proton = 1.67 X 10-27 Vg
Solution:

Mass of the proton, m = 1.67 x 10-27 kg velocity of the proton, v = 4 x 105 m.s-1; charge, q = 1.6 x 1049 C.

Electromagnetism Example 5 A beam of protons with velocity

∵ The component of velocity perpendicular to the field is vsinθ, the radius of the helical path,

⇒ \(r=\frac{m \nu \sin \theta}{q B}\)

= \(\frac{\left(1.67 \times 10^{-27}\right)\left(4 \times 10^5\right) \sqrt{3}}{\left(1.6 \times 10^{-19}\right) \times 0.4 \times 2}\)

= 9 x 10-3 m

= 0.9 cm

During the time period of the revolution,

⇒ \(T=\frac{2 \pi r}{\nu \sin \theta}\)

= \(\frac{2 \times 3.14 \times 0.009 \times 2}{4 \times 10^5 \times \sqrt{3}}\)

= 1.63 x 10-7 s

The pitch of the helix,

p = vcosθ x T =4 x 105 x \(\frac{1}{2}\) x 1.63 x 10-7

= 3.26 x 10-2 m

= 3.26 cm

Magnetic Effect Of Current And Magnetism

Electromagnetism Action Of A Magnet On Current

Oersted’s experiment deals with the influence of a current-carrying conductor on a magnet. The magnet, in reaction, also exerts equal but opposite force on the current-carrying conductor.

So, it is possible to move a current-carrying wire in a magnetic field. This kind of motion is very important for practical purposes. This phenomenon is the subject matter of electrodynamics.

Force on a Current-Carrying Conductor in a Magnetic Field:

The Lorentz force acting on a charge q moving with velocity \(\vec{v}\) in a magnetic field \(\vec{B}\) is,

⇒ \(\vec{F}=q \vec{v} \times \vec{B}\)

When current flows in a circuit, the drift motion of free electric charges through the circuit is considered as the cause of that current. Naturally, the magnetic force acting on those free charges in a magnetic field acts as the force on the current-carrying conductor.

Let the current through a circuit = I; a V element of length \(\vec{dl}\) of that circuit is considered.

Electromagnetism Force on a Current Carrying Conductor in a magnetic field

The magnetic field at the position of this element of the circuit = \(\vec{B}\). If dq amount of charge crosses dl length in time dt, current the circuit, \(I=\frac{d q}{d t}\)

Velocity of the charge, \(\vec{v}=\frac{d \vec{l}}{d t}\)

So, the magnetic force acting on the small part \(\vec{dl}\)

⇒ \(d \vec{F}=d q \vec{v} \times \vec{B}=d q \cdot \frac{d l}{d t} \times \vec{B}=\frac{d q}{d t} d \vec{l} \times \vec{B}\)

∴ \(d \vec{F}=I d \vec{l} \times \vec{B}\) ….(1)

If the angle between \(d \vec{l} \text { and } \vec{B}\) be θ, then

\(dF=|d \vec{F}|=I d l B \sin \theta=B I d l \sin \theta\)….(2)

So, the magnetic force acting on the whole circuit or on a finite part of it,

⇒ \(F=\int d F=\int B I d l \sin \theta\)…(3)

Generally, for different elements- of the circuit, both B and θ may change. Hence, in the absence of some special symmetry, the integral of equation (3) becomes very complicated.

Special cases:

The current carrying wire is at right angles with a uniform magnetic field: For any small part of the wire, θ = 90° or sinθ = 1. If the magnetic field is uniform, B = constant. If a current carrying wire of length is placed in this uniform magnetic field, according to equation (3) the force acting on the wire will be,

F = BIl….(4)

Electromagnetism The current carrying wire is at right angles

The current-carrying wire is placed parallel to the magnetic field: For any small part of the wire, 0 = 0° or sinθ = 0.

So, according to equation (3), the force acting on the wire is,

F = 0….(5)

Note that, the units of l or dl, I, B, and F are m, A, Wb.m-2, and N, respectively.

Fleming’s left-hand rule:

If a long straight current-carrying conductor is kept at right angles with a uniform magnetic field, i.e., if for each element of the wire, the angle between \(\vec{dl}\) and \(\vec{B}\) is θ = 90°, then according to the rule of cross-product we can say from equation (1) that the three vectors \(\vec{dl}\), \(\vec{B}\) and \(\vec{dF}\) will be mutually perpendicular.

Fleming’s left-hand rule is a handy tool for determining the direction of the magnetic force acting on a current-carrying conductor, perpendicular to the magnetic field.

Statement: If the forefinger, middle finger, and thumb of the left hand are stretched mutually perpendicular to each other, such that the forefinger points to the direction of the magnetic field and the middle finger points to the direction of current, then the thumb points to the direction of force experienced by the conductor.

Electromagnetism Fleming's left hand rule.

This left-hand rule is also known as the motor rule.

Barlow’s wheel: We can demonstrate the action of a magnet on an electric current with the help of this experimental arrangement.

Description: B is the Barlow’s wheel made of a thin copper plate having a number of sharp teeth. The wheel B is kept vertical such that its one sharp tooth remains in contact with mercury kept in a container placed in between two poles of a strong horse-shoe magnet (NS). Now the wheel and the mercury inside the container (M) are connected to a source of current E.

Electromagnetism Barlow's wheel

Working principle: The magnetic field acts from the JV-pole to the S-pole of the magnet parallel to the horizontal platform.

Since the tooth of the wheel remains in contact with mercury, the circuit is closed as soon as the battery is switched on. As a result, the current flows in a downward direction.

From Fleming’s left-hand rule, we see that a magnetic force acts on the current toward the right. Due to this force, the tooth will be deflected towards the right. As a result, the circuit breaks.

Hence magnetic force will no longer act on the wheel. But due to inertia of motion, the next tooth comes in contact with mercury and in a similar way will be deflected towards the right. Hence, a continuous anticlockwise motion will be observed in Barlow’s wheel.

Discussions:

1. To get an effective result, the wheel should be very light. Moreover, the magnetic field and the electric current should be very strong.

2. If the direction of either the magnetic field or the electric current is reversed, the rotation of the wheel will also be reversed; but if both of them are reversed then the rotation of the wheel will be in the same direction.

3. The machine which converts electrical energy into rotational energy (usually mechanical energy) is called a motor. So, Barlow’s wheel is a motor, although it has no practical use.

Torque on a Current Loop in a Uniform Magnetic Field:

PQRS is a rectangular conductor. Its length, PQ = RS =l and breadth, QR = PS = b

So, the area of the rectangular face of the conductor, <A= Tb. This rectangular conductor is placed in a magnetic field B in such a way that

  1. PQ and RS 316 perpendicular to the magnetic field
  2. The surface PQRS is parallel to the magnetic field.

Electromagnetism Torque on a Current Loop in a Uniform magnetic field

If a current is sent through the conductor,

1. No force acts on the arms QR and PS as they are parallel to the magnetic field,

2. Magnitude of the force (\(\vec{F}\)) acting on each of the arms PQ and RS, F = BIl.

Applying Fleming’s left-hand rule, we see that these two equal forces are downward and upward, respectively and hence they constitute a couple.

Since the perpendicular distance between these two forces is QR = PS = b, the moment of the couple, i.e., the torque acting on the coil is,

⇒ \(\tau=B I l \cdot b=B I A\)….(1)

and due to this torque, the coil starts to rotate. Instead of a single turn, if the conducting coil has N turns, torque acting on it,

⇒ \(\tau\) = BNIA….(2)

Equations (1) and (2) show that the torque r depends only on area A of the coil. This means that the shape of the coil is not important. Instead of a rectangular coil, a coil of any other shape of equal area may be used to obtain the same torque.

Vector representation of torque: We know that any plane surface can be treated as a vector, such that the magnitude of the vector is equal to the magnitude of its area and its direction is along the normal to that plane.

So, the quantity A in equation (1) can be expressed as a vector \(\vec{A}\) the direct arbitrary position of the rotating loop under the influence of torque. If the angle between \(\vec{A}\) and the magnetic field \(\vec{B}\) is θ, from equation (1) we get,

⇒ \(\tau=B I A \sin \theta\)….(3)

The vector representation of the above equation is,

⇒ \(\vec{\tau}=I \vec{A} \times \vec{B}\)…..(4)

So, the torque acting on a conducting coil having N turns will be

⇒ \(\vec{\tau}=N I \vec{A} \times \vec{B}\)….(5)

Electromagnetism Vector representation of torque

Magnetic Effect Of Current And Magnetism

Electromagnetism Numerical Examples

Example 1. 2A current is flowing through a circular coil of radius 10 cm, made of insulated wire and having 100 turns.

  1. If the circular plane of the conductor is kept at right angles to the direction of a magnetic field of 0.2 Wb.m-2, determine the force acting on the coil.
  2. If the conductor is placed parallel to the magnetic field, determine the torque acting on it.

Solution:

Here,N = 100, r = 10 cm = 0.1 m,

B = 0.2 Wb m-2.

∴ Circumference of the circular loop,

L = 2πr

= 2 x I x 0.1

= 0.628 m

Area of the circular loop,

A = πr2 = π x (0.1)2

= 0.0314 m2

1. The whole circumference of the conductor is normal to the magnetic field; hence magnetic force

= NILB

= 100 x 2 x 0.628 x 0.2

= 25.12 N

[It should be noted that, in this situation, if the circular plane is taken as a vector, its direction will be perpendicular to that plane, i.e., parallel to the magnetic field. As a result, no torque will act on the conductor.]

2. If the circular plane is taken as a vector, its direction will be perpendicular to the plane, i.e., normal to the magnetic field.

As a result, the torque acting on the conductor

= NIAB

= 100 x 2 x 0.0314 x 0.2

= 1.256 N.m

Example 2. The radius of a circular coil having 100 turns Is 5 cm and a current of 0.5 A is flowing through this coil. If it is placed in a uniform magnetic field of strength 0.001 T, then what torque will act on the coil when the plane of the coil is

  1. Parallel to the magnetic field
  2. Inclined at 30° with the magnetic field
  3. Perpendicular to the magnetic field?

Solution:

Number of turns = 100

Current, I = 0.5A

Radius of the circular coil, r = 5 cm = 5 x 10-2m

∴ Area of the coil, A = πr2 = 3.14 x (5 x 10-2)2m2

Magnetic field intensity, B = 0.0001 T

Now torque acting on the current carrying coil due to magnetic field,

⇒ \(\tau=n B I A \sin \theta\)

1. When the coil is parallel to the malefic field, then θ = 90°

So, \(\tau=n B I A \sin 90^{\circ}\)

= 100 x 0.001 x 0.5 x 3.14 x 25 x 10-4

= 3.93 x 10-4 N.m

2. When the coil makes an angle 30° with the field, then θ = 60°

So, \(\tau=n B I A \sin 60^{\circ}\)

⇒ \(=100 \times 0.001 \times 0.5 \times 3.14 \times 25 \times 10^{-4} \times \frac{\sqrt{3}}{2}\)

= 3.4 X 10-4 N.m

3. When the coil is perpendicular to the field, θ = 0°.

∵ \(\tau=0\)

Example 3. On a smooth plane inclined at 30° with the horizontal, a thin current-carrying metallic rod is placed parallel to the horizontal ground. The plane is In a uniform magnetic field of 0.15 T along the vertical direction. For what value of current can the rod remain stationary? The mass per unit length of the rod is 0.30kg/m.
Solution:

Along the inclined plane, the component of the magnetic force acting on the conductor and the component of the weight of the conductor will bring equilibrium. So,

BIlcosd = mgsinθ….(1)

Here, force on the conductor BIl acts horizontally towards the right.

Now, from equation (1),

⇒ \(I=\frac{m g \sin \theta}{B l \cos \theta}=\frac{m g}{B l} \tan \theta\)

∵ \(\frac{m}{l}=0.30 \mathrm{~kg} / \mathrm{m}, g=9.8 \mathrm{~m} / \mathrm{s}^2\)

B = 0.15 T, θ = 30°

∴ \(I=\frac{0.30 \times 9.8}{0.15} \tan 30^{\circ}\)

⇒ \(=2 \times 9.8 \times \frac{1}{\sqrt{3}}=11.3 \mathrm{~A}\)

Electromagnetism Example 3 On a smooth plane inclined

Magnetic Effect Of Current And Magnetism

Electromagnetism Action Of Current On Current

Like parallel currents: Let two straight parallel conductors PQ and RS be kept horizontally and I1 and I2 be the currents flowing through them in the same direction

Electromagnetism like parallel currents

According to the corkscrew rule, for current I1, the magnetic field B at any point O on the wire RS will act downwards. If Fleming’s left-hand rule is applied at point O on wire RS, it is seen that the wire RS will experience a force F towards the wire PQ.

As Reactiion, the wire PQ also will experience the same force F towards the wire RS. So, we can conclude that two parallel currents attract each other.

Unlike parallel currents: Suppose I1 and I2 currents are flowing through the wires PQ and RS, respectively mutually opposite directions. Applying Fleming’s left-hand rule similarly, it is found that the wire RS experiences a force F away from the wire PQ. The wire PQ also experiences an equal but opposite force that acts on It away from the wire RS. So, we can conclude that two unlike parallel currents repel each other.

The magnitude of the force of attraction or repulsion: Let r be the perpendicular distance between two long, straight parallel conductors kept in a vacuum or air.

Due to current I1 the magnetic field at a distance of r will be

⇒ \(B=\frac{\mu_0}{4 \pi} \cdot \frac{2 I_1}{r}\)

From the relation F = BIl, we get, the force acting on the unit length of the wire placed at r, carrying current I2, which is

⇒ \(\left.F=\frac{\mu_0}{4 \pi} \cdot \frac{2 I_1 I_2}{r} \text { [Putting } B=\frac{\mu_0}{4 \pi} \cdot \frac{2 I_1}{r} \text { and } l=I_2\right]\)….(1)

Here, the unit of r is m, I1, and I2 is A, F is N.m-1, and μ0 = 4π x 10-7 H.m-1.

This equation gives the definition of international ampere. If I1 = I2 = 1 A and r = 1 m,

⇒ \(F=\frac{\mu_0}{4 \pi} \times \frac{2 \times 1 \times 1}{1}=\frac{4 \pi \times 10^{-7}}{4 \pi} \times 2\)

= 2 x 10-7 N

1 ampere: Two long straight parallel conducting wires, having negligible cross sections and carrying equal currents are kept 1 m apart from each other. The steady direct current for which each wire experiences a force of 2 x 10-7 N per unit length, is called 1 ampere.

Expressions in Gaussian system: Substituting \(I_1 \rightarrow i_1, I_2 \rightarrow i_2 \text { and } \mu_0 \rightarrow 4 \pi\) in equation (1) we get,

⇒ \(F=\frac{2 i_1 i_2}{r}\)

Here unit of r is cm, F is dyn, and i1 and i2 emu

Obliquo currents:

1. If currents through two oblique straight conductors lying in the same plane converge to or diverge from their point of intersection, they attract each other

Electromagnetism Obliquo currents

2. If currents through two oblique straight conductors lying in the same plane are such that, one of them is directed towards and the other directed away from their point of intersection, they repel each other.

Experimental Demonstration Roget’s Vibrating Spiral:

Description: A long elastic spring made of insulated copper wire is suspended from a rigid support. A small spherical copper bob, attached at the lower end of the spring, just touches the mercury kept in a vessel. The fulcrum at the upper end of the spring and mercury in the vessel are joined to a battery.

Electromagnetism Experimental Demonstration Roget’s vibrating sprial

Working principle: When the battery is switched on, a unidirectional parallel current flows through the turns of the spring and hence the turns attract each other, resulting in the contraction of the spring.

Naturally, the copper bob remains no longer in contact with mercury and hence the circuit is cut off. As soon as the bob leaves the surface of the mercury, the attraction between the turns of the spring no longer exists.

Hence, the spring elongates due to its own weight, the copper sphere touches the mercury again and closes the circuit. In this way, alternate compression and elongation of the spring goes on and hence the spring vibrates continuously.

This experiment proves that a number of unidirectional parallel currents attract each other.

Magnetic Effect Of Current And Magnetism

Electromagnetism Numerical Examples

Example 1. Two very long- conducting wires are kept at a distance of 4 cm from each other in a vacuum. Currents flowing through the wires are 25A and 5A, respectively. Find the length of each conductor, which experiences a force of 125 dynes.
Solution:

Here, I1 = 25 A, I2 = 5 A, r = 4 cm = 0.04 m

So, the force acting per unit length of the wire,

⇒ \(F=F=\frac{\mu_0}{4 \pi} \cdot \frac{2 I_1 I_2}{r}=\frac{4 \pi \times 10^{-7}}{4 \pi} \cdot \frac{2 \times 25 \times 5}{0.04}\)

= 625 x 10-6 N.m-5

For 125 dyn or 125 x 10-5 N force, the effective length of each wire,

⇒ \(l=\frac{125 \times 10^{-5}}{625 \times 10^{-6}}=2 \mathrm{~m}\)

Example 2. Two long straight parallel conducting wires kept 0.5 m apart, carry 1 A and 3 A currents, respectively.

  1. What is the force acting per unit length of tire two wires?
  2. At what position in the plane of the wires, the resultant magnetic field will be zero?

Solution:

1. Force acting per unit length,

⇒ \(F=\frac{\mu_0}{4 \pi} \cdot \frac{2 I_1 I_2}{r}=10^{-7} \times \frac{2 \times 1 \times 3}{0.5}\)

= 1.2 x 10-6 N.m-1

According to the corkscrew rule, the magnetic fields produced between two wires are mutually opposite in direction. Let any point P in this region be at a distance x from the first wire.

∴ The magnetic field at point P due to the first wire

⇒ \(\frac{\mu_0}{4 \pi} \cdot \frac{2 I_1}{x}\)

and magnetic field at the point P due to the second wire

⇒ \(\frac{\mu_0}{4 \pi} \cdot \frac{2 I_2}{(0.5-x)}\)

If the resultant magnetic field at the point P is zero, then

⇒ \(\frac{\mu_0}{4 \pi} \cdot \frac{2 I_1}{x}=\frac{\mu_0}{4 \pi} \cdot \frac{2 I_2}{(0.5-x)} \quad\)

or, \(\frac{0.5-x}{x}=\frac{I_2}{I_1}=\frac{3}{1}\)

or, 3x = 0.5 – x

or, 4x = 0.5

or, x = \(\frac{0.5}{4}\)

= 0.125 m

So, at any point on the plane between the wires, 0.125 m away from the first wire, the resultant magnetic field will be zero

Example 3. Two long parallel conductors, kept at a distance d, carry currents I1 and I2, respectively. The mutual force acting between them. Now, the current one is doubled and its direction is also reversed. If the distance of separation between them is made 3d, what will be the force acting between the two conductors?
Solution:

For the unit length of the conductors,

⇒ \(F=\frac{\mu_0}{4 \pi} \cdot \frac{2 I_1 I_2}{d}\)

If the current in the first conductor is doubled and its direction is reversed, it will be -2I1.

∴ In the second case

⇒ \(F^{\prime}=\frac{\mu_0}{4 \pi} \cdot \frac{2\left(-2 I_1\right) I_2}{3 d}\)

= \(-\frac{2}{3} \frac{\mu_0}{4 \pi} \cdot \frac{2 I_1 I_2}{d}\)

= \(-\frac{2}{3} F\)

Example 4. A long horizontal wire AB, which is free to move in a vertical plane and carries a steady current of 20 A, is in equilibrium at a height of 0.01 m over another parallel long wire CD, which is fixed in a horizontal plane and carries a steady current of 30A. Show that when AB is slightly depressed, it executes simple harmonic motion. Find its period of oscillations.

Electromagnetism Example 4 A long horizontal wire

Solution:

Force per unit length on wire AB due to the magnetic field produced by wire CD

⇒ \(\frac{\mu_0}{4 \pi} \frac{2 I_1 I_2}{r}\)

where Ix = 20 A, I2 = 30 A and distance between AB and CD, r = 0.01m.

So force on the whole of wire AB due to wire CD,

⇒ \(\left.F=\frac{\mu_0}{4 \pi} \cdot \frac{2 I_1 I_2}{r} \times L \text { [where } L=\text { length of wire } A B\right]\)

Since wire AB is in equilibrium, the force F must balance its weight by acting downwards. This is possible only if the two wires repel each other and hence I1 and I2 are flowing in opposite directions.

Accordingly, \(F=\frac{\mu_0}{4 \pi} \cdot \frac{2 I_1 I_2}{r} \times L=M g\)…..(1) [where M=mass of wire AB]

If we consider that the wire is depressed by a small distance d so that the height of AB over CD becomes (r-d), then the force on wire AB increases to,

⇒ \(F_1=\frac{\mu_0 2 I_1 I_2}{4 \pi(r-d)} \times L=\frac{\mu_0 I_1 I_2}{2 \pi r}\left(1-\frac{d}{r}\right)^{-1} L\)

= \(=\frac{\mu_0 I_1 I_2}{2 \pi r}\left(1+\frac{d}{r}\right) L\)

Now on releasing the wire AB, it moves under the effect of restoring force f.

⇒ \(f=F_1-M g=F_1-F=\frac{\mu_0 I_1 I_2}{2 \pi r}\left[1+\frac{d}{r}-1\right] L\)

= \(\frac{\mu_0 I_1 I_2 L}{2 \pi r^2} d\)

Acceleration of the wire AB, \(a=\frac{f}{M}=\frac{\mu_0 I_1 I_2 L}{2 \pi r^2 M} d\)

Since, except d all other physical quantities are constants

Hence the motion of the wire is simple harmonic.

So time period,

⇒ \(T=2 \pi \sqrt{\frac{2 \pi r^2 M}{\mu_0 I_1 I_2 L}}=2 \pi \sqrt{\frac{2 \pi r^2}{\mu_o I_1 I_2 L} \times \frac{\mu_0 I_1 I_2 L}{2 \pi r g}}\) [putting the value of M from equation (1)]

∴ \(T=2 \pi \sqrt{\frac{r}{g}}=2 \pi \sqrt{\frac{0.01}{9.8}}\)

= 0.2s

Magnetic Effect Of Current And Magnetism

Electromagnetism Galvanometer

A galvanometer is an instrument used to detect and measure the current in a circuit.

A current measuring instrument can be constructed using the heating effect, the chemical effect, or the magnetic effect of current. However, to measure a direct current (dc), the magnetic effect is most advantageous for practical purposes.

The two types of galvanometers used widely in laboratories are:

1. Moving magnet galvanometer: The basic principle of this instrument is the action of electric current on a magnet.

Example: Tangent galvanometer, Since galvanometer, Helmholtz double coil galvanometer.

This type of galvanometer is rarely used due to its disadvantageous setup and low sensitivity. The discussion is out of our syllabus.

2. Moving coil galvanometer: The basic principle of this instrument is the action of a magnet on an electric current.

Example: D’Arsonval galvanometer, table galvanometer.

Moving Coil Galvanometer:

With a moving coil galvanometer, we can detect and measure even a very weak current (about 10-9A) in a circuit. This galvanometer.

Electromagnetism Moving Coil Galvanomete

Description of a moving coil:

ABCD: A coil of insulated copper wire wound over a rectangular frame made of cane or aluminum.

I: A cylinder of soft iron. The axis of the cylinder coincides with the axis of the coil and the cylinder is fitted inside the coil.

NS: Two magnetic poles made of soft iron attached with a strong permanent magnet. The gap between the two poles is cylindrical and in this gap, the coil is placed in such a way that the axis of the gap coincides with the axis of the coil.

Due to this special shape of the magnetic poles, the magnetic field always remains parallel to the plane of the coil.

Now this coil is fitted with a mechanical system such that:

  1. The circuit current to be measured can be sent directly through the coil
  2. A very accurate measurement of the deflection of the coil due to the flow of current can be done. For this purpose, two types of mechanical arrangements are widely used

1. Suspended-coil galvanometer or D’Arsonval galvanometer: This type of arrangement.

W: Phosphor-bronze wire. The coil ABCD is hung by this wire.

M: A small mirror attached to the phosphor-bronze thread W. Ift he coil rotates through an angle θ, and the mirror also rotates through θ. So, if a ray of light falls on the mirror from a stationary source of light, the reflected ray rotates through an angle of 20. If the displacement of the reflected light spot on a scale, kept at a distance D from the mirror, is d then,

⇒ \(2 \theta=\frac{d}{D} \quad(\text { when, } d \ll D)\)

or, \(\theta=\frac{d}{2 D}\)….(1)

For this arrangement, it is also called a mirror galvanometer.

b This arrangement is shown. Two magnetic poles are not drawn here.

HH: Two hairsprings. Two ends of the coil are connected with these two hairsprings. The circuit current to be determined enters the coil through the two bearings P and Q attached to HH.

R: A pointer. With the rotation of the coil, it also rotates. When no current flows through the galvanometer, the pointer remains vertical. It can move on either side of this position depending on the direction of the current.

The deflection of the pointer gives an idea of the magnitude of the current. The tire value of the rotation is obtained from a graduated circular scale. Titus it is also called a pointer galvanometer.

Theory: Let the number of turns of the rectangular coil = N, the area of its surface A, the magnetic field parallel to the coil = D, and current through the coil = I. The torque acting on the coil, \(\tau=B N I A\) Due to torque, the coil rotates from its equilibrium position.

On the other hand, the phosphor-bronze thread in a suspended coil galvanometer or the hairsprings in a table galvanometer always tries to return the coil to its equilibrium position by virtue of its elastic property.

The value of this restoring torque \(\tau\)‘ is directly proportional to the tire angle of deflection of the coil. So, if the tire coil comes to rest at an angular displacement of θ

⇒ \(r=r^{\prime} \text { or, } B N I A=c \theta\)

or, \(I=\frac{c}{B N A} \cdot \theta\)….(2)

Here, c = restoring torque per unit twist = constant. From this equation, the tire value of the current can be obtained.

Using equation (1) for a suspended coil galvanometer we get,

⇒ \(I=\frac{c}{2 B N A D} d\)….(3)

Discussions:

1. Relation between current and angle of deflection: From equation (2), it is seen that current (I) is directly proportional to the angle of deflection (θ), i.e., a linear relation exists between them. Thus, the current measuring scale of this kind of galvanometer is always uniform.

2. Setting of the galvanometer: This galvanometer contains no magnetic needle and hence it is not necessary to set this galvanometer magnetic meridian plane. Rather, it can be placed in any position.

3. Galvanometer sensitivity: A moving coil galvanometer is very sensitive and current can be measured very accurately with it. Thus, this type of galvanometer is widely used for current measurement.

Suspended-type galvanometers are highly sensitive. The current through the coil for which the displacement of the light spot on a scale kept at a distance of 1 m from the mirror is 1 .mm, which is called the sensitivity of the galvanometer. Its unit is A mm-1

The sensitivity of this kind of galvanometer is about 10-9 A.mm-1.So, the galvanometer will be more sensitive for a lesser value of sensitivity.

On the other hand, a table galvanometer is very durable in spite of our lower sensitivity. Different table galvanometers are so designed that their pointers give full-scale deflection for a current range of 50μA to 15 mA

4. Condition of sensitivity:

From equation (2) we get, \(\frac{\theta}{I}=\frac{B N A}{c}\)

So, for a given current I, to get greater deflection θ, the value of \(\frac{B N A}{c}\) should be greater.

  1. Using a powerful magnet the value of B can be increased.
  2. A rectangular coil of multiple turns (N) and of sufficiently large surface area (A) has to be used. However, with an increase of N and A, the resistance of the galvanometer also increases and the coil becomes very heavy.
  3. A suspension thread should be chosen for which c is sufficiently small and hence a very thin wire should be used which can bear the weight of the coil. A suspension wire made of phosphor bronze serves this purpose

5. Limitation: The main disadvantage of a moving coil galvanometer is overload. If the current is more than its endurance limit and is passed through this galvanometer, its phosphor bronze wire or the hairsprings may get damaged permanently.

Magnetic Effect Of Current And Magnetism

Electromagnetism Use Of Galvanometer As An Ammeter Or A Voltmeter

Ammeter: The instrument which measures current in an electrical circuit is called an ammeter. An ammeter is always connected in series with a circuit so that the entire current in the circuit can pass through it, Again, its resistance should be very low so that when it is joined in a circuit, the main current in that circuit does not decrease appreciably.

So, an ammeter milliammeter or microammeter) is a low-resistance instrument connected in series with an electrical circuit. The resistance of an Ideal ammeter should be zero.

Voltmeter: The instrument used for measuring the potential difference between two points of an electrical circuit is called a voltmeter. To equalize the potential difference between those two points with the potential difference between the two ends of the voltmeter, it should be connected in parallel with those two points in the circuit.

Again, the resistance of the voltmeter should be very high so that no appreciable fraction of the main current passes through the voltmeter. So, a voltmeter (or millivoltmeter) is an instrument of high resistance connected in parallel with an electrical circuit. The resistance of an ideal voltmeter should be infinite.

Electromagnetism Voltmeter

Disadvantages of using a galvanometer: The basic instrument used for measuring current is a moving magnet galvanometer or a moving coil galvanometer. In spite of this, there are some problems with using these instruments directly in laboratory experiments.

1. Some time is spent on leveling and setting of these galvanometers.

2. The magnitude of the current cannot be obtained directly from any scale. To calculate the magnitude of current we have to use the galvanometer formula after measuring the angle of deflection.

3. Usually the resistance of a galvanometer is of intermediate order (in most of cases, from 100Ω to 500Ω). When this galvanometer is used to measure current or potential difference, the results turn out to be erroneous; because it is not an ideal one to use as an ammeter as its resistance is not too low, and again it is not an ideal one as a voltmeter as its resistance is not too high.

Transformation of Galvanometers:

To make a galvanometer fit for everyday use in the laboratory, it should be effectively converted into an ammeter or a voltmeter. Thus, a galvanometer is the primary instrument while an ammeter or a voltmeter is the secondary instrument.

To construct an ammeter or a voltmeter, a moving coil table galvanometer is generally used. For this

The resistance of the galvanometer, G;

The current IG, for the full-scale deflection of the pointer in the circular scale, should be known beforehand.

Galvanometer to ammeter: Let the resistance of the galvanometer = G and the current required for the full-scale deflection of the pointer = IG

Electromagnetism galvanometer to ammeter

Now, a low resistance in parallel, i.e., a shunt S is connected with the galvanometer. The whole system is kept in a box covered with glass in such a way that

The pointer and the circular scale remain visible from the outside,

The terminals A and B are outside the box. The points A and B are connected with the external circuit. If the main circuit current corresponds to a full-scale deflection of the galvanometer, the galvanometer is effectively converted into an ammeter suitable for measuring maximum current I. Note that, due to the parallel combination of G and S, the equivalent resistance of the ammeter thus formed becomes sufficiently small.

Galvanometer to ammeter Calculation:

VX – VY = IG.G = IS.S [here, Is stands for shunt current]

So, \(S=\frac{I_G}{I_S} \cdot G\)

Again, \(I=I_G+I_S \quad \text { or, } \quad I_S=I-I_G\)

Hence, \(S=\frac{I_G}{I-I_G} \cdot G\)….(1)

So, to convert the galvanometer into an ammeter to measure a maximum current I, a shunt S as obtained from equation (1), should be connected in parallel with the galvanometer.

Range of the ammeter: In equation (1), I > IG; hence when an ammeter is constructed from a galvanometer, its range always increases and never decreases. If this range becomes n times, \(\frac{I}{I_G}=n\); so from equation (1) we get,

⇒ \(S=\frac{1}{I / I_G-1} \cdot G=\frac{G}{n-1}\) ….(2)

Resistance of the ammeter: The equivalent resistance of G and S is the resistance (R) of the ammeter.

So, \(\frac{1}{R}=\frac{1}{G}+\frac{1}{S}=\frac{1}{G}+\frac{n-1}{G}=\frac{n}{G} \text { i.e., } R=\frac{G}{n}\)…(3)

Galvanometer to voltmeters: In this case, a high resistance R is connected in series with the galvanometer. The two points A and B are connected with the external circuit.

So, if the current in the external circuit is IG, the galvanometer current will also be and as a result, the pointer gives full-scale deflection. Hence, In this condition, the potential difference (VA – VB) between the two points A and B will show full-scale deflection of the pointer.

So, the galvanometer will then be effectively converted into a voltmeter. Due to the series combination of G and R, the equivalent resistance of the voltmeter becomes sufficiently large.

Electromagnetism Resistance of the ammeter

Galvanometer to voltmeters Calculation:

VA – VB = V = IG (G+R)

or, \(G+R=\frac{V}{I_G} \quad .. R=\frac{V}{I_G}-G\)….(4)

So, to convert the tire galvanometer into a voltmeter fit for measuring a maximum potential difference V, high resistance of magnitude R, as obtained from equation (4), should be connected in series.

Range of voltmeter: In equation (4), V > IGG (= VG), and hence if a galvanometer is converted into a voltmeter, the range of the voltage always increases and never decreases.If this range becomes n -times, then \(\frac{V}{V_G}=n\); So from equation (4) we get,

⇒ \(R=G\left(\frac{V}{I_G \cdot G}-1\right)=G\left(\frac{V}{V_G}-1\right)=G(n-1)\)…(5)

Resistance of the voltmeter: Naturally, the resistance of the newly formed voltmeter,

RV = G + R = G + G(n-l) = nG….(6)

Differences between an ammeter and a voltmeter:

Electromagnetism differences between an ammeter and a voltmeter

Theoretically moving magnet or suspended-coll galvanometers may be converted to an ammeter or voltmeter in a similar way. However due to the practical difficulties associated with such arrangements only moving coil table galvanometers are converted to be used as an ammeter or voltmeter.

Magnetic Effect Of Current And Magnetism

Electromagnetism Numerical Examples

Example 1. A galvanometer of resistance 10Ω gives lull scale deflection for a current of 10 mA. How can this galvanometer be used?

  1. As an ammeter to measure current of range 0-2 A
  2. As a voltmeter having a voltage range of 0-5 V?

Solution:

Resistance of the galvanometer, G = 10-11; maximum current, IG = 10 mA = 0.01 A

1. Connecting a shunt S in parallel to the galvanometer, if the instrument is used between points A and B, an ammeter of current range 0-1 will be obtained.

In the given question, I = 2 A.

Electromagnetism Example 1 A galvanometer of resistance

Now, IS.S= IG.-G

or, \(S=\frac{I_G}{I_S} G=\frac{I_G}{I-I_G} \cdot G=\frac{0.01}{2-0.01} \times 10\)

= 0.0503Ω

This is the required value of the shunt resistance.

2. A voltmeter having a voltage range of zero to (VA – VG) is obtained if a resistance R is connected in series with the galvanometer and the instrument is used between points A and B.

Electromagnetism Example 1 A galvanometer of resistance.

In the given problem, VA – VB = 5V

Now, VA – VB = IG (G + R)

or, \(R=\frac{V_A-V_B}{I_G}-G=\frac{5}{0.01}-10\)

= 500-10

= 490Ω

Example 2. Full-scale deflection occurs, in a moving coil galvanometer of resistance 36Ω when 100 mA current flows through it. What arrangement should be made to convert it into a voltmeter in the 0-5V range? Draw the necessary circuit diagram.

Solution:

Resistance of the galvanometer = 36Ω, maximum current, IG = 100 mA = 0.1 A.

Connecting a resistance R in series with the galvanometer and using the equipment in between the two points A and B, we shall get a voltmeter to measure voltage difference ranging from 0 to (VA– VB).

Here, VA – VB = 5V

Now, VA – VB = IG(G+R)

or, \(R=\frac{V_A-V_B}{I_G}-G=\frac{5}{0.1}-36\)

= 50-36

= 14Ω

This is the required value of the resistance R.

Example 3. A millivoltmeter of range ‘0-50 mV’ and resistance 50Ω is to be converted into an ammeter of range 0-1 A. How can it be done?
Solution:

Resistance of the millivoltmeter, R = 50 Ω; maximum voltage, V = 50 mV = 0.05 V.

Connecting a shunt S in parallel with the millivoltmeter and using this combination in between points A and B, we shall get an ammeter to measure a maximum current I.

Here, I = 1 A.

Now, IS. S = IV.R

or, \(S=\frac{I_V}{I_S} \cdot R=\frac{I_V}{I-I_V} \cdot R\)

⇒ \(\frac{\frac{V}{R}}{I-\frac{V}{R}} \cdot R=\frac{V}{I-\frac{V}{R}}=\frac{0.05}{1-\frac{0.05}{50}}=\frac{0.05}{1-0.001}=0.05 \Omega\)

This is the required value of the shunt resistance.

Electromagnetism Example 3 A millivoltmeter of range

Example 4. How would you convert a voltmeter that can measure up to 150 V to an ammeter that can measure current up to 8R The Resistance of the voltmeter is 300Ω.
Solution:

Resistance of the voltmeter, R = 300Ω; maximum voltage, V = 150 V.

∴ Maximum current through the voltmeter,

⇒ \(I_V=\frac{V}{R}=\frac{150}{300}=0.5\)

Connecting a shunt S in parallel with the voltmeter and using this combination in between the points A and B, we shall get an ammeter to measure a maximum current I.

Here, I = 8 A

Now, IS.S = IV.R

or, \(S=\frac{I_V}{I_S} \cdot R=\frac{I_V}{I-I_V} \cdot R=\frac{0.5}{8-0.5} \times 300=\frac{0.5}{7.5} \times 300\)

= 20Ω

This is the required value of the shunt resistance.

Example 5. A galvanometer of resistance 100Ω gives full-scale deflection for a current of 10 mA. What is the value of, the shunt to be used to convert it into an ammeter which can measure current up to 10 A?
Solution:

Resistance of the galvanometer, G= 100Ω; maximum current, IG = 10 mA = 0.01 A. Connecting a shunt S in parallel with the voltmeter and using this combination in between the points A and B, we shall get an ammeter useful to measure a maximum current I.

Here, I = 10A and IS.S = IG.G

or, \(S=\frac{I_G}{I_S} \cdot G=\frac{I_G}{I-I_G} \cdot G=\frac{0.01}{10-0.01} \times 100=\frac{1}{9.99}\)

= 0.1001Ω

Example 6. A moving coil galvanometer of resistance 50Ω gives full-scale deflection for a current of 50 mA. How can this galvanometer used to convert it into a voltmeter that can measure voltage up to 200 V?
Solution:

Resistance of the galvanometer, G = 50Ω; maximum current, IG= 50 mA = 0.05 A. Connecting a resistance R in series with the galvanometer and using the equipment in between the two points A and B, we shall get a voltmeter to measure the voltage difference ranging from 0 to ( VA– VB)

Here, VA – VB = 200V

Now VA – VB = IG(G+R)

or, \(R=\frac{V_A-V_B}{I_G}-G=\frac{5}{0.1}-36=14 \Omega\)

This is the required value of resistance R.

Magnetic Effect Of Current And Magnetism

Electromagnetism Conclusion

  • Two like poles repel each other and two unlike poles attract each other.
  • Repulsion is the conclusive test of magnetization.
  • The region surrounding a magnet in which the influence of the magnet is felt is called the magnetic field of the magnet.
  • A magnetic line of force in a magnetic field is defined as a continuous line such that the litigant drawn at any point on it gives the direction of the magnetic field at that point.
  • The direction of deflection of a magnetic needle due to the flow of electric current can be determined by
    • Ampere’s swimming rule
    • Right-hand thumb rule.
  • The right-hand thumb rule is also called Oersted’s rule. For determination of the direction of the magnetic field at a point near a current-carrying conductor)
    • Maxwell’s corkscrew rule
    • The right-hand grip rule is used. The ratio of the magnetic field (B) acting at a point by a substance to the magnetic permeability (μ) of that substance is called the magnetic intensity or magnetizing field (H) at that point.

Units of B and H:

Electromagnetism units of B and H

Relation between SI and CGS units:

  • 1A.m-1 = 4π x 10-3 Oe , 1Wb.m-2 = 104 G
  • The current which, when flowing through a conducting wire of length 1 cm bent in the form of an arc of a circle of radius 1 cm, produces a magnetic field of 1 Oe at the center of the arc, is called an electromagnetic unit of current.
  • 1 emu of current =10 A.
  • Ampere’s circuital law: The line integral of the magnetic field vector along a closed path in a magnetic field is equal to the product of the total current enclosed by the closed path and the permeability of the vacuum.
  • \(\text { i.e., } \oint \vec{B} \cdot \overrightarrow{d l}=\mu_0 I\)
  • Here, I is the total current enclosed by the closed path.
  • If a long, insulated conducting wire is wrapped over the surface of a cylinder in such a way that each circular turn is perpendicular to the axis of the cylinder, the coil thus formed is called a solenoid.
  • If a long, insulated conducting wire is wound on a donut-shaped core of uniform cross-section having a circular axis such that each turn is perpendicular to its axis, the coil thus formed is called a toroid. A toroid is also called an endless solenoid.
  • If a positively charged particle of magnitude 1 C moves with velocity 1m s-1 in a magnetic field and experiences a maximum force of 1 N, then the magnitude of the magnetic field is called 1 T or 1 Wb.m-2.
  • The magnetic force experienced by a moving charged particle in a magnetic field is a no-work force.
  • The speed and kinetic energy of a charged particle does not change when a magnetic force acts on it.
  • If the magnitude and direction of a magnetic field throughout a certain region remain constant, the field is called a uniform magnetic field otherwise, it is a non-uniform magnetic field.
  • A charged particle follows a circular path inside a uniform magnetic field when it enters the field perpendicularly. A charged particle follows a helical path inside a uniform magnetic field when it enters the tire field obliquely.

Fleming’s left-hand rule: If the tire forefinger of the middle finger and the thumb of the left hand are stretched mutually perpendicular to each other, such that the forefinger points the direction of the magnetic field and the tire middle finger points the direction of the current, the thumb points the direction of the force acting on tire conductor.

  • Two like parallel currents attract each other and two unlike parallel currents repel each other.
  • Two long, linear, and parallel conducting wires of negligible cross sections are kept 1 nr apart from each other. The steady direct current flowing in each wire for which both of them experience a force of 2 x 10-7 N.m-1 is called 1 A current.
  • Galvanometers used in the laboratory are:
    • Moving magnet galvanometer-examples: Tangent galvanometer, sine galvanometer.
    • Moving coil galvanometer-examples: D’Arsonval galvanometer, table galvanometer.
  • According to Biot-Savart law or Laplace’s law, die magnitude of magnetic field ⇒ \(\delta \vec{B}\) at a point due to a small element δ1 of a current carrying wire is,
  • ⇒ \(\delta B \propto \frac{I \delta l \sin \theta}{r^2} \text { or, } \delta B=k \frac{I \delta l \sin \theta}{r^2}\)
  • where r = distance of the point from the element and
  • θ = angle between the current element \(\delta \vec{B}\) and position vector \(\vec{r}\).
  • In SI, the conventional form of Biot-Savart law is,
  • ⇒ \(\delta B=\frac{\mu_0}{4 \pi} \cdot \frac{I \delta l \sin \theta}{r^2}\)
  • [μ0 = magnetic permeability of vacuum = 4π x 10-7 H.m-1]
  • Generally, \(B=\mu H \quad \text { or, } \quad H=\frac{1}{\mu} B\)
  • The magnetic field at any point near a straight conductor,
  • ⇒ \(B=\frac{\mu_0}{4 \pi} \frac{I}{r}\left(\sin \theta_1+\sin \theta_2\right)\)
  • Magnetic field due to an Infinite straight conductor,
  • ⇒ \(B=\frac{\mu_0}{4 \pi} \cdot \frac{2 l}{r}\)
  • The magnetic field at the center of a circular conductor of N turns,
  • ⇒ \(B=\frac{\mu_0 I N}{2 r}[r=\text { radius of the circle }]\)
  • The magnetic field at a point on the axis of a circular conductor of N times,
  • ⇒ \(B=\frac{\mu_0 N I}{2} \cdot \frac{r^2}{\left(r^2+x^2\right)^{3 / 2}}\)
  • [where, x = distance of the point from the center of the coil]
  • Magnetic field due to a long, straight solenoid at any point on its axis,
  • ⇒ \(B=\mu_0 n I\)
  • [n = number of turns per unit length of the solenoid = \(\frac{N}{L}\)
  • Magnetic field due to a toroid,
  • ⇒ \(B=\mu_0 n I\)
  • [n = number of turns per unit length of the toroid = \(\frac{N}{2 \pi r}\)]
  • If N number of conductors carrying current I are enclosed by a closed loop then according to circuital law,
  • ⇒ \(\oint \vec{B} \cdot d \vec{l}=\mu_0 N I\)
  • Force on a charged particle moving with a velocity (\(\vec{v}\)) in a uniform magnetic field (\(\vec{B}\)) is
  • ⇒ \(\vec{F}=q \vec{v} \times \vec{B}\)
  • The radius of the circular path described by a moving charged particle entering a uniform magnetic field perpendicularly,
  • ⇒ \(r=\frac{m v}{q B}\)
  • [m = mass of the charged particle, v = velocity of the particle, q = charge of the particle and B = magnetic field]
  • Time period of revolution, \(T=2 \pi \frac{m}{q B}\)
  • Number of complete revolutions in the circular path, i.e., frequency of circular motion,
  • ⇒ \(n=\frac{1}{T}=\frac{1}{2 \pi}\left(\frac{q}{m}\right) B\)
  • This is known as cyclotron frequency.
  • The kinetic energy of the charged particle ejected from the outlet of the cyclotron device = \(\frac{q^2 B^2 R^2}{2 m}\) [ R = radius of the dee of cyclotron]
  • A charged particle, when it enters a magnetic field obliquely, follows a helical or spiral path. The pitch of this spiral or helical path
  • = time period x linear velocity
  • = circumference of the circular path x cotθ
  • Force experienced by a particle of charge + q in a uniform electric field
  • ⇒ \(\vec{F}=q \vec{E}\)
  • For a charge q moving in an electromagnetic field with velocity \(\vec{v}\), the forces acting are,
    • Electric force, \(\vec{F}_e=q \vec{E} \quad[\vec{E}=\text { electric field }]\)
    • Magnetic force, \(\vec{F}_m=q \vec{v} \times \vec{B}[\vec{B}=\text { magnetic field }]\)
  • Resultant Lorentz force, \(\vec{F}=q(\vec{E}+\vec{v} \times \vec{B})\)
  • In the case of a current-carrying conductor in a magnetic field, the resultant magnetic force acting on the whole circuit or a finite part of the circuit,
  • ⇒ \(F=\int d F=\int B I d l \sin \theta\)
  • The torque acting on a rectangular conductor having N turns placed in a uniform magnetic field,
  • ⇒ \(\tau=B N I A\) [I = current-strength, A = area of cross-section of the rectangular conductor]

Vectorial form:

  • ⇒ \(\vec{\tau}=N I \vec{A} \times \vec{B}\)
  • If the perpendicular distance between two long parallel conductors kept in vacuum or in air be r, the force of attraction or repulsion per unit length of the wire is,
  • ⇒ \(F=\frac{\mu_0}{4 \pi} \cdot \frac{2 I_1 I_2}{r}\) [I1 , I2 are the currents in the two wires]
  • In the case of a moving coil galvanometer, if the number of turns in the tire coil is N, the area of the plane of coil A, the magnetic field parallel to coil B, current the of the coil I, and the coil comes to rest at an angle θ, then
  • ⇒ \(I=\frac{c}{B N A} \theta\)
  • Here, c = restoring torque per unit deflection = constant
  • Required shunt to be connected with a galvanometer in parallel to convert it to an ammeter,
  • ⇒ \(S=\frac{I_G}{I-I_G} \cdot G\)
  • [where S = shunt resistance, G = galvanometer resistance, IG = maximum galvanometer current, I = main current in circuit]
  • If the range of an ammeter is to be increased n -times,
  • ⇒ \(S=\frac{G}{n-1}\)
  • To convert a galvanometer into a voltmeter for measuring a maximum voltage of V, the resistance to be connected in series with the galvanometer,
  • ⇒ \(R=\frac{V}{I_G}-G\)
  • If the range of the voltmeter is to be increased n -times, then
  • R = G(n-l)

Units of some magnetic quantities:

Electromagnetism units of some magnetic quantities

  • When two current-carrying loops make an angle θ with each other, then net magnetic field at their center,
  • ⇒ \(B=\sqrt{B_1^2+B_2^2+2 B_1 B_2 \cos \theta}\)

Electromagnetism net magnetic field

  • The value of the magnetic field at at point on the center of separation of two parallel conductors carrying equal currents in the same direction is zero.
  • If a current-carrying circular loop is turned into a coil having n identical turns then the magnetic field at the center of the coil becomes n² times the previous electric field i.e., Bn = n²B1 where Bn = magnetic field at the center of the coil having n turns; B1 = magnetic field at the center of the circular loop.

Magnetic Effect Of Current And Magnetism

Electromagnetism Very Short Questions and Answers

Question 1. What is the nature of lines of force in a uniform magnetic field?
Answer: Parallel and equispaced

Question 2. In a uniform magnetic field, lines of force are equispaced – straight lines.
Answer: Parallel

Question 3. How is the direction of a magnetic field $ at a point related to the magnetic line of force passing through that point?
Answer:

⇒ \(\vec{B}\) is tangential to the line of force at that point

Question 4. A magnetic needle is kept below a very long conducting wire. If the current is sent through the wire from north to south, in which direction will the north pole of the needle be deflected?
Answer: Toward the east

Question 5. A magnetic needle is placed below a very long conducting wire. If the current is sent through the wire from east to west, in which direction will the north pole of that needle be deflected?
Answer: No deflection occurs

Question 6. How does the magnetic field at a point near a long straight current-carrying conductor vary with the current and the distance of the point?
Answer: Proportional, inversely proportional

Question 7. When 1 A current flows through a circular conductor, the magnetic: fields generated at its centre are 10-7 T. For what value of current, will the magnetic field be 10-6 T?
Answer: 10 A

Question 8. Which physical quantity has the unit Wb.m-2? Is it a scar lar or a vector quantity?
Answer: Magnetic field or magnetic induction \(\vec{B}\) it is a vector quantity

Question 9. What is the unit of magnetic permeability μ0 of vacuum?
Answer: H.m-1

Question 10. What is the magnetic field produced at a distance of 1 m from a long, straight conductor carrying 1 A current?
Answer: 2 x 10-7 T

Question 11. The magnetic field at a distance of 1 m from a long, straight conductor is 10-7 T. What is the current through the conductor?
Answer: 0.5 A

Question 12. Consider the circuit where APB and AQB are semi-circles. What is the magnetic field at the center C of the circular loop?

Electromagnetism magnetic field at the at the center C of the circler loop

Answer: Zero

Question 13. A solenoid carrying 1 A current has a length of 1 m and contains 10000 turns. What is the magnetic field on the axis of the solenoid?
Answer: 4π x 10-3T

Question 14. What will be the nature of the path of a charged particle when it enters a uniform magnetic field 5 normally?
Answer: Circular

Question 18. An electron and a proton enter a uniform magnetic field perpendicularly with the same speed. How many times larger will be the radius of the proton’s path than that of the electron’s path? Given: the proton is 1840 times heavier than the electron
Answer: 1840

Question 19. Can a stationary charge produce a magnetic field?
Answer: No

Question 20. What is the magnitude of force experienced by a stationary charge placed in a uniform magnetic field?
Answer:

⇒ \(\vec{F}_m=q \vec{v} \times \vec{B}=0(… \vec{v}=0)\)

Question 21. Does any force act on a magnetic north pole ifit is brought near a negatively charged conductor at rest?
Answer: No

Question 22. Does any force act on a charge moving in a magnetic field?
Answer:

⇒ \(\text { yes, } \vec{F}=q \vec{v} \times \vec{B}\)

Question 23. A long straight wire is carrying a current. An electron starts its motion on a line parallel to the wire in a direction same as that of the current. What will be the direction of the force on the electron?
Answer: Away from the wire in a normal direction

Question 24. A charge q moves with velocity \(\vec{v}\) at an angle 6 to a magnetic field P. What is the force experienced by the particle?
Answer: qvBsinθ

Question 25. An electron moving with a velocity of 107m.s-1 enters a uniform magnetic field of 1 T along a direction parallel to the field. What would be its trajectory?
Answer: Undeviated straight line

Question 26. A certain proton moving through a magnetic field region experiences maximum force. When does this occur?
Answer: The velocity of the proton is perpendicular to the magnetic field

Question 27. An electron beam projected along the positive x-axis experiences a force, due to a magnetic field, along the positive y-axis. What is the direction of the magnetic field?
Answer: Positive z-axis

Question 30. What is the mutual action between two, unlike parallel currents?
Answer: Repulsion

Question 31. Two long and straight parallel wires carry a current 1 A each. If the distance between the two wires is 1 m, what will be the force acting per unit length on them?
Answer: 2 x 10-7 N.m-1

Question 32. What type of galvanometer is used to prepare an ammeter or a voltmeter in the laboratory?
Answer: Moving coil table galvanometer

Question 33. In the case of a moving coil galvanometer, what is the relation between the current I and the angle of deflection θ?
Answer: Proportional

Question 34. How is a galvanometer converted into an ammeter?
Answer: By connecting a rightly chosen low-resistance shunt in parallel to it

Question 35. How should a resistance be connected with a galvanometer to convert it into a voltmeter?
Answer: In series

Question 36. What is the nature of the magnetic field in a moving coil galvanometer?
Answer: Radial

Magnetic Effect Of Current And Magnetism

Electromagnetism Assertion Reason Type

Direction: Those questions have Statement 1 and Statement 2, Of the four choices given below, choose the one that best describes the two statements.

  1. Statement 1 is true, Statement 2 is true; Statement 2 is a correct explanation for statement I.
  2. Statement 1 is true, Statement 2 Is true; Statement 2 is not a correct explanation for Statement 1.
  3. Statement 1 is true, Statement 2 is false.
  4. Statement 1 is false, and Statement 2 Is true.

Question 1.

Statement 1: A magnetic needle that can rotate in a horizontal plane undergoes a deflection when current is passed through a conducting wire, placed above and parallel to it.

Statement 2: A magnetic field is developed around a current-carrying conductor.

Answer: 1. Statement 1 is true, Statement 2 is true; Statement 2 is a correct explanation for statement I.

Question 2.

Statement 1: A long straight conductor attracts iron filings when a current is passed through it.

Statement 2: A magnetic field is developed around a current-carrying conductor.

Answer: 4. Statement 1 is false, Statement 2 Is true.

Question 3.

Statement 1: I = 1 A and OA = 1 cm. The magnetic field developed at A is 10-5 Wb.m-2.

Electromagnetism the magnetic field developed

Statement 2: If I am the current flowing through a long straight conductor, then the magnetic field developed at a distance r from the conductor is \(B=\frac{\mu_0 I}{2 \pi r}\)

Answer: 2. Statement 1 is true, Statement 2 Is true; Statement 2 is not a correct explanation for Statement 1.

Question 4.

Statement 1: Neither the magnetic field vector \(\vec{B}\) nor the
magnetic intensity vector \(\vec{H}\) of a magnetic field depend on
the nature of the medium.

Statement 2: If the magnetic permeability of a medium is \(\mu \text { then } \vec{H}=\frac{1}{\mu} \vec{B}\)

Answer: 4. Statement 1 is false, Statement 2 Is true.

Question 5.

Statement 1: A galvanometer of resistance G is converted to an ammeter by increasing its range n times. The resistance of the ammeter is \(\frac{G}{n}\).

Statement 2: A shunt of resistance has \(\frac{G}{n-1}\) to be connected in parallel with a galvanometer in order to increase its range n times.

Answer: 1. Statement 1 is true, Statement 2 is true; Statement 2 is a correct explanation for statement I.

Question 6.

Statement 1: The velocity of injection of a charged particle, being accelerated in a cyclotron remains constant irrespective of the applied magnetic field.

Statement 2: In a cyclotron the charged particle is accelerated only due to the applied electric field because magnetic force is a no-work force.

Answer: 4. Statement 1 is false, Statement 2 Is true.

Question 7.

Statement 1: If an electron and a proton are projected with equal momentum in a uniform transverse magnetic field, then the curvature of their padis is equal.

Statement 2: The mass of a proton is much higher than that of an electron.

Answer: 2. Statement 1 is true, Statement 2 Is true; Statement 2 is not a correct explanation for Statement 1.

Question 8.

Statement 1: A charged particle moves perpendicular to a magnetic field. Its kinetic energy remains constant, but momentum changes.

Statement 2: A magnetic force acts on the charged particle.

Answer: 2. Statement 1 is true, Statement 2 Is true; Statement 2 is not a correct explanation for Statement 1.

Magnetic Effect Of Current And Magnetism

Electromagnetism Match The Column

Question 1. Match the electromagnetic quantities in column A with the rules or formulae in column B.

Electromagnetism Match the Columns 1

Answer: 1-B, 2-C, 3-D, 4-A

Question 2. The BC of the conductor is semicircular. Match the values of the magnetic field developed at O given in column 2 with the corresponding segments given in column 1.

Electromagnetism Match the Columns 2

Electromagnetism Match the Columns 2.

Answer: 1-A, 2-D, 3-B, 4-C

Question 3. A galvanometer of resistance 100Ω shows full-scale deflection for a current of 10 mA. Match the resistances in column 2 necessary to convert the galvanometer to an ammeter or voltmeter as given in column 1.

Electromagnetism Match the Columns 3

Answer: 1-D, 2-A, 3-C, 4-B

Question 4. Current in each of the two conductors parallel to each other and at a distance r is I (column 1). The magnetic fields at a point on the same plane and equidistant from the two conductors are given in column 2 (μ0 = 4π x 10-7 H.m-1)

Electromagnetism Match the Columns 4

Answer: 1-A, 2-C, 3-D, 4-B

Question 5. A current-carrying wire. An electron at a point P moves with a velocity \(\vec{v}\). The direction of \(\vec{v}\) is given in column A and its deflection in column B. Match them.

Electromagnetism Match the Columns 5

Electromagnetism Match the Columns 5.

Answer: 1-B, 2-C, 3-D,

WBCHSE Class 12 Physics Electric Energy And Power Notes

Electric Energy And Power Introduction

In the preceding chapters, different types of electrical circuits, their constructions and uses have been discussed. The subject of discussion of this chapter is the measurement of electrical energy consumed in different electrical circuits and the power of different effective elements of a circuit. The practical idea about electrical power and energy is essential for systematic planning and control of the following subjects

  1. How much electrical energy is to be generated in a generating station,
  2. Howdie energy is to be distributed in different areas,
  3. How the loss of energy is to be minimized,
  4. What types of electrical appliances are to be used in the residence office and factory and how they should be used so that, necessary work will be obtained at minimum cost?

Electric Energy And Power Electrical Work

From the discussion in statical electricity, we know that, if Q amount of charge flows through a section under the potential difference V, then the amount of electrical work done is given by,

W = Q x V….(1)

In this equation, if Q is measured in coulomb'(C) and V in volt (V), then the unit of IV is the joule (I), i.e., the relation of the unit is

I = C x V

Read and Learn More Class 12 Physics Notes

Definition: The amount of work done to carry a 1-coulomb charge through a potential difference of 1 volt is called 1 joule.

Besides this, we know that, if current I flow through a conductor for time t, then the amount of charge flowing through it is given by,

Q = It…..(2)

So, combining equations (1) and (2) wo haw.

The electrical work done in the conductor,

W = VIt…..(3)

This work is the amount of electrical energy consumed by the conductor.

WBCHSE Class 12 Physics Electric Energy And Power And Notes

Active and Passive Electrical Devices:

The electrical work done by an electrical source in a circuit, l.e., the electrical energy sent in a circuit is used up generally in two ways:

Electrical energy is converted only to treat energy in the connecting wires and in some devices of the circuit.

In most of the cases, this heat energy has no use, i.e., this energy is dissipated. These devices are called passive devices of the circuit and their resistances are called passive resistances.

But in our daily lives, heat energy is utilized in many ways; e.g., electric heaters, electric irons, etc. are very useful appliances.

Yet, the resistance of diesel appliances will also be called passive resistance in conformity with the present discussion.

Electrical energy may be converted to other forms of energy.

For Example, from electrical energy, mechanical energy is obtained in an electric fan and chemical energy is obtained during the charging of a storage cell.

These types of devices are called active devices. However, in these appliances, some amount of heat is evolved. So, it may be said that the active devices possess some passive resistance also.

Electric Energy And Power Active and Passive Electrical Devices

Let V be the potential difference between the points ll and C in the circuit. If current 1 flows through the circuit for time t, then electrical work,

IV = Vlt

In section AB of the circuit, there is an active electrical device (e.g., an electric fan) and in tire section DC there is a passive device whose resistance is R . This R also Includes (he resistance that generates heat in the active device.

Now, VA – VB = V0

and, VB – VC = V’

VAC = VA – VC

= (VA– VB) + (VB– VC)

= V0 + V’

= V (say)

∴ V = V0 + V’

or, Vlt = V0lt+ V’lt

Again, in the section BC, according to Ohm’s law,

V’ = IR

or, V’t = IR- It

= I²Rt

So, Vlt= V0It+I²Rt….(4)

This equation (4) can be expressed in the following form:

Electrical work in the circuit = transformed active energy + exothermic energy

If energy and heat are expressed in the same unit (e.g., in SI, both have the same unit joule or), the amount of exothermic energy and heat produced (H) will be equal

i.e., in that case, H = I²Rt

But in many cases, calorie (CGS) is used as the unit of heat. In that case, Joule’s law regarding the relation between heat and work, i.e., W = JH is to be used. In that case,

J = mechanical equivalent of heat or Joule’s equivalent
of heat

= 4.2 J.cal-1

= 4.2 x 107 erg.cal-1

In this condition, if exothermic work or energy is W’, then heat produced,

⇒ \(H=\frac{W^{\prime}}{J}\)

i.e., \(H=\frac{I^2 R t}{J}\)….(5)

If work or energy is expressed in joule (J) and heat in calorie (cal), then J = 4.2 J.cal-1

i.e., \(\frac{1}{J}=\frac{1}{4.2}\)

So, the practical form of the equation (5) is

⇒ \(H=\frac{t^2 R t}{4.2}=0.24 I^2 \mathrm{Rt}\)….(6)

the expense of electrical work Is known as the soulful effect or Joule heating.

Three laws regarding Joule effect are offlined easily from equation (5) or (6):

⇒ \(H \propto I^2\); when R and t are constants;

⇒ \(H \propto R\); when I and t are constants;

⇒ \(H \propto t\); when I and R are constants.

Mechanical equivalent of heat:

Prom equation (5), we have,

⇒ \(J=\frac{I^2 R t}{H}\)

Now, if I = 1 , R = 1 and t = 1

we have, J = \(\frac{1}{H}\)

From the tills relation, the mechanical equivalent of heat Is defined in current electricity.

Definition: The reciprocal of the heat produced in one second in a conductor of unit resistance, for the passage of unit current through it, is called the mechanical equivalent of heat.

In electricity, J = 4.2 J.cal-1 means:

The amount of heat produced in 1Ω resistance, for the passage of 1 A current for 1 s through It = \(\frac{1}{4.2}\) cal = 0.24 cal.

Electric Energy And Power Numerical Examples

Example 1. 2A current was sent through a coil of resistance 100Ω for 30 min. Determine the amount of heat produced, the quantity of charge passed, and the amount of work done.
Solution:

Here, I = 2A, R = 100Ω t = 30min = 30 x 60s

∴ The amount of heat produced,

⇒ \(H=\frac{I^2 R t}{J}=\frac{(2)^2 \times 100 \times 30 \times 60}{4.2}\)

= 1.71 x 105 cal

The quantity of charge flowing,

Q = It

=2 x 30 x 60

= 3600C

The amount of work done,

W = QV = QIR

= 3600 x 2 x 100

= 7.2 x 10s J

Example 2. Two separate circuits are made with resistances r1 and r2 connected to the same storage battery. What should be the internal resistance (r) of the storage battery for which an equal amount of heat is produced in the external circuits?
Solution:

For the first connection, current Ix = \(I_1=\frac{E}{r+r_1}\) [E = emf of the battery]

So, the heat produced in resistance rx in time t,

⇒ \(H_1=\frac{I_1^2 r_1 t}{J}=\frac{E^2 r_1 t}{\left(r+r_1\right)^2 J}\)

Similarly, the heat produced at the same time in the resistance r2,

⇒ \(H_2=\frac{E^2 r_2 t}{\left(r+r_2\right)^2 J}\)

According to the question, H1 = H2

∴ \(\frac{E^2 r_1 t}{\left(r+r_1\right)^2 J}=\frac{E^2 r_2 t}{\left(r+r_2\right)^2 J}\)

or, \(r_1\left(r+r_2\right)^2=r_2\left(r+r_1\right)^2\)

or, \(r_1\left(r^2+2 r r_2+r_2^2\right)=r_2\left(r^2+2 r r_1+r_1^2\right)\)

or, \(r_1 r^2+r_1 r_2^2=r_2 r^2+r_2 r_1^2\)

or, \(r^2\left(r_1-r_2\right)=r_1 r_2\left(r_1-r_2\right)\)

or, \(r^2=r_1 r_2\)

or, \(r=\sqrt{r_1 r_2}\)

Example 3. A heating coll of resistance 5Ω is connected to a cell. The internal resistance of the cell is 20Ω. Calculate the value of the shunt to be introduced, so that, the energy consumed in the heating coil will be \(\frac{1}{9}\)th of the previous value.
Solution:

Let E be the emf of the cell and S be the shunt.

Here, the internal resistance of the cell, r = 20Ω, and external resistance, R = 5Ω.

Class 12 Physics Unit 2 Current Electricity Chapter 3 Electric Energy And Power Example 3 A heating coll of resistance

In the absence of the shunt, current flowing in the circuit,

⇒ \(I_1=\frac{E}{R+r}\)

In time t, energy consumed in the resistance R,

⇒ \(W_1=I_1^2 R t=\frac{E^2 R t}{(R+r)^2}\)

Now, if shunt S is connected, current flowing in the circuit,

⇒ \(I_2=\frac{E}{\frac{R S}{R+S}+r}=\frac{E(R+S)}{R S+r(R+S)}\)

∴ Current flowing in the resistance R,

⇒ \(I_R=I_2 \cdot \frac{S}{R+S}=\frac{E S}{R S+r(R+S)}\)

∴ In time t, energy consumed in the resistance R

⇒ \(W_2=I_R^2 R t=\frac{E^2 S^2 R t}{[R S+r(R+S)]^2}\)

According to the question,

⇒ \(W_2=\frac{W_1}{9} \quad \text { or, } W_1=9 W_2\)

∴ \(\frac{E^2 R t}{(R+r)^2}=9 \cdot \frac{E^2 S^2 R t}{[R S+r(R+S)]^2}\)

or, \(\frac{1}{(R+r)^2}=\frac{9 S^2}{[R S+r(R+S)]^2}\)

or, \(\frac{1}{R+r}=\frac{3 S}{R S+r(R+S)} \text { or, } R+r=\frac{R S+r R+r S}{3 S}\)

or, \(S=\frac{r R}{2(R+r)}=\frac{20 \times 5}{2(20+5)}=\frac{20 \times 5}{2 \times 25}=2 \Omega\)

Example 4. The rate of energy consumed in 5Ω resistance is 10 J.s-1. What will be the rate of energy consumed in 4Ω resistance?
Solution:

In the adjoining if, VA-VB= V, then,

⇒ \(I_1=\frac{V}{5} \text { and } I_2=\frac{V}{4+6}=\frac{V}{10}\)

Class 12 Physics Unit 2 Current Electricity Chapter 3 Electric Energy And Power Example 4 The rate ofenergy consumed

∴ \({Energy consumed in 1 \mathrm{~s} in 4 \Omega resistance}{Energy consumed in 1 \mathrm{~s} in 5 \Omega resistance}\)

⇒ \(\frac{I_2^2 \cdot 4}{I_1^2 \cdot 5}=\frac{\left(\frac{V}{10}\right)^2 \cdot 4}{\left(\frac{V}{5}\right)^2 \cdot 5}=\frac{1}{5}\)

So, energy consumed in 1s in 4 resistance

= \(\frac{1}{5}\) x energy consumed in 5Ω resistance

= \(\frac{1}{5}\) x 10

= 2J

i.e., the rate of energy consumed in 4Ω resistance is 2 J.s-1

Example 5. Water boils in an electric kettle for 10 minutes after being switched on. How will you modify the heating coil to boil water in 5 minutes using the same source
Solution:

For the source of power of the same emf, V = constant.

As an equal amount of heat is produced in the two cases, we have

⇒ \(\frac{V^2 t_1}{R_1}=\frac{V^2 t_2}{R_2} \text {, because heat produced } \propto \frac{V^2 t}{R}\)

∴ \(R_2=R_1 \frac{t_2}{t_1}=R_1 \times \frac{5 \mathrm{~min}}{10 \mathrm{~min}}=\frac{1}{2} R_1\)

This means that a heating coil with half the resistance compared to the initial one, is to be used.

Electric Energy And Power Electrical Power

If an electrical source sends current I for time t, in a circuit under potential difference V, then the electrical work done in the circuit,

W = Vlt

We know that the rate of work done with respect to time is called power.

∴ \(P=\frac{W}{t}=\frac{V I t}{t}\)

or, P = VI …(1)

i.e., electrical power = potential difference x current

Unit of power: The units of current and potential difference are volt (V) and ampere (A) respectively. Again, the unit of work is joule (J) and the unit of power is joule/second (J/s) or watt (W).

So, from equation (1) we have,

watt = volt x ampere ….(2)

Definition: If the potential difference at the two terminals of an electrical appliance is 1 volt and the current passing through it is 1 ampere, then the power of the appliance is 1 watt.

We have seen that the relation between the electrical work done in an electrical circuit or in a portion of the circuit is electrical work = active energy + exothermic energy

or, Vlt = V0It + I2Rt

For t= 1s; VI = V0I+I2R

Equation (3) expresses that, electrical power applied = active power + power consumed in the production of heat

The active energy/power is of little concern to us. We are only interested in the latter, for which we shall find an alternative expression.

An alternative expression of the power of passive resistance: If V’ is the terminal potential difference and I is the current flowing through a passive resistance R, then according to Ohm’s law,

V’ = IR

The power consumed in this resistance is,

P’ = I²R [equation (3)]

= IR.I

= V’I, as already shown.

Since, \(I=\frac{V^{\prime}}{R}\)

∴ \(P^{\prime}=\left(V^{\prime} \cdot \frac{V^{\prime}}{R}\right)=\frac{V^{\prime 2}}{R}\)

So, \(P^{\prime}=V^{\prime} I=I^2 R=\frac{V^{\prime 2}}{R}\) …(4)

In most electrical circuits, there is no active device. All the resistances are passive. So, the entire power applied is consumed in the form of heat. In that case, the applied potential difference and the potential difference across the passive resistance (or resistances) become equal to each other.

That is, in this condition, V = V’ and P = P’. So equation (4) can be written as,

⇒ \(P=V I=I^2 R=\frac{V^2}{R}\)…(5)

It is evident from equations (4) and (5) that, the power consumed in the external resistance can be expressed in three different ways. P = VI is the general expression of power and it is applicable in all types of electrical appliances.

On the other hand, the relations \(P=I^2 R \text { or, } P=\frac{V^2}{R}\) are used as the expressions for power consumed in exothermic devices.

Wastage of energy in the transmission of electric power. Suppose, electrical power is to be transmitted over long distances from a generating station. Usually, a long line wire is used for this purpose.

Now, as current flows through this wire, a huge amount of heat is generated due to the Joule effect, i.e., a large amount of electrical energy is wasted as heat during transmission. So, the most important condition for transmitting the electrical power from one place to another is that the wastage of energy should be reduced as much as possible.

A solution to minimize wastage of energy: From the expression of the power P = VI, it can be easily understood that the higher the value of V, the lower will be the value of I.

Again, from equation (3), it is seen that, if the value of I is small, the value of power consumed in the production of heat (I2R) will also diminish, i.e., the wastage of energy will also diminish. So, a high-voltage power transmission allows for lesser resistive losses over long distances in wiring.

In actual practice, electrical energy is transmitted along the long line wire at a high voltage, 11000 V or more. At the place where the supply is required, the high voltage is stepped down to 220 V or 440 V by a step-down transformer.

Horsepower or hp: This is a large unit of power. From the discussion of mechanical energy and power, we know,

1 hp = 746 W

Generally, a horsepower unit is used to express the power of those machines (motor, pump, etc.), where electrical energy is transformed into mechanical energy.

Power Consumed in an Electrical Circuit:

Suppose, in an electrical circuit, the emf of the battery is E. If the internal resistance of the battery is r and R is the equivalent resistance of all the resistances used in the external circuit, current in the circuit, \(I=\frac{E}{R+r}\)

Class 12 Physics Unit 2 Current Electricity Chapter 3 Electric Energy And Power Power Consumed in an electrical circuit

∴ Power consumed in the circuit, \(P_0=I^2(R+r)=\frac{E^2}{R+r}\)

Maximum power in the external circuit: A part of the total power, in the circuit equivalent to I²r, is consumed due to the internal resistance of the battery and it cannot be utilized in any other way. The best part of the power is spent in the external circuit and is utilized for running different electrical appliances connected in the external circuit. Since the equivalent resistance of the external circuit is R, the available power in the external circuit,

⇒ \(P=I^2 R=\frac{E^2 R}{(R+r)^2}\)

Class 12 Physics Unit 2 Current Electricity Chapter 3 Electric Energy And Power Maximum power in the extrnal circuit

In case of constant emf and internal resistance of the battery (E = constant, r = constant), the condition of availability of maximum power in the external circuit is given by,

⇒ \(\frac{d P}{d R}=0 \quad \text { or, } \frac{d}{d R}\left[\frac{E^2 R}{(R+r)^2}\right]=0\)

or, \(E^2 \cdot \frac{1(R+r)^2-2(R+r) R}{(R+r)^2}=0\)

or, (R + r)²-2R²-2Rr = 0

or, r²-R² = 0

or, R = r

i.e., maximum power is obtained in the external circuit, if the equivalent resistance of the external circuit is equal to the internal resistance of the battery. The value of maximum power is

⇒ \(P_{\max }=\frac{E^2 R}{(R+R)^2}=\frac{E^2}{4 R}=\frac{E^2}{4 r}\)

Electric Energy And Power Numerical Examples

Example 1. A 20Ω resistor can dissipate a maximum of 2kW power as heat without being damaged. Should this resistor be connected directly across a 300 V dc source of negligible Internal resistance?
Solution:

If the resistor is connected directly with a 300Vdc source, its power would be,

⇒ \(P=\frac{V^2}{R}=\frac{(300)^2}{20}\)

= 4500W

= 4.5W

Since, this power is greater than 2 kW, so the resistor should not be directly connected to a 300V dc source.

Example 2. Three resistors of equal resistances which en connected in series across voltage sources, dissipate 1.00 watatt of power. What would be the power dissipated, if the resistors are connected in parallel across the same source of emf
Solution:

If each resistance is r, then the equivalent resistance in series, R1 = 3r; and the equivalent resistance in parallel

⇒ \(R_2=\frac{r}{3}\)

In both combinations, the potential difference across the two ends is equal to V (say).

∴ In case of series combination, power, \(P_1=\frac{V^2}{R_1}\) and in case of parallel combination, power, \(P_2=\frac{V^2}{R_2}\)

∴ \(\frac{P_1}{P_2}=\frac{R_2}{R_1} \quad \text { or, } P_2=P_1 \cdot \frac{R_1}{R_2}=100 \times \frac{3 r}{\frac{r}{3}}\)

= 900W

Example 3. The coil of a heater connected to a 200V line consumes a power of 100 W. The coil Is divided Into two equal parts. The two parts are combined In parallel and connected to a 200 V line. What will be the power consumed by the new combination?
Solution:

If R is the resistance of the coll, then power summed in the potential difference V, \(P=\frac{V^2}{R}\)

In the first case, \(100=\frac{(200)^2}{R}\)

In the second case, the resistance of each of the two equal parts = \(\frac{R}{2}\)

So, the equivalent resistance in the parallel combination

⇒ \(\frac{\frac{R}{2} \times \frac{R}{2}}{\frac{R}{2}+\frac{R}{2}}=\frac{R}{4}\)

So, power consumed,

⇒ \(P_2=\frac{(200)^2}{\frac{R}{4}}=4 \times \frac{(200)^2}{R}\)

= 4 x 100

= 400W

Example 4. The power consumed in the circuit. Is 150 W. What is the value of it?

Electric Energy And Power The power consumed in the circuit

Solution:

Equivalent resistance,

⇒ \(r=\frac{R \cdot 2}{R+2}=\frac{2 R}{R+2}\)

So, power, \(P=\frac{V^2}{r}=\frac{V^2(R+2)}{2 R}\)

or, \(\frac{R+2}{R}=\frac{2 P}{V^2} \quad\)

or, \(1+\frac{2}{R}=\frac{2 \times 150}{(15)^2}=\frac{4}{3}\)

or, \(\frac{2}{R}=\frac{4}{3}-1=\frac{1}{3}\)

or, R = 6Ω

Example 5. With a ceil of emf 1.5 V and of internal resistance of 0.1Ω when connected with a resistor and an ammeter of negligible resistance in series, the ammeter shows a 2.0 A steady current. Find

  1. The rate of energy dissipated within the cell
  2. The power consumed in

Solution:

1. The rate of energy dissipated within the cell

= El = 1.5 x 2.0

= 3W

2. \(I=\frac{E}{R+r}\); R = resistance of the resistor

or, \(R=\frac{E}{I}-r\)

= \(\frac{1.5}{2}-0.1\)

= 0.65Ω

∴ Power consumed in the resistor,

= (2.0)² x 0.65

= 2.6 W

Example 6. A balanced Wheatstone bridge has resistances 100Ω, ion, 500Ω, and 50Ω respectively in its four arms. Determine the ratio of powers consumed in its different arms.
Solution:

Wheatstone Bridge has been shown.

Class 12 Physics Unit 2 Current Electricity Chapter 3 Electric Energy And Power Example 6 A balanced Wheatstone bridge has resistances

Let \(I_1=\frac{V}{P+Q} \text { and } I_2=\frac{V}{R+S}\)

∴ \(\frac{I_1}{I_2}=\frac{R+S}{P+Q}=\frac{500+50}{100+10}=5 \quad \text { or, } I_1=5 I_2\)

∴ The required ratio

⇒ \(I_1^2 P: I_1^2 Q: I_2^2 R: I_2^2 S\)

⇒ \(\left(5 I_2\right)^2 \times 100:\left(5 I_2\right)^2 \times 10: I_2^2 \times 500: I_2^2 \times 50\)

= 2500: 250: 500: 50

=50: 5: 10: 1

Example 7. A factory requires a power of 90kW. The energy is transmitted to the factory through a 2.5 n line wire. If 10% of the power generated is lost in transmission, calculate

  1. The transmission line current
  2. The potential difference at the power generating station
  3. The potential drop due to line resistance

Solution:

Here, \(\begin{aligned}
\frac{\text { power of the generating station }}{\text { power required for the factory }} & =\frac{100}{(100-10)} \\
& =\frac{100}{90}
\end{aligned}\)

or, power of the generating station = 90 x \(\frac{100}{90}\) = 100 kW

Power lost =100-90

= 10kW

= 10000 W

This power is lost in the transmission line.

So, according to the equation, P = I²R

Current in the transmission line,

⇒ \(I=\sqrt{\frac{P}{R}}=\sqrt{\frac{10000}{2.5}}=\sqrt{4000}\)

= 63.25A(approx.)

According to the equation, P = VI,

potential difference at the generating station,

⇒ \(V=\frac{P}{I}=\frac{10000}{63.25}=1581 \mathrm{~V}(\text { approx })\)

Potential drop due to line resistance,

V’ =IR

= 63.25 x 2.5

= 158.1V (approx.)

Example 8. Electrical energy is transmitted at the rate of 2.2 MW through the line wire. The resistance of the line wire is 25Ω. Calculate the percentage of heat dissipation of the electrical energy for each line voltage

  1. 22000V
  2. 110kV.

Solution:

VI = 2.2 MW

= 2.2 x 106W

1. \(V=22000 \mathrm{~V} ; I=\frac{V I}{V}=\frac{2.2 \times 10^6}{22000}=100 \mathrm{~A}\)

∴ Percentage of heat dissipation = \(\frac{I^2 R}{V I} \times 100 \%\)

⇒ \(\frac{(100)^2 \times 25}{2.2 \times 10^6} \times 100 \%\)

= 11.4%

2. \(V=110 \mathrm{kV}=110000 \mathrm{~V} ; I=\frac{V I}{V}=\frac{2.2 \times 10^6}{110000}=20 \mathrm{~A}\)

∴ Percentage of heat dissipation = \(\frac{I^2 R}{V I} \times 100 \%\)

⇒ \(\frac{(20)^2 \times 25}{2.2 \times 10^6} \times 100 \%\)

0.45%

[Obviously, it is seen that the dissipation of energy becomes less if the line voltage is high.]

Electric Energy And Power Commercial Units Of Electrical Work And Electrical Energy

Watt. hour (W.h): Work done or electrical energy consumed in 1 hour by an electrical appliance having power 1W is called 1W.h.

∴ lW h = lWx lh = 1W x 3600 s =3600J

It is obvious that, W.h = W x h

= V x A x h

i.e., if 1 A current flows for lh under the potential difference of IV, the amount of electrical energy spent will be 1W.h.

kiloWatt. hour (kW.h): Work done or electrical energy spent in lh by an electrical appliance having power lkW is called lkW h or 1 BOT unit.

Obviously, this is a bigger unit of electrical energy. The electricity supplied to us for our use is measured by this unit. Many times, kW h or BOT unit is simply called unit

lkW.h = 1000 W h = 1000 W x lh

= 1000 W X 3600 s

= 3.6 X 106 W.S

= 3.6 X 106J

⇒ \(\mathrm{kW} \cdot \mathrm{h}=\frac{\mathbf{W} \times \mathbf{h}}{1000}=\frac{\mathrm{V} \times \mathrm{A} \times \mathrm{h}}{1000}\)

For Example, if an electric bulb marked 100 W glows for 20h then, the amount of electrical energy spent = \(=\frac{100 \times 20}{1000}=2 \mathrm{~kW} \cdot \mathrm{h}\)

= 2 unit

So, in this case, the value of the electric meter will rise up by 2 units.

Electric Energy And Power Numerical Examples

Example 1. In a house, there are 10 lamps of 40 W each, 5 fans of 80 W each, and a TV set of 80 W. They run for 6 hours a day. Find the consumption of electrical energy in a month of 30 days. What is its value in BOT unit
Solution:

Total power = (10 x 40) + (5 x 80) + 80

= 880 W.

Total time = 6 x 30

= 180 h

= 180 x 3600 s

Energy consumed in a month = 880 x 180 x 360

= 5.7 x 108J

Its value in BOT unit = \(\frac{\mathrm{W} \times \mathrm{h}}{1000}=\frac{880 \times 180}{1000}\)

= 158.4

Example 2. In an evening college, there are 100 bulbs of 60 W each, 80 bulbs of 100 W each, and 70 fans of 100 W each. They run for 5h, 4h and 4h respectively per day. If each kW.h costs ₹ 0.50, calculate the electric bill of the college for a month
Solution:

The electrical energy consumed by 60 W bulb

= \(\frac{(60 \times 100) \times 5 \times 30}{1000}\)

= 900kW.h

Electrical energy consumed by 100 W bulbs

⇒ \(\frac{(100 \times 80) \times 4 \times 30}{1000}\)

= 960kW.h

Electrical energy consumed by fans

⇒ \(\frac{(100 \times 70) \times 4 \times 30}{1000}\)

= 840 kW.h

∴ Cost of electrical energy = (900 + 960 + 840) x 0.50

= ₹ 1350

Example 3. In a house, there are 20 lamps of 60W each, and 10 fans that operate in 0.5A current. If the main power supply is 220V, the expense per kW h is 50 paise and each application runs 6 h per day then calculate the electric bill of
Solution:

November month has 30 days.

∴Total time = 30 x 6h = 180h ;

50 paise = ₹ 0.5

Power of each fan =220 x 0.5 = 110W

∴ Electrical energy

⇒ \(\frac{(60 \times 20+110 \times 10) \times(30 \times 6)}{1000}\)

⇒ \(\frac{2300 \times 30 \times 6}{1000}\)

= 23 x 18 kW.h

∴Electric bill of November = ₹(23 x 18) x 0.5

= ₹ 207

Example 4. There are six 40 W and two 100 W lamps, four 40 W fans, and a 1000 W electric heater in the house. If in April, each lamp runs for 5 hours a day, each fan for 15 hours a day, and the heater for 2 hours a day, what will be the electric bill for that month? It may be supposed that the main supply voltage is 200 V and the cost of each BOT unit = ₹ 1.50. Which one of the given three fuse wires of rating 5 A, 10 A, and 15 A will be appropriate for connection in the main switch?
Solution:

Number of days in April = 30

Electrical energy consumed by the lamp

⇒ \(\frac{(6 \times 40+2 \times 100) \times 5 \times 30}{1000}\)

= 66 BOT unit

Electrical energy consumed by the fans

⇒ \(\frac{(4 \times 40) \times 15 \times 30}{1000}\)

= 72 BOT unit

The electrical energy consumed by the heat

⇒ \(=\frac{1000 \times 2 \times 30}{1000}=60 \mathrm{BOT} \text { unit }\)

∴ Monthly expenditure on electricity

= (66 + 72 + 60) X 1.50 = ₹ 297

Again, the total power of the appliances

= (6 x 40) + (2 x 100) + (4 x 40) + 1000

= 1600 W

If all the appliances run simultaneously, according to the relation, P = V x I,

we have, \(I=\frac{P}{V}=\frac{1600}{200}=8 \mathrm{~A}\)

So, a 10 A fuse will be appropriate for connection in the main switch
because a 5 A fuse will melt and there is no need for a 15 A fuse

Example 5. The power of a small electric motor is \(\frac{1}{8}\) HP. If it is connected to a 220 V supply line, how much current will it j draw? If the motor runs for 80 hours, what will be the cost? j (Each BOT unit costs 70 paise)
Solution:

Power, P = \(\frac{1}{8}\) HP = \(\frac{1}{8}\) x 746 W

According to the relation, P = V x I,

we have, \(I=\frac{P}{V}=\frac{1}{8} \times 746 \times \frac{1}{220}=0.424 \mathrm{~A}\)

Number of BOT units = \(\frac{1}{8} \times 746 \times 80 \times \frac{1}{1000}\)

∴ \(\text { Cost }=\frac{1}{8} \times 746 \times 80 \times \frac{1}{1000} \times 70 \text { paise }\)

= ₹ 5.22 (approx.)

Example 6. A heating coil of resistance 100 fl is connected for 30 min to 220 V. By this time, determine

  1. Amount of charge flowing,
  2. Amount of electrical energy consumed
  3. Amountofheatgenerated. Determine the cost of consumed electrical energy if 1 kW.h costs ₹1.

Solution:

1. Amount of charge flowing,

⇒ \(Q=I t=\frac{V}{R} t=\frac{220}{100} \times 30 \times 60=3960 \mathrm{C}\)

2. Amount of electrical energy consumed,

W = Q.V

= 3960 x 220

= 8.712 x 105 J

3. Amount of heat generated,

⇒ \(H=\frac{W}{J}=\frac{8.712 \times 10^5}{4.2}=2.074 \times 10^5 \mathrm{cal}\)

= 2.074 x 105 cal

4. Cost of consumed electrical energy = \(\frac{8.712 \times 10^5}{3600 \times 1000} \times 1\)

= ₹ 0.24

= 24 paise

Electric Fuse:

Every metal, such as copper, aluminum, etc. used as the element of connecting wires in electrical circuits and electrical applications has a definite melting point.

Due to a short circuit or any other reason, if an excessively high current flows in the circuit, the temperature of the wire may rise to its melting point.

As a result, the whole circuit may be damaged and may cause fire. A fuse inserted in the circuit may prevent an accidental fire. It is prepared from an alloy (e.g., 3 parts lead and 1 part tin) having a comparatively low melting point.

This fuse is placed in an insulating box and is connected in series with the main circuit. Before the current of the main circuit reaches the danger point, the temperature of the fuse reaches its melting point.

As a result, the fuse melts down and the main circuit is disconnected. Hence, there is no chance of damage to the main circuit

Circuit breaker:

This is an automatic switch system that disconnects the circuit before being damaged due to short-circuit or overload. At the very moment the circuit is disconnected, the switch of the breaker moves to the ‘off’ position.

After replacing the defective instrument for which the circuit was broken, the switch is made ‘on current flows as usual through the circuit. In different types of circuit breakers, different physical phenomena are used.

For Example, in the circuit through which high current flows, a bimetallic strip bends due to heating and disconnects the circuit; or if a high current flows through an electromagnet, it attracts a soft-iron plate of the circuit and thus the circuit is cut off.

The main difference between a fuse wire and a circuit breaker is that, the fuse wire is burnt off and needs to be replaced with a new wire every time the fuse melts.

On the other hand, a circuit breaker is not at all damaged and hence there is no need to replace it. Once the fault has been repaired, the contacts must again be closed to restore power to the interrupted circuit.

The circuit breaker which is used in low voltage lines (0-1000 V) is generally called a Mechanical or Miniature Circuit Breaker (MCB).

Nowadays, in place of fuse wire, MCB is widely used. The switch gear is generally used as a circuit breaker in the supply line of high voltage (11000 V or more) from the generating station.

Highest safe current: If l is the length of the fuse wire, r is its radius, and \(\rho\) its specific resistance, then the resistance of the wire is given by,

⇒ \(R=\rho \frac{l}{\pi r^2}\)

So, heat generated in the wire per second is,

⇒ \(H=\frac{l^2 R}{J}=\frac{l^2 \rho l}{J \pi r^2}\)

This heat is used in two ways:

1. One part increases the temperature of the tire wire,

2. The other part radiates to the surroundings from the outer surface of the wire.

Thus, a situation arises when the total heat generated in the wire, radiates to the surroundings. So, the temperature does not increase further and reaches a fixed value. If this temperature lies just below the tire melting point of the material of the fuse wire, then the tire current flowing in the tire wire at this stage is called the tire’s highest safe current.

Now, the outer surface area of the tire wire = 2πrl.

If h is the tire amount of radiant heat from the unit area of the tire wire per second, then the total radiant heat from the outer surface of the wire per second is, H’ = 2πrlh.

So, for the highest safe current I,

⇒ \(H=H^{\prime}\)

or, \(\frac{I^2 \rho l}{J \pi r^2}=2 \pi r l h \quad\)

or, \(I^2=\frac{2 \pi^2 J h}{\rho} \cdot r^3\)

i.e., \(I^2 \propto r^3 \text { or, } I \propto r^{3 / 2}\)

It is to be noted that, the expression for highest safe current I is independent of the length I of the fuse wire.

Electric Energy And Power Rating Of Electrical Instruments

Voltage rating: Bach and every electrical instrument depends on potential differences. Each instrument is marked with a fixed value of potential difference, indicating the range of potential difference within which it can be run safely. This is called voltage rating.

For Example, the voltage rating of each domestic appliance is 220 V. This means that, if the potential difference between two ends of the appliance is 220 V, then its efficiency becomes maximum.

The efficiency of the appliance decreases if for any reason the supply voltage Is diminished, i.e., the electric lamp glows dimly, the electric fan moves comparatively slowly, etc.

On the other hand, if the voltage exceeds 220 V, there is a chance of damage to the appliance.

This is to note that, the appliances heaving the same voltage rating should be connected in parallel while used simultaneously in a circuit. In that case, the potential difference across each application remains the same.

For this reason, daily appliances are all connected in parallel combination with a 220 V supply line.

Watt rating: Different electrical appliances, even running at the same voltage may not consume the same electrical energy in unit time.

Hence, the power of the appliance, i.e., the electrical energy consumed per second should be mentioned.

This is called the watt rating of electrical appliances. Thus 220V-100W on the body of a lamp or 220V-2000W on the body of a heater refers to the voltage and watt ratings of the appliance.

Significance of ratings:

1. Maximum capability: Voltage rating indicates at what voltage, the appliance does maximum work, and watt rating indicates the rate of consumption of electrical energy at that voltage.

For Example, 220V-100W indicates diet, if the voltage at the two ends of the lamp is 220 V, it will glow with maximum brightness, consuming energy at the rate of 100 W

2. Calculation of current and resistance: It is possible to calculate the current and resistance of the appliance when it operates with maximum efficiency.

Significance of rating Example:

⇒ \(I(\text { current })=\frac{P(\text { power })}{V(\text { potential difference })}=\frac{100 \mathrm{~W}}{220 \mathrm{~V}}=0.45 \mathrm{~A}\)

⇒ \(R(\text { resistance })=\frac{V}{I}=\frac{220 \mathrm{~V}}{0.45 \mathrm{~A}}=489 \Omega\)

Safety of supply line: Whether an electrical appliance is safe or not for a supply line, is known from its rating. Consider the case of a 220 V-2000 W electric heater.

Current drawn by the heater = \(\frac{2000}{20}\) = 9.1 A (approx.)

So, this heater cannot be used in the house, where the meter and fuse are installed for 5 A current. If it is used, the supply line will be damaged.

Power in a combination of more than one electrical device:

Parallel combination: Suppose, and R2 are the resistances of two electrical appliances. Their watt ratings are P1 and P2 respectively and the voltage rating of both of them is V.

If they are connected in parallel to an electric source of voltage V, then the voltage at the two ends of both the appliances will be V. The two appliances will function at full power and the total power consumed by the circuit will be,

P = P1 + P2

Class 12 Physics Unit 2 Current Electricity Chapter 3 Electric Energy And Power Parallel combination

For any number of electrical appliances connected in parallel, P = P1+P2 + P3+…. obviously, total power will be greater than that of each appliance.

Household electrical appliances are usually connected in
parallel.

Now, \(P_1=\frac{V^2}{R_1} \text { and } P_2=\frac{V^2}{R_2}\)

∴ \(\frac{P_1}{P_2}=\frac{R_2}{R_1}\)….(1)

For Example, if two electric lamps of 220 V-100 W and 220 V-60 W are connected in parallel and the combination is joined to a 220 V line, the 100 W lamp will glow brighter and the total power of the circuit will be, P – 100 + 60 = 160 W.

Series combination: Now the two appliances are nected in series and this combination is connected to the same electric source. The same current, say I, will flow through both of them. So, their powers will be respectively,

P’1 = I²R1 and P’2 = I²R2

So, \(\frac{P_1^{\prime}}{P_2^{\prime}}=\frac{R_1}{R_2}\) …..(2)

From equations (1) and (2) we have,

⇒ \(\frac{P_1^{\prime}}{P_2^{\prime}}=\frac{P_2}{P_1}\)

So, if \(P_1>P_2, P_1^{\prime}<P_2^{\prime}\)

i.e., in series combination, the appliance having a higher watt rating will consume less power.

For Example, if two electric lamps having the power of 100 W and 60 W are connected in series, the power of the 100 W lamp will be comparatively lower and it will glow with less brightness.

Again, according to,

⇒ \(I=\frac{V}{R_1+R_2}\)

So, the total power of the circuit,

⇒ \(P^{\prime}=I^2\left(R_1+R_2\right)=\frac{V^2}{R_1+R_2}\)

∴ \(\frac{1}{P^{\prime}}=\frac{R_1+R_2}{V^2}=\frac{R_1}{V^2}+\frac{R_2}{V^2}=\frac{1}{P_1}+\frac{1}{P_2}\)

For any number of electrical appliances connected in series,

⇒ \(\frac{1}{P^{\prime}}=\frac{1}{P_1}+\frac{1}{P_2}+\frac{1}{P_3}+\cdots\)

Obviously, the total power P’ will be less than that of each application.

For Example, if two electric lamps of 220 V-100 W and 220 V-60 W are connected in series and joined to a 220 V supply line, the 60 W lamp will glow brighter and the total power of the circuit will be less than 60 W. Here

⇒ \(\frac{1}{P^{\prime}}=\frac{1}{100}+\frac{1}{60} \quad\)

or, \(P^{\prime}=\frac{100 \times 60}{100+60}\)

= 37.5 W^

Rating of a resistor: No resistor can endure excess current over a certain designated value depending upon its nature. If the limit is exceeded, the excessive heat generated in the resistor will bum it. We may say that, as soon as the power consumption due to the production of heat crosses a certain limit, the resistor gets damaged. This maximum power tolerance is known as the watt rating of the resistor.

For Example, a resistor with a rating lW-100Ω means a 100Ω resistance and of maximum power tolerance 1W. We know, power,

P = VI = IR.I = I²R

So maximum current tolerance of the resistor Is,

⇒ \(I=\sqrt{\frac{P}{R}}=\sqrt{\frac{1}{100}}=\frac{1}{10} \mathrm{~A}\)

Under this condition, the potential difference between the two ends of the resistor is,

⇒ \(V=I R=\frac{1}{10} \times 100\)

= 10V

Therefore, before joining a circuit, the rating of a resistor should be known. In that case, it can be connected in such a way that the current should always remain below

⇒ \(\frac{1}{10}\) A through it

Electric Energy And Power Numerical Examples

Example 1. A 220V-60W electric bulb is connected To a 220 V line. What is the resistance of the filament of the bulb, when it is turned on?
Solution:

Power, \(P=V I=V \cdot \frac{V}{R}=\frac{V^2}{R} \quad[∵ V=I R]\)

∴ \(R=\frac{V^2}{P}=\frac{220 \times 220}{60}\)

= 806.67Ω

Example 2. The resistance of a hot tungsten filament is about 10 times that in its normal state. What will be the resistance of a 100 W-200 V tungsten lamp in its normal state?
Solution:

Resistance of the lamp when it is hot,

⇒ \(R=\frac{V^2}{P}=\frac{(200)^2}{100}=400 \Omega\)

So, resistance of the lamp in its normal state = \(\frac{400}{10}=40 \Omega\)

Example 3. The main meter of a house is marked 10 A-220 V. How many 60 W electrical amps can be used safely in this line?
Solution:

Maximum power of the household line,

P = VI

= 220 X 10

= 2200 W

Maximum number of 60 W lamp = \(\frac{2200}{6}\) = 36.67

So, a maximum of 36 lamps can be used at a time.

Example 4. A 220V-100W electric lamp fuses above 150 power. What should be the maximum tolerable voltage for the lamp?
Solution:

Power, \(P=\frac{V^2}{R}\)

If we ignore the change or resistance to the Increase of power,

We have, \(P \propto V^2\)

So, for two different powers, \(\frac{P_1}{P_2}=\frac{v_1^2}{v_2^2}\)

i.e., \(\cdot V_2=V_1 \sqrt{\frac{P_2}{P_1}}=220 \times \sqrt{\frac{150}{100}}\)

= 269.4 V

So, the maximum tolerable voltage for the lamp Is 269.4 V.

Example 5. In order to run a 60V-120W lamp safely in a 220V dc line, a resistor of what minimum magnitude should be placed in series with it?
Solution:

When the lamp runs safely In the given dc line, the maximum current is,

⇒ \(I=\frac{120}{60}=2 \mathrm{~A}\)

So, this 2 A current will also flow through the resistance placed in series.

Again, the potential difference at the two ends of the resistance

= 220-60

= 160V.

∴ The minimum value of the resistance = \(\frac{160}{2}\)

= 80Ω

Example 6. Draw a household circuit having a 1200 W toaster, a 1000 W oven, an 800 W heater, and a 1500 W cooler. The circular has a heavy-duty wire and a 20 A circuit breaker. Will the circuit breaker trip, if all the appliances are operated simultaneously in a 220 V supply voltage?
Solution:

The household circuit. When all the appliances are operated simultaneously, the total po is, P = 1200 + 1000 + 800 + 1500

= 4500 W.

Now, P = VI

or, \(I=\frac{P}{V}=\frac{4500}{220}\)

= 20.45A

Class 12 Physics Unit 2 Current Electricity Chapter 3 Electric Energy And Power Example 6 household circuit

But the circuit breaker is rated for 20 A.

So, if all the appliances are operated simultaneously, the circuit breaker will trip.

Example 7. Two lamps of 200 W and 100W arc connected in series in 200 V mains. Assuming the resistance of the two lamps remains unchanged, calculate the power consumed by each of them.
Solution:

⇒ \(P=\frac{V^2}{R} \quad \text { or, } R=\frac{V^2}{P}\)

So, the resistance of the first lamp, \(R_1=\frac{200 \times 200}{200}=200 \Omega\)

and resistance of the second lamp, \(R_2=\frac{200 \times 200}{100}=400 \Omega\)

Now, if we connect the two lamps in series with 200 V mains, current flowing through each of them,

⇒ \(I=\frac{V}{R_1+R_2}=\frac{200}{200+400}=\frac{1}{3} \mathrm{~A}\)

∴ Power of the first lamp, \(P_1=I^2 R_1=\frac{1}{3} \times \frac{1}{3} \times 200=22.2 \mathrm{~W}\)

Power of the second lamp, \(P_2=I^2 R_2=\frac{1}{3} \times \frac{1}{3} \times 400=44.4 \mathrm{~W}\)

Example 8. Two electric bulbs each designed to operate with a power of 500 W In a 220 V line are connected in series in a 110 V line. What will be the power generated by each bulb?
Solution:

Power, \(P=\frac{V^2}{R}\)

So, the resistance of each bulb,

⇒ \(R=\frac{V^2}{P}=\frac{(220)^2}{500}=\frac{484}{5} \Omega\)

When connected in series, the potential difference at the two ends of each bulb = \(\frac{110}{2}\)

=55V.

∴ \(\frac{(55)^2}{\frac{484}{5}}=\frac{55 \times 55 \times 5}{484}\)

= 31.25W

Example 9. If the supply voltage drops from 220 V to 200 V , what would be the percentage reduction in heat produced by a 220 V-1000 W heater? Neglect the change of resistance. If the change of resistance is taken into consideration, would the reduction of heat produced be smaller or larger than the previously calculated value?
Solution:

We know, \(P=\frac{V^2}{R}\)

So, \(R=\frac{V^2}{P}\)

If the variation of resistance is ignored,

In the first case, \(R=\frac{V_1^2}{P_1}\)

In the second case, \(R=\frac{V_2^2}{P_2}\)

i.e., \(\frac{V_1^2}{P_1}=\frac{V_2^2}{P_2} \text { or, } \frac{P_2}{P_1}=\frac{V_2^2}{V_1^2}\)

or, \(\frac{P_2-P_1}{P_1}=\frac{V_2^2-V_1^2}{V_1^2}=\frac{\left(V_2+V_1\right)\left(V_2-V_1\right)}{V_1^2}\)

= \(\frac{(200+220)(200-220)}{220 \times 220}\)

or, \(\frac{P_2-P_1}{P_1}=-\frac{420 \times 20}{220 \times 220}\)

= -0.1736 (approx.)

So, the power of the heater will reduce by 17.36%; i.e., the percentage reduction of heat produced is 17.36%. In the above calculation, the decrease of resistance with voltage drop was ignored. But actually, with a decrease in the temperature of the coil, its resistance also decreases. Hence according to the relation P = \(P=\frac{V^2}{R}\), power Increases, i.e., heat supplied by the heater also increases. So, the reduction of heat produced will be lower than 17.36%.

Example 10. The emf of the cell, E = 20V. The rating of each resistance R1 and R2, is 1W-100Ω. What should be the minimum value of the resistance R in the circuit? Also, find its minimum watt rating

Electric Energy And Power Example 10 the minimum value of the resistance

Solution:

According to the formula,

P = I²R

or, \(I=\sqrt{P / R}\)

The maximum current that can flow through each of the resistance R1, and R2 is,

⇒ \(I_1=I_2=\sqrt{\frac{1}{100}}=\frac{1}{10} \mathrm{~A}\)

∴ The maximum current that can flow through the resistance R,

⇒ \(I=I_1+I_2=2 \times \frac{1}{10}=\frac{1}{5} \mathrm{~A}\)

On die other hand, equivalent resistance of R1 and R2.

⇒ \(\frac{R_1 R_2}{R_1+R_2}=\frac{100 \times 100}{100+100}=50 \Omega\)

Now, \(I=\frac{E}{R+50} \quad\)

or, \(\frac{1}{5}=\frac{20}{R+50}\)

or, R + 50 = 100

∴ R = 100-50 = 50Ω

i.e., to keep the value of I lower than \(\frac{1}{5}\) A, R sliould be greater than 50Ω. Power consumption for the minimum value of R,

⇒ \(P=I^2 R=\left(\frac{1}{5}\right)^2 \times 50=2 \mathrm{~W}\)

Example 11. If a 220 V-1000 W lamp is connected to 110 V line then what will be the power consumed by it?
Solution:

Resistance of the lamp, \(R=\frac{V_1^2}{P}=\frac{(220)^2}{1000} \Omega\)

∴ Power consumed by the lamp when it is connected to 110V
line,

⇒ \(P_2=\frac{V_2^2}{R}=\frac{(110)^2 \times 1000}{(220)^2}=250 \mathrm{~W}\)

Example 12. 2.2 kW power is supplied through a line of 10Ω resistance under a 22000 V voltage difference. What is the rate of heat dissipation in the line?
Solution:

⇒ \(I=\frac{P}{V}=\frac{2.2 \times 10^3}{22000} \quad\left[∵ 2.2 \mathrm{~kW}=2.2 \times 10^3 \mathrm{~W}\right]\)

= 0.1 A

∴ Rate of heat dissipation = I²R

= (0.1 )2 x 10

= 0.1 W

Example 13. The potential difference between the two ends of an electric lamp is decreased by 1%. Neglecting the change in its resistance, calculate the percentage increase or decrease in the power of the lamp.
Solution:

We know power, \(P=\frac{V^2}{R}\)

∴ InR = 21nV-InR

By differentiating \(\frac{d P}{P}=2 \cdot \frac{d V}{V}[∵ R=\text { constant, } d R=0]\)

According to die problem, \(\frac{d V}{V}=-1 \%=-\frac{1}{100}\)

∴ \(\frac{d P}{P}=-\frac{2}{100}=-2 \%\)

i.e., the power of the lamp will be decreased by 2%.

Example 14. 15 kW power is supplied through a line of 0.5Ω resistance under 250 V potential difference. Find the efficiency of the supply in percentage.
Solution:

Power, P = 15 kW

= 15000 W

So, currently in the supply line.

⇒ \(I=\frac{P}{V}=\frac{15000}{250}=60 \mathrm{~A}\)

Power loss due to inactive resistance in the line

= I²R = (60)² x 0.5

= 1800W;

Therefore, total power in line = 15000 + 1800

= 16800 W

∴ \(\text { Efficiency }=\frac{\text { effective power }}{\text { total power }} \times 100 \%\)

⇒ \(\frac{15000}{16800} \times 100 \%=89 \%\)

Example 15. Two incandescent lamps (25 W, 120 V) and (100 W, 120 V) are connected in series across a 240 V supply. Assuming that the resistances of the lamps do not vary with current, find the power dissipated in each lamp after the connection.
Solution:

The resistance of the lamp is 25 W-120 V,

⇒ \(R_1=\frac{120^2}{25}=576 \Omega\)

The resistance of the lamp \(100 \mathrm{~W}-120 \mathrm{~V}, R_2=\frac{120^2}{100}=144 \Omega\)

When the lamp is connected in series, the equivalent resistance of the circuit,

R’ = 576 + 144 = 720Ω

∴ Current flowing through the circuit = \(=\frac{240}{720}=0.33 \mathrm{~A}\)

∴ Power dissipated in die first bulb =(0.33)²x576 = 62.72 W

∴ Power dissipated in die second bulb = (0.33)² x 144

= 15.68 W

Electric Energy And Power Synopsis

  • The amount of work done to carry a 1-coulomb charge through a potential difference of 1 volt is called 1 joule.
  • The reciprocal of the heat produced in one second in a conductor of unit resistance, for the passage of unit current through it, is called the mechanical equivalent of heat.
  • If in a resistor, electrical energy is totally transformed into heat energy, the resistor is then called a passive resistor.
  • Electrical power is to be transmitted from one place to another at high voltage and low current. This is the most important condition to minimize heat loss.
  • If the potential difference at the two terminals of an electrical appliance is 1 volt and the current passing through it is 1 ampere, then the power of the appliance is l watt.
  • Work done or electrical energy spent in 1 hour by an electrical appliance having power 1W is called 1Wh. Work done or electrical energy spent in lh by an electrical appliance having power lkW is called lkW-h or 1BOT unit.
  • In writing 220V-100W on the body of an electric lamp or 220V-2000W on the body of an electric heater, both voltage rating and watt rating are preferred.
  • If Q charge flows through a section under the potential difference V, then the amount of electrical work done,
  • W = QV.
  • The relation between joule (I), coulomb (C), and volt (V) is J = C x V [∵ W= QV]
  • The heat evolved in a conductor of resistance R due to current I flowing through it for time t,
  • \(H=\frac{I^2 R t}{J}\)
  • where, J = mechanical equivalent of heat
  • = 4.2 x 107 erg/cal (In CGS)
  • = 4.2 J/cal
  • = 1 (In SI)
  • i.e., \(H=0.24 I^2 R t=0.24 \frac{V^2 t}{R}\) [∵ V = IR]

Class 12 Physics Unit 2 Current Electricity Chapter 3 Electric Energy And Power Relation of electrical power P with V, I and R and relation between the units

  • Current in a total circuit,
  • I = \(\frac{E}{R+r}\) , where, E = emf of the cell, R = external resisR + r tance, r = internal resistance
  • Power consumption in the circuit, \(P_0=I^2(R+r)=\frac{E^2}{R+r}\)
  • Output power, \(P=I^2 R=\frac{E^2 R}{(R+r)^2}\)
  • lW.h = 3600J, lkW.h = 3.6 x 106 J
  • \(1 W \cdot h=\frac{W \times h}{1000}=\frac{V \times A \times h}{1000}\)
  • For a parallel combination of a number of electrical appliances, equivalent power, P = P1 + P2 + P3+….
  • For a series combination of a number of electrical appliances, equivalent power,
  • \(P^{\prime}=\frac{1}{\frac{1}{P_1}+\frac{1}{P_2}+\frac{1}{P_3}+\cdots}\)
  • If a heating coil of resistance R, consumes power P, when voltage V is applied to it, then by keeping V constant if it is cut in n equal parts then the resistance of each part will be R/n and the power consumed by each part P’ = nP.
  • Power consumed by a n equal resistors is parallel is n2 times that of power consumed in series if V remains the same i.e., Pp = n2Ps.

Electric Energy And Power Very Short Questions and Answers

Question 1. A current I flows through a potential drop V across a conductor. What is the rate of heat production?
Answer: VI

Question 2. A small heating element connected to a 10 V dc supply draws a current of 5 A. Find the electric power supplied to the heater.
Answer: 50W

Question 3. A 220 V – 1000 W electric heater is connected in parallel with a 220 V – 60 W electric lamp, and the combination is fed by a 220 V main. Now, if the lamp is replaced by another 220 V – 100 W lamp, what will be the change in the rate of heat generation in the heater?
Answer: No change

Question 4. Two wires having resistances R and 2R are connected in series. If current is allowed to pass through the combination, what will be the ratio of power consumed in the two resistances?
Answer: 1: 2

Question 5. Two wires having resistances R and 2R are connected in parallel. If current is allowed to pass through the combination, what will be the ratio of power consumed in the two resistances?
Answer: 2: 1

Question 6. A 240 V- 1000 W lamp and a 240 V- 100 W lamp—which of these two has a thinner filament?
Answer: 240 V – 100 VV lamp

Question 7. Two resistances, each of magnitude 2Ω, are connected in series and a potential difference of 2V is applied at the two terminals of the combination. What is the power of the combination?
Answer: 1W

Question 8. Two resistances each of magnitude 2Ω are connected in parallel and a potential difference of 2V is applied at the two ends of the combination. What is the power of the combination?
Answer: 4W

Question 9. What is the resistance of an electric bulb marked 220 V-100 W in an incandescent state?
Answer: 484Ω

Question 10. An electric lamp is marked 240V -60 W. What is the resistance of the lamp in an incandescent state?
Answer: 960Ω

Question 11. A 220V -60 W electric lamp is connected to a 220V supply line. Determine the resistance of the lamp in an incandescent state.
Answer: 8.67Ω

Question 12. Which one of two electrical appliances, rated 100 W-200 V and 60 W-200 V, would have a higher resistance?
Answer: 60 W-220 V

Question 13. What is the largest voltage you can safely put across a 98Ω, 0.5 W resistor?
Answer: 7V

Electric Energy And Power Fill In The Blanks

1. A 2Ω resistance is connected to a source of constant emf. Another 2Ω resistance is connected in parallel to the previous one. The power consumed in the circuit becomes double

2. Two resistances are connected in series. If the current is made to pass through the combination, power consumed in the larger resistance will be higher

3. Two resistances are connected in parallel. If the current is made to pass through the combination power consumed in the larger resistance will be lower

4. A 220 V-100 W lamp and a 220V-60W lamp are connected in parallel. If the current is made to pass through the combination, the brightness of the first lamp will be higher

5. A 220 V-100 W lamp and a 220V-60W lamp are connected in series. If the current is made to pass through the combination, the brightness of the first lamp will be lower

Electric Energy And Power Assertion Reason Type

Direction: These questions have Statement 1 and Statement 2. Of the four choices given below, choose the one that best describes the two statements.

  1. Statement 1 is true, Statement 2 is true; Statement 2 is a correct explanation for Statement 1
  2. Statement 1 is true, Statement 2 is true; Statement 2 is not a correct explanation for Statement 1
  3. Statement 1 is true, Statement 2 is false
  4. Statement 1 is false, Statement 2 is true

Question 1.

Statement 1: Electric current is distributed in different branches of a circuit in such a way, that the total heat evolved in the circuit is the lowest.

Statement 2: The transformation of electrical energy into heat energy in a circuit is less probable than its transformation into other forms of energy.

Answer: 3. Statement 1 is true, Statement 2 is false

Question 2.

Statement 1: An external circuit can draw a maximum power of 9 W from a source of emf 6 V and internal resistance 1 n.

Statement 2: The condition, for which an external circuit of resistance R draws the maximum power from a source of internal resistance r, is R = r.

Answer: 1. Statement 1 is true, Statement 2 is true; Statement 2 is a correct explanation for Statement 1

Question 3.

Statement 1: The power consumed would be 50 W by each of two 200V-100W lamps when their series combination is driven by a potential difference of 200V.

Statement 2: If P is the power consumed by a series combination of some electrical devices of power P1, P2, P3,…., then, \(\frac{1}{P}=\frac{1}{P_1}+\frac{1}{P_2}+\frac{1}{P_3}+\cdots\)

Answer: 4. Statement 1 is false, Statement 2 is true

Question 4.

Statement 1: A fuse wire of diameter 0.5 mm can withstand a maximum current of I A. For a current of 8 A, a fuse wire made of the same alloy should have a diameter of 2 mm.

Statement 2: The radius r of a fuse wire and the maximum safe current I that may pass through it are related as \(I \propto r^{3 / 2}\)

Answer: 1. Statement 1 is true, Statement 2 is true; Statement 2 is a correct explanation for Statement 1

Question 5.

Statement 1: The coil resistance of a 200 V-100 W electric fan is 20Ω. A power of 5 W is lost as heat when the fan rotates at its maximum speed.

Statement 2: If, for an electrical device the current is I and the terminal potential difference is V, the power consumed = VI

Answer: 2. Statement 1 is true, Statement 2 is true; Statement 2 is not a correct explanation for Statement 1

Electric Energy And Power Match The Columns

Question 1. Each of the two cells in the circuit has emf 5 V and internal resistance 2Ω. Match the following two columns for this circuit.

Class 12 Physics Unit 2 Current Electricity Chapter 3 Electric Energy And Power Match The Column 1

Class 12 Physics Unit 2 Current Electricity Chapter 3 Electric Energy And Power Match The Column 1.

Answer: 1-B, 2-D, 3-A, 4-C

Question 2. The cell in the circuit has negligible internal resistance. The resistance values are in Ω. Match the following two columns for this circuit.

Class 12 Physics Unit 2 Current Electricity Chapter 3 Electric Energy And Power Match The Column 2

Class 12 Physics Unit 2 Current Electricity Chapter 3 Electric Energy And Power Match The Column 2.

Answer: 1-C, 2-A, 3-D, 4-B

Question 3. The ratings of a few electrical elements are given in Column A, and Column B shows the values of maximum safe currents. Match the two columns.

Class 12 Physics Unit 2 Current Electricity Chapter 3 Electric Energy And Power Match The Column 3

Answer: 1-B, 2-A, 3-D, 4-C

WBCHSE Class 12 Physics Kirchhoff’s Laws And Electrical Measurement Notes

Kirchhoff’s Laws And Electrical Measurement Kirchhoff’s Laws

Kirchhoff formulated the following two laws which enable us to find the distribution of current in complicated electrical circuits (or networks of conductors).

Kirchhoff’s First Law or Kirchhoff’s Current Law (KCL):

Statement: In an electrical circuit 9or network of wires) the algebraic sum of currents through the conductors as meeting at a point is zero, i.e., \(\Sigma i=0\)

Kirchhoff’s Current Law Explanation of the law: Let us consider the number of wires connected at point A. Currents i1, i2, i3, and i4 flow through these wires in the directions. Here currents i1 and i2 are approaching point A while currents i3 and i4 are leaving point A. Taking current entering point A as positive current while current leaving point A as negative current and applying the first law we can write,

i1 + i2 – i3 – i4 = 0…(1)

Class 12 Physics Unit 2 Current Electricity Chapter 2 Kirchhoff’s Laws And Electrical Measurement Explanation of the law

Kirchhoff’s Current Law Discussion:

Node analysis: Any connecting point in an electrical circuit is called a node. By applying the first law of Kirchhoff, the analysis of an electrical circuit is called node analysis.

Conservation of electric charge: If we write equation (1) in the form i.e., i1 + i2 = i3 + i4, we can say that the sum of the currents approaching a connecting point = the sum of the currents leaving the point. If the current flows for time t we have

Read and Learn More Class 12 Physics Notes

⇒ \(i_1 t+i_2 t=t_3 t+i_4 t \quad \text { or, } q_1=q_2=q_3+q_4\)

i.e., the sum of the charges approaching the connecting point = the sum of the charges leaving the point.

The significance of the above equation is discussed below:

There cannot be any accumulation of charge at any connecting point in an electrical circuit

The charge cannot be created or destroyed, i.e., the total charge will remain constant

Kirchhoff’s Second Law or Kirchhoff’s Voltage Law (KVL):

Statement: The algebraic sum of the product of the current and resistance in any closed loop of a circuit is equal to the algebraic sum of electromotive force acting in that loop, i.e.,

⇒ \(\Sigma i r=\Sigma e\)

Explanation of the law: A closed-loop ACBDA within an electrical circuitIn the loop, current i1 is in a clockwise direction while current- i2 is in an anticlockwise direction. Taking clockwise current as positive and anticlockwise current as negative, i1 becomes positive and i2 becomes negative.

Class-12-Physics-Unit-2-Current-Electricity-Chapter-2-Kirchhoffs-Laws-And-Electrical-Measurement-Kirchhoffs-Second-Law-or-Kirchhoffs-voltage-law

Again, the electromotive force of the sources that send currents in clockwise and anticlockwise directions in the closed loop are taken as positive and negative respectively.

So emf e1 becomes negative and emf e2 becomes positive. Hence, for the closed-loop ACBDA,

i1r1-i2r2 = -e1 + e2 ….(2)

We can write by applying Kirchhoff’s first law to the point A

i-i1-i2 = 0 on i2 = i – i1

So, equation (2) can be written as

i1r1 -(i-i1)r2

= –1+ e2

on i1(r1 + r2) = e2– e1 + ir2

or, \(i_1=\frac{\left(e_2-e_1\right)+i r_2}{r_1+r_2}\)….(3)

Knowing the values of die quantities of equation (3) we can determine i1 and i2.

WBCHSE Class 12 Physics Kirchhoff’s Laws And Electrical Measurement Notes

Kirchhoff’s Voltage Law Discussion:

Mesh analysis: A complicated circuit formed by a number of adjacent loops resembles a mesh which can be analyzed one by one by applying Kirchhoff’s second law.

S3 Conservation of energy: We know that inside a source of electricity, other forms of energy are converted into electrical energy, and in an external circuit, electrical energy is converted into other forms of energy.

The amount of electrical energy developed inside a source of electricity to send a one-coulomb charge in a circuit is the electromotive force of the source. Again the amount of electrical energy spent in an external circuit due to the flow of one coulomb charge is the potential difference of the external circuit According to the second law of Kirchhoff,

⇒ \(\Sigma i r=\Sigma e \text { i.e., } \Sigma e=\Sigma v\) [potential difference, v = ir]

i.e., electrical energy developed in any circuit due to unit charge = electrical energy spent in the circuit

So, Kirchhoff’s second law obeys the law of conservation of energy

Kirchhoff’s Laws And Electrical Measurement Kirchhoff’s Laws Numerical Examples

Example 1. Two cells, one of emf 1.2 V and internal resistance 0.5Ω, the other of emf 2 V and internal resistance 0.1Ω are connected in parallel and the combination is connected in series with an external resistance of 5Ω. What is the current through this resistor?
Solution:

Applying Kirchhoff’s first law to point A we have,

I1 + I2 – I = 0

or, I2 = I-I1 …..(1)

Applying Kirchhoff’s second law to the loop ACBDA we have,

-I1 x 0.5 + 12 x 0.1 = -1.2 + 2

-I1 x 0.3 -5-(I-I1) x 0.1 = 0.8

or, 0.1 I – 0.6I1 = 0.8 …….(2)

For the loop ADBFA,

-I2 x 0.1 – I x 5 = -2

or, (I-I1) x 0.1 + 5I = 2….(3)

Class 12 Physics Unit 2 Current Electricity Chapter 2 Kirchhoff’s Laws And Electrical Measurement Example 1 internal and external resistance

Now, multiplying equation (3) by 6 and subtracting equation (2) from it we have,

⇒ \(30.5 I=11.2 \quad \text { or, } I=\frac{11.2}{30.5}=0.367 \mathrm{~A}\)

Example 2. Determine the current flowing through the resistor of resistance 200Ω and the potential difference across its ends.

Class 12 Physics Unit 2 Current Electricity Chapter 2 Kirchhoff’s Laws And Electrical Measurement Example 2 potential differences

Solution:

Applying Kirchhoff’s first law to point B we have,

I-I1-I2 = 0…..(1)

Applying Kirchhoff’s second law to the loop ABCA we have,

100I1 + 200I = 10

or, 10I1 + 20I = 1….(2)

For the loop ADBA,

-200I-150I2 = -30

200I+ 150(I-I1) = 30

350I- 150I1 = 30

or – 15I1 + 35I = 3….(3)

From the equations (2) and (3) we get,

⇒ \(130 I=9 \text { or, } I=\frac{9}{130} \mathrm{~A}=69.2 \mathrm{~mA}\)

⇒ \(V_{A B}=\frac{9}{130} \times 200=\frac{180}{13} V \cdot 13.84\) V

Example 3. Twelve equal wires, each of resistance r ohm, are connected so as to form the frame of a cube. An electric current enters this cube at one corner and leaves from the diagonally opposite corner. Calculate the total resistance between the two corners.
Solution:

Let ADGCHEFB be the frame of the cube formed by twelve equal wires each of resistance r. Let the total current entering at the comer A and leaving the diagonally opposite comer B be 6x.

Class 12 Physics Unit 2 Current Electricity Chapter 2 Kirchhoff’s Laws And Electrical Measurement Example 3 twelve equal wires

Therefore, the 12 wires of the cubic framework (mesh) will have equivalent resistance,

⇒ \(R=\frac{V}{I}=\frac{V}{6 x}\) …..(1)

As the resistance of each wire is the same, the current 6x is divided at A into three equal parts, one along AD, the other along AE and the third along AC.

At points D, E, and C the current is again divided into two equal parts. At points F, G, and H the currents combine so that the current in each of the arms FB, GB, and HB is 2x.

The currents at point B again combine. If V is the potential difference across A and B, then taking the path ACHBA and applying Kirchhoff’s second law we get,

2x.r + x.r + 2x.r = V

or, 5xr = V

From equations (1) and (2) we have,

6xR = 5xr

or, R = \(\frac{5}{6}\)r

Node Voltage Loop Current:

Node voltage: During the analysis of any circular we can consider a potential for every connecting point i.e., node. This is called node voltage. For analysis circuit with the help of node voltage the following rules are adopted:

1. The potential of any connecting point of the circuit may be taken as zero, because for the calculation of current the potential difference between two points Is required whatever may be the Individual potential of the two points. In the potential of C Is taken ns zero.

Class 12 Physics Unit 2 Current Electricity Chapter 2 Kirchhoff’s Laws And Electrical Measurement node voltage

2. If there is no source, of electricity or resistance (or condenser or inductance) between any two points, the potential of the two points is equal.

3. The potential of the negative terminal of a source of electricity is less than that of the positive terminal by the amount equal to the emf of the source. For Example, in the potential of A = V; the potential of B = (V- 5) V.

4. Current through any resistance is determined by applying the formula \(\frac{V}{R}\) = I. For Example, in the current through BC

⇒ \(I=\frac{V_B-V_C}{10}=\frac{(V-5)-0}{10}=\frac{V-5}{10}\)

5. After calculating the currents, Kirchhoff’s first law can be applied at various connecting points.

Loop Current: During the analysis of any circuit a separate current for every closed loop may be considered. This is called loop current.

For analysis of a circuit with the help of loop current the following rules are adopted:

1. Loop current is to be shown with a definite direction. For Example, in current i1 for the loop, ABCD is shown in the clockwise direction

Class 12 Physics Unit 2 Current Electricity Chapter 2 Kirchhoff’s Laws And Electrical Measurement loop current

2. For the calculation of the current flowing in every branch of a loop the current in the adjacent loop is to be taken into account. For Example, in the current through AB, CD, and DA = i1, but current through BC = i1– i2.

3. Next Kirchhoff’s second law can be applied to the loop.

Kirchhoff’s Laws And Electrical Measurement Numerical Examples

Example 1. Determine the current flowing through the resistance of 5Ω as shown In the circuit. 

Class 12 Physics Unit 2 Current Electricity Chapter 2 Kirchhoff’s Laws And Electrical Measurement Example 1 the current flowing through the resistance

Solution:

Potentials of different connecting points are taken in the following way:

VA = V; VB = V-1.2;

VC = V – 2 and VD = 0

Current along \(B D=\frac{(V-1.2)-0}{0.5}=\frac{V-1.2}{0.5} \mathrm{~A}\)

Current along \(C D=\frac{(V-2)-0}{0.1}=\frac{V-2}{0.1} \mathrm{~A}\)

Current along \(A D=\frac{V-0}{5}=\frac{V}{5} \mathrm{~A} .\)

Applying Kirchhoff’s first law to the connecting point D we have,

⇒ \(\frac{V-1.2}{0.5}+\frac{V-2}{0.1}+\frac{V}{5}=0\)

or, \(\frac{10(V-1.2)+50(V-2)+V}{5}=0\)

or, 10V- 12 + 50V- 100 + V = 0

or, \(61 V=112 \text { or, } V=\frac{112}{61} \mathrm{~V}\)

∴ Current through resistance of \(5 \Omega=\frac{V}{5}=\frac{112}{61 \times 5}=0.367 \mathrm{~A}\)

Example 2. The resistances of the ammeter and the voltmeter are 10Ω and 900Ω respectively. What are the readings of the ammeter and the voltmeter?

Class 12 Physics Unit 2 Current Electricity Chapter 2 Kirchhoff’s Laws And Electrical Measurement Example 2 resistances of the ammeter and the volmeter

Solution:

In the circuit, currents flowing in the three loops have been shown as i1, i2, and i3 (clockwise).

From the second law of Kirchhoff’s, we get, for the first loop,

100i1 + 100(i1– i2) = 0

or, 2i1– i2 = 0 …..(1)

for the second loop,

100(i2-i1) + 10i2 + 100(i2– i3) = 12

or, – 100i1 + 210i2 – 100i3 = 12…(2)

fol the third loop,

100(i3 -i2) + 900 i3 = 0.

or, 10i3-i2 = 0 ….(3)

From (1) and (3),

i2 = 2i1 = 10i3

i1 = 5i3

Substituting these values in equation (2) we get,

-100 x 5i3 + 210 x 10i3 – 100i3 = 12

or, 1500i3 = 12

or, \(i_3=\frac{12}{1500}=\frac{4}{500} \mathrm{~A}\)

∴ Reading of voltmeter

= \(i_3 \times 900=\frac{4}{500} \times 900\)

= 7.2 V

and reading of ammeter

= \(i_2=10 i_3=10 \times \frac{4}{500}\)

= 0.08 A

Kirchhoff’s Laws And Electrical Measurement Potential Divider Or Potentiometer

Potential Divider Or Potentiometer Description: A wire of uniform resistance is connected to two binding screws A and B. A slider or jockey J can move along the wire, remaining in contact with it. J is connected with the binding screw C. This arrangement is generally called a potential divider or potentiometer

Class 12 Physics Unit 2 Current Electricity Chapter 2 Kirchhoff’s Laws And Electrical potential divider or potentiometer

Suppose, the length of the resistor AB = L; resistance = R, the

length of the portion AJ = l; its resistance = r.

Since the resistance of the resistor is uniform,

⇒ \(\frac{L}{l}=\frac{R}{r} \quad \text { or, } \quad r=\frac{l}{L} \cdot R\)….(1)

Uses:

Determination of the emf of a cell: Suppose, a source of emf E0 is connected between the two ends A and B of the potentiometer wire of resistance R. The cell whose emf E is to be measured is connected between the points A and C through a galvanometer G. It is to be noted that the positive pole of the cell is connected at A.

Now, suppose that the current in the circuit ACJA due to the effective emf VAC is ix and the current in that circuit due to the cell = i2. These two currents flow in opposite directions.

The slider J is shifted from point to point and placed at a position for which the current through the galvanometer is zero.

Clearly at that condition i1 = i2. If R0 is the total resistance of the circuit ACJA, then,

⇒ \(i_1=\frac{V_{A C}}{R_0} \quad \text { and } i_2=\frac{E}{R_0}\)

So for zero galvanometer current, \(\frac{V_{A C}}{R_0}=\frac{E}{R_0}\)

It is evident that the current l passes through AJ. So,

⇒ \(V_{A J}=\frac{E_0}{R} \cdot \frac{l}{L} \cdot R_i=\frac{l}{L} \cdot E_0 \quad \text { i.e., } V_{A C}=\frac{l}{L} E_0\)….(2) [length of 1 AB = L, length of AJ =l]

or, \(E=V_{A C}=\frac{l}{L} E_0\)…(3)

If we already know the values of E0 and L then by noting the value of l from the position of the slider, the emf E of the cell can be determined from the equation (3).

Comparison of the emfs of two cells: From equation (3) we can write, \(\frac{E}{E_0}=\frac{l}{L}\). So the ratio of the emfs of two cells can be determined with a potentiometer even though the emf of each cell may not be known.

Determination of the internal resistance of a cell: For the length of the potentiometer wire AJ1 = l1 (when the slider is placed at the point J1, if the galvanometer current is zero, according to the equation (3),

⇒ \(E=\frac{l_1}{L} E_0\)

Now, a known resistance R is to be connected parallel to the cell. Under this condition, the potentiometer circuit measures the potential difference V across R. If the galvanometer current is zero for the length of the potentiometer wire AJ2 = l2 (when the slider is placed at J2

⇒ \(V=\frac{l_2}{L} E_0\)…(4)

Class 12 Physics Unit 2 Current Electricity Chapter 2 Kirchhoff’s Laws And Electrical Measurement Determination of the internal resistance of a cell

Therefore, \(\frac{V}{E}=\frac{l_2}{l_1}\left[\text { since, } E=\frac{l_1}{L} E_0\right]\)

Again if r is the internal resistance of the cell, E is the emf and I is the current in the circuit due to the cell, then

E = I(R + r) and V = IR

⇒ \(\text { i.e., } \frac{V}{E}=\frac{R}{R+r}\)

So, \(\frac{l_2}{l_1}=\frac{R}{R+r} \quad \text { or, } R+r=\frac{l_1}{l_2} \cdot R \quad \text { or, } r=\left(\frac{l_1}{l_2}-1\right) R\)

Therefore, to determine the internal resistance r of the cell, we have no need to know the values of L, E0, or E.

Measurement of the potential difference between two points of an electric circuit: The potential difference between any two points of a circuit car i be measured by using a potentiometer in the same way as the emf of a cell is measured.

One of the two points (M and N) of an electrical circuit across which the potential difference is to be measured, is connected to the end A of the potentiometer wire, and the other point is connected to the jockey J through a galvanometer G.

The two poles of the battery of emf E0 are connected to the ends A and B of the potentiometer wire in such a tray that the current i1 in the part AM due to the batten’ of emf E0 and the cu rent i2 due to the external circuit are opposite to each other.

Now changing the position of the jockey J from point to point we observe the position of the jockey. for which the galvanometer current is zero. In this position i1 = i2. Let the total resistance of the circuit MNJA be R0

Class 12 Physics Unit 2 Current Electricity Chapter 2 Kirchhoff’s Laws And Electrical Measurement Measurement of the potential difference between two points of an electric circuit

Then \(\left.i_1=\frac{V_{A J}}{R_0} \text { [length of } A B=L \text {, length of } A J=l\right]\)

and \(i_2=\frac{V_{M N}}{R_0} \quad \text { So, } V_{M N}=V_{A J}=\frac{l}{L} E_0\)….(5)

If the values of E0 and L are known, then by knowing the value of I from the position of J, the value of VMN i.e., the potential difference between M and N can be determined from the equation (5).

Condition of effectiveness: In this arrangement, the potentiometer behaves as a voltmeter. So like any voltmeter, the resistance of the potentiometer should be very high.

So the condition of measurement of potential difference by a potentiometer is that the resistance of the potentiometer wire between the two points A and J must be many times greater than the resistance between the two points M and N of the experimental circuit If it is not so, then as soon as the potentiometer is connected, the equivalent resistance between the points M and N of the experimental circuit will be reduced to a large extent and much error will be found in the measured value of VMN.

Kirchhoff’s Laws And Electrical Measurement Potential Divider Or Potentiometer Numerical Examples

Example 1. The length of the wire of a potentiometer is 100 cm and the emf of a standard cell connected to it is E volt. While measuring the emf of a battery having an Internal resistance of 0.5Ω. the null point is obtained at a length of 30 cm. Determine the emf of the battery.
Solution:

When the annuli point is obtained, no .current flows through the battery. So there is no internal potential drop. In this case, the internal resistance of the battery has no influence. So if E’ is the emf of the battery, then

⇒ \(\frac{E}{100}=\frac{E^{\prime}}{30} \quad \text { or, } \frac{E}{E^{\prime}}=\frac{10}{3}\)

or, E’ = 0.3E

Example 2. A potential difference of 220 V is applied at the two ends of a rheostat of 12000Ω. A voltmeter of resistance 6000 fl is connected between points A and D. If point D divides AB in the ratio of 1:4 what will be the reading of the voltmeter?

Class 12 Physics Unit 2 Current Electricity Chapter 2 Kirchhoff’s Laws And Electrical Measurement Example 2 A potential difference

Solution:

⇒ \(\frac{A D}{B D}=\frac{1}{4} \quad \text { or, } \frac{A D+B D}{B D}=\frac{1+4}{4} \text { or, } \frac{A B}{B D}=\frac{5}{4}\)

or, \(B D=\frac{4}{5} A B=\frac{4}{5} \times 12000=9600 \Omega\)

∴ AD = AB – BD

= 12000-9600

= 2400Ω

If the voltmeter is connected between A and D, the equivalent resistance of the circuit

= \(9600+\frac{2400 \times 6000}{2400+6000}\)

= \(9600+\frac{24 \times 6000}{84}\)

⇒ \(\left(9600+\frac{2 \times 6000}{7}\right) \Omega\)

∴ Current \(I=\frac{220}{9600+\frac{2 \times 6000}{7}} \mathrm{~A}\)

∴ \(V_{B D}=I \times B D=\frac{220}{9600+\frac{2 \times 6000}{7}} \times 9600\)

⇒ \(\frac{220}{1+\frac{2 \times 6000}{7 \times 9600}}=\frac{220}{1+\frac{5}{28}}=\frac{220 \times 28}{33}=\frac{560}{3} \mathrm{~V}\)

⇒ \(V_{A D}=220-\frac{560}{3}=\frac{100}{3}\)

= 33.3V

Since the voltmeter is connected between points A and D, its reading will be 33.3 V.

Example 3. In a potentiometric arrangement, a cell is connected to the potentiometer wire of 60 cm in length to make the 1 deflection zero in the galvanometer. Now if the cell is shunted with a 6Ω resistor, a null point is found in the 50 cm length of the wire. What is the internal resistance of the cell?
Solution:

Here, shunt R = 6Ω, l1 = 60cm l2 = 50cm.

∴ Internal resistance of the cell

⇒ \(r=\left(\frac{l_1}{l_2}-1\right) R\)

= \(\left(\frac{60}{50}-1\right) \times 6\)

= \(0.2 \times 6\)

= \(1.2 \Omega\)

Example 4. A length of potentiometer wire of 188 cm balances the emf of a cell In the n circuit and its length of 135 cm when the cell tins a conductor of resistance 8Ω connected between Its terminals. Find the Internal resistance of
Solution:

Let the emf of the cell = E, and Its internal resistance =r. When a resistance R Is connected between Its terminals (here, R = 8Ω), the coll is subjected to an internal drop of potential Then, the terminal potential difference, the cell.

V=E-Ir

= \(E-\frac{E r}{R+r}\)

= \(E\left(1-\frac{r}{R+r}\right)\)

= \(E \frac{R}{R+r}\)

From the given data,

⇒ \(\frac{E}{V}=\frac{155}{135} \quad\)

or, \(\frac{R+r}{R}=\frac{155}{135} \quad\)

or, \(1+\frac{r}{R}=\frac{155}{135}\)

∴ \(r=R\left(\frac{155}{135}-1\right)=8 \times \frac{20}{135}\)

= 1.185Ω

Kirchhoff’s Laws And Electrical Measurement Wheatstone Bridge

Description: Four resistances P, Q, S, and R form the four arms AB, BC, CD, and DA respectively of a quadrilateral ARCD.

Class 12 Physics Unit 2 Current Electricity Chapter 2 Kirchhoff’s Laws And Electrical Measurement Wheatstone bridge

A battery B Is connected between A and C and a galvanometer of resistance G Is connected between B and D, This complete arrangement Is called a Wheatstone bridge circuit. In this circuit the first arm AB has resistance P, the second arm HC has resistance Q, the third arm AD has resistance it, and the fourth arm DC has resistance S.

Working principle: The resistances P, Q, R, and S for the four arms of the bridge are so selected that the galvanometer deflection is zero i.e., galvanometer current IG = 0. This Is called the null condition or balanced condition of the bridge.

In the null condition, the relation of the resistances is:

⇒ \(\frac{P}{Q}\) = \(\frac{R}{S}\) ….(1)

Of the four resistances, if three are known, the fourth resistance can be determined from the above relation. Usually, the unknown resistance is placed in the fourth arm i.e.,.in place of S.

Proof of the relation P/Q = R/S: In the null condition, IG = 0. So, the potential difference between the two ends of the galvanometer,

VB – VD = IG.G = 0 [G = resistance of galvanometer]

or, VB = VD

For IG = 0, suppose current in resistance P = current in resistance Q = I1 and current resistance R = current resistance S = I2.

∴ VA-VB = I1P; VA-VD = I2R

and VB-VC = I1Q; VD-VC = I2S

Now, Since VB = VD,

∴ I1P = I2R…(2)

and I1Q = I2S….(3)

Now, by dividing equation (2) by equation (3) we have,

⇒ \(\frac{P}{Q}\) = \(\frac{R}{S}\)

On the basis of this relation, the arms P and Q are called ratio arms.

Discussion:

1. Wheatstone bridge is used for measurement of ordinary resistances (from 1Ω to about 1000Ω).

2. If a low resistance (less than 1Ω) is placed in the position of S, the resistances of the connecting wires become nearly equal to S. So the value of S becomes erroneous due to the resistance of the connecting wires. Therefore, for measurements of low resistances Wheatstone bridge is not used.

3. Its high resistance (higher than 1000Ω) Is placed In the position of .S’, and a very small current will pass through it. So in the null condition, the value of I1 Is much greater than I2 he., almost the entire current passes through P and Q. In that case, even when the bridge Is unbalanced, no appreciable current flows through the galvanometer, and the galvanometer deflection stays at zero.

Then, It becomes difficult to Identify the actual null condition. In other words, the sensitivity of the bridge decreases. So, Wheatstone bridge Is not used for measurement of high resistances.

4. If the bridge Is sensitive enough, even In slightly off-balance conditions the galvanometer deflection is visible distinctly.
The conditions of sensitiveness of this bridge are:

  1. The galvanometer must be very sensitive. Even for the passage of a very feeble current through the galvanometer, the deflection of the pointer should be distinct.
  2. The four resistances P, Q, R, and S as well as the resistance of the galvanometer G should be of almost equal value i.e., P ≈ Q ≈ R ≈ S ≈ G.

It is to be noted that, the null condition does not depend on galvanometer resistance G, but the sensitivity of the bridge relies on G. Current can flow through the galvanometer easily if G has a lesser value.

On the other hand, if the number of turns of the coll of the galvanometer is increased, the pointer of the galvanometer deflects more, and G also increases. So a galvanometer is designed for optimum sensitivity.

Various types of galvanometers are available based on which circuit it is connected. In the Wheatstone bridge, if galvanometer resistances are nearly equal to the resistances of four arms i.e., P ≈ Q ≈ R ≈ S ≈ G bridge becomes most sensitive.

5. A galvanometer is a very sensitive instrument for the detection and measurement of current. While making adjustments for the balanced condition of the bridge, there is a possibility of high current passing through the galvanometer. Thus the galvanometer may get damaged. For this reason, in the beginning, it is judicious

  1. To connect a high resistance in series with the galvanometer, or
  2. To connect a shunt in parallel with the galvanometer.

Later when the bridge comes close to the balanced condition,

  1. The resistance can be removed or a shunt can be added to it, or
  2. The shunt which is connected with the galvanometer may be removed.

6. The balanced condition does not depend on the emf of the battery, its internal resistance, or any resistance connected with the battery. However, generally, a cell of low emf (e.g., Leclanche cell) Is used. Otherwise, due to the flow of large currents, the resistances of P, Q, R, and S may increase due to the heating effect.

7. If the position of the battery and the galvanometer interchanged, the battery exists between B and D and the galvanometer between A and C. Then Q, S, P, and R become the first, second, third, and fourth arms respectively of the bridge. So in this case the null condition is

⇒ \(\frac{Q}{S}=\frac{P}{R}\).

Obviously, this condition is identical to the condition \(\frac{P}{Q}=\frac{R}{S}\). So it can be said that the balanced condition of the Wheatstone bridge remains unchanged in spite of the interchange of the positions of the battery and the galvanometer. For this reason, the diagonals AC and BD of the bridge are called mutually conjugate.

8. The sensitivity of the bridge is not the same in the two mutually conjugate positions. The bridge becomes more sensitive if the galvanometer is connected between the junction of the two smaller resistances and the junction of the two larger resistances.

9. When the equivalent resistance of the bridge is calculated in a balanced condition, the resistance of the galvanometer i.e., the resistance of the arm BD should not be taken into consideration.

Practical applications of Wheatstone bridge: For measurement of various electrical quantities in the laboratory, the Wheatstone bridge and some of its modified forms are extensively used. Two familiar forms of Wheatstone bridge for measurement of resistances in the laboratory are

  1. Post office box and
  2. Metre bridge. (Discussion on post office box is beyond our present syllabus.)

Kirchhoff’s Laws And Electrical Measurement Wheatstone Bridge Numerical Examples

Example 1. Resistances of the ratio arms of a Wheatstone bridge are 100Ω and 10Ω respectively. An unknown resistance is placed in the fourth arm and the galvanometer current becomes zero when 153Ω resistance is placed in the third arm. What is the value of unknown resistance?
Solution:

Here, P = 100Ω, Q = 10Ω and R = 153Ω

Since, \(\frac{P}{Q}=\frac{R}{S} \quad\)

∴ \(S=\frac{R Q}{P}=\frac{153 \times 10}{100}, 15.3 \Omega\)

Example 2. Five resistances are connected. What is the effective resistance between points A and B?

Class 12 Physics Unit 2 Current Electricity Chapter 2 Kirchhoff’s Laws And Electrical Measurement Example 2 five resistances are connected

Solution:

⇒ \(\text { Here, } \frac{2 \Omega}{3 \Omega}=\frac{4 \Omega}{6 \Omega}\)

So, the circuit between points A and B is a balanced Wheatstone bridge. So no current flows through the resistance of 7Ω. In this condition to calculate the effective resistance, the resistance of 7Ω will be ignored.

Equivalent resistance of 2Ω and 3Ω = 2 + 3 = 5Ω

Equivalent resistance of 4Ω and 6Ω = 4 + 6 = 10Ω

Now at points A and B these 5Ω and 10Ω resistances are connected in parallel.

So, the effective resistance = \(\frac{5 \times 10}{5+10}=\frac{50}{15}=\frac{10}{3} \Omega\)

Example 3. In a Wheatstone bridge, a resistance and an unknown resistance are placed in the third arm and fourth arm respectively. Current through galvanometer becomes zero when the ratio of resistances of first and second arms is 3: 2. Find the value of unknown resistance.
Solution:

⇒ \(\text { Here, } R=15 \Omega \text { and } \frac{P}{Q}=\frac{3}{2}\)

since \(\frac{P}{Q}=\frac{R}{S} \quad\)

∴ \( S=R \cdot \frac{Q}{P}=15 \times \frac{2}{3}=10 \Omega\)

Example 4. The resistances of the four arms of a Wheatstone bridge are 100Ω, 10Ω, 300Ω, and 30Ω respectively. A battery of emf 1.5 V and negligible internal resistance is connected to the bridge. Calculate the current flowing through each resistance.
Solution:

Here, P = 100Ω,

Q = 10Ω, R = 300Ω, S = 30Ω and E = 1.5 V

Since, \(\frac{100}{10}=\frac{300}{30}\)

i.e., \(\frac{P}{Q}=\frac{R}{S}\), the bridge Is in a balanced condition.

∴ Current in the resistance P = current in the resistance Q

⇒ \(\frac{V_{A B}}{P+Q}=\frac{1.5}{100+10}=\frac{1.5}{110}\)

= 0.0136A

Again, current in the resistance R = current in the resistance S

⇒ \(\frac{V_{A B}}{R+S}=\frac{1.5}{300+30}=\frac{1.5}{330}\)

= 0.0045A

Class 12 Physics Unit 2 Current Electricity Chapter 2 Kirchhoff’s Laws And Electrical Measurement Example 4 the resistances of the four arms

Example 5. Every resistance is of magnitude r. What is the equivalent resistance between A and B?

Class 12 Physics Unit 2 Current Electricity Chapter 2 Kirchhoff’s Laws And Electrical Measurement Example 5 equivalent resistance between A and B

Solution:

The Wheatstone bridge circuit is the equivalent circuit.

Here, P = Q- R = S = r

∴ \(\frac{P}{Q}=\frac{R}{S}\)

i.e., the circuit is in a balanced condition. So for the calculation of equivalent resistance, the value of CD is of no use.

Now, resistance along the path ADB = r+ r = 2r and resistance along the path ACB = r+ r = 2r

Since ADB and ACB are connected in parallel, the equivalent resistance between the points A and B is

⇒ \(R^{\prime}=\frac{2 r \times 2 r}{2 r+2 r}=\frac{4 r^2}{4 r}=r\)

Class 12 Physics Unit 2 Current Electricity Chapter 2 Kirchhoff’s Laws And Electrical Measurement Example 5 equivalent resistance between A and B.

Example 6. Each resistance in the given circuit in is of value R. Calculate the equivalent resistance of the circuit with respect to the points A and JB.

Class 12 Physics Unit 2 Current Electricity Chapter 2 Kirchhoff’s Laws And Electrical Measurement Example 6 equivalent resistance of the circuit

Solution:

The equivalent circuit.

Now proceeding in the same way as done in Example 1, it can be shown that equivalent resistance =R.

Class 12 Physics Unit 2 Current Electricity Chapter 2 Kirchhoff’s Laws And Electrical Measurement Example 6 equivalent resistance of the circuit.

Example 7. A coil of wire is kept in melting ice. Its resistance measured by a Wheatstone bridge is 5Ω. If the coil is heated to I00°C and another wire of resistance 100Ω is connected in parallel to it, the balanced condition of the bridge remains unchanged. Determine the temperature coefficient of resistance of the wire of the coil.
Solution:

If the resistance of the coil at 100°C is R, then the equivalent resistance of the parallel combination of R and 100Ω is obviously 5Ω

∴ \(\frac{R \times 100}{R+100}=5 \quad\)

or, 95 R=500

or, \(R=\frac{100}{19} \Omega\)

Again, if or be the temperature coefficient of resistance of the wire of the coil, then

R = R0(l + αt)

or, \(\alpha=\frac{\frac{R}{R_0}-1}{t}=\frac{\frac{100}{19 \times 5}-1}{100}=\frac{5}{9500}\)

= 5.26 x 10-4 °C-1

Example 8. An electrical circuit. Calculate the potential difference across the resistance 400Ω as will be measured by the voltmeter V of resistance 400Ω.

Class 12 Physics Unit 2 Current Electricity Chapter 2 Kirchhoff’s Laws And Electrical Measurement Example 8 electrical circuit

Solution:

The equivalent circuit.

The equivalent resistance between B and C = \(\frac{400 \times 400}{400+400}\)

= 200Ω

This is a Wheatstone bridge circuit, where

⇒ \(\frac{\text { resistance of the side } A B(P)}{\text { resistance of the side } B C(Q)}=\frac{100}{200}\)

= \(\frac{\text { resistance of side } A D(R)}{\text { resistance of side } D C(S)}\)

Class 12 Physics Unit 2 Current Electricity Chapter 2 Kirchhoff’s Laws And Electrical Measurement Example 8 electrical circuit.

So, the bridge is in a balanced condition, i.e., no current flows in the arm BD. Under this condition the reading of the voltmeter, V is

⇒ \(V_B-V_C=I Q=\frac{V_A-V_C}{P+Q} \times Q\)

⇒ \(\frac{10}{100+200} \times 200=\frac{20}{3}\)

= 6.67V

Example 9. ABCD is a Wheatstone bridge in which the resistance of the arms AB, BC, CD, and DA are respectively 2Ω,4Ω,6Ω, and 8&&. Points A and C are connected to the terminals of a cell of emf 2 V and have negligible internal resistance. Points B and D are connected to a galvanometer of resistance 50Ω. Using KirchhofFs laws find the current flowing through the galvanometer
Solution:

In the circuit of let VC = 0; then VA = 2V;

again, if VB = V1 and VB-VD = V, then VD=V1-V

Now, applying Kirchhoff’s 1st law at junction B, we get

⇒ \(\frac{V_1-2}{2}+\frac{V_1}{4}+\frac{V}{50}=0\)

or, \(\frac{50 V_1-100+25 V_1+2 V}{100}=0\)

or, 75V1 + 2V = 100…..(1)

Again applying Kirchhoff’s 1st law at junction D, we get

⇒ \(\frac{V_1-V-2}{8}+\frac{V_1-V}{6}+\frac{-V}{50}=0\)

or, \(\frac{75 V_1-75 V-150+100 V_1-100 V-12 V}{600}=0\)

or, 175V1 – 187V = 150…(2)

Doing (1) x 7- (2) X 3 we get

575 V = 250

or, \(V=\frac{250}{575} \mathrm{~V}\)

∴ \(I_G=\frac{V}{50}=\frac{250}{575 \times 50}=\frac{1}{115}\)

= 0.0087A

= 8.7mA

Class 12 Physics Unit 2 Current Electricity Chapter 2 Kirchhoff’s Laws And Electrical Measurement Example 9 wheatstone bridge

Example 10. We determine the value of the fourth resistance to be 8Ω with the help of a Wheatstone bridge with three known resistances 100Ω, 10Ω, and 80Ω respectively. If the emf of the cell and its internal resistance are 2 V and 1.1Ω respectively, find the current passing through the cell.
Solution:

The bridge is in a balanced condition because \(\frac{100}{10}=\frac{80}{8}\) galvanometer current is zero. In this case, the effective circuit. Total resistance of the circuit

⇒ \(R=1.1+\frac{(100+10)(80+8)}{(100+10)+(80+8)}\)

= \(1.1+\frac{110 \times 88}{198}\)

= 50Ω

Class 12 Physics Unit 2 Current Electricity Chapter 2 Kirchhoff’s Laws And Electrical Measurement Example 10 wheatstone bridge with three known resistances

∴ Current through the cell, i.e., the main current of the circuit,

I = \(\frac{2}{50}\) = 0.04A

Metre Bridge:

It is another practical form of the Wheatstone Bridge. It also helps in measuring ordinary resistances very easily.

From the null condition of Wheatstone bridge, we have,

⇒ \(S=R \cdot \frac{Q}{P}\)…(1)

  1. In the meter bridge, the steps that are followed are mentioned below.
  2. The unknown resistance is placed in the fourth arm S.
  3. The magnitude of R, i.e., the resistance of the third arm is
    kept constant.
  4. By increasing or decreasing the ratio \(\frac{Q}{P}\), the bridge is brought to a balanced condition.
  5. Now by using equation (1) the unknown resistance S is calculated.

Description:

The meter bridge circuit. It consists of a one-metre-long thin uniform wire AB made of manganin or German silver. The wire is stretched and fixed to the two points a and f of two copper strips ab and fe and is placed along a meter scale M over a wooden board. There is another copper strip cd.

The gaps be and de is called the left gap and the right gap, respectively. A jockey J is connected to the midpoint of the copper strip cd through a galvanometer G and its sharp end touches the wire AB and can be moved along it when required.

A resistance box R is inserted in the left gap (third arm of the bridge). The resistance S to be determined is introduced in the right gap (fourth arm of the bridge). A cell E is connected to the bridge through a commutator C.

Working principle: An equivalent meter bridge circuit. Obviously, it is a Wheatstone bridge circuit.

Class 12 Physics Unit 2 Current Electricity Chapter 2 Kirchhoff’s Laws And Electrical Measurement Working principle

The direction of current through the wire AB can be reversed with the help of the commutator C. The left portion of the wire with respect to the position of the jockey is the first arm of the bridge and the right portion of the wire is the second arm, i.e., resistance of the portion AJ of the wire =P and resistance of the portion JB, of the wire = Q.

A suitable resistance R is introduced in ‘the resistance Now the moved along the wire till a null point is reached. At this balanced condition, let the length of AJ =l cm.

Length of JB = (100-l) cm [∵ AB = 1 m = 100 cm]

If ρ is the resistance per unit length of the wire then,

⇒ \(P=\rho l \text { and } Q=\rho(100-l)\)

∴ \(\frac{Q}{P}=\frac{100-l}{l} \text { and } S=R \cdot \frac{Q}{P}=R \cdot \frac{100-l}{l}\)…..(2)

Knowing l from the scale, S can be calculated from the equation (2). Taking different resistances from box R the experience is repeated.

Discussion:

Material of the wire: The wire of the meter bridge is generally made of alloys like manganin, German silver, etc. The resistance of a uniform manganin wire of length 1 m and diameter 0.5 mm is about 2Ω. The bridge becomes sensitive enough for this magnitude of resistance of the wire. On the other hand, the resistance of a copper wire of the same length and diameter is about 0.1Ω. The bridge is not at all sensitive if such a low resistance is used. For this reason, copper wire is never used in a meter bridge.

Thermoelectric effect:

The copper strips at the ends A and B of the wire come in contact with manganin or German silver. Now, if current passes through the junctions of two dissimilar metals, heat is evolved at one junction and is absorbed at the other due to the thermoelectric effect.

So, the wire AB does not remain in thermal equilibrium and hence its resistance per unit length, ρ is not uniform. Therefore, when we apply equation (2) to calculate the unknown resistance, an error occurs.

To remove this deficit is necessary to reverse the direction of the current through the wire AB. Then, heat will be absorbed at the end where it evolved originally and vice versa.

In this way, the thermal equilibrium of the wire AB will be maintained. To reverse the direction of current in the wire AB, a commutator C is used.

End error:

Though the resistance of the copper strips is very small, it is finitely non-zero.

The bridge wire is soldered at the two ends A and B with the copper strips. So some resistance exists in these two places.

The 0 and 100 cm marks of the meter stale (M) may not coincide accurately with the ends ll and B of the bridge wire. Hence it is assumed that some additional resistance exists at the two ends of the bridge wire.

This is called end resistance. So when we measure an unknown resistance, we face some end errors.

End correction: The end resistance may be assumed to be equivalent to the resistance of a certain length of the bridge wire. Suppose, the end resistance of end A = resistance of Ax cm of the bridge wire and end resistance of end B = resistance of Az cm of the bridge wire

i.e., resistance of l1 length of the bridge wire = resistance of (l1 + λ1) length and resistance of l2 length of the bridge wire = resistance of (l2 + λ2) length

In the position of the null point be l1 cm then we have from equation (2),

⇒ \(\frac{S}{R}=\frac{\left(100-l_1\right)+\lambda_2}{l_1+\lambda_1}\)…(3)

Now, by interchanging the position of R and S, if the null point is obtained at l2 cm, then we have

⇒ \(\frac{R}{S}=\frac{\left(100-l_2\right)+\lambda_2}{l_2+\lambda_1}\) …..(4)

If we use two known resistances R and S, we can find out λ1 and λ2 by solving equations (3) and (4). Next, using the values of λ1 and λ2, the unknown resistance can be measured accurately.

Position of the null point: The bridge becomes sensitive if the position of the null point lies between 40 cm and 60 cm of the bridge wire. But it is better to avoid the null point at 50 cm because in that case no change is observed in the reading of the null point on interchanging the positions of R and S.

Kirchhoff’s Laws And Electrical Measurement Numerical Examples

Example 1. In a meter bridge experiment, a null point Is obtained at a length of 39.8 cm when a 2Ω resistance Is placed In the left gap and a 3Ω resistance in the right gap. If the two resistances are interchanged, the null point is obtained at 60.8 cm. Calculate the end errors of the bridge.
Solution:

Suppose, the resistance per unit length of the bridge wire = ρΩ.cm-1. End resistance of the left end of the bridge

= resistance λ1 of cm of the wire. End resistance of the right end of the bridge

= resistance of λ2 cm. of the wire. These two are the end errors of the bridge.

Therefore, iΩ be the position of the null point, then according to the relation \(\frac{P}{Q}=\frac{R}{S}\) we have,

⇒ \(\frac{R}{S}=\frac{\left(l+\lambda_1\right) \rho}{\left(100-l+\lambda_2\right) \rho}=\frac{l+\lambda_1}{100-l+\lambda_2}\)

In the first case,

R = 2Ω,S = 3Ω and l = 39.8 cm

∴ \(\frac{2}{3}=\frac{39.8+\lambda_1}{60.2+\lambda_2}\)

or, 3λ1 – 2λ2 = 1 ….(1)

Again, in the second case,

R = 3Ω,S = 2Ω and l = 60.8 cm

∴ \(\frac{3}{2}=\frac{60.8+\lambda_1}{39.2+\lambda_2}\)

or, 2λ1 – 3λ2 = -4 …(2)

Solving (1) and (2) we get,

λ1 = 2.2 and λ2 = 2.8

So, the left-end resistance and the right-end resistance of the bridge are equal to the resistances of 2.2 cm and 2.8 cm of the bridge wire respectively

Example 2. In the left gap of a meter bridge, there is a coil of copper and in the right gap, there is a fixed resistance. If the coil of copper is dipped in ice the balance point is obtained at 41.2 cm of the bridge wire. Next, the coil is taken off from ice and placed in a container of hot water. Now the balance point is shifted by a distance of 8.1 cm towards the right. What is the temperature of hot water? (Temperature coefficient of resistance of copper = 42.5 x 10-4 °C-1.)
Solution:

Suppose, fixed-resistance = R, the temperature coefficient of resistance of copper = aaa, the resistance of the coil at 0°C = R0, the position of the balance point = l0, the resistance of the coil at t°C = Rt, and the position of the balance point =l.

∴ Rt = R0(l + at)

According to the principle of the meter bridge,

In case of \(0^{\circ} \mathrm{C}, \frac{R_0}{R}=\frac{l_0}{100-l_0}\)….(1)

In case of \(t^{\circ} \mathrm{C}, \frac{R_0(1+\alpha t)}{R}=\frac{197}{100-l}\) …(2)

Dividing (2) by (1) we have,

⇒ \(1+\alpha t=\frac{l}{l_0} \times \frac{100-l_0}{100-l}\)

⇒ \(\frac{49.3}{41.2} \times \frac{100-41.2}{100-49.3} \cdot[l=41.2+8.1=49.3]\)

⇒ \(\frac{49.3}{41.2} \times \frac{58.8}{50.7}=1.388\)

∴ \(t^t=\frac{1.388-1}{\alpha}=\frac{0.388}{42.5 \times 10^{-4}}\)

= 91.3°C

Example 3. In a meter bridge, the balance point is found to be at 40 cm from one end when the resistor at the end is 15Ω. Find the resistance on the other side.
Solution:

If p be the resistance per 1 cm length of the metre wire, then in the present case,

P = 40ρΩ, Q = (100-40)ρ = 60ρΩ, R = 15Ω,

S =?

In balance condition \(\frac{P}{Q}=\frac{R}{S}\)

or, \(S=R \frac{Q}{P}=15 \times \frac{60 \rho}{40 \rho}\)

= 22.5Ω

Example 4. In the circuit given, E1 = 6 V, E2 = 2 V, E3 = 3 V, C’ = 5μF, = 2R2 = 6Ω, R3 = 2R4 = 4Ω. Find the current in R3 and the energy stored in the capacitor

Class 12 Physics Unit 2 Current Electricity Chapter 2 Kirchhoff’s Laws And Electrical Measurement Example 4 energy stored in the capacitor

Solution:

Let the distribution of currents in the various branches be as shown below.

Class 12 Physics Unit 2 Current Electricity Chapter 2 Kirchhoff’s Laws And Electrical Measurement Example 4 energy stored in the capacitor.

Considering the closed loop CDEHC

⇒ \(I_2 R_3=E_1 \quad \text { or, } 4 I_2=6 \quad \text { or, } I_2=1.5 \mathrm{~A}\)

∴ Current through the resistance = 1.5 A

Considering the closed-loop AHEFGA,

⇒ \(\left(I_1-I_2\right) \times R_2-I_2 R_3+\left(I_1-I_2\right) R_4=-E_2-E_3\)

or, \(\left(I_1-1.5\right) \times 3-1.5 \times 4+\left(I_1-1.5\right) \times 2=-2-3\)

∴ I1 = 1.7A

Let VA and VC be the potentials at points A and C respectively, As the current flows from point A to C along the path AHC,

VA = VC + potential drop across R2 + E2

or, VA-VC= (I1-I2)R2+E2

= (1.7- 1.5) X 3+ 2

∴ VA-VC = 2.6 V

Through branch ABC current = 0. Hence the potential difference across the capacitor is equal to the potential difference between the points A and C.

∴ Potential differences across the capacitor,

VA-VC = 2.6V

∴ Energy storedin the capacitor,

⇒ \(U=\frac{1}{2} C V^2=\frac{1}{2} \times 5 \times 10^{-6} \times(2.6)^2\)

= 1.69 x 10-5 J

Example 5. If the resistance X and Y. (X < Y) are placed in the two gaps of a meter bridge, a null point is obtained at a length of 20 cm. Keeping Y unchanged if a resistance 4X is placed in place of X what will be the position of the null point?
Solution:

In the first case, \(\frac{X}{Y}=\frac{20}{100-20}\)

or, Y = 4X

So, if resistance 4X is placed in place of X, the resistances of the two gaps will be equal, since Y = 4X.

Therefore, the null point will be in the middle of the bridge wire i.e., at 50 cm

Example 6. The distance between the positions of two null points obtained in a meter bridge wire of length 100 cm, by interchanging a known resistance of 2.5Ω and an unknown resistance in the two gaps, is 28.6 cm. Find the value of the unknown resistance.
Solution:

In the first case, if the unknown resistance r is kept at the

left gap and resistance of 2.5Ω at the right gap, the position of null point l1.

∴ \(\frac{r}{2.5}=\frac{l_1}{100-l_1}\)…(1)

In the second case, after interchanging the resistances, the position of the null point l2

∴ \(\frac{2.5}{r}=\frac{l_2}{100-l_2}\)…(2)

According to the question,

l1-l2 = 28.6…(3)

By (1) x (2) we get,

⇒ \(1=\frac{l_1 l_2}{\left(100-l_1\right)\left(100-l_2\right)}\)

or, 104 – 100l1 – 100l2 + l1l2 = l1l2

or, l1 + l2 = 100….(4)

Solving equation (3) and (4),

l1 = 643

and l2 = 100-l1 = 35.7

∴ The value of the unknown resistance

⇒ \(r=2.5 \times \frac{l_1}{100-l_1}=2.5 \times \frac{64.3}{100-64.3}=4.5 \Omega\)

Kirchhoff’s Laws And Electrical Measurement Very Short Questions and Answers

Question 1. In which part of an electrical circuit can Kirchhoff’s law of
Answer: At the meeting points

Question 2. In which part of an electrical circuit can Kirchhoff’s law of voltage be applied?
Answer: At The Closed Loops

Question 3. If the potential of point C be 12 V, what will be the potential of the point D?

Class 12 Physics Unit 2 Current Electricity Chapter 2 Kirchhoff’s Laws And Electrical Measurement potential of the point C and D (2)

Answer: 3V

Question 4. At a junction of three wires, the inward currents through two of the wires are 1 A and 2 A. What is the inward current through the third wire?
Answer: -3 A

Question 5. AB = 1 m. What will be the length of the part AC for the null condition of the galvanometer?

Class 12 Physics Unit 2 Current Electricity Chapter 2 Kirchhoff’s Laws And Electrical Measurement null condition of the galvanometer

Answer: 75cm

Question 6. Why do we prefer a potentiometer with a longer wire?
Answer: Proportional error in the recorded reading will be less

Question 7. What kind of cell should be used in a Wheatstone bridge circuit?
Answer: Leclanche cell

Question 8. Of battery and galvanometer segments which one should be closed first during the operation of a Wheatstone bridge circuit?
Answer: Battery

Question 9. The resistance of each of the four arms of a Wheatstone bridge is 10Ω and the resistance of the galvanometer is 500Ω. What is the equivalent resistance of the combination?
Answer: 10Ω

Question 10. The resistances in the left and the right gaps of a metre bridge circuit are 3Ω and 2Ω respectively. For what length of the bridge wire IN the null point found If there In no end error?
Answer: 60cm

Question 11. Keeping a resistance of 2Ω In the left gap of a metre bridge, an unknown resistance Is placed In the right gap and the null point is obtained at a distance of 40.0 cm. If no end error Is present, what will be the value of the unknown resistance?
Answer:

Kirchhoff’s Laws And Electrical Measurement Fill In The Blanks

1. Kirchhoff’s law of current expresses the principle of conservation of charge

2. With the help of a potentiometer, the emf of a cell can be measured accurately, in this experiment lost volt of the cell becomes zero

3. A battery of steady emf 2.0 V is connected across the two ends of a potentiometer wire. With the help of this arrangement, the emf of more than 2V of a cell can not be determined.

Kirchhoff’s Laws And Electrical Measurement Assertion Reason Type

Direction: These questions have Statement 1 and Statement 2. Of the four choices given below, choose the one that best describes the two statements.

  1. Statement 1 is true, Statement 2 is true; Statement 2 is a correct explanation for Statement 1.
  2. Statement 1 is true, Statement 2 is true; Statement 2 is not a correct explanation for Statement 1.
  3. Statement 1 is true, Statement 2 is false.
  4. Statement 1 is false, and Statement 2 is true.

Question 1.

Statement 1: A balance is obtained at the position of 40 cm on the meter wire, when 2Ω and 3Ω resistances are put in the left and right gaps respectively, of a meter bridge.

Statement 2: The balanced condition of a Wheatstone bridge is P/Q = R/S.

Answer: 1. Statement 1 is true, Statement 2 is true; Statement 2 is a correct explanation for Statement 1.

Question 2.

Statement 1: An alloy like manganin or German silver is used, instead of copper, as the material of the wire of a meter bridge.

Statement 2: The temperature coefficient of resistance is very low for alloys.

Answer: 2. Statement 1 is true, Statement 2 is true; Statement 2 is not a correct explanation for Statement 1.

Question 3.

Statement 1: A potentiometer arrangement is more suitable than a voltmeter arrangement for the accurate measurement of the emf of an electric cell.

Statement 2: It is possible to connect an electric cell in a potentiometer circuit in such a way that no current passes through the cell.

Answer: 1. Statement 1 is true, Statement 2 is true; Statement 2 is a correct explanation for Statement 1.

Question 4.

Statement 1: If the resistances of the first two arms P and Q of a balanced Wheatstone bridge are exchanged, the balanced condition is not disturbed.

Statement 2: The balanced condition of a Wheatstone bridge is independent of the resistance of the galvanometer used.

Answer: 4. Statement 1 is false, Statement 2 is true.

Question 5.

Statement 1: Kirchhoff’s voltage law indicates that the static field is conservative.

Statement 2: The potential difference between two points in a circuit does not depend on the path.

Answer: 1. Statement 1 is true, Statement 2 is true; Statement 2 is a correct explanation for Statement 1.

Question 6.

Statement 1: In the balanced Wheatstone bridge,

⇒ \(R_{A C}=\frac{(P+Q)(R+S)}{(P+Q+R+S)}\)

Class 12 Physics Unit 2 Current Electricity Chapter 2 Kirchhoff’s Laws And Electrical Measurement balanced Wheatstone bridge

Statement 2: This is because B and D are at the same potential.

Answer: 2. Statement 1 is true, Statement 2 is true; Statement 2 is not a correct explanation for Statement 1.

Question 7.

Statement 1: In an electrical circuit the algebraic sum of currents meeting at a point is zero.

Statement 2: In the case of the flow of current in an electrical circuit total energy is conserved.

Answer: 2. Statement 1 is true, Statement 2 is true; Statement 2 is not a correct explanation for Statement 1.

Kirchhoff’s Laws And Electrical Measurement Match The Columns

Question 1. A balance is obtained at 25 cm of a meter bridge when resistances X and Y are placed in the left and the right gaps respectively. Then, during repetitions of the experiment, the left and the right gaps are filled respectively, with the resistances of column A. Column B shows the positions of, the balance points

Class 12 Physics Unit 2 Current Electricity Chapter 2 Kirchhoff’s Laws And Electrical Measurement Match the columns 1

Answer: 1-D, 2-B, 3-A, 4-C

Question 2. Match the following two columns to specify the instrument to be used for the given electrical purposes

Class 12 Physics Unit 2 Current Electricity Chapter 2 Kirchhoff’s Laws And Electrical Measurement Match the columns 2

Answer: 1-B, 2-D, 3-A, 4-C

WBCHSE Class 12 Physics Capacitance And Capacitor Notes

Capacitance And Capacitor Capacitance

Ifabodyis heated, its temperature rises. Similarly, the conductor is positively charged, and its potential increases if different materials are heated equally, their rise in temperature is not equal because the thermal capacities of the bodies are different. Similarly, even if different conductors are charged equally, their increase in potential may not be equal. The potential of a conductor depends not only on the amount of charge possessed by it but also on its shape, surface area, nature of the surrounding medium, and presence of other conductors.

For a given conductor its potential is always proportional to its charge. If a conductor is charged with Q and as a result, its potential is raised by an amount V, then

Q ∝ V or, Q = CV

The proportionality constant C is known as the capacitance or capacity of the conductor. Its value depends on shape, surface area, nature of the surrounding medium, and presence of other conductors.

From equation (1) we have,

C = \(\frac{Q}{V}\)…(2)

i.e., capacitance = \(\frac{amount of charge}{rise of potential}\)

If V = 1, then C = Q.

Read and Learn More Class 12 Physics Notes

Definition: The capacitance or capacity of a conductor is defined as the charge required to raise its potential by unity.

Units of Capacitance:

In the CGS system:

In the CGS system, a unit of capacitance is esu of capacitance. In the above equation (2), if Q = 1 esu of charge and V = 1 esu of potential, then’ C = 1 esu of capacitance, i.e., if 1 esu of charge raises the potential of a conductorby1 esu, the capacitance of the conductor is defined as 1 esu. This unit is also known as statfarad (statF).

In SI: SI unit of capacitance is farad (F). It is the practical unit of capacitance. The capacitance of a conductor is said to be 1 farad if 1 coulomb of charge is required to raise the potential of the conductor by 1 volt.

Therefore, IF = \(\frac{1 \mathrm{C}}{1 \mathrm{~V}}\)

Since farad is a very large unit, smaller units like microfarad and picofarad are most frequently used as the unit of capacitance.

1 microfarad (μF) = 10-6 farad (F)

1 picofarad (pF) = 10-12 farad (F)

Relation between farad and esu of capacitance:

1C = 3 X 109 esu of charge

IV = esu of potential

∴ \(1 \mathrm{~F}=\frac{1 \mathrm{C}}{1 \mathrm{~V}}=\frac{3 \times 10^9 \text { esu of charge }}{\frac{1}{300} \text { esu of potential }}\)

= 9 x 1011 esu of capacitance or statF

1μF = 10-6F = 9 x 10 esu of capacitance

lpF = 10-12F = 0.9 esu of capacitance

Dimension of capacitance

⇒ \(C=\frac{Q}{V}=\frac{Q}{W / Q}=\frac{Q^2}{W}\)

∴ \([C]=\frac{\left.T^2\right|^2}{M L^2 T^{-2}}=M^{-1} L^{-2} T^4 I^2\)

WBCHSE Class 12 Physics Capacitance And Capacitor Notes

Capacitance And Capacitor Capacitance Numerical Examples

Example 1. The capacitance of a spherical conductor is 1μF and the charge on it is -10C. What is its potential in the air?
Solution:

We know, V = 2;

here C = 1μF = 10-6F,

Q = -10C

∴ \(V=\frac{-10}{10^{-6}}=-10^7 \mathrm{~V}\)

Example 2. The potential of a conductor having 40 esu of capacitance is raised by 10 esu. What is the charge on the conductor? How much charge is to be given to another conductor, having capacitance three times that of the first conductor, to raise its potential three times that of the first one?
Solution:

Charge given to the first conductor,

Q1 = C1V1

= 40 x 10

= 400 esu of charge

The capacitance of the second conductor,

C2 = 3C1

= 3 x 40

= 120 esu of capacitance

The potential rise of the second conductor,

V2 = 3 x 10

= 30 esu of potential

Charge to be given to the second conductor,

Q2 = C2V2

= 120 x 30

= 3600 esu of charge

Factors Affecting Capacitance of a Conductor:

A conductor, at a potential V and having a charge Q, has a capacitance, C = \(\frac{Q}{V}\).

For constant Q, \(C \propto \frac{1}{V}\) Hence, the factors affecting V also affect the value of C of a conductor. The value of the capacitance of a conductor depends on the following factors.

  1. Surface area and shape of the conductor,
  2. Nature of the surrounding medium
  3. Presence of other conductors (especially earthed ones).

Surface area and shape of the conductor:

A conductor of greater size, i.e., a larger surface area has larger capacitance. The potential of a conductor decreases with the increase of its surface area and hence its capacitance increases.

Experiment:

A thin tin sheet is suspended from a charged ebonite rod. At the bottom of the sheet, a heavy metal rod is attached. This rod keeps the sheet stretched.

The tin sheet Is connected to the disc of a gold-leaf electroscope by a metal wire. If the sheet Is given a definite amount of charge, the leaves of the gold-leaf electroscope spread apart.

The divergence of the leaves indicates the potential of the sheet. NowIf the tin sheet is rolled up to some extent with the help of the ebonite rod, the divergence of the leaves of the electroscope will Increase.

It indicates that the potential of the sheet has increased, So, the charge of the sheet remains constant, and its capacitance decreases.

If the of the tin sheet is increased, the divergence of the leaves decreases, i.e., the potential of the sheet decreases and its capacitance increases. So capacitance of a conductor depends on its surface area.

Class 12 Physics Unit 1 Electrostatics Chapter 4 Capacitance and Capacitor Surface area and shape of the conductor

If the experiment is performed with conductors of the same surface area but of different shapes, it will be found that the spreading of the leaves of the gold-leaf electroscope are different So capacitance of a conductor also depends on its shape.

Nature of the surrounding medium:

If the conductor is surrounded by some dielectric medium other than air, the capacitance of the conductor increases. The effect increases with the increase of the dielectric constant of the medium.

Experiment:

A charged metal sheet A placed on an insulating stand is connected to the disc of an uncharged gold-leaf electroscope. The divergence of the leaves of the electroscope is observed. The amount of deflection indicates the potential of the metal sheet.

Class 12 Physics Unit 1 Electrostatics Chapter 4 Capacitance and Capacitor Nature of the surrounding medium

Now a dielectric slab, say a thick glass slab, is brought slowly near sheet A. It is found that the spreading of the leaves diminishes. So, the potential of A has decreased, i.e., its capacitance has increased.

Again if the dielectric slab is removed from the vicinity of sheet A, the electroscope leaves will spread out to the same extent as earlier.

In this case, the dielectric medium is polarised from induction due to the metal sheet A. So opposite charges are developed on the two sides of the glass slab.

Induced negative charge reduces the potential of sheet A and positive charge raises the potential of A.

However, due to the close proximity of the negative charge, its effect on sheet A predominates. So as a whole, the potential of sheet A diminishes a little, and hence its capacitance increases.

Presence of other conductors:

The capacitance of a conductor depends on the presence of other conductors near it. If an uncharged conductor is present in the vicinity of the charged conductor under test, its capacitance increases. This effect becomes pronounced if the neighboring conductor is earthed.

Experiment:

If a positively charged conductor A placed on an insulating stand is connected to the disc of an uncharged gold-leaf electroscope, the leaves of the electroscope spread out. The amount of divergence indicates the potential of conductor A.

Now another uncharged conductor B placed on an insulating stand is brought near A, it will be found that the divergence of the leaves diminishes a little. So, the potential of A has decreased a little.

If conductor B is removed, the leaves spread to the same extent as earlier. From this, it is understood that if B is brought near A, the potential of A diminishes, and its capacitance increases.

Class 12 Physics Unit 1 Electrostatics Chapter 4 Capacitance and Capacitor Presence of other conductors

The reason is that an induced negative charge is developed at the nearer end and a positive charge at the far end of B due to the inducing charge of A.

The induced negative charge reduces the potential of A and the induced positive charge enhances its potential. But due to the proximity of the negative charge its effect on A predominates. So as a whole, the potential of conductor A diminishes a little, and hence its capacitance increases.

Now if conductor B is earthed, it will be found that the divergence of the leaves decreases considerably. This proves that the potential of conductor A is highly reduced.

If conductor B is removed from the vicinity of the coil now ductor A, the leaves of the electroscope will spread out to the same extent as earlier. So it is proved that if conductor B is earthed, the potential of conductor A diminishes a lot and hence its capacitance increases to a large extent.

The reason is that, if B is connected to the earth, the induced positive charge being free charge moves to the earth. Under this condition, due to the presence of only a negative charge in B, the potential of A diminishes a lot and hence its capacitance increases to a large extent.

The capacitance of a Spherical Conductor:

Let us consider a spherical conductor of radius R charged with Q’ amount of charge. The charge Q is uniformly distributed over the surface of the sphere. Potential at the surface of the sphere, as we know, is the same as that produced by an isolated point charge Q placed at the center of the sphere.

The potential of the sphere is given by,

⇒ \(V=\frac{1}{4 \pi \epsilon_0} \cdot \frac{Q}{R}\) [∈0 = permittivity of air or vacuum]

∴ The capacitance of the sphere,

⇒ \(C=\frac{Q}{V}=4 \pi \epsilon_0 R\)

We know, \(\frac{1}{4 \pi \epsilon_0}=9 \times 10^9 \mathrm{~N} \cdot \mathrm{m}^2 \cdot \mathrm{C}^{-2}\)

So, in vacuum or air, the capacitance of the sphere,

⇒ \(C=\frac{R}{9 \times 10^9} \text { farad }\)

In CGS system, replacing \(\epsilon_0 \text { by } \frac{1}{4 \pi}\), we have, C = R.

Hence the capacitance in the CGS unit of a spherical conductor placed in the air (or vacuum) is numerically equal to its radius in centimeters. For this reason, the capacitance CGSunit is sometimes expressed in centimeters.

Unit of ∈0: From the relation C = 4π∈0R,we have,

⇒ \(\epsilon_0=\frac{C}{4 \pi R}\)

So, the unit of ∈0

⇒ \(=\frac{\text { unit of } C}{\text { unit of } R}=\frac{\mathrm{F}}{\mathrm{m}}=\text { farad } / \text { metre }\left(\mathrm{F} \cdot \mathrm{m}^{-1}\right)\)

Using this simpler unit, we may write,

= 8.854 x 10-21 F.m-1

That this unit F m-1 is identical to the unit C².N-1.m-2 of ∈0, used earlier, is shown here

⇒ \(\mathrm{F} \cdot \mathrm{m}^{-1}=\frac{\mathrm{F}}{\mathrm{m}}=\frac{\mathrm{C}}{\mathrm{V} \cdot \mathrm{m}}=\frac{\mathrm{C}}{\frac{\mathrm{J}}{\mathrm{C}} \cdot \mathrm{m}}=\frac{\mathrm{C}^2}{\mathrm{~J} \cdot \mathrm{m}}\)

⇒ \(\frac{\mathrm{C}^2}{(\mathrm{~N} \cdot \mathrm{m}) \cdot \mathrm{m}}=\frac{\mathrm{C}^2}{\mathrm{~N} \cdot \mathrm{m}^2}=\mathrm{C}^2 \cdot \mathrm{N}^{-1} \cdot \mathrm{m}^{-2}\)

Capacitance And Capacitor Factors Affecting Capacitance of a Conductor Numerical Examples

Example 1. The radius of the__earth is 6400km. Determine its capacitance in μF.
Solution:

The radius of the earth = 6400 km = 6400 x 10³ m.

Capacitance, \(C=4 \pi \epsilon_0 R=\frac{1}{9 \times 10^9} \times 6400 \times 10^3\)

⇒ \(=\frac{64}{9} \times 10^{-4}=\frac{6400}{9} \times 10^{-6} \mathrm{~F}\)

= 711.1μF

Example 2. A metal sphere has a diameter of 1 m. What will be the amount of charge required to raise its potential by 2.7 x 106 V?
Solution:

Radius, R = \(\frac{1}{2}\) = 0.5 m;

⇒ \(C=4 \pi \epsilon_0 R=\frac{1}{9 \times 10^9} \times 0.5=\frac{0.5}{9 \times 10^9} \mathrm{~F}\)

∴ The amount of charge required,

⇒ \(Q=C V=\frac{0.5}{9 \times 10^9} \times\left(2.7 \times 10^6\right)\)

= 1.5 X 10-4

= 150 X 10-6 C

= 150μC

Example 3. Is it possible for a metal sphere of radius 1 cm to hold a charge of IC?
Solution:

Radius, r = 1 cm = 0.01 m

So, capacitance of the sphere

⇒ \(C=4 \pi \epsilon_0 r=\frac{1}{9 \times 10^9} \times 0.01=\frac{1}{9} \times 10^{-11} \mathrm{~F}\)

∴ The potential of the sphere,

⇒ \(V=\frac{Q}{C}=\frac{1}{\frac{1}{9} \times 10^{-11}}=9 \times 10^{11} \mathrm{~V}\)

At this very high potential, the sphere will discharge in the sureo rounding air, i.e., it will not be able to hold the charge of 1 C.

Example 4. SI The diameter of the spherical liquid drop is 2mm and its charge is 5 x 10-6 esu.

  1. What is the potential and its surface?
  2. If two such liquid drops coalesce to form a bigger drop, what will be the potential on its surface?

Solution:

1. In the CGS system, the radius of the spherical conductor = its capacitance (numerically).

∴  The capacitance of the spherical liquid drop,

C = 0.1 statF [∵ Radius = 1mm = 0.1 cm ]

∴  Potential on the surface of the liquid drop,

⇒ \(V=\frac{Q}{C}=\frac{5 \times 10^{-6}}{0.1}=5 \times 10^{-5} \mathrm{statV}\)

= 5 x 10-5 x 300 V

= 0.015 V

2. Let the radius of the bigger drop be R.

According to the question,

⇒\(\frac{4}{3} \pi R^3=2 \times \frac{4}{3} \pi(0.1)^3\)

or, \(R^3=2 \times(0.1)^3\)

or, \(R=0.1 \times 2^{\frac{1}{3}}\)

= 0.1 x 1.26

= 0.126 cm

Total charge,

Q = 2 x 5 x 10-6

= 10~5 esu of charge.

∴  Potential on the surface of the bigger liquid drop,

⇒ \(V=\frac{Q}{C}=\frac{10^{-5}}{0.126}\)

= 7.94 x 10-5 x 300 statV

= 7.94 x 10-5 x 300 V

= 0.0238 V

Capacitance And Capacitor Potential Energy Of A Charged Conductor

A certain amount of work has to be done in order to charge a conductor. The energy spent for doing that work remains stored in the charged conductor as potential energy. Essentially, the electric field of the conductor stores this energy.

Calculation:

Let a conductor be charged with Q and let its capacitance be C. The potential of the conductor is V. During charging we assume that the whole amount of charge is not given to the conductor at a time, rather it is charged gradually.

At first, the charge of the conductor is zero so its potential is also zero. Gradually its potential increases due to the accumulation of charges. So at the time of charging the conductor has no particular potential. Its potential becomes V when its charge is Q.

Initial potential = 0; final potential = V

Average of these potentials = \(\frac{0+V}{2}=\frac{V}{2}\)

Therefore, work done = average potential x charge

⇒ \(\frac{V}{2} \times Q=\frac{1}{2} Q V=\frac{1}{2} C V \cdot V\) [∵ Q = CV]

⇒ \(\frac{1}{2} C V^2\)

= \(\frac{1}{2} C \times\left(\frac{Q}{C}\right)^2\)

= \(\frac{1}{2} \cdot \frac{Q^2}{C}\)

This work is stored in the charged conductor as potential energy.

∴ The potential energy of a charged conductor

⇒ \(\frac{1}{2} Q V\)

= \(\frac{1}{2} C V^2\)

= \(\frac{1}{2} \frac{Q^2}{C}\)

If C, V, and Q are expressed in esu, the unit of potential energy will be erg. Again C, V, and Q are expressed in farad, volt, and coulomb, respectively, the unit of potential energy will be joule.

Derivation using calculus: Let at any moment the charge on the conductor be q and its potential be v.

Evidently, q = Cv

When a charge dq is given to the conductor, the work done against the repulsive force due to potential v is given by,

⇒ \(d W=v d q=\frac{q}{C} d q\)

Hence the total work done to impart Q amount of charge is,

⇒ \(W=\int d W=\int_0^Q \frac{q}{C} d q=\frac{1}{2} \frac{Q^2}{C}=\frac{1}{2} C V^2=\frac{1}{2} Q V\)

This work is stored as potential energy in the charged conductor.

Capacitance And Capacitor Distribution Of Charge Between Two Conductors

Two conductors at the same potential:

Let us consider two insulated uncharged conductors A and B of capacitances C1 and C2, respectively. They are connected by a fine metal wire. Under this condition, if a charge Q is given to this combination, it will be distributed between the two conductors.

Class 12 Physics Unit 1 Electrostatics Chapter 4 Capacitance and Capacitor Two conductors at the same potential

Let us consider that conductor A has obtained a charge Q1 and conductor B a charge Q2 As the two conductors are connected to each other, they have the same potential. Let their common potential be V.

Q = Q1 + Q2

and \(V=\frac{Q_1}{C_1}\)

= \(\frac{Q_2}{C_2}\)

= \(\frac{Q_1+Q_2}{C_1+C_2}\)

= \(\frac{Q}{C_1+C_2}\)

⇒ \(\left.\begin{array}{ll}
∴ & Q_1=C_1 V=Q \cdot \frac{C_1}{C_1+C_2} \\
\text { and } & Q_2=C_2 V=Q \cdot \frac{C_2}{C_1+C_2}
\end{array}\right\}\)…(1)

Again, \(\frac{Q_1}{Q_2}=\frac{C_1}{C_2}\)….(2)

Therefore, a charge on each conductor is proportional to its capacitance. If the two conductors have the same capacitance, the given charge will be shared equally between them. If they have radii r1 cm and r2 cm, then C1 = r1 and C2 = r2 (in the CGS system).

In that case,

⇒ \(\left.\begin{array}{l}
Q_1=Q \cdot \frac{r_1}{r_1+r_2} \\
Q_2=Q \cdot \frac{r_2}{r_1+r_2}
\end{array}\right\}\)….(3)

∴ \(\frac{Q_1}{Q_2}=\frac{r_1}{r_2}\)…(4)

Therefore, the charge on each spherical conductor is proportional to its radius.

We know that the surface density of charge on the surface of a charge is the same everywhere.

If σ1 and σ2 be the surface densities of charge of the two conductors, then

⇒ \(\sigma_1=\frac{Q_1}{4 \pi r_1^2} \text { and } \sigma_2=\frac{Q_2}{4 \pi r_2^2}\)

∴  \(\frac{\sigma_1}{\sigma_2}=\frac{Q_1}{Q_2} \cdot \frac{r_2^2}{r_1^2}=\frac{r_1}{r_2} \times \frac{r_2^2}{r_1^2}=\frac{r_2}{r_1}\)…(5)

Therefore, the surface density of the charge of a spherical conductor is inversely proportional to its radius.

Two conductors initially at different potentials:

Let us consider two insulated conductors A and B. They have capacitances C1 and C2 and they are given charges Q1 and Q2 separately.

So under this condition,

potential of the conductor A, \(V_1=\frac{Q_1}{C_1} \text { or, } Q_1=C_1 V_1\)

and potential of the conductor B, \(V_2=\frac{Q_2}{C_2} \text { or, } Q_2=C_2 V_2\)

∴  The total charge of the conductors,

Q = Q1 + Q2

= C1V1 + C2V2…(6)

If the two conductors are connected by a thin metal wire, a positive charge will flow from the conductor at a higher potential to that at a lower potential and this flow of charge will continue till their potentials become equal.

Suppose, V1 > V2; then a charge will flow from A to B. Let V be the common potential after connection. During this flow of charge, the total charge of the system remains constant.

So, total charge before connection = total charge after connection

i.e„ Q = C1V1 + C2V2

= C1V + C2V

= (C1 + C2)V

or, \(V=\frac{C_1 V_1+C_2 V_2}{C_1+C_2}\)….(7)

After connection, if A and B contain charges q x and q2 respectively, then

⇒ \(\left.\begin{array}{l}
q_1=C_1 V=C_1 \cdot \frac{C_1 V_1+C_2 V_2}{C_1+C_2}=\frac{C_1}{C_1+C_2} \cdot Q \\
q_2=C_2 V=C_2 \cdot \frac{C_1 V_1+C_2 V_2}{C_1+C_2}=\frac{C_2}{C_1+C_2} \cdot Q
\end{array}\right\}\)….(8)

As V1 > V2, the charge lost by A,

q’1 =C1V1-C1V = C1(V1-V)

⇒ \(C_1\left(V_1-\frac{C_1 V_1+C_2 V_2}{C_1+C_2}\right)\)

or, \(q_1^{\prime}=\frac{C_1 C_2\left(V_1-V_2\right)}{C_1+C_2}\)….(9)

Again, the charge gained by B,

⇒ \(q_2^{\prime}=C_2 V-C_2 V_2=C_2\left(\frac{C_1 V_1+C_2 V_2}{C_1+C_2}-V_2\right)\)

or, \(q_2^{\prime}=\frac{C_1 C_2\left(V_1-V_2\right)}{C_1+C_2}\)…..(10)

From equations (9) and (10) we get, q1 = q2, i.e., the charge gained by conductor B is equal to that lost by conductor A.

Loss of energy due to sharing of charge:

Before connection, the total energy of the two conductors

⇒ \(\frac{1}{2} C_1 V_1^2+\frac{1}{2} C_2 V_2^2\)

After connection, the total energy of them

⇒ \(\frac{1}{2} C_1 V^2+\frac{1}{2} C_2 V^2=\frac{1}{2}\left(C_1+C_2\right) V^2\)

⇒ \(\frac{1}{2}\left(C_1+C_2\right) \cdot\left(\frac{C_1 V_1+C_2 V_2}{C_1+C_2}\right)^2\)

⇒ \(\frac{1}{2} \cdot \frac{\left(C_1 V_1+C_2 V_2\right)^2}{C_1+C_2}\)

Therefore, loss of energy due to the sharing of charge

⇒ \(\frac{1}{2} C_1 V_1^2+\frac{1}{2} C_2 V_2^2-\frac{1}{2} \cdot \frac{\left(C_1 V_1+C_2 V_2\right)^2}{C_1+C_2}\)

⇒ \(\frac{1}{2} C_1 V_1^2+\frac{1}{2} C_2 V_2^2-\frac{1}{2} \cdot \frac{\left(C_1 V_1+C_2 V_2\right)^2}{C_1+C_2}\)

⇒ \(\frac{1}{2} \cdot \frac{C_1 C_2}{C_1+C_2} \times\left(V_1-V_2\right)^2\)….(11)

Now, C1 and C2 are both positive quantities, and (V1 – V2)², being a perfect square, is also positive. So, the relation (11) is positive.

Hence, there is always a loss of energy in the electric field of the conductors due to the sharing of charges.

According to the law of conservation of energy, this loss of energy must be converted to some other form, usually as heat in the connecting wire. This Joss is partly converted into light and sound in addition to heat if sparkling occurs.

Capacitance And Capacitor Distribution Of Charge Between Two Conductor Numerical Examples

Example 1. A conductor of capacity 4 units, charged with 100 units of positive charge is connected to another conductor of capacity 2 units, charged with 20 units of negative charge. What is the change In the potential of each conductor? What will be the charges for each of them after the connection?
Solution:

Capacity of the first conductor; C1 = 4 units; charge, Q1 = 100 units.

∴ \(\text { Potential, } V_1=\frac{Q_1}{C_1}\)

= \(\frac{100}{4}\)

= 25units

Capacity of the second conductor C2 = 2 units; charge, Q2 = -20 units.

∴ \(\text { Potential, } V_2=\frac{Q_2}{C_2}\)

= \(\frac{-20}{2}\)

= -10 units

After connection, suppose, the common potential of the two conductors becomes equal to V.

∴ \(V=\frac{Q_1+Q_2}{C_1+C_2}\)

= \(\frac{100-20}{4+2}\)

= \(\frac{80}{6}\)

= \(\frac{40}{3}\)

= 13.33 units

So, the change of potential of the first conductor

= 25-13.33

= 11.67 units

The change of potential of the second conductor

= 13.33- (-10)

= 2333 units

Residual charge in the first conductor after connection,

q1 = C1V

= 4 x \(\frac{40}{3}\)

= 5333 units

Residual charge in the second conductor after connection,

q2 = C2V

= 2 x \(\frac{40}{3}\)

= 26.67 units

Example 2. An insulated metallic vessel full of water Is charged with a potential of 3V. Drops of water are trickling from an orifice at the bottom of the vessel What is the amount of charge contained In each spherical drop of radius 1mm?
Solution:

Potential of the metal vessel full of water, V = 3 V Radius of a water drop, R = 1 mm = 10-3m

∴ The capacitance of the water drop,

⇒ \(C=4 \pi \epsilon_0 \mathrm{R}=\frac{10^{-3}}{9 \times 10^9}=\frac{1}{9} \times 10^{-12} \mathrm{~F}\)

∴ Charge of each water drop,

⇒ \(Q=C V=\frac{1}{9} \times 10^{-12} \times 3=3.3 \times 10^{-13} \mathrm{C}\)

Example 3. The radii of two insulated metal spheres are 5 cm and 10 cm. They are charged up to potentials of 10 esu and 15 esu, respectively. If the two spheres are connected with one another, what will be the loss of energy?
Solution:

The radius of the first sphere = 5 cm

Capacitance, C1 = 5 statF

and potential, V1 = 10 statV

Charge, Q1 = C1V1

= 5 x 10

= 50 state

The radius of the second sphere = 10 cm

∴ Capacitance, C2 = 10 statF

and potential, V2 = 15 statV

Charge, Q2 = C2V2

= 10 x 15

= 150 statC

The total charge of the two spheres,

Q = Q1 + Q2

= 50 + 150

= 200 statC

Equivalent capacitance of the combination of two spheres,

C = C1 + C2

= 5 + 10

= 15 statF

If V is the common potential of the two spheres after connection, then

⇒ \(V=\frac{\text { total charge of the two spheres }}{\text { total capacitance of the two spheres }}\)

⇒ \(\frac{200}{15}\)

= \(\frac{40}{3} \mathrm{statV}\)

The total energy of the two spheres before connection,

⇒ \(E_1=\frac{1}{2} C_1 V_1^2+\frac{1}{2} C_2 V_2^2\)

⇒ \(\frac{1}{2}\left[5 \times(10)^2+10 \times(15)^2\right]\)

= 1375 erg.

The total energy of the two spheres after connection,

⇒ \(E_2=\frac{1}{2} C V^2=\frac{1}{2} \times 15 \times\left(\frac{40}{3}\right)^2\)

= 1333.33 erg

∴ Loss of energy due to connection

= 1375-1333.33

= 41.67 erg

Example 4. A metal sphere of radius 10 cm is charged up to a potential of 80 esu. After sharing its charge with another sphere, their common potential becomes 20 esu. What is the radius of the second sphere?
Solution:

The radius of the first sphere = 10 cm

∴ Capacitance C1 = 10 statF

and potential, = 80 statV

∴ Charge, Q1 = C1V1

= 10 x 80

= 800 state

After connection, the common potential, V = 20 statV

∴ \(V=\frac{\text { total charge of the two spheres }}{\text { equivalent capacitance of the two spheres }}\)

or, \(20=\frac{800}{10+C_2}\left[C_2=\text { capacitance of the second sphere }\right]\)

or, 10+ C2 = 40

or, C2 = 30 statF

Example 5. One thousand similar electrified raindrops merge into a single one so that their total charge remains unchanged. Find the change in the total electrostatic energy of the drops, assuming that all the drops are spherical and the small drops were initially at large distances from one another
Solution:

Suppose, each drop of radius r contains a charge Q.

∴ Capacitance, C = r

and energy, \(E_1=\frac{1}{2} \cdot \frac{Q^2}{C}=\frac{Q^2}{2 r}\)

The total energy of 1000 drops,

⇒ \(E=1000 E_1=\frac{500 Q^2}{r}\)

If R is the radius of the large drop then,

⇒ \(\frac{4}{3} \pi R^3=1000 \times \frac{4}{3} \pi r^3\)

or, R = 10r

Charge of the large drop = 1000 Q.

∴ The energy of the large drop,

⇒ \(E_2=\frac{1}{2} \cdot \frac{(1000 Q)^2}{R}\)

= \(\frac{5 \times 10^5 Q^2}{10 r}\)

= \(\frac{5 \times 10^4 Q^2}{r}\)

∴ \(\frac{E_2}{E}=\frac{5 \times 10^4}{500}=100\)

or, E2 = 100E

∴ Change in electrostatic energy of the drops

E2-E = 100E-E

= 99E

= 99 x initial energy

Example 6. Two equally charged soap bubbles of equal volume join together to form a large bubble. If each small bubble had a potential V, find the potential of the result
Solution:

Let the radius of each small bubble be r, the charge be Q, and the radius of the large bubble be R.

∴ \(\frac{4}{3} \pi R^3=2 \times \frac{4}{3} \pi r^3 \text { or, } R=2^{1 / 3} \cdot r\)

The potential of each small bubble,

V = \(\frac{Q}{r}\)

or, Q = Vr

∴ The potential of the large bubble

⇒ \(\frac{\text { total charge }}{\text { radius }}\)

⇒ \(\frac{2 Q}{R}\)

= \(\frac{2 V r}{2^{1 / 3} r}\)

= \(2^{2 / 3} V\)

Example 7. Eight spherical liquid drops join to form a large drop. The diameter of each drop is 2 mm and the charge 5μ statC. What is the potential on the surface of the large drop?
Solution:

Radius of a small drop, r = 1 mm =0.1 cm

Suppose, the radius of the large drop is R.

∴ \(\frac{4}{3} \pi R^3=8 \times \frac{4}{3} \pi r^3\)

or, R = 2r

= 2 x 0.1

= 0.2 cm

∴ The capacitance of large drop, C = R = 0.2 statF

The total charge of the small drops,

Q = 8 X 5

= 40μstatC

= 40 X 10-6 statC

∴ Potential on the surface of the large drop,

⇒ \(V=\frac{Q}{C}=\frac{40 \times 10^{-6}}{0.2} \text { statV }\)

= 2 X 10-4 x 300 V

= 0.06 V

Example 8. The ratio of the capacitances of two conductors A and B is 2: 3. The conductor A gains a certain amount of charge and shares it with B. Compare the initial energy of A with the total energy of A and B.
Solution:

Let the capacitance of the conductor A be 2C and that of the conductor B be 3C.

The amount of charge gained by A is Q.

Let the common potential of A and B after sharing of charge be V

∴ \(V=\frac{Q}{2 C+3 C}\)

= \(\frac{Q}{5 C}\)

The energy of the conductor A before sharing of charge,

⇒ \(E_A=\frac{1}{2} \cdot \frac{Q^2}{2 C}\)

= \(\frac{Q^2}{4 C}\)

Total energy of the conductors A and B after sharing the charge,

⇒ \(E_A=\frac{1}{2} \cdot \frac{Q^2}{2 C}\)

= \(\frac{Q^2}{4 C}\)

∴ The required ratio = \(\frac{E_A}{E}\)

= \(\frac{\frac{Q^2}{4 C}}{\frac{Q^2}{10 C}}\)

= \(\frac{5}{2}\)

Example 9. Each of the 27 identical mercury drops is charged to a potential of 10V. If the drops coalesce to form a big drop, what will be its potential? Calculate the ratio of the energy of the big drop to that of a small drop.
Solution:

Let the radius of each small drop be r.

∴ Capacitance, C1 = 4π∈0r

∴ Charge of each small drop, q = C1V1

= 10C1

∴ Total charge, Q = 27q = 27 x 10C1

= 270C1

If R is the radius of the big drop, then according to the question,

⇒ \(\frac{4}{3} \pi R^3=27 \times \frac{4}{3} \pi r^3 \text { or, } R=3 r\)

∴ The capacitance of the big drop,

C2 = 4π∈0.3r = 3C1

∴ The potential of the big drop,

⇒ \(V_2=\frac{Q}{C_2}=\frac{270 C_1}{3 C_1}=90 \mathrm{~V}\)

The energy of a small drop,

⇒ \(E_1=\frac{1}{2} C_1 V_1^2=\frac{1}{2} \times C_1 \times(10)^2\)

The energy of the big drop,

⇒ \(E_2=\frac{1}{2} C_2 V_2^2=\frac{1}{2} \times 3 C_1 \times(90)^2\)

∴ \(\frac{E_2}{E_1}=\frac{3 C_1}{C_1} \times \frac{(90)^2}{(10)^2}=\frac{243}{1}\)

∴ E2: E1 = 243: 1

Example 10. Charges of 10-2 C and 5 x 10-2.C are put on two metal spheres of radii 1 cm and 2 cm respectively. If they are connected with a metal wire, what will be the final charge on the smaller sphere?
Solution:

Here, the radius of the first sphere, R% = 1 cm = 0.01 m, and the radius of the second sphere, R2 = 2 cm = 0.02 m.

∴ The capacitance of the first sphere, C1 = 4π∈0R1

and capacitance of the second sphere, C2 = 4π∈0R2

The total amount of charge before and after connection,

Q1 + Q2 = Q

= (C1 + C2) V [V = common potential]

∴ \(\dot{V}=\frac{Q_1+Q_2}{C_1+C_2}\)

The final charge on the smaller (first) sphere,

⇒ \(q_1=C_1 V=C_1 \frac{Q_1+Q_2}{C_1+C_2}\) [Given, Q1 = 10-2C and Q2 = 5 x 10-2C]

⇒ \(4 \pi \epsilon_0 R_1 \times \frac{10^{-2}+5 \times 10^{-2}}{4 \pi \epsilon_0 R_1+4 \pi \epsilon_0 R_2}\)

⇒ \(R_1 \times \frac{6 \times 10^{-2}}{R_1+R_2}\)

= \(0.01 \times \frac{6 \times 10^{-2}}{0.01+0.02}\)

= 0.02C

Example 11. The capacitance and potential, respectively, of conductor A are 10 units and 50 units; those of conductor B are, respectively, 5 units and 65 units. Find out the charges on the two conductors after they
Solution:

Initially, the charge on conductor A,

Q2 = C1V1

= 10 x 50

= 500 unit

and charge on conductor B,

Q2 = C2V2

= 5 x 65

= 325 unit

The common potential after the two conductors are connected,

⇒ \(V=\frac{Q_1+Q_2}{C_1+C_2}=\frac{500+325}{10+5}=\frac{825}{15}\)

= 55 unit

Now, charge on conductor A,

q1 = C1V

= 10 X 55

= 550 unit

and charge on conductor B,

q2 = C2V

= 5 X 55

= 275 unit

Example 12. A spherical liquid drop of capacitance I/<F breaks Into drops of the same radius. What Is the capacitance of each of these smaller drops?
Solution:

Let R = radius of the Initial drop; r = radius of each of 8 smaller drops.

∴ \(\frac{4}{3} \pi R^3=8 \times \frac{4}{3} \pi r^3 \text { or, } r=\frac{R}{2}\)

The capacitance of the bigger drop = 4π∈0R

= 1μF

∴ The capacitance of each small drop

=4π∈0r

⇒ \(4 \pi \epsilon_0 r=4 \pi \epsilon_0 \frac{R}{2}=\frac{1}{2} \cdot 4 \pi \epsilon_0 R=\frac{1}{2} \times 1 \mu \mathrm{F}\)

= 0.5μF

Example 13. Two isolated metallic solid spheres of radii R and 2R are charged in such a way that both of these have the same charge density. The spheres are placed far away from each other and are connected by a thin conducting wire. Find the new charge density on the bigger sphere
Solution:

Let or be the charge density of the two spheres.

So, charge of the first sphere = q1 = 4πR2σ and charge of the second sphere = q2 = 4π(2R)²σ = 16πR²σ

When they are connected with a wire, let q1 and q2 be the new charges.

Then we may write

q’1 +q’2 = q1 + q2 ….(1)

Since the two spheres are at the same potential,

⇒ \(\frac{1}{4 \pi \epsilon_0} \frac{q_1^{\prime}}{R}=\frac{1}{4 \pi \epsilon_0} \frac{q_2^{\prime}}{2 R} \quad \text { or, } q_1^{\prime}=\frac{q_2^{\prime}}{2}\)

In the equation (1), by substituting q1, we have

⇒ \(\frac{q_2^{\prime}}{2}+q_2^{\prime}=q_1+q_2\)

or, \(q_2^{\prime}=\frac{2}{3}\left(q_1+q_2\right) \text { or, } q_2^{\prime}=\frac{2}{3}\left(4 \pi R^2 \sigma+16 \pi R^2 \sigma\right)\)

∴ \(q_2^{\prime}=\frac{40 \pi R^2 \sigma}{3}\)

Therefore, the new charge density of the bigger sphere,

⇒ \(\sigma^{\prime}=\frac{q_2}{4 \pi(2 R)^2}=\frac{40 \pi R^2 \sigma}{3 \times 16 \times \pi R^2}=\frac{5}{6} \sigma\)

Capacitance And Capacitor Capacitor And Its Principle

Capacitor:

A capacitor (originally known as con denser) is an arrangement by which the capacitance of a conductor can be increased.

It is used for storage of charge. Hence, a capacitor can alternatively be defined as an arrangement that can store a certain amount of charge.

Usually, a counselor uses the Ilia principle of artificially looking at Ilia capacitance of mi Insulnlod clinical conductor by bringing another method of conductor noor It.

Construction of capacitor:

A capacitor Is basically an arrangement of an Insulated conductor and an earthed conductor held close to each other and separated by air or a non-conducting (dielectric) medium. The shape of the two conductors Is usually the same, e.g.,

In the case of a parallel plate capacitor, parallel metal plates are placed close to each other. Again, a spherical capacitor consists of two concentric spheres and a cylindrical capacitor of two co-axial cylinders.

Working principle of capacitor: An insulated metal plate A is connected to an electrical machine, Suppose, the potential of the plate is +V when It Is fully charged, ff C be the capacitance of the plate, the charge on the plate will be,

Q = CV

Class 12 Physics Unit 1 Electrostatics Chapter 4 Capacitance and Capacitor Working principle of capacitor

Now if a similar plate B is placed in front of plate A, then due to induction, a negative charge is induced on the inner surface of B and a positive charge on its outer surface.

The induced negative charge, being nearer, lowers the positive potential of plate A. Thus, the capacitance of plate A increases a little (since C = \(\frac{Q}{V}\)). Hence plate A takes a slight additional charge from the electrical machine and raises its potential again to V.

Now if B is earthed, the positive charge on the far side of it moves to the earth and the influence of positive charges is absent, the potential of A falls further. So the capacitance of A increases further and consequently, it will now be able to receive a greater amount of charge from the machine.

So in this way, the capacitance of an insulated charged conductor can be increased with the help of another earthed conductor, placed in its vicinity.

Factors affecting the capacitance of a capacitor:

1. Overlapping nrcu of the plates: The Greater the surface area of the plates, the greater its capacitance. The capacitance decreases with the decrease of the overlapping area.

2. Distance between the two conducting plates: Capacitance increases with the decrease of this distance and vice versa.

3. Nature of the intervening medium between the two ducting plates: Instead of air, if the intervening space of the two plates is filled up with an insulator, e.g., paraffin, glass, paper, etc., the capacitance of a capacitor increases.

Uses of capacitors: Extensive uses of tire capacitors are found in different electrical circuits. In the case of different circuits in AC, the capacitor is almost an indispensable part. Capacitors are used in electronic instruments, radio, television, telephone, the flash circuit of a camera, etc.

Discussions:

1. Charge of a capacitor: Charge of a capacitor means die magnitude of charge on any one of its plates. If one plate possesses a charge + Q, then the other plate will contain a charge -Q. So total charge = +Q + (-Q) = 0. Here, Q is called the charge of a capacitor, it is not the total charge.

2. Ideal capacitor:

If a capacitor is connected to a source of high potential, it is charged to that potential. The capacitor is called an ideal one if it is not discharged automatically even if the source of potential is removed. It preserves its acquired charge without any leakage.

3. Maximum limit of the potential of a capacitor:

A capacitor cannot be charged to any high potential at will. If the value of the potential exceeds a certain maximum limit, the intervening medium loses its insulating properties. Consequently, electric discharge takes place between the capacitor and the intervening medium.

4. Any charged conductor is a capacitor:

Any charged conductor may be considered as a capacitor. The floor or the walls of the room act as the earthed conductor in this case.

5. Circuit symbol of capacitor:

Two parallel lines of the same size in an electrical circuit diagram, represent a capacitor Symbol of a variable capacitor.

Class 12 Physics Unit 1 Electrostatics Chapter 4 Capacitance and Capacitor Circuit symbol of capacitor

Charging and discharging of a capacitor:

When a battery Is connected to a series resistor and capacitor, charges begin to accumulate on the capacitor.

This Is called the charging of a capacitor. After removal of the battery, the capacitor loses Its accumulated charge through the resistor gradually. Tills Are called discharging of a capacitor.

The two plates A and B of a capacitor are connected to a buttery of electromotive force E through a resistor. Electrons from the negative pole of (lie battery move to plate H.

Simultaneously, a How of electrons starts from plate A to the positive pole of the battery. This produces a charging current.

As negative charges on plate B and positive charges on plate A keep on accumulating, (ho potential difference between die two plates increases.

So the plates act as a cell and consequently, a tendency of electron flow Is established In the direction opposite to that of the initial electron flow. As a result, the die charging current decreases.

When the potential difference between the two plates A and II becomes equal to the end of the battery, the charging current ceases to flow.

Then it is said that the capacitor has become fully charged. So at the start, the charging current is maximum and afterward, It gradually decreases. When the capacitor is fully charged, the charging current becomes zero.

Class 12 Physics Unit 1 Electrostatics Chapter 4 Capacitance and Capacitor Charging and discharging of a capacitor

After removal of the battery from the circuit, l.e„ during discharging electrons from plate 2 flow to plate A and begin to neutralize the positive charge of plate A.

Thus again a current flows in the circuit. This is called discharging and its direction is opposite to that of the charging current- After a while all the electrons of plate B neutralize all the positive charges of plate A.

Then the discharging current becomes zero and the capacitor is said to be completely discharged. So at the start, discharging current is. maximum and It decreases gradually and becomes zero when the capacitor is completely discharged.

In fact, no capacitor is an ideal one. A fully charged capacitor loses its charge in the course of time even if the two plates of it are not connected by a conducting wire.

Finally, the tire capacitor becomes completely discharged. Of course, in this case, the discharging action continues for a long time.

Potential and Capacitance of a Capacitor:

Potential of a capacitor: The potential difference between the two conducting plates of a capacitor is called the potential of a capacitor. Generally, the potential of a capacitor means the potential attained by the insulated plate of the capacitor due to the charge given to it, the grounded plate of the capacitor being at zero potential.

The capacitance of a capacitor:

The capacitance of a capacitor means the capacitance of the insulated conducting plate of the capacitor. So it may be defined as the amount of charge that must be given to the insulated plate to raise its potential by unity. If a charge Q raises its potential by Vi its capacitance, C = \(\frac{Q}{V}\).

Therefore, the capacitance of a capacitor may be defined as the ratio of the magnitude of the charge on any one of the two plates to die difference of potential between them, i.e., the capacitance of a capacitor

= \(\frac{\text { charge on a conducting plate of the capacitor }}{\text { difference of potential between the two plates }}\)

The capacitance of a capacitor is assigned a value of 1 faradic 1 coulomb of charge is required to maintain a potential difference of 1 V between the two conductors or plates of the capacitor.

Here capacitance is always a positive quantity and it does not depend on the nature of charge and potential. The capacitance of a conductor and that of a capacitor are expressed in the same unit.

Usually, a capacitor is rated, bearing the mark of the magnitude of its capacitance and the maximum potential difference that can be applied safely between its two plates.

A capacitor rated 0.04μF 220V means that its capacitance is 0.04μF and the maximum potential difference to be applied between its two plates is 220 V. If it is used in a potential difference higher than 220 V, it may get damaged.

Capacitance And Capacitor Dielectrics

Now we discuss the materials that do not conduct electricity and can be inserted between the plates of a capacitor.

Substances that have no free electrons cannot conduct electricity. They are called insulators or dielectrics. When they are placed in an electric field, charges are induced on their surfaces.

Classification of Dielectrics:

Dielectrics are classified into two groups according to the position of charge within their molecules: O non-polar substance and 0 polar substance.

Non-polar substance: A substance in which the center of negative charges (electrons) coincides with that of positive charges (protons) in each of its molecules, is called a non-polar substance.

In the absence of an external electric field, these molecules do not possess any permanent electric dipole moment Thus they are called non-polar molecules.

In the presence of an external electric field, a relative displacement occurs between the centers of positive and negative charge distributions. Thus a non-polar molecule when subjected to an electric field, acquires an electric dipole moment These types of dipoles are called induced dipoles.

Class 12 Physics Unit 1 Electrostatics Chapter 4 Capacitance and Capacitor Non-polar substance

Polar substance: A substance in which the center of negative charges (electrons) does not coincide with that of positive charges (protons) in each of its molecules, is called a polar substance.

So, even in the absence of an external electric field, each of these molecules possesses a permanent electric dipole moment So They are called polar molecules.

Example: water (H2O), ammonia (NH3).

Generally, the dipole moments of different molecules of a polar substance are randomly directed. So the resultant dipole moment of a polar substance is zero.

But when subjected to an electric field, each molecule of a polar substance experiences a torque and tends to fall in line with the direction of field lines of the external electric field. As a result, the sample of the polar substance acquires a resultant electric dipole moment.

Class 12 Physics Unit 1 Electrostatics Chapter 4 Capacitance and Capacitor Polar substance

So polar and non-polar molecules behave in a similar manner when subjected to an external electric field.

Polarisation of a Dielectric:

A conductor In an external electric field: If a conductor is placed in an external electric field the free electrons of the conductor orient themselves in a direction opposite to the electric field. This transfer continues until finally, the induced electric field balances the external electric field. In that case, no further displacement of charges takes place i.e., an equilibrium has been reached. So the resultant electric field \(\vec{E}\) inside a conductor is zero.

Class 12 Physics Unit 1 Electrostatics Chapter 4 Capacitance and Capacitor A conductor In an external electric field

A dielectric in an electric field:

If a dielectric is placed in an external electric field \(\left(\vec{E}_0\right)\), the dipoles align themselves along the lines of force. So an electric field \(\left(\vec{E}_p\right)\) is generated inside the dielectric whose direction is opposite to that of the applied external field \(\left(\vec{E}_0\right) \cdot \vec{E}_P\) is less than \(\left(\vec{E}_0\right)\).

As a dielectric has no free electrons, the external field \(\left(\vec{E}_0\right)\) is not completely balanced by the internal field \(\left(\vec{E}_p\right)\) set up inside the dielectric. So at any point inside a dielectric the resultant intensity \((\vec{E})\) is less than the external field intensity \(\left(\vec{E}_0\right) \cdot \vec{E}_P\) but it does not become zero as in a conductor.

The alignment of the molecules of a dielectric, which behave like electric dipoles under the influence of an external field, is known as electric polarization.

It is observed that one face of the dielectric acquires a net positive charge and the other, a negative. This is because the charges between the two dotted lines neutralize each other’s effect, thus leaving an unbalanced negative charge on the left face and a positive charge on the right face of the dielectric,

Class 12 Physics Unit 1 Electrostatics Chapter 4 Capacitance and Capacitor A dielectric in an electric field

The random arrangement of the molecules of a dielectric has been shown in the absence of any external electric field. Alignment of the molecules along the field lines under the influence of the external field has been shown.

Shows that inside a dielectric, electric field intensity reduces due to electric polarization. The resultant intensity of the electric field inside the dielectric is given by

⇒ \(\vec{E}=\vec{E}_0-\vec{E}_p\)

The electric polarization is directly proportional to die resultant electric field In the dielectric

The capability of storing the charge of a capacitor, i.e., its capacitance can be increased by using a suitable dielectric substance between its two plates.

For Example, air, paraffin, glass, sulfur, mica, paper, etc. are the dielectric substances used as intervention medium in a parallel plate capacitor. The increase in the capacitance of a capacitor depends on a property of dielectric materials, termed as dielectric constant (k).

Definition: The dielectric constant of a material is the ratio of the capacitance of a capacitor filled with the given dielectric material to the capacitance of a similar capacitor without any medium.

So, dielectric constant,

k = \(\frac{capacitance of capacitor with the dielectric as the intervening medium}{capacitance of the same capacitor without anymedium}\)

The dielectric constant is also known as specific inductive capacity (SIC).

The capacitor without any medium between its two plates has only a vacuum between the plates.

By the statement that the dielectric constant of glass is 8.5, we mean that the capacitance of a capacitor will increase 8.5 times if glass is used as a dielectric instead of a vacuum. Naturally, the dielectric constant of a vacuum is 1. The dielectric constant of dry air is 1.000586(≈1).

Capacitance And Capacitor Capacitance Of Some Standard Capacitors

Parallel Plate Capacitor:

It consists of two similar metal plates held parallel to each other, separated by a certain distance. The space in between the two plates contains air or any dielectric, e.g., glass, mica, etc.

Consider two parallel plates A and B separated by a distance d. The area of each plate is a. Plate A is charged with a charge +Q while plate B is grounded. The dielectric constant of the medium between the plates is K.

Class 12 Physics Unit 1 Electrostatics Chapter 4 Capacitance and Capacitor Parallel Plate Capacitor

Now the surface density of charge on plate A will be

⇒ \(\sigma=\frac{Q}{\alpha}\)

The inner face of plate B is charged to -Q due to induction. If the area of the plates is large compared to the die distance between them, the electric lines of force between the plates are straight and parallel except near their ends.

Consequently, the intensity of the electric field between the plates may be taken to be uniform. The slight deviation from this uniformity near the edges may be neglected.

The plates A and B can be considered to be infinite plates with respect to any point in between the two plates. So, the electric field at that point due to the positive charge on plate A is,

⇒ \(E_1=\frac{\sigma}{2 \kappa \epsilon_0} \text {, along } A B\)

The electric field at that internal point will be in the same direction as AB, also due to the negative charge on plate B. Its magnitude is

⇒ \(E_2=\frac{\sigma}{2 \kappa \epsilon_0}\)

Therefore, at all points between the plates A and B, the electric field is

⇒ \(E=E_1+E_2=\frac{\sigma}{2 \kappa \epsilon_0}+\frac{\sigma}{2 \kappa \epsilon_0}=\frac{\sigma}{\kappa \epsilon_0}\)

If V is the potential difference between the plates, then

V = work done to bring a unit positive charge from plate B to plate A

= force acting on the unit charge x distance

= intensity of the electric field x distance

⇒ \(E \cdot d=\frac{\sigma}{\kappa \epsilon_0} \cdot d\)

If C is the capacitance of the capacitor, then

⇒ \(C=\frac{Q}{V}=\frac{\sigma \alpha}{\frac{\sigma}{\kappa \epsilon_0^V} \times d}=\frac{\kappa \epsilon_0 \alpha}{d}\)…(3)

⇒ \(\text { [In CGS units, } C=\frac{K \alpha}{4 \pi d} \text { ] }\)

For air, K = 1

Hence, \(C=\frac{\epsilon_0 \alpha}{d}=\frac{8.854 \times 10^{-12} \times \alpha}{d}\)…(4)

Dependence of the capacitance of a parallel plate capacitor on various factors:

Area of the plates: The capacitance is directly proportional to the area of the plates, i.e., C ∝ a.

1. Distance between the plates: The capacitance of a parallel plate capacitor is inversely proportional to the distance between the plates, i.e., C oc \(\frac{1}{d}\)

2. The nature of the medium between the plates: The capacitance is directly proportional to the permittivity or dielectric constant of the medium between the plates, i.e., C ∝ K.

3. The relation (3) clearly indicates that the capacitance does not depend on the charge Q or the potential V of the capacitor. Only the shape and the intervening medium determine its capacitance.

4. If the common overlapping area between the two plates can be changed by using a hinge arrangement, then changes, and as a result, the capacitance C changes. This technique may be used to design a capacitor, of variable capacitance.

Special case: If n number of parallel plates are alternately connected to form a multi-plate capacitor, the capacitance will be,

⇒ \(C=\frac{(n-1) \kappa \epsilon_0 \alpha}{d}\)

Class 12 Physics Unit 1 Electrostatics Chapter 4 Capacitance and Capacitor Special case

where a = area of each plate,

d = distance between two consecutive plates

The capacitance of a parallel plate capacitor with compound diet dice:

A and B are two parallel plates of which A is charged and B is grounded. Let a be the area of each plate, d the distance between them, and +Q the charge on plate A.

So the surface density of charge on each plate is \(\sigma=\frac{Q}{\alpha}\)

Class 12 Physics Unit 1 Electrostatics Chapter 4 Capacitance and Capacitor Capacitance of a parallel plate capacitor with compound dielectric

The space between the plates is now filled with two media of permittivites e1 and e2. The thickness of the two layers are (d-1) and t respectively. The intensity of the electric field between the plates may be taken as uniform. If the dielectric constants of the two media are k1 and k2, the intensity of the electric field will be given by,

⇒ \(E_1=\frac{\sigma}{\kappa_1 \epsilon_0}\)

and \(E_2=\frac{\sigma}{\kappa_2 \epsilon_0} ; \epsilon_0\) = permittivity of air or vacuum

∴ The potential difference between the plates Is,

V = E1(d-t) + E2t

⇒ \(\frac{\sigma}{\kappa_1 \epsilon_0}(d-t)+\frac{\sigma}{\kappa_2 \epsilon_0} t=\frac{\sigma}{\epsilon_0}\left[\frac{d-t}{\kappa_1}+\frac{t}{\kappa_2}\right]\)

So, the capacitance of the capacitor,

⇒ \(\frac{Q}{V}\)

⇒ \(\frac{\sigma \alpha}{\frac{\sigma}{\epsilon_0}\left[\frac{d-t}{\kappa_1}+\frac{t}{\kappa_2}\right]}=\frac{\epsilon_0 \alpha}{\left(\frac{d-t}{\kappa_1}+\frac{t}{\kappa_2}\right)}\)

If k1 = 1 (for air) and k2 = k (say),

⇒ \(C=\frac{\epsilon_0 \alpha}{d-t+\frac{t}{\kappa}}=\frac{\epsilon_0 \alpha}{d-\left(t-\frac{t}{\kappa}\right)}\)…(6)

1. Since \(\left(t-\frac{t}{K}\right)\) is a positive quantity, equation (6) shows that the capacitance of a parallel plate capacitor increases with the insertion of any dielectric medium between the plates.

2. If a dielectric of thickness t is inserted in between the two plates of a parallel plate air capacitor, the capacitance of the capacitor becomes equal to that of a capacitor having separation between the plates reduced by \(\left(t-\frac{t}{K}\right)\), i.e., the separation between the two plates effectively decreases by the amount \(\left(t-\frac{t}{K}\right)\).

If we want to get the previous value of the capacitance, the distance between the plates is to be increased. If this increase is x, then

⇒ \(x=t-\frac{t}{K}=t\left(1-\frac{1}{K}\right)\)

3. If n number of dielectric slabs of dielectric constants k1,k2,…,Kn of thicknesses t1, t2…., tn be inserted between the two parallel plates, the capacitance of the capacitor so formed is given by,

⇒ \(C=\frac{\epsilon_0 \alpha}{\frac{t_1}{\kappa_1}+\frac{t_2}{\kappa_2}+\cdots+\frac{t_n}{\kappa_n}}=\frac{\epsilon_0 \alpha}{\sum_1^n \frac{t}{\kappa}}\)

Energy Stored in a Charged Capacitor:

During the charging of an uncharged capacitor, electrons are removed from one plate and transferred to the other gradually. Initially, the charge of the capacitor is zero and so the potential difference between the plates is also zero. As soon as an electron is transferred from one plate to the other, an electric field builds up in the space between the capacitor plates.

This field opposes further transfer. Thus as the charge accumulates on the capacitor plates, increasingly larger amounts of work is to be done to transfer more electrons.

Hence the potential difference increases due to the accumulation of charges. The energy spent for doing that work remains stored as potential energy in the electric field between the two plates of the capacitor.

Calculation: Suppose, at any moment, the charge of a capacitor be q and the potential difference between the two plates is v.

The capacitance of the capacitor, C = \(\frac{q}{v}\)

Further, when some charge of amount dq is given to the capacitor, the work done against the repulsive force due to the existing charge on the capacitor plate,

⇒ \(d W=v d q=\frac{q}{C} d q\)

∴ To give Q the amount of charge, the total work done

⇒ \(W=\int d W\)

= \(\int_0^Q \frac{q}{C} \cdot d q\)

⇒ \(=\frac{1}{2} \cdot \frac{Q^2}{C}\)

= \(\frac{1}{2} C V^2\)

= \(\frac{1}{2} Q V\)

where V = final potential difference between the plates.

This work is stored as potential energy in the capacitor

Energy stored in a charged parallel plate capacitor:

Let us consider a charged parallel plate capacitor.

Here,

a = area of each plate,

d = separation between the plates,

Class 12 Physics Unit 1 Electrostatics Chapter 4 Capacitance and Capacitor Energy stored in a charged parallel plate capacitor

K = dielectric constant of the material between the plates,

σ = surface density of charge on each plate

So, volume between the plates = αd and amount of charge on each plate (Q) = σa.

The capacitance of this parallel plate capacitor,

⇒ \(C=\frac{K \epsilon_0 \alpha}{d}\)

where, ∈0 = permittivity of air or vacuum

Therefore, the energy stored in this charged capacitor,

⇒ \(U=\frac{1}{2} \frac{Q^2}{C}\)

= \(\frac{1}{2}\left(\sigma^2 \alpha^2\right) \frac{d}{\kappa \epsilon_0 \alpha}\)

= \(\frac{\sigma^2 \alpha d}{2 \kappa \epsilon_0}\)

The unit of U is joule (J). This energy is stored in the electric field between the plates of the capacitor.

[In the CGS system, the expression for U is obtained by replacing

⇒ \(\epsilon_0 \text { by } \frac{1}{4 \pi} . \text { So, } U=\frac{2 \pi \sigma^2 a d}{\kappa}\); its unit is erg.]

Energy stored per unit volume or energy density between the plates:

Energy stored per unit volume,

⇒ \(u=\frac{U}{\alpha d}=\frac{\sigma^2}{2 \kappa \epsilon_0}\)

This is called the energy density in the electric field of the
capacitor.

Now, the electric field is uniform, except at the ends, inside a parallel plate capacitor, provided the plate area is very large compared to the separation between the plates.

From Gauss’ theorem, we already know that the uniform electric field between the two plates of a parallel plate capacitor, neglecting end effects, is

⇒ \(E=\frac{\sigma}{\kappa \epsilon_0} ; \text { then } \sigma=\kappa \epsilon_0 E\)

So, the energy density between the two plates is,

⇒ \(u=\frac{\left(\kappa \epsilon_0 E\right)^2}{2 \kappa \epsilon_0}=\frac{1}{2} \kappa \epsilon_0 E^2\)…(1)

For vacuum or air, K = 1 . Then equation (1) becomes,

⇒ \(u=\frac{1}{2} \epsilon_0 E^2\)…(2)

[In the CGS system, the equation (1) and (2) become, due to the replacement of
\(\epsilon_0 \text { by } \frac{1}{4 \pi}, u=\frac{1}{8 \pi} \kappa E^2 \text { and } u=\frac{1}{8 \pi} E^2\)]

In SI, the unit of u is J.m-3 [In the CGS system, it is erg.cm-3 ]

Dimension of \(u=\frac{\text { dimension of energy }}{\text { dimension of volume }}\)

⇒ \(\frac{M L^2 T^{-2}}{L^3}\)

= \(M L^{-1} T^{-2}\)

The expressions (1) and (2) of energy density in an electric field have been derived by considering the special case of a parallel plate capacitor. However, it can be proved that these two expressions are quite general expressions – true not only for parallel plate capacitors but also for electric fields of any other type.

These expressions give the energy density, i.e. energy in a unit volume around any, point in an electric field of any type. The rigorous proofs of the expressions are beyond the scope of our present discussions.

Energy density around a point In an electric field.’ Suppose, a point charge q is placed at a point 0. Another point P in air, is at a distance r from 0. Then the electric field at P,

⇒ \(E=\frac{1}{4 \pi \epsilon_0} \frac{q}{r^2}\)

So, the energy density of the tire electric field around point P is,

⇒ \(u=\frac{1}{2} \epsilon_0 E^2=\frac{1}{2} \epsilon_0 \frac{q^2}{16 \pi^2 \epsilon_0^2 r^4}=\frac{q^2}{32 \pi^2 \epsilon_0 r^4}\)

Then, in a small volume v around the point P, the energy stored is,

⇒ \(U=u v=\frac{q^2 v}{32 \pi^2 \epsilon_0 r^4}\)

Class 12 Physics Unit 1 Electrostatics Chapter 4 Capacitance and Capacitor Energy density around a point In an electric field

Capacitance And Capacitor Capacitance Of Some Standard Capacitors Numerical Examples

Example 1. The area of each plate of a parallel plate glass capacitor is 314 cm2. Its plates are separated distance 1cm. What will be the radius of a sphere having a capacitance equal to that of this capacitor? [k of glass = 8 ].
Solution:

The capacitance of the sphere

⇒ \(C=\frac{\kappa \alpha}{4 \pi d}=\frac{8 \times 314}{4 \times \pi \times 1} \approx 200 \mathrm{statF}\)

So, the radius of the sphere = 200 cm.

Example 2. The area of each plate of a parallel plate capacitor is 22 cm² and the plates are kept separated by a paraffin paper of thickness 1 mm. The specific inductive capacity (SIC) of paraffin paper is 2. What are the capacitance of the capacitor and the surface density of charge under a potential difference of 330 V?
Solution:

The capacitance of the capacitor

⇒ \(C=\frac{\kappa \alpha}{4 \pi d} \quad\left[\text { Here, } \kappa=2 ; \alpha=22 \mathrm{~cm}^2 ; d=0.1 \mathrm{~cm}\right]\)

⇒ \(\frac{2 \times 22}{4 \pi \times 0.1}=35 \mathrm{statF}\)

∴ \(Q=C V=35 \times \frac{330}{300} {statC}\left[∵ V=330 \mathrm{~V}=\frac{330}{300} \text { statV }\right]\)

∴ Surface density of charge,

⇒ \(\sigma=\frac{Q}{\alpha}=\frac{35 \times 330}{300 \times 22}\)

= 1.75 statC.cm-2

Example 3. The distance between the two plates of a parallel plate air capacitor is d. A piece of metal of thickness \(\frac{d}{2}\) and of area equal to that of the plates is inserted between the plates. Compare the capacitances in the two cases.
Solution:

In the first case, the capacitance of the parallel plate capacitor,

⇒ \(C_1=\frac{\epsilon_0 \alpha}{d}\)

We know that the intensity of the electric field inside a metal is zero. So, if a piece of metal of thickness \(\frac{d}{2}\) is inserted between the plates now becomes \(\left(d-\frac{d}{2}\right)=\frac{d}{2}\). Therefore, the capacitance of the capacitor becomes

⇒ \(C_2=\frac{\epsilon_0 \alpha}{d / 2}=\frac{2 \epsilon_0 \alpha}{d}\)

∴ \(\frac{C_1}{C_2}=\frac{\left(\epsilon_0 \alpha\right) / d}{\left(2 \epsilon_0 \alpha\right) / d}\)

= \(\frac{1}{2}\)

Example 4. The conducting plates of a parallel plate capacitor are separated by 2 cm from each other. A dielectric slab (K = 5) of thickness 1 cm is inserted between the two plates. The distance between the plates is now so changed that the capacitance of the capacitor remains the same. What will be the new distance between the plates?
Solution:

Let the present distance between the plates be d. According to the question,

⇒ \(\frac{\epsilon_0 \alpha}{2}=\frac{\epsilon_0 \alpha}{\left[d-t+\frac{t}{\kappa}\right]}\) [t = thickness of the dielectric slab = 1 cm; K = 5 ]

or, \(\frac{1}{2}=\frac{1}{\left[d-1+\frac{1}{5}\right]}\)

or, \(2=\left[d-1+\frac{1}{5}\right]=d-\frac{4}{5}\)

or, d = \(\frac{14}{5}\)

= 2.8 cm

Example 5. The surface area of each plate of a parallel plate capacitor is 50 cm2. They are separated by 2mm in air. It is connected with a 100V power supply. Now a dielectric (K = 5) is inserted between its two plates. What will happen

  1. If the voltage source remains connected and
  2. If the voltage sources are absent during this insertion?

Solution:

We know, that the capacitance of a parallel plate capacitor is

⇒ \(C=\frac{k \epsilon_0 \alpha}{d}\)

For air, the capacitance,

⇒ \(C_1=\frac{\epsilon_0 \alpha}{d}\) [∈0 = 8.854 x 10-12 F.m-1, a = 50cm2 = 5 X 10-2m2, d = 2 mm = 2 X 10-3 m]

⇒ \(\frac{8.854 \times 10^{-12} \times 5 \times 10^{-3}}{2 \times 10^{-3}}\)

= 2.21 x 10-11 F

For the dielectric (k = 5), the capacitance,

⇒ \(C_2=\frac{\kappa \epsilon_0 \alpha}{d}\)

= 5 x 2.21 x 10-11

= 1.11 x 10-10 F

1. If a dielectric is inserted, the capacitance of a parallel plate capacitor increases. Since the capacitor is still connected to the power supply, its potential will remain constant.

When the intervening medium is air, charge on the capacitor,

Q0 = capacitance x potential

= 2.21 X 10-11 X 100

= 2.21 X 10-9 C

When the intervening medium is the dielectric (k = 5), charge on the capacitor,

Q = 1.11 x 10-10 x 100

= 1.11 x 10-8C

∴ Change in charge of the capacitor

= Q – Q0

= 1.11 x 10-8 – 2.21 x 10-9

= 8.89 x 10-9 C

Change in potential difference = 0.

2. If the battery is removed, the charge stored remains the same.

So change in charge of the capacitor = 0.

According to the question, the potential difference between the plates of the capacitor before the insertion of the dielectric = 100 V.

After the insertion of the dielectric, the potential difference between the plates is,

⇒ \(V=\frac{Q_0}{C_2}=\frac{2.21 \times 10^{-9}}{1.11 \times 10^{-10}}\)

= 19.91 V

So, the potential difference decreases.

Change in potential difference

= 100 – 19.91

= 80.09 V

Example 6. Each of the two square plates of a capacitor has sides of length l. The angle d between the two plates is very small, If the medium between the plates is air and the minimum distance between them is t, determine the capacitance of the capacitor

Class 12 Physics Unit 1 Electrostatics Chapter 4 Capacitance and Capacitor Example 6 two square plates

Solution:

The average distance between the plates,

⇒ \(d=\frac{t+(t+l \sin \theta)}{2}=t+\frac{l}{2} \theta\) [∵ θ is very small, sinθ ≈ 0]

∴ The capacitance of the capacitor,

⇒ \(C=\frac{\epsilon_0 \alpha}{d}=\frac{\epsilon_0 l^2}{t+\frac{l}{2} \theta}=\frac{\epsilon_0 l^2}{t\left(1+\frac{l \theta}{2 t}\right)}=\frac{\epsilon_0 l^2}{t}\left(1+\frac{l \theta}{2 t}\right)^{-1}\)

⇒ \(\approx \frac{\epsilon_0 l^2}{t}\left(1-\frac{l \theta}{2 t}\right)\)

Example 7. A parallel plate air capacitor has a capacitance of 2pF. Now, the separation between the plates is doubled, and the space is filled with wax. If the capacitance rises to 6 pF, what is the dielectric constant of wax?
Solution:

Initial separation between the plates = d; area of each plate = α.

∴ Capacitance in the 1st case,

⇒ \(C_1=\frac{\epsilon_0 \alpha}{d}\)

The final separation between the plates = 2d; dielectric constant of wax = k.

∴ Capacitance in the 2nd case,

⇒ \(C_2=\frac{\kappa \epsilon_0 \alpha}{2 d}\)

∴ \(\frac{C_1}{C_2}=\frac{2}{\kappa} \text { or, } \kappa=2 \times \frac{C_2}{C_1}=2 \times \frac{6 \mathrm{pF}}{2 \mathrm{pF}}=6\)

Example 8. The area of each plate of a parallel plate capacitor is A – 600 cm² and their separation is d = 2.0 mm. The capacitor is connected to a 200 V DC source. Find out

  1. The uniform electric field between the plates In the SI unit and
  2. The surface density, of charge on any plate. Given, ∈0 = 8.85 x 10-12F m-1

Solution:

Here, A = 600 cm2

= 600 x 10-4m2

= 6 x 10-2m2;

d = 2.0 mm

= 2 x 10-3 m.

1. The uniform electric field between the plates,

⇒ \(E=\frac{V}{d}=\frac{200}{2 \times 10^{-3}}=10^5 \mathrm{~V} \cdot \mathrm{m}^{-1}\)

2. If cr = surface density of charge on any plate, then

⇒ \(E=\frac{\sigma}{\epsilon_0}\)

∴ \(\sigma=\epsilon_0 E=\left(8.85 \times 10^{-12}\right) \times 10^5\)

= 8.85 x 10-7 C m-2

Example 9. The potential of a capacitor increases from zero to 150 V when a charge of 10 esu Is imparted to it. What will be the energy stored in the capacitor?
Solution:

Energy stored within the capacitor

⇒ \(\frac{1}{2} Q V=\frac{1}{2} \times 10 \times \frac{150}{300}\)

= 2.5 erg.

Example 10. Find out the energy content in a volume of 1 cm³ around a point, situated in the electric field of a point charge of 10 C, at a distance of 2 m in air from the position of the point charge. Given,∈0 = 8.85 x 10-12 F.m-1
Solution:

The electric field at the referred point due to the point charge,

⇒ \(E=\frac{1}{4 \pi \epsilon_0} \frac{q}{r^2}[\text { Here, } q=10 \mathrm{C}, r=2 \mathrm{~m}]\)

∴ Energy density at the point

⇒ \(u=\frac{1}{2} \epsilon_0 E^2=\frac{1}{2} \epsilon_0 \cdot \frac{1}{16 \pi^2 \epsilon_0^2} \frac{q^2}{r^4}=\frac{q^2}{32 \pi^2 \epsilon_0 r^4}\)

∴ The energy content in a volume of 1 cm3, i.e., 10-6 m3,

U = u x 10-6

⇒ \(\frac{q^2}{32 \pi^2 \epsilon_0 r^4} \times 10^{-6}\)

⇒ \(\frac{10^2 \times 10^{-6}}{32 \times(3.14)^2 \times\left(8.85 \times 10^{-12}\right) \times 2^4} \approx 2240 \mathrm{~J}\)

Example 11. Estimate the percentage change of the energy stored in a parallel plate capacitor, if the separation between its plates is reduced by 10%, keeping the voltage of the charging source unchanged.
Solution:

Let d = initial separation between the plates.

∴ Final separation,

d’ = d – (10% of d)

= d – \(\frac{d}{10}\)

= 0.9d

The initial and final values of the energy stored in the capacitor are,

⇒ \(U=\frac{1}{2} C V^2=\frac{1}{2} \frac{\epsilon_0 \alpha}{d} V^2 \text { and } U^{\prime}=\frac{1}{2} \cdot \frac{\epsilon_0 \alpha}{0.9 d} V^2\)

∴ Percentage increase in energy

⇒ \(\frac{U^{\prime}-U}{U} \times 100=\left(\frac{U^{\prime}}{U}-1\right) \times 100=\left(\frac{1}{0.9}-1\right) \times 100\)

⇒ \(\frac{0.1}{0.9} \times 100=\frac{100}{9}\)

= 11.1 %

Example 12. A 900 pF capacitor is charged to 100 V by a battery. How much energy is stored in the capacitor?
Solution:

Here, C = 900 pF

= 900 x 10-12F

=9 x 10-10F;

Energy stored within the capacitor,

E = \(\frac{1}{2}\)CV2

= \(\frac{1}{2}\) x (9 X 1010) X (100)2

= 4.5 x 10-6J

Example 13. The capacitance of a parallel plate air capacitor is C. The capacitor is immersed halfway into an oil of dielectric constant 1.6 with the plates perpendicular to the surface of the oil. What will be the capacitance of this capacitor?
Solution:

The capacitance of the half of the capacitor immersed in oil,

⇒ \(C_1=\frac{\kappa \epsilon_0}{d} \cdot \frac{\alpha}{2}=\frac{1.6 \epsilon_0}{d} \cdot \frac{\alpha}{2}\)

The capacitance of the other half of the capacitor in air,

⇒ \(C_2=\frac{\epsilon_0}{d} \cdot \frac{\alpha}{2}\)

Net capacitance = \(C_1+C_2=\frac{1.6 \epsilon_0}{d} \cdot \frac{\alpha}{2}+\frac{\epsilon_0}{d} \cdot \frac{\alpha}{2}=\frac{2.6 \epsilon_0}{d} \cdot \frac{\alpha}{2}\)

⇒ \(\frac{1.3 \epsilon_0}{d} \cdot \alpha=1.3 C\left[∵C=\frac{\epsilon_0 \alpha^2}{d}\right]\)

Class 12 Physics Unit 1 Electrostatics Chapter 4 Capacitance and Capacitor Example 13 The capacitance of a parallel plate air capacitor

Capacitance And Capacitor Combination Of Capacitors

Series combination: in this type of combination of capacitors, the first plate of the first capacitor is joined to the electric source, its second plate is joined to the first plate of the second capacitor, the second plate of the second capacitor is joined to tire first plate of tire third capacitor and so on. The second plate of the last capacitor is grounded, the rest of the system being kept insulated.

Class 12 Physics Unit 1 Electrostatics Chapter 4 Capacitance and Capacitor Series combination

Calculation of equivalent capacitance: Let three capacitors of capacitances C1, C2, and C3 be connected in series. Now a charge +Q be given from a source to the first plate A of the first capacitor, this will induce a charge -Q on the other plate B of this capacitor and a charge +Q on the first plate C of the second capacitor, and so on. All the capacitors will have the same charge Q. The final free positive charge from the last plate of the system moves to the earth.

If V1, V2, and V3 are the potential differences across the capacitors C3, C2, and C3 and V is the potential difference between the first plate A and the last plate F of the combination, then

V= V1+ V2 + V3 …(1)

For the first capacitor, \(V_1=\frac{Q}{C_1}\)

For the second capacitor, \(V_2=\frac{Q}{C_2}\)

For the third capacitor, \(V_3=\frac{Q}{C_3}\)

∴ From equation (1) we have,

⇒ \(V=\frac{Q}{C_1}+\frac{Q}{C_2}+\frac{Q}{C_3}\)

or, \(V=Q\left(\frac{1}{C_1}+\frac{1}{C_2}+\frac{1}{C_3}\right)\)…..(2)

Now suppose that instead of this combination, a single capacitor is used such that the same charge (Q) given to this new capacitor produces the same potential difference ( V) between its two plates. This single capacitor is known as the equivalent capacitor of the combination and its capacitance is known as the equivalent capacitance

If C is the equivalent capacitance of the series combination of the capacitors C1, C2, and C3, then

V = \(\frac{Q}{C}\)…(3)

From equations (2) and (3) we get,

⇒ \(\frac{Q}{C}=Q\left(\frac{1}{C_1}+\frac{1}{C_2}+\frac{1}{C_3}\right)\)

or, \(\frac{1}{C}=\frac{1}{C_1}+\frac{1}{C_2}+\frac{1}{C_3}\)…(4)

For n number of capacitors connected in series, the equivalent capacitance C is given by,

⇒ \(\frac{1}{C}=\frac{1}{C_1}+\frac{1}{C_2}+\cdots+\frac{1}{C_n}\)…(5)

Thus if several capacitors are connected in series, the reciprocal of the capacitance of the equivalent capacitor is equal to the sum of the reciprocals of the capacitances of the individual capacitors.

Now, \(\frac{1}{C}=\frac{1}{C_1}+\frac{1}{C_2}+\frac{1}{C_3}+\cdots+\frac{1}{C_n}\)

∴ \(\frac{1}{C}>\frac{1}{C_1}, \frac{1}{C_2}, \frac{1}{C_3}, \cdots, \frac{1}{C_n}\)

or, \(C<C_1, C_2, C_3, \cdots, C_n\)

Clearly, for series combinations, the equivalent capacitance is always less than any individual capacitance in the series.

Parallel combination:

In this type, of combination the first plates, i.e., the insulated plates of all the capacitors are connected to a common point A, and the second plates, i.e., the grounded plates to another common point B. Point A is connected to an electric source and point B is earthed

Class 12 Physics Unit 1 Electrostatics Chapter 4 Capacitance and Capacitor Series combination

Calculation of equivalent capacitance: Three capacitors of capacitances C1, C2, and C3 connected in parallel. The insulated plates of the three capacitors are connected to an electric source having potential V and other plates are earthed. So the potential difference between the two plates of each capacitor is V. A charge +Q drawn from the supply divides into Q1, Q2, and Q3 according to the capacity of the different capacitors. So,

Q = Q1 + Q2+ Q3….(6)

For the first capacitor,

Q1 = C1V

For the second capacitor,

Q2 = C2V

For the third capacitor,

Q3 = C3V

∴ From the equation (6) we get,

Q = C1V + C2V + C3V

or, Q = V(C1 + C2 + C3)….(7)

If the capacitors connected in parallel are replaced by a single capacitor so that the same potential difference V is produced if charge +Q is given to its insulated plate, the single capacitor is known as the equivalent capacitor of the combination and its capacitance is known as the equivalent capacitance. If C is the equivalent capacitance of the parallel combination of the capacitors C1, C2, and C3, then

Q = CV…..(8)

From equations (7) anil1(8) we have

CV = V(C1 + C2 + C3)

or, C = C1 + C2 + C3….(9)

For n number of capacitors connected in parallel, the equivalent capacitance C is

C = C1 + C2 + …. + Cn ,

Thus the equivalent capacitance of the capacitors joined in parallel is equal to the sum of their individual capacitances.

Clearly, the equivalent capacitance of a number of capacitors in parallel is greater than any individual capacitance in the combination.

Capacitors are connected in parallel when a large capacitance for a small potential is required.

Unit 1 Electrostatics Chapter 4 Capacitance And Capacitor Combination Of Capacitors Numerical Examples

Example 1. H A condenser Is composed of 21 circular plates placed one after the other. The diameter of each plate is 10 cm. The consecutive plates are separated by 0.2 mm thick mica sheets of dielectric constant 6. If the alternate circular plates are connected, calculate the capacitance of the condenser μF.
Solution:

The condenser is composed of 21 circular plates and alternate plates are connected. So here we get 20 identical capacitors connected in parallel whose capacitance is

⇒ \(C=\frac{20 \kappa \epsilon_0 \alpha}{d}\)

⇒ \(\frac{20 \times 6 \times 8.854 \times 10^{-12} \times \pi \times 2.5 \times 10^{-3}}{2 \times 10^{-4}}\)

= 4.17 x 10-8

F = 0.0417μF

[Here, k = 6, a = n x (5)2 cm2 = n x 2.5 x 10-3 m2, d = 2 x 10-4m, ∈0 = 8.854 x 10-12 C2.N-1.m2 ]

Example 2. A condenser is composed of 200 circular tin plates placed one after the other. The consecutive plates are separated by 0.5 mm thick mica sheets of dielectric constant 6. If the alternate tin plates are connected and the capacitance of the entire condenser is 0.4μF, what is the radius of each tin plate?
Solution:

The condenser is composed of 200 circular plates and alternate plates are connected. So, here we get 199 identical capacitors connected in parallel. Now capacitance of each capacitor is,

⇒ \(C=\frac{\kappa \alpha}{4 \pi d}=\frac{\kappa \pi r^2}{4 \pi d}=\frac{\kappa r^2}{4 d}\)

∴ \(\frac{\kappa r^2}{4 d} \times 199=0.4 \times 10^{-6} \times 9 \times 10^{11}\) [∵ 0.4μF =0.4 x 10-6F and IF = 9 x 1011 statF]

or, \(\frac{6 r^2}{4 \times 0.05} \times 199=3.6 \times 10^5\)

or, \(r^2=\frac{3.6 \times 10^5 \times 4 \times 0.05}{6 \times 199}\)

or, r-2 = 60.3

or, r = 7.76

So, the radius of each tin plate = 7.76 cm.

Example 3. The equivalent capacitances of the parallel and the series combinations of two capacitors are 5μF and 1.2μF, respectively. Calculate the capacitances of each capacitor
Solution:

Let the capacitances of the two capacitors be C1μF and C2μF. According to the question,

C1 + C2 = 5…(1)

and \(\frac{C_1 C_2}{C_1+C_2}=1.2\)

or, C1C2 = 1.2 x 5

= 6 ….(2)

or, C1(5-C1)-6 = 0 [with the help of equation (1)]

or, \(C_1^2-5 C_1+6=0\)

or, (C1-3)(C1-2) = 0

∴ C1 = 3 or, 2

If C1 = 3;

C2 = 5-3

= 2

and if C1 = 2;

C2 = 5-2

= 3

So, the capacitances of the two capacitors are 3μF and 2μF.

Example 4. Two capacitors of capacitances 20μF and 60μF are connected in series. If the potential difference between the two ends of the combination is 40 V, calculate the terminal potential differences of each capacitor
Solution:

If C is the equivalent capacitance of the combination,
then

⇒ \(C=\frac{20 \times 60}{20+60}=15 \mu \mathrm{F}=15 \times 10^{-6} \mathrm{~F}\)

∴ The total charge of the combination,

Q = CV

= 15 x 10-6 x 40

= 6 x 10-4C

Since the two capacitors are connected in series, the charge on each capacitor is equal to the total charge of the combination, i.e., 6 x 10-4C.

∴ The potential difference between the two plates of the capacitor having capacitance C1 is,

⇒ \(V_1=\frac{Q}{C_1}=\frac{6 \times 10^{-4}}{20 \times 10^{-6}}\)

= 30 V

Again, the potential difference between the two plates of the capacitor having capacitance C2 is,

⇒ \(V_2=\frac{Q}{C_2}=\frac{6 \times 10^{-4}}{60 \times 10^{-6}}\)

= 10V

Example 5. A charged condenser is made to share its charge with an uncharged condenser of twice its capacitance. Find the sum of the energy of the two condensers.
Solution:

Let the capacitance of the charged condenser be C and let its charge be Q.

Before sharing of charge, the energy of the charged condenser is,

⇒ \(E_1=\frac{Q^2}{2 C}\)

The capacitance of the other condenser = 2C; since the charged condenser is made to share its charge with the second condenser, it is clear that the condensers are connected in parallel with each other.

∴ Equivalent capacitance of them

= C + 2C

= 3C

∴ Energy of the combination = \(=\frac{1}{2} \cdot \frac{Q^2}{3 C}=\frac{1}{3} \cdot \frac{1}{2} \frac{Q^2}{C}=\frac{1}{3} E_1\)

Example 6. A spherical drop of water carries a charge of 10 X 10-12 C and has a potential of 100 V at its surface.

  1. What is the radius of the drop?
  2. If eight such charged drops combine to form a single drop, what will be the potential at the surface of the new drop?

Solution:

1. Charge of a spherical water drop,

Q = 10 x 10-12C

= 10 x 10-12 x 3 x 109 esu of charge

= 3 x 10-2 esu of charge

Potential of the drop, \(V=\frac{100}{300}=\frac{1}{3} \text { stat } \mathrm{V}\)

The capacitance of the drop,

⇒ \(C=\frac{Q}{V}=\frac{3 \times 10^{-2}}{\frac{1}{3}}\)

= 0.09 statF

In CGS units, the radius of a spherical conductor = its capacitance.

The radius of the drop, r = 0.09 cm.

2. If R is the i&dius of the large drop, then

⇒ \(\frac{4}{3} \pi R^3=8 \times \frac{4}{3} \pi r^3\)

or, R = 2r

= 2x 0.09

= 0.18 cm

Total charge, Q = 8 x 3 x 10-2

= 24 x 10-2 statC

∴ Potential at the surface of the new drop,

⇒ \(V=\frac{Q}{C}=\frac{Q}{R}=\frac{24 \times 10^{-2}}{0.18}=\frac{4}{3}\)

= 1.33 statV

= 1.33 X 300 V

= 400 V

Example 7. Three plates of the same size form a system of capacitors. Each plate has an area a. The intermediate differences between the plates are d1 and d2, respectively. The space between the first two plates is occupied by a dielectric of constant and that between the second and third plates by a dielectric of constant K2. Calculate the capacitance of the system.
Solution:

The three plates form two capacitors connected in series.

The capacitance of the first capacitor,

⇒ \(C_1=\frac{\kappa_1 \epsilon_0 \alpha}{d_1}\)

The capacitance of the second capacitor

⇒ \(C_2=\frac{\kappa_2 \epsilon_0 \alpha}{d_2}\)

If C is the equivalent capacitance of the whole system, then

⇒ \(\frac{1}{C}=\frac{1}{C_1}+\frac{1}{C_2}=\frac{d_1}{\kappa_1 \epsilon_0 \alpha}+\frac{d_2}{\kappa_2 \epsilon_0 \alpha}=\frac{1}{\epsilon_0 \alpha}\left(\frac{d_1}{\kappa_1}+\frac{d_2}{\kappa_2}\right)\)

∴ \(C=\epsilon_0 \alpha \cdot \frac{1}{\frac{d_1}{\kappa_1}+\frac{d_2}{\kappa_2}}\)

Class 12 Physics Unit 1 Electrostatics Chapter 4 Capacitance and Capacitor Example 7 Three plates of the same size form a system of capacitors

Example 8. The capacitance of a parallel plate air capacitor is 9 pF. The separation between the plates is d. The intermediate space is filled up by two dielectric media. The widths of them are,\(\frac{d}{3} \text { and } \frac{2 d}{3}\), and their dielectric constants are 3 and 6, respectively. Find the capacitance of the parallel plate capacitor.
Solution:

As the capacitors are connected in series the equivalent capacitance,

⇒ \(C_{\mathrm{eq}}=\frac{C_1 C_2}{C_1+C_2}\)

We know that,

⇒ \(C_1=\kappa_1 \frac{\epsilon_0 A}{\frac{d}{3}} \text { and } C_2=\kappa_2 \frac{\epsilon_0 A}{\frac{2 d}{3}}\) [where A = area of each plate]

Class 12 Physics Unit 1 Electrostatics Chapter 4 Capacitance and Capacitor Example 8 The capacitance of a parallel plate air capacitor

Putting the values of C1 and C2 in equation (1),

⇒ \(C_{\mathrm{eq}}=\frac{\frac{\epsilon_0 A}{\frac{d}{3}} \times \frac{\epsilon_0 A}{\frac{2 d}{3}} \times \kappa_1 \kappa_2}{\frac{\epsilon_0 A}{\frac{d}{3}}\left(\kappa_1+\frac{\kappa_2}{2}\right)}\)

∵ \(\frac{\epsilon_0 A}{d}=9 \mathrm{pF} \text { and } \kappa_1=3, \kappa_2=6\)

So, \(C_{\text {eq }}=\frac{3 \times 9 \times 3 \times \frac{9}{2} \times 3 \times 6}{3 \times 9\left(3+\frac{6}{2}\right)}\)

= 40.5 pF

Example 9. Three capacitors having capacitances 1μF, 2μF, and 3μF are Joined in series. A potential difference of 1100 V is applied to the combination. Find the charge and potential difference across each capacitor.
Solution:

If C is the equivalent capacitance of the combination, then,

⇒ \(\frac{1}{C}=\frac{1}{1}+\frac{1}{2}+\frac{1}{3}=\frac{11}{6} \text { or, } C=\frac{6}{11} \mu \mathrm{F}=\frac{6}{11} \times 10^{-6} \mathrm{~F}\)

∴ The total charge of the combination,

Q = CV

= \(\frac{6}{11}\) x 10-6 x 1100

= 6 x 10-4 c

Since the capacitors are connected in series, the charge on each capacitor is equal to the total charge, i.e., 6 x 10-4 C. Potential difference across the plates of the first capacitor,

⇒ \(V_1=\frac{Q}{C_1}=\frac{6 \times 10^{-4}}{1 \times 10^{-6}}=600 \mathrm{~V}\)

Similarly, for the other two capacitors, respectively,

⇒ \(V_2=\frac{Q}{C_2}=\frac{6 \times 10^{-4}}{2 \times 10^{-6}}=300 \mathrm{~V}\)

and, \(V_3=\frac{Q}{C_3}=\frac{6 \times 10^{-4}}{3 \times 10^{-6}}=200 \mathrm{~V}\)

Example 10. Two capacitors having capacitances 0.1μV and 0.01μF are joined in series. A potential difference of 22 V is applied to the combination. If the capacitors are now joined in parallel, what will be the change in stored energy?
Solution:

If the capacitors are joined in series, their equivalent capacitance,

⇒ \(C_s=\frac{C_1 \times C_2}{C_1+C_2}=\frac{0.1 \times 0.01}{0.1+0.01}=\frac{1}{110} \mu \mathrm{F}=\frac{10^{-6}}{110} \mathrm{~F}\)

∴ Energy stored in the combination,

⇒ \(E_1=\frac{1}{2} C_s V^2=\frac{1}{2} \times \frac{10^{-6}}{110} \times(22)^2 \mathrm{~J}\)

= 22 erg

If the capacitors are joined in parallel, their equivalent capacitance,

Cp = C1 + C2

= 0.1 + 0.01

= 0.11μF

= 0,11 x 10-6F

∴ Energy stored in this combination,

⇒ \(E_2=\frac{1}{2} C_p V^2=\frac{1}{2} \times 0.11 \times 10^{-6} \times(22)^2 \mathrm{~J}\)

= 266.2 erg

The change in stored energy = E2-E1

= 266.2- 22

= 244.2 erg

∴ Stored energy increases by 244.2 erg.

Example 11. Five capacitors have been arranged in a circuit. The capacitance of each capacitor is C. Determine the effective capacitance between points A and B.

Class 12 Physics Unit 1 Electrostatics Chapter 4 Capacitance and Capacitor Example 11 Five capacitors have been arranged in a circuit

Solution:

The equivalent circuit is like that,

Class 12 Physics Unit 1 Electrostatics Chapter 4 Capacitance and Capacitor Example 11 Equivalent circuitc

From symmetries see that P and Q are equipotential points. So no current will pass through C3.

So, the effective capacitance between A and B \(=\frac{C}{2}+\frac{C}{2}=C\)

Example 12. Show that the equivalent capacitance of an infinite circuit formed by the repetition of a similar loop made of two similar capacitors, each of capacitance C = (√5 + l) μF, is 2μF.

Class 12 Physics Unit 1 Electrostatics Chapter 4 Capacitance and Capacitor Example 12 the equivalent capacitance

Solution:

Let the capacitance of the infinite circuit be C’.

So the circuit on the right-hand side starting from the second loop is also infinite. Therefore, its capacitance will also be C’.

In this case, the equivalent circuit will be that

Class 12 Physics Unit 1 Electrostatics Chapter 4 Capacitance and Capacitor Example 12 the equivalent capacitance.

∴ \(\frac{1}{C^{\prime}}=\frac{1}{C}+\frac{1}{C^{\prime}+C}\)

or, \(\frac{1}{C^{\prime}}-\frac{1}{C^{\prime}+C}=\frac{1}{C}\)

or, \(\frac{C^{\prime}+C-C^{\prime}}{C^{\prime}\left(C^{\prime}+C\right)}=\frac{1}{C}\)

or, \(C^{\prime 2}+C^{\prime} C-C^2=0\)

or, \(C^{\prime}=\frac{-C \pm \sqrt{C^2+4 C^2}}{2}=\frac{(-1 \pm \sqrt{5}) C}{2}\)

Neglecting the negative value of C’ we have

⇒ \(C^{\prime}=\frac{(\sqrt{5}-1) C}{2}=\frac{(\sqrt{5}-1)}{2} \cdot(\sqrt{5}+1)\)

= 2μF

Example 13. Twelve capacitors, each of capacitance 10μF, are inserted at the sides of a cube made of conducting wires. Determine the equivalent capacitance between A and B.

Twelve capacitors, each of capacitance

Solution:

Let a charge Q be given to the circuit from a source. The distribution of charge in the different capacitors has been shown

Let C’ be the equivalent capacitance between the points A and B.

Capacitance of each capacitor = C = 10μF

∴ \(V_{\text {A}}-V_B=\frac{Q}{C^{\prime}}\)….(1)

Again considering the path AMNB we have,

⇒ \(V_A-V_B=\frac{Q}{\frac{3}{C}}+\frac{Q}{C}+\frac{Q}{C}=\frac{Q}{3 C}+\frac{Q}{6 C}+\frac{Q}{3 C}=\frac{5}{6} \cdot \frac{Q}{C}\)…(2)

Class 12 Physics Unit 1 Electrostatics Chapter 4 Capacitance and Capacitor Example 13 Twelve capacitors, each of capacitance.

∴ From equations (1) and (2) we have,

⇒ \(\frac{Q}{C^{\prime}}=\frac{5}{6} \cdot \frac{Q}{C}\)

or, \(C^{\prime}=\frac{6 C}{5}=\frac{6}{5} \times 10\)

= 12μF

Example 14. Two identical parallel plate air capacitors are connected to a battery. At first, the key S is closed and then It is opened. The spaces between the two capacitors are now filled up with a dielectric trie, having dielectric constant 3. Determine the ratio of the energy stored in the two capacitors before and after insertion of the dielectric.

Capacitance and Capacitor two identical parallel plate air capacitors

Solution:

When the key was closed, the potential of each capacitor was equal.

Let this value ofpotential be V.

Energy of the capacitor A = \(\frac{1}{2}\) CV²

and energy of the capacitor B = \(\frac{1}{2}\) CV²

∴ Total energy = \(\frac{1}{2} C V^2+\frac{1}{2} C V^2=C V^2\)

When the key is opened and the intervening space is filled up with the dielectric, the capacitance of each capacitor becomes 3C. Since the capacitor A is still connected to the battery, its potential remains at V.

∴ The energy of the capacitor A

⇒ \(\frac{1}{2} \times 3 C \times V^2\)

= \(\frac{3}{2} C V^2\)

The total charge of the capacitor B, Q = CV

This charge of B remains unaltered even after opening the key and inserting the dielectric.

Energy of \(B=\frac{1}{2} \cdot \frac{Q^2}{3 C}\)

= \(\frac{1}{2} \frac{C^2 V^2}{3 C}\)

= \(\frac{C V^2}{6}\)

∴ Energy of the two capacitors \(\frac{3}{2} C V^2+\frac{1}{6} C V^2\)

= \(\frac{5}{3} C V^2\)

∴ The required ratio = \(\frac{C V^2}{\frac{5}{3} C V^2}\)

= \(\frac{3}{5}\)

Example 15. Determine the capacitance A of the capacitor C when the equivalent capacitance between A and B is 1μF. The unit of all the capacitances is μF.

Capacitance and Capacitor equivalent capacitance

Solution:

The two capacitors of capacitance 6μF and 12μF are connected in series. If C1 is their equivalent capacitance, then

⇒ \(C_1=\frac{6 \times 12}{6+12}=4 \mu \mathrm{F}\)

This C1 and the capacitor of capacitance 4μF are connected in parallel. If C2 is their equivalent capacitance, then

C2 = 4 + 4

= 8μF

Again C2 and the capacitor of capacitance 1μF are connected in series. If C3 be their equivalent capacitance, then

⇒ \(C_3=\frac{8 \times 1}{8+1}=\frac{8}{9} \mu \mathrm{F}\)

On the other side, the capacitors of capacitance 2μF each arein parallel. If C4 is their equivalent capacitance, then

C4 = 2 + 2

= 4μF

Again C4 and the capacitor of capacitance 8mmF are in series.If C5 be their equivalent capacitance, then

⇒ \(C_5=\frac{4 \times 8}{4+8}=\frac{8}{3} \mu \mathrm{F}^{\prime}\)

Now C3 and C5 are in parallel combination. Their equivalent capacitance is given by,

⇒ \(C_6=\frac{8}{9}+\frac{8}{3}=\frac{32}{9} \mu \mathrm{F}\)

This C6 and C are in series and their equivalent capacitance = 1μF.

∴ \(\frac{\frac{32}{9} \times C}{\frac{32}{9}+C}=1\)

or, \(\frac{32}{9} \times C=\frac{32}{9}+C\)

or, \(\frac{23}{9} C=\frac{32}{9}\)

or, \(C=\frac{32}{23}=1.39 \mu \mathrm{F}\)

Example 16. Three capacitors A, B, and C are connected in such a way that their equivalent capacitance Is equal to the capacitance of B. The capacitances of A and B are 10μF and 30μF respectively and C ≠ 0. Determine three possible values of C and also show how the capacitors are to be connected in the three cases.
Solution:

The capacitor B cannot be joined in parallel to any combination of the capacitors A and C because in that case, the equivalent capacitance will be greater than the capacitance of B. The following three combinations are possible.

Class 12 Physics Unit 1 Electrostatics Chapter 4 Capacitance and Capacitor Example 16 equivalent capacitance

1. A and B are connected in series. Now C is connected in parallel with the series combination of A and B.

According to the question,

⇒ \(\frac{10 \times 30}{10+30}+C=30\)

or, \(\frac{300}{40}+C=30\)

or, C = 22.5μF

2. B and C are connected in series. Now A is connected in parallel with the series combination of B and C.

According to the question,

⇒ \(\frac{30 \times C}{30+C}+10=30\)

or, \(\frac{30 \times C}{30+C}=20\)

or, C = 60μF

3. A and B are connected in parallel. Now C is connected in series with the parallel combination of A and B.

According to the question,

⇒ \(\frac{40 \times C}{40+C}=30\)

or, 3(40 + C) = 4C

or, C = 120μF

Example 17. Two parallel plate capacitors of capacitances C and 2C are connected parallel and charged to a potential difference V. The battery then disconnected and the region between the plates of the capacitor C is completely filled with a material of dielectric constant k. What is the potential difference across the capacitors now?
Solution:

If charges Q1 and Q2 are given to the capacitors, then

Q1 = CV; Q2 = 2CV

∴ Total charge = Q1 + Q2

= CV+ 2CV

= 3CV

If the region between the plates of capacitor C is filled with a material of dielectric constant K, its capacitance will be kC.

∴ Total capacitance = kC+2C = (k + 2)C

∴ Potential difference across the capacitors = \(\frac{3 C V}{(\kappa+2) C}=\frac{3 V}{\kappa+2}\)

Example 18. A potential difference of 20 V is applied across a parallel combination of three Identical capacitors. If the total charge in the combination is 30 C, determine the capacitance of each capacitor. What will be the charge of the series combination of these three capacitors with the same potential difference?
Solution:

Let the capacitance of each capacitor be C.

The potential difference across each capacitor when they are connected in parallel combination, V = 20 V.

∴ Charge on the three capacitors,

Q1 = Q2 = Q3 = CV

∴ Q1 + Q2 + Q3 = 3CV

or, 30 = 3 X C X 20

= 60C

or, C = \(\frac{1}{2}\)

= 0.5F

∴ The capacitance of each capacitor = 0.5 F

In the second case when the capacitors are connected in series, let us suppose that the equivalent capacitance is C’

∴ \(\frac{1}{C^{\prime}}=\frac{1}{C}+\frac{1}{C}+\frac{1}{C}=\frac{3}{C}\)

or, \(C^{\prime}=\frac{C}{3}=\frac{0.5}{3}=\frac{1}{6} \mathrm{~F}\)

In this case, the total charge of the combination,

⇒ \(Q_i^{\prime}=C^{\prime} V\)

= \(\frac{1}{6} \times 20\)

= \(\frac{10}{3}\)

= 3.33 C

Example 19. The capacitance of a parallel plate air capacitor is C. Now, half the areas of its plates are vertically dipped in oil of dielectric constant 1.6. What would be Its capacitance?
Solution:

The upper half would still be an air capacitor. As the plate areas are halved, its capacitance, C1 = \(\frac{C}{2}\). The lower half, immersed in oil, would have a capacitance, C2 = \(\frac{kC}{2}\). C1 and C2 would form a parallel combination; the equivalent capacitance,

\(C^{\prime}=C_1+C_2=\frac{C}{2}+\frac{\kappa C}{2}=\frac{C}{2}(1+\kappa)\)

= \(\frac{C}{2}\)(1 + 1.6)

= 1.3C

Example 20. Each of the plates of a parallel plate capacitor is a circular disc of radius 5 cm. Find out its capacitance if the separation between the plates is 1 mm.
Solution:

The radius of each plate, r = 5 cm = 0.05 m.

Separation between the plates, d = 1 mm = 0.001 m.

∴ Capacitance, \(C=\frac{\epsilon_0 \alpha}{d}=\frac{\epsilon_0 \pi r^2}{d}\)

= \(\frac{\left(8.85 \times 10^{-12}\right) \times 3.14 \times(0.05)^2}{0.001}\)

= 6.95 X 100-11 F

= 69.5 pF

Example 21. A 2μF capacitor is charged to a potential of 20 V. Another 3μF uncharged capacitor is connected in parallel with the first capacitor. What would be the minimum potential difference of the combination? Find out the charges on the two capacitors
Solution:

Initially, the charge on the first capacitor,

Q1 = C1V1

= (2 x 10-6) x 20

= 40 x 10-6 C

and charge on the second capacitor, as it was uncharged, Q2 = 0.

∴ Net charge, Q = Q1 + Q2

= 40 x 10-6 C

Equivalent capacitance of the parallel combination,

C = C1 + C2

= (2 + 3)μF

= 5 X 10-6 F

∴ Terminal potential difference of the combination,

⇒ \(V=\frac{Q}{C}=\frac{40 \times 10^{-6}}{5 \times 10^6}=8 \mathrm{~V}\)

Charge on the first capacitor,

Q1 = C1V

= (2 x 10-6)x 8

= 16 x 10-6 C

= 16μC

and charge on the second capacitor,

Q2 = C2V

= (3 X 10-6) X 8

= 24 X 10-6 C

= 24pC

Example 22. A 20μF capacitor is charged to a potential of 20 V and is then connected to an uncharged 10μF capacitor. Find out the common potential and the ratio of the energies stored in the two capacitors.
Solution:

Here, the capacitance of the first capacitor,

C1 = 20μF

= 20 x 10-6 F

The capacitance of the second capacitor,

C2 = 10μF

= 10 x 10-6 F

Initially, the charge on the first capacitor,

Q1 = C1V1

= (20 X 10-6) X 20

= 400 X 10-6 C

and the charge on the second capacitor, as it was uncharged, Q2 = 0.

∴ Net charge, Q = Q1 + Q2

= 400 x 10-6 C

A parallel combination of the capacitors attains a common potential, say V.

Equivalent capacitance,

C = C1 + C2

= (20 + 10) x 10-6 F

= 30 x 10-6 F

⇒ \(V=\frac{Q}{C}=\frac{400 \times 10^{-6}}{30 \times 10^{-6}}=\frac{40}{3}\)

= 13.33V

The ratio of the energies stored in the two capacitors is,

⇒ \(\frac{E_1}{E_2}=\frac{\frac{1}{2} C_1 V^2}{\frac{1}{2} C_2 V^2}=\frac{C_1}{C_2}=\frac{20}{10}=\frac{2}{1}\)

Example 23. How should three capacitors of capacitances 3μF, 3μF and 6μF, be connected to get an equivalent capacitance of 5μF?
Solution:

For a parallel connection of all three capacitors, the 1 equivalent capacitance = 3 + 3 + 6

= 12μF; it is greater than 5μF.

Again, for a series connection, if the equivalent capacitance is C, then

⇒ \(\frac{1}{C_s}=\frac{1}{3}+\frac{1}{3}+\frac{1}{6}=\frac{5}{6}\)

or, \(C_s=\frac{6}{5}=1.2 \mu \mathrm{F}\) ; it is less than 5μF.

So, the above two connections are not applicable. Then, the following four arrangements are possible.

Class 12 Physics Unit 1 Electrostatics Chapter 4 Capacitance and Capacitor Example 23 three capacitors of capacitances

Now, for arrangement (1), the equivalent capacitance is clearly greater than 6μF; so it cannot be 5μF.

For the arrangement (2), equivalent capacitance

= \(\frac{(6+3) \times 3}{(6+3)+3}=2.25 \mu \mathrm{F}\)

For the arrangement (3), equivalent capacitance

= \(\frac{(3+3) \times 6}{(3+3)+6}=3 \mu \mathrm{F}\)

For the arrangement (4), equivalent capacitance

= \(3+\frac{3 \times 6}{3+6}=5 \mu \mathrm{F}\)

So, the arrangement (4) represents the desired connection.

Example 24. The capacitance of a parallel plate capacitor with plate area A and separation s is C. The space between the plates is filled with two wedges of dielectric constants k1 and k2 Find the capacitance of the resultant capacitor.

Class 12 Physics Unit 1 Electrostatics Chapter 4 Capacitance and Capacitor Example 24 The capacitance of a parallel plate capacitor

Solution:

Let the length of the capacitor plates be l and width b.

Area of each plate, A = lb

Now consider a small element of the capacitor of width dx at a distance x from the end. Its area, dA = bdx

The capacitance of this small element

⇒ \(d C=\frac{\epsilon_0 d A}{\frac{y}{\kappa_1}+\frac{s-y}{\kappa_2}}\)

According,

⇒ \(\frac{y}{x}=\frac{s}{l}=\tan \theta\)

∴ \(d C=\frac{\epsilon_0 b d x}{\frac{x \tan \theta}{\kappa_1}+\frac{s-x \tan \theta}{\kappa_2}}\)

∴ \(C=\int_0^l d C=\int_0^l \frac{\epsilon_0 b(d x) \kappa_1 \kappa_2}{\kappa_2 x \tan \theta+\kappa_1(s-x \tan \theta)}\)

Putting tanθ = \(\frac{s}{l}\)

⇒ \(C=\int_0^l \frac{\epsilon_0 A \kappa_1 \kappa_2 d x}{s \kappa_2 x+(l-x) s \kappa_1}=\frac{\epsilon_0 A \kappa_1 \kappa_2}{s\left(\kappa_2-\kappa_1\right)} \log _e \frac{\kappa_2}{\kappa_1}\)

Class 12 Physics Unit 1 Electrostatics Chapter 4 Capacitance and Capacitor Example 24 The capacitance of a parallel plate capacitor.

Example 25. Consider a parallel plate capacitor of plate separation d. Each plate has the length l and the width a. A dielectric slab of permittivity e and thickness d is partially inserted between the plates. The plates are kept at a constant potential difference V. If x is the length of the dielectric slab within the plates, determine the force exerted on the slab.

Class 12 Physics Unit 1 Electrostatics Chapter 4 Capacitance and Capacitor Example 25 parallel plate capacitor

Solution:

As per the diagram, the system may be regarded as the parallel combination of two capacitances C1 and C2. The part containing the dielectric of permittivity e has capacitance q and the other part (through free space) has capacitance C2.

Hence, \(C_1=\frac{\epsilon a x}{d}\)

and \(C_2=\frac{\epsilon_0 a(l-x)}{d}\)

∴ Equivalent capacitance is,

⇒ \(C=C_1+C_2\)

= \(\frac{a}{d}\left[\left(\epsilon-\epsilon_0\right) x+\epsilon_0 l\right]\)

Now electrostatic energy of the system is

⇒ \(U=\frac{1}{2} C V^2\)

= \(\frac{1}{2} \frac{a}{d}\left[\left(\epsilon-\epsilon_0\right) x+\epsilon_0 l\right] V^2\)

∴ V is constant, the force experienced by the slab is,

⇒ \(F_x=\frac{d U}{d x}\)

= \(\frac{a}{2 d}\left(\epsilon-\epsilon_0\right) V^2\)

Example 26. A parallel plate capacitor of capacitance C Is connected to a battery to charge to a potential V. Similarly, another capacitor of capacitance 2C is charged to a potential 2V. Now the batteries are removed, and the two capacitors are connected in parallel by joining the positive plate of one with the negative plate of the other. Find out the final energy of the system.
Solution:

Charge on the first capacitor of capacitance C and connected to the battery of voltage V is,

Q1 = CV

Similarly charge on the second capacitor,

Q2 = (2C) x (2 V)

= 4CV

Since the plates of opposite polarity are connected together, the common potential is,

⇒ \(V^{\prime}=\frac{Q_2-Q_1}{C_1+C_2}=\frac{4 C V-C V}{C+2 C}=V\)

Now equivalent capacitance,

C’ = C+2C

= 3C

So final energy of the configuration,

⇒ \(U_f=\frac{1}{2} C^{\prime}\left(V^{\prime}\right)^2\)

= \(\frac{1}{2} \times 3 C \times V^2\)

= \(\frac{3}{2} C V^2\)

Example 27.  In the circuit, the values of the capacitances of the four capacitors are C1 = C, C2 = 2C, C3 = 3C and C4 = 4C. Find out the ratio between the charges on C2 and C4

Class 12 Physics Unit 1 Electrostatics Chapter 4 Capacitance and Capacitor Example 27 values of the capacitances of the capacitors

Solution:

Here C1 = C, C2 = 2C, C3 = 3C and C4 = 4C.

C1, C2, and C3 are connected in series, so equivalent capacitance is,

⇒ \(\frac{1}{C_{123}}=\frac{1}{C}+\frac{1}{2 C}+\frac{1}{3 C}=\frac{11}{6 C}\)

∴ \(C_{123}=\frac{6 C}{11}\)

Let q1, q2, q3 and q4 be the charges of the capacitors C1, C2, C3 and C4 respectively.

As C1, C2, and C3 are in series, the charge on them is the same.

∴ \(q_1=q_2=q_3=C_{123} \times V=\frac{6 C V}{11}\)

Again charge on capacitor C4 is,

Q4 = C4 x V

= 4CV

So, \(\frac{\text { charge on } C_2}{\text { charge on } C_4}=\frac{\frac{6 C V}{11}}{4 C V}\)

∴ \(\frac{q_2}{q_4}=\frac{6}{4 \times 11}=\frac{3}{22}\)

∴ q2: q4 = 3: 22

Example 28. Five capacitors, capacitance 10μF, form a network. The network is connected to a 100 V dc supply. Calculate the equivalent capacitance between A and B, and the charge accumulated in the network.

Class 12 Physics Unit 1 Electrostatics Chapter 4 Capacitance and Capacitor Example 28 Calculate the equivalent capacitance

Solution:

The given network is a balanced bridge so the middle part (CD) does not play any role.

So equivalent capacitance of branch ACB is,

⇒ \(C_{A C B}=\frac{10 \times 10}{10+10}=5 \mu \mathrm{F}\) [∴ capacitors in branches AC and CB are in series]

Similarly equivalent capacitance of branch ADB is,

⇒ \(C_{A D B}=\frac{10 \times 10}{10+10}=5 \mu \mathrm{F}\)

∴ Equivalent capacitance for the whole network is,

Cwhole = (5 + 5)μF

= 10μF

Now charge accumulated in branch ACB is,

qACB = CACB x V

= 5 x 10-6 x 100C

and charge accumulated in branch ADB,

qADB = CADB x V

= 5 x 10-6 x 100 C

∴ Total charge accumulated in the whole network,

q = qACB + qADB

= 10 x 10-6 x 100 C

= 10-3 C

Example 29. In the network of the capacitance of each capacitor is 1μF. Determine the equivalent capacitance between P and Q.

Class 12 Physics Unit 1 Electrostatics Chapter 4 Capacitance and Capacitor Example 29 capacitance of each capacitor

Solution:

Let C1, C2, C3, C4, …∞ be the capacitances of the capacitors contained in the first row, second row, third row, fourth row, and so on.

Since the capacitors are in series in each row, the equivalent capacitances for the rows are given by,

C1 = 1μF

⇒ \(C_2=\frac{1 \times 1}{1+1}=\frac{1}{2} \mu \mathrm{F}\)

⇒ \(\frac{1}{C_3}=\frac{1}{1}+\frac{1}{1}+\frac{1}{1}+\frac{1}{1}=4\)

∴ \(C_3=\frac{1}{4} \mu \mathrm{F}\)

Similarly,

⇒ \(\frac{1}{C_4}=\frac{1}{1}+\frac{1}{1}+\frac{1}{1}+\frac{1}{1}+\frac{1}{1}+\frac{1}{1}+\frac{1}{1}+\frac{1}{1}=8\)

∴ \(C_4=\frac{1}{8} \mu \mathrm{F}\)

Hence, the equivalent capacitance of the infinite number of rows,

⇒ \(C_{\mathrm{eq}}=C_1+C_2+C_3+C_4+\cdots+\infty\) [∵ they are connected in parallel]

∴ \(C_{\mathrm{eq}}=1+\frac{1}{2}+\frac{1}{4}+\frac{1}{8}+\cdots \infty\)

∴ Sum of the infinite geometric progression is given by,

⇒ \(C_{\mathrm{eq}}=\frac{1}{1-\frac{1}{2}}\)

∴ \(C_{\mathrm{eq}}=2 \mu \mathrm{F}\)

This is equivalent capacitance.

Example 30. A 0.1 F capacitor is charged by a 10 V battery. After disconnecting the battery, this charged capacitor is connected with an uncharged capacitor. If the charge is equally shared between the two, then what will be the energy stored in the two capacitors? Compare this energy with the energy stored initially in the first capacitor.
Solution:

Let C1 and C2 be the capacitances of the capacitors and their voltages be V1 and V2 respectively.

Given C1 = 0.1 F,

initially V1 = 10 V

and V2 = 0.

∴ The initial energy stored in the two capacitors is,

⇒ \(U_i=\frac{1}{2} C_1 V_1^2+\frac{1}{2} C_2 V_2^2\)

= \(\frac{1}{2}\) x 0.1 x (10)² + 0

= 5J

It Q is the initial charge on capacitor C1, its initial energy is,

⇒ \(U_i=\frac{Q^2}{2 C_1}\)

When the two capacitors are connected together, the charge is distributed equally, so the charge on each capacitor is \(\frac{Q}{2}\). Since the potential difference (in a parallel connection) across the two capacitors is also the same, it follows that their capacitances are equal.

Thus, C1 = C2 = C (say)

Also, Q1 = Q2 = \(\frac{Q}{2}\)

∴ The final energy stored in the two capacitors is

⇒ \(U_f=\frac{Q_1^2}{2 C_1}+\frac{Q_2^2}{2 C_2}=\frac{\left(\frac{Q}{2}\right)^2}{2 C}+\frac{\left(\frac{Q}{2}\right)^2}{2 C}=\frac{Q^2}{4 C}\)

But, \(U_i=\frac{Q^2}{2 C}, \text { hence } U_f=\frac{U_i}{2}=2.5 \mathrm{~J}\)

Thus, \(\frac{U_f}{U_i}=\frac{2.5}{5}=\frac{1}{2}\)

Example 31. The equivalent capacitance of two capacitors connected in series and in parallel are Cs and Cp respectively. Determine the capacitance of each capacitor.
Solution:

Let C1 and C2 be the values of the two capacitances.

According to the question,

C1 + C2 = Cp…(1)

⇒ \(\frac{C_1 C_2}{C_1+C_2}=C_s\)…(2)

From equations (1) and (2),

\(\frac{C_1 C_2}{C_p}=C_s\)

∴ \(C_2=\frac{C_p C_s}{C_1}\)

Using this value in equation (1)

⇒ \(C_1+\frac{C_p C_s}{C_1}=C_p \quad\)

or, \(C_1^2-C_p C_1+C_p C_s=0\)

∴ \(C_1=\frac{C_p \pm \sqrt{C_p^2-4 C_p C_s}}{2}\)

Using the positive sign (+) for C1, we have,

So, the two capacitances are,

⇒ \(\frac{1}{2}\left[C_p+\sqrt{C_p^2-4 C_p C_s}\right] \text { and } \frac{1}{2}\left[C_p-\sqrt{C_p^2-4 C_p C_s}\right]\)

Unit 1 Electrostatics Chapter 4 Capacitance And Capacitor Different Types Of Capacitors

Nowadays, different types of capacitors are used for practical purposes. Some of them are discussed below.

Mica capacitor or block capacitor:

This is actually a parallel combination of a few parallel plate capacitors having a fixed capacitance. It consists of a number of metal sheets with mica as a dielectric between them. The alternate sheets are connected to A while the others are to B. If a potential difference is applied between A and B, the capacitor becomes charged.

If α be the area of each metal plate, d is the distance between two plates, K is the dielectric constant of mica and n is the number of metal plates, then the capacitance of such a capacitor in SI is given by

Class 12 Physics Unit 1 Electrostatics Chapter 4 Capacitance and Capacitor Mica capacitor or block capacitor

⇒ \(C=\frac{(n-1) \kappa \epsilon_0 \alpha}{d}\)

This capacitor looks like a block. So it is often called a block capacitor. This type of capacitor is mainly used in wireless receiving sets.

Paper capacitor: Paper coated with paraffin is a suitable dielectric. In between two aluminum or tin foils, paraffin-waxed paper sheets are inserted. The foils are then rolled into a cylindrical shape for the economy of space. This type is essentially a single parallel plate capacitor. It is cheaper than a mica capacitor. Of course one of its disadvantages is that it cannot work at high potential difference.

Class 12 Physics Unit 1 Electrostatics Chapter 4 Capacitance and Capacitor Paper capacitor

In low-voltage ac circuits, having semiconductor diodes and transistors in particular, paper capacitors are widely used.

Variable air capacitor: It is also one type of parallel plate capacitor whose capacitance can be varied at will. It consists of two sets (F and M) of parallel plates made of aluminum or brass. The set is fixed while the other set M can be turned with the help of a knob K.

As the knob turns, the set M either moves into the spacings of the set F or comes out of them, thereby changing the area of overlap of tire plates. Since the capacitance of a capacitor depends on the area of overlap of the plates, the capacitance also changes.

Class 12 Physics Unit 1 Electrostatics Chapter 4 Capacitance and Capacitor Variable air capacitor

It is used in radio sets or wireless receiving sets and other electronic instruments where variation of capacitance is required. Electrolytic capacitor: It consists of a pair of aluminum plates partly immersed into a solution of ammonium borate [NH4B5O6].

The two aluminum plates are connected to the positive and negative terminals of a source of steady current. Due to electrolysis, a fine layer (10-6 cm) of aluminum oxide [Al2O3] is formed on the positive plate.

Class 12 Physics Unit 1 Electrostatics Chapter 4 Capacitance and Capacitor Electrolytic capacitor

This layer acts as a dielectric medium. The solution of ammonium borate acts as an extension of the other plate. Since the capacitance of the capacitor is inversely proportional to the thickness of the dielectric and the thickness of the aluminum oxide layer is very small, the capacitor has a large capacitance, often a few thousand microfarads.

The plate on which aluminum oxide is deposited must be connected to the positive potential. Otherwise, the capacitor would be damaged. For this reason, the anode plate is marked with a + sign or a red dot.

Further, the maximum voltage applicable is restricted by the thickness of the oxide layer. This is also marked on the capacitor. Higher voltage would break the layer and destroy the device. Electrolytic capacitors are largely used as filters in rectifier circuits.

Electrostatic Machines

Classification of Electrostatic Machines:

Electrostatic machines can generate large quantities of electric charges rapidly. These machines are also used to set up high-potential differences. Electrostatic machines are of two types

  1. Frictional machine
  2. Induction machine.

Frictional machines were not very effective. So after the invention of induction machines, frictional machines became obsolete. The principle of action of induction machines depends on the principle of electrostatic induction. In this chapter, we shall discuss a familiar electrostatic machine known as the Van de Graaff generator.

Van de Graaff Generator:

In 1931, Van de Graaff invented this machine. With the help of this machine, very high potential difference (up to a few million volts) can be produced. The principle of action of this machine is based on the discharging action of points and on the property of a collection of charges of a hollow conductor. This machine is very useful at atomic research centers. At present, many changes and modifications of this machine have been introduced.

Description: The sketch of a Van de Graaff generator. A and B are two hollow spherical conductors.

Class 12 Physics Unit 1 Electrostatics Chapter 4 Capacitance and Capacitor Van de Graaff Generator

These are placed on two big insulating stands (X, X). P1 and P2 form two pairs of pulleys. The pulleys are situated at the centers of the spherical conductors and are connected by two electric motors.

Silk or rubber belt moves in the path shown by arrows along the body of each pair of pulleys. The belt enters the spherical conductor through the hole Sx and comes out through the hole S2. C, D, F, and G are four pointed conductors. The pointed ends are directed towards the belt.

The positive charge is supplied to the small sphere (N) placed in front of the pointed conductor D and the negative charge to the small sphere (M) placed in front of the pointed conductor C with the help of a dc generator. The pointed conductors F and G are connected with the spheres A and B respectively.

Working principle:

A positive charge on the small sphere (N) in front of the pointed conductor D induces a negative charge on D and the induced positive charge, being free, moves to the earth. The pointed conductor D discharges the negative charge to the belt in front of it.

The belt, being a non-conductor, does not distribute the charge all over its body; it remains connected to one place. The belt carries the charge upwards and when the charge comes near the pointed conductor G, it induces a positive charge on G and a negative charge on sphere B. Very soon the positive charge on G gets neutralised by the negative charge on the belt. So sphere B is charged with negative electricity.

Similarly, due to the negative charge on the small sphere (M) in front of the pointed conductor C, sphere A is charged with positive electricity.

Discharge of electricity and its remedy:

Due to the continuous movement of the belt by the electric motor, a large quantity of charge accumulates on the two spheres A and B and the potential difference between them increases quickly.

Due to continuous increases in the potential difference between the two spheres, an electric discharge may start in the neighboring air, as air cannot bear high potential differences under normal pressure.

To avoid pointed-end discharge, the spheres and the belts are made very smooth. The whole instrument is installed in a large metallic case connected to the earth and air inside the case is pumped out with the help of an exhaust pump.

Next, the entire case is filled up with nitrogen or freon gas under high pressure. Because, even under high potential difference, the tendency of nitrogen or freon molecules to be ionized, is low.

Uses:

1. Production of high energy charged particles, in nuclear research.

2. Production of hard X-rays. In Science City, Kolkata there is a Van de Graaff generator for display

 

Unit 1 Electrostatics Chapter 4 Capacitance And Capacitor Synopsis

The capacitance of a conductor is equal to the charge necessary to increase its potential by unity.

Units of capacitance:

Class 12 Physics Unit 1 Electrostatics Chapter 4 Capacitance and Capacitor Units ofcapacitance

  • If 1 esu of charge raises the potential of a conductor by 1 esu, the capacitance of the conductor is defined as 1 esu of capacitance or 1 stat farad (statF).
  • If 1 coulomb of charge is required to raise the potential of a conductor by 1 volt, the capacitance of the conductor is defined as 1 farad.
  • 1 C = 3 x 109 esu of charge; 1 V = \(\frac{1}{300}\) esu ofpotential difference; so,
  • IF = \(\frac{1C}{1V}\) = 9 x 1011 esu of capacitance or statF
  • An arrangement by which the capacitance of an insulated charged conductor placed in the vicinity of another conductor (usually earthed) is increased artificially, is called a capacitor.
  • When a capacitor is connected to a battery, charges begin to accumulate on it. This is called the charging of a capacitor. After disconnection of the battery, if any conduction occurs between the two plates of the capacitor, it begins to lose charge. This is called the discharging of a capacitor.
  • The potential difference between the two conducting plates of a capacitor is called the potential of a capacitor.
  • The capacitance of a capacitor is equal to the charge given to the insulated conductor of the capacitor to raise its potential by unity.
  • Dielectric is an insulator. Examples—are air, paraffin, glass, sulphur, mica, ebonite, etc.
  • The dielectric constant of an insulator,
  • k = \(\frac{capacitance of a capacitor with the dielectric ns the intervening medium}{capacitance of the same capacitor with vacuum ns the intervening medium}\)
  • Effectively, the dielectric constant of air or vacuum is 1.
  • To charge a capacitor, a certain amount of work is to be done. The energy spent for doing that work remains stored as potential energy in the electric field between the two plates of the capacitor.
  • For a series combination, the equivalent capacitance is always less than any of the capacitances in the series.
  • The equivalent capacitance of capacitors joined in parallel is greater than any of the capacitances in the combination.
  • Electrostatic machines can generate large quantities of electric charge rapidly. These machines are also used to set up high-potential differences.
  • Electrostatic machines arc of two types:
    • Frictional machine and
    • Induction machine.
  • The working principle of an induction machine is based on electrostatic induction. Van de Graaff generator is an electrostatic machine of this type.
  • With the help of a Van de Graaff generator, a very high potential difference (up to a few million volts) can be produced.
  • The working principle of a Van de Graaff generator depends on the discharging action of points and on the property of collection of charge by hollow conductors.
  • Van de Graaff generator is used
    • To produce high energy charged particles which are required for nuclear research
    • To produce hard X-rays.
  • If charge Q given to a conductor raises its potential by V,then Q ∝ For, Q = CVwhere C is the capacitance of the conductor.
  • The capacitance of a spherical conductor,
    C = 4π∈0kR (in SI)
  • [Here, R = radius of spherical conductor, K = dielectric constant of the surrounding medium]
  • The capacitance (in CGS) of a spherical conductor air or vacuum is numerically equal to its radius in centimetres.
  • The energy of a charged conductor,

⇒ \(U=\frac{1}{2} Q V\)

= \(\frac{1}{2} C V^2\)

= \(\frac{1}{2} \frac{Q^2}{C}\)

where Q = charge, V = potential.

  • If two conducting spheres of radii rx and r2, connected by a conducting wire, are given a charge Q, then charges on the spheres are,

⇒ \(Q_1=Q \cdot \frac{r_1}{r_1+r_2} ; Q_2=Q \cdot \frac{r_2}{r_1+r_2}\)

  • Two conductors having capacitances C1 and C2, are charged to potentials V1 and V2. If they are connected, the common potential becomes,

⇒ \(V=\frac{C_1 V_1+C_2 V_2}{C_1+C_2}\)

= \(\frac{Q_1+Q_2}{C_1+C_2}\)

Q1 and Q2 are their charges before connection. Loss of energy due to sharing of charge

⇒ \(\frac{1}{2} \frac{C_1 C_2}{C_1+C_2}\left(V_1-V_2\right)^2\)

  • The capacitance of a parallel plate capacitor,

⇒ \(\left.C=\frac{\kappa \epsilon_0 \alpha}{d} \text { (in } \mathrm{SI}\right)\)

⇒ \(C=\frac{\kappa \alpha}{4 \pi d} \text { (in CGS system) }\)

where α = area of each plate; d = distance between the plates.

If a combination is formed by connecting alternately n number of equispaced parallel plates, the capacitance of such a block capacitor

⇒ \(C=\frac{(n-1) \kappa \epsilon_0 \alpha}{d}(\text { in SI) }\)

⇒ \(C=\frac{(n-1) k \alpha}{4 \pi d} \text { (in CGS system) }\)

The capacitance of a parallel plate capacitor with compound dielectric,

⇒ \(C=\frac{\epsilon_0 \alpha}{\frac{d_1}{\kappa_1}+\frac{d_2}{\kappa_2}} \text { (in SI) }\)

where k1 and k2 are the dielectric constants of the dielectrics of thickness d1 and d2, respectively.

Energy stored in a charged capacitor,

⇒ \(U=\frac{1}{2} \cdot \frac{Q^2}{C}=\frac{1}{2} C V^2=\frac{1}{2} Q V\)

[where Q = charge of the capacitor and V = potential difference between the capacitorplates]

The potential energy of a parallel plate capacitor

⇒ \(U=\frac{1}{2} \cdot \frac{\alpha \sigma^2 d}{\kappa \epsilon_0} \text { (in SI); }\)

⇒ \(U=\frac{2 \pi \sigma^2 \alpha d}{\kappa} \text { (in CGS system) }\)

Energy density between the capacitor plates,

⇒ \(u=\frac{1}{2} \kappa \epsilon_0 E^2\)

[where E = electric field between the two plates of the capacitor, k = dielectric constant of the medium between the plates.]

For vacuum or air, K = 1.

Then u = \(\frac{1}{2} \epsilon_0 E^2\)

  • For a series combination of capacitors, the equivalent capacitance is given by the following equation:

⇒ \(\frac{1}{C}=\frac{1}{C_1}+\frac{1}{C_2}+\cdots+\frac{1}{C_n}=\sum_1^n \frac{1}{C_n}\) [n = number of capacitors in series]

  • The equivalent capacitance of the capacitors joined in parallel,

⇒ \(C=C_1+C_2+\cdots+C_n=\sum_1^n C_n\) [n = number of capacitors in parallel]

  • Charge in different quantities due to the Introduction of a dielectric slab of dielectric constant K between the plates of a parallel plate capacitor after the capacitor has been chanted with the help of a battery.

Class 12 Physics Unit 1 Electrostatics Chapter 4 Capacitance and Capacitor Change in different quantities and different cases

  • In n number of similar electrically charged drops coalesce to form a larger drop,
    • Charge of the large drop =nx charge of each small drop
    • Capacitance of eh large drop = n1/3 x capacitance of each small drop
    • The surface potential of the large drop = n2/3 x surface potential of each small drop
    • The energy of the large drop = n5/3 x energy of each small drop
    • Surface density of charge of the large drop = n1/3 x surface density of each small drop
  • A capacitor of capacitance C can be charged by connecting
    it is in series with a battery of emf E and a resistance R. The charge on the capacitor after a time t,

Class 12 Physics Unit 1 Electrostatics Chapter 4 Capacitance and Capacitor Example 27 values of the capacitances of the capacitors

⇒ \(q=C E\left(1-e^{-\frac{t}{R C}}\right)=q_0\left(1-e^{-t / \tau}\right)\)

Maximum charge on the capacitor, q0 = CE

The time constant of the charging circuit, \(\tau\) = RC

  • In the circuit, when the key Ky is pressed, the battery of emf E is connected in series with the capacitor C and resistor R, and the capacitor starts charging. After some time, the key Ky is switched off and the key K2 is pressed, then the capacitor starts discharging

Class 12 Physics Unit 1 Electrostatics Chapter 4 Capacitance and Capacitor Time constant of the discharging circuit

The charge on the capacitor after a time t,

⇒ \(q=C E e^{-\frac{t}{C R}}=q_0 e^{-t / \tau}\)

The time constant of the discharging circuit, \(\tau\) = RC

Unit 1 Electrostatics Chapter 4 Capacitance And Capacitor Very Short Questions and Answers

Question 1. What is the name of the physical quantity whose unit is coulomb volt-1?
Answer: Capacitance

Question 2. What is the radius of a conducting sphere of capacitance 10μF?
Answer: 9 cm

Question 3. Two copper spheres of the same radius, one of them being hollow and the other solid, are charged to the same potential. Which of them does contain a greater amount of charge?
Answer: Both contain the same amount of charge

Question 4. If a charged soap bubble expands, what will be the change in its potential?
Answer: The potential will decrease

Question 5. What is the unit of dielectric constant?
Answer: No unit

Question 6. The surface area of a conducting sphere is 10.18 cm2. If it is placed in the air, what will be its capacitance in the picofarad?
Answer: 1

Question 7. n small drops of the same size are initially at the. same potential. They coalesce to form a big drop. What is the ratio of the capacitance of this big drop to that of any of these small drops?
Answer: n1/3

Question 8. Two conductors of capacitances C1 and C2 are connected by a conducting wire. Some amount of charge is given to the system. How is the ratio of charges acquired by the conductors related to their capacitances?
Answer: C1/C2

Question 9. Two conductors, of capacitances C1 and C2, initially have charges q1 and q2 at potentials V1 and V2, respectively. They are now connected by a thin conducting wire. What quantity the net energy loss of the system would be proportional to?
Answer: (V1-V2)2

Question 10. Two spheres of radii r and 2r have charges 2q and q on them, respectively. If they are connected by a copper wire, what will be the amount of charge flowing through the wire?
Answer: q

Question 11. A capacitor is marked as 0.05μF 200V. What is the maximum charge it can accumulate without being damaged?
Answer: 10-3C

Question 12. What is the dielectric constant of conducting materials?
Answer: Infinite

Question 13. What happens when the space between the two plates of a capacitor is filled with a conducting material?
Answer: It will be discharged completely

Question 14. What is the permittivity ofmica if its dielectric constant is 5.4?
Answer: 4.78 x 10-11C2N-1m-2

Question 15. What is the unit of the permittivity of vacuum, ∈0?
Answer: C2.N-1 m-2

Question 16. Write down the dimensional formula of ∈0
Answer: M-1L-3T4l2

Question 17. s the permittivity of any medium greater or less than that of vacuum?
Answer: Greater

Question 18. If a dielectric is placed in an electric field, what change in the intensity of the electric field takes place inside the dielectric?
Answer: Electric field intensity decreases

Question 19. The space between the two plates of a parallel plate air capacitor is filled with an insulator. What will be the nature of the change in its capacitance?
Answer: Increase

Question 20. The space between the two plates of an isolated charged parallel plate air capacitor is filled with an insulator. What will be the nature of the change of the charge accumulation?
Answer: No change

Question 21. In a parallel plate capacitor, the capacitance increases from 4μF to 80μF when a dielectric medium is introduced between the plates. What is the dielectric constant of the medium?
Answer: 20

Question 22. What will be the effect on the capacity of a parallel plate capacitor when the area of each plate is doubled and the distance between them is also doubled?
Answer: No change

Question 23. The distance between the plates of a parallel plate capacitor is d. A metal plate of thickness d/2 is placed between the plates. What will be its effect on the capacitance of the system?
Answer: Capacitance Is Doubled

Question 24. Two protons A and B are placed between two parallel plates having a potential difference V. Will these protons experience equal or unequal forces?
Answer: Equal forces

Class 12 Physics Unit 1 Electrostatics Chapter 4 Capacitance and Capacitor Two protons A and B are placed

Question 25. How can a capacitance of 10μF be designed from a few supplied 2μF capacitors?
Answer: Five of them are to be connected in parallel

Question 26. How can two capacitors be connected so that the charges on them are equal?
Answer: In series

Question 27. How can two capacitors be connected so that the potential differences between their plates are equal?
Answer: In parallel

Question 28. A combination is formed by connecting alternately n number of equidistant parallel plates. The capacitance between any two consecutive plates is C. What will be the equivalent capacitance of the combination?
Answer: (n-1)C

Question 29. Two charged conductors, each of which is effectively a capacitor, are connected by a conducting wire. Which type of combination of capacitors is this—series or parallel?
Answer: Parallel

Question 30. Three capacitors of equal capacitance, when connected in series, have a net capacitance C1. When connected parallel, they have a net capacitance of C2. What is the value of C1/C2?
Answer: 1/9

Question 31. Two plates of an isolated charged capacitor are connected by a copper wire. What will happen to the energy stored in the capacitor?
Answer: It will be zero

Question 32. The separation between the two plates of an isolated charged parallel plate air capacitor is d. The capacitor stores an energy U. Now a metal plate of thickness d/2 and of area equal to that of the capacitor plates is introduced in the intermediate space. What will be the energy stored in the capacitor?
Answer: U/2

Question 33. The separation between the two plates of an isolated charged parallel plate air capacitor is d. The capacitor stores an energy U. Now an insulating plate of thickness d/2, of dielectric constant K, and of area equal to that of the capacitor plates is introduced in the intermediate space. What will be energy stored in the capacitor?
Answer: \(\frac{K+1}{2 K} U\)

Question 34. n capacitors, each of capacity C, are connected in parallel and to a source of V volt. What will be the energy storedin the arrangement?
Answer: \(\frac{1}{2} n C V^2\)

Question 35. n capacitors, each of capacity C, are connected in series and to a source of V volt. What will be the energy storedin the arrangement?
Answer: \(\frac{1}{2 n} C V^2\)

Unit 1 Electrostatics Chapter 4 Capacitance And Capacitor Fill In The Blanks

1. The radius of the earth is 6400 km. Its capacitance in microfarad is 711.1

2. The intensity of the electric field in a dielectric decrease due to electric polarisation

3. A metal plate of negligible thickness is introduced between the two plates of a parallel plate air capacitor. The capacitance will remain the same

4. The distance between the two plates of an isolated charged parallel plate air capacitor is increased. The potential difference between the plates will increase

5. The space between the plates of a capacitor is filled up with a liquid of specific inductive capacity K. The capacitance will change by a factor of k

6. V a de Graaff generator is used to produce hard X-ray

Unit 1 Electrostatics Chapter 4 Capacitance And Capacitor Assertion Reason Type

Direction: These questions have Statement 1 and Statement 2. Of the four choices given below, choose the one that best describes the two statements.

  1. Statement 1 is true, statement 2 is true; Statement 2 is, a correct explanation for statement 1.
  2. Statement 1 is true, Statement 2 is true; Statement 2 is not a correct explanation for Statement 1.
  3. Statement 1 is true, statement is false.
  4. Statement 1 is false, and Statement 2 is true.

Question 1.

  1. Statement 1: Two capacitors of the same capacity are first connected in parallel and then in series. The ratio of equivalent capacitances in two cases is 2: 1.
  2. Statement 2: The equivalent capacitance is less than any of the capacitances in series.

Answer: 4. Statement 1 is false, Statement 2 is true.

Question 2.

  1. Statement 1: If the distance between the parallel plates of a capacitor is halved and the dielectric constant is made three times then the capacitance becomes 6 times.
  2. Statement 2: The capacitance of the capacitor does not depend on the nature of the material of the plates of the capacitor.

Answer: 2. Statement 1 is true, Statement 2 is true; Statement 2 is not a correct explanation for Statement 1.

Question 3.

  1. Statement 1: The larger the sphere, the larger is its capacity, and the smaller the sphere, the smaller is its capacity.
  2. Statement 2: The capacitance of a spherical conductor is directly proportional to its radius.

Answer: 1. Statement I is true, statement II is true; Statement 2 is, a correct explanation for statement I.

Question 4.

  1. Statement 1: Dielectric has no significance in a parallel plate capacitor.
  2. Statement 2: A Dielectric is an insulator that can be easily polarised on the application of an electric field.

Answer: 4. Statement 1 is false, Statement 2 is true.

Question 5.

  1. Statement 1: The force with which one plate of a parallel plate capacitor is attracted towards the other plate is equal to the square of surface charge density per e per unit area.
  2. Statement 2: The total amount of charge that resides, on the unit surface area is known as surface charge density.

Answer: 4. Statement 1 is false, Statement 2 is true.

Question 6.

  1. Statement 1: The capacity of a conductor, under given circumstances, remains constant irrespective of the charge present on it.
  2. Statement 2: Capacity depends on the size and shape of the conductor and also on the surrounding medium.

Answer: 1. Statement 1 is true, statement 2 is true; Statement 2 is, a correct explanation for statement 1.

Question 7.

  1. Statement 1: A dielectric slab is inserted between the ‘ plates of an isolated charged capacitor. The charge on the capacitor will remain the same.
  2. Statement 2: Charge on an isolated system is conserved.

Answer: 1. Statement 1 is true, statement 2 is true; Statement 2 is, a correct explanation for statement I.

Question 8.

  1. Statement 1: The potential energy of a capacitor is obtained at the cost of chemical energy from the battery used for charging the capacitor.
  2. Statement 2: In a battery potential energy is converted to chemical energy.

Answer: 3. Statement 1 is true, the statement is false.

Unit 1 Electrostatics Chapter 4 Capacitance And Capacitor Match The Columns

Question 1. A capacitor of capacitance C is charged to a potential V. Now, it is connected to a battery of emf E. Based on this information match the entries of column 1 with entries of column 2 in the following table.

Class 12 Physics Unit 1 Electrostatics Chapter 4 Capacitance and Capacitor Match the columns 1

Class 12 Physics Unit 1 Electrostatics Chapter 4 Capacitance and Capacitor Match the columns 1.

Answer: 1-B, 2-C, 3-A, 4-D

Question 2. Mathematical expressions of some physical quantities and their corresponding units are given in column 1 and column 2 respectively.

Class 12 Physics Unit 1 Electrostatics Chapter 4 Capacitance and Capacitor Match the columns 2

Answer: 1-B, 2-A,C, 3-D, 4-A,C

Question 3. 

Class 12 Physics Unit 1 Electrostatics Chapter 4 Capacitance and Capacitor Match the columns 3

Answer: 1-A, 2-A,B, 3-D, 4-C

Question 4. 

Class 12 Physics Unit 1 Electrostatics Chapter 4 Capacitance and Capacitor Match the columns 4

Answer: 1-B,C 2-A,D 3-D, 4-C

Question 5. In the area of each plate is A. Match the following.

Class 12 Physics Unit 1 Electrostatics Chapter 4 Capacitance and Capacitor Match the columns 5.

Class 12 Physics Unit 1 Electrostatics Chapter 4 Capacitance and Capacitor Match the columns 5

Answer: 1-E, 2-D, 3-B, 4-A

WBCHSE Class 12 Physics Electric Current and Ohm’s Law Notes

Ohm’s Law Introduction

An electric current is a flow of charges—which is often carried by electrons through a wire. It can also be carried by ions. Electric current is a physical quantity that can be measured and expressed numerically. The principal function of any source of electricity is to send current in an external circuit. In this chapter, we shall discuss the electric current following Ohm’s law

Ohm’s Law Electromotive Force Or Emf

To maintain the flow of water in a pipe external force is to be applied. Similarly, to maintain the flow of current in a conductor some sort of force is required. The physical quantity supplied by the source of electricity is known as the electromotive force. Force and electromotive force are two different physical quantities. i.e., an electromotive force is really not a force, as understood in mechanics.

In the sources of electricity like electric cell, dynamo etc. when even an external energy is converted to electrical energy, an electromotive force is said to have developed.

Definition: The amount of electrical energy produced for transferring a unit positive charge from the lower to the higher potential inside an electric source is called the electromotive force or emf of the source i.e.,

electromotive force = \(\frac{amotmt ofelectrical energy}{quantity ofcharge transferred}\)

Unit: According to the above definition the unit of electromotive force is J.C-1. Also, joule/coulomb is known to be equal to volt [see Chapter ‘Electric Potential’]. So the unit of electromotive force is volt (V) which is also the unit of electric potential and potential difference.

The usual definition of electromotive force: When a source of electricity does not send current in an external circuit i.e., when the circuit is open, the potential difference between the two ends of the source is called its electromotive force.

WBCHSE Class 12 Physics Electric Current And Ohms Law Notes

Ohm’s Law Classification Of The Sources Of Electricity

Electric cell: The source of electricity which does not contain any moving machinery inside it is generally called an electric cell. For Example—chemical cells, solar cells, photoelectric cells, and nuclear cells.

Dynamo: The source of electricity which has a moving machinery is called a dynamo. In this chapter, we shall discuss only the chemical cell.

Different Types of Chemical Cells:

1. Primary cell: In this type of cell chemical energy is converted into electrical energy. Since we get electrical energy without the help of any electrical source, it is called a primary cell.

Examples: Voltaic cell, Leclanche cell, dry cell, Daniell cell. The active components of this cell gradually decay if it sends current in the external circuit for some time and ultimately the cell becomes inactive. The electrochemical reactions taking place inside this cell are irreversible. This implies that the inactive components cannot be made active by a reversible process. The cell is to be abandoned unless the components are replaced.

2. Secondary cell: In this type of cell too, chemical energy is converted into electrical energy. But unlike a primary cell, it can be reused several times. For this, the external current source is needed to recharge it.

Example: Lead-acid accumulator, Alkali accumulator.

The active components of this cell gradually decay as the cell sends current in the external circuit. But here the electro-chemical reactions are reversible. With the help of reverse reactions, the inactive components of the cell can be made active again. To start reverse reactions, current from an external source of electricity, having an electromotive force greater than that of the cell, is allowed to pass through the cell.

This cell acts in two steps:

1. Discharging:

The act of sending current in the external circuit by this cell is called discharging of the cell. During discharging chemical energy is converted into electrical energy.

2. Charging:

The act of activating the practically inactive cell by sending a current in it from an external source in the reverse direction is called charging of the cell. During charging, electrical energy is converted into chemical energy.

During charging, electrical energy is converted into energy which is stored in the cell and conveniently can be converted into electrical energy. Hence this cell is also called a storage cell or accumulator.

3. Standard cell:

As long as this cell is active its electromotive force does not change. So compared with the constant value of the emf of this cell, emfs of other cells can be determined or different electrical instruments can be calibrated. This cell is almost never used as a source of electricity. At present the internationally recognized standard cell is the Weston cadmium cell. Emf of this cell at 20°C is 1.01830 V.

Symbol Of a cell: All cells are shown by drawing two parallel lines of unequal lengths. The long line represents the positive pole and the short line represents the negative pole of the cell.

Electric Current and Ohm's Law Symbol Of a cell

Ohm’s Law Ohm’s Law

In the year 1826 German scientist Georg Simon Ohm established the relationship between the potential difference across a conductor and the current through it.

The law states:

With that temperature and other physical conditions remaining constant, the current flowing through a conductor is directly proportional to the potential difference between its two ends. Let AB be the part of a current-carrying conductor. Also let the potential at A and B be VA and VB respectively, VA being greater than VB. Naturally, current will flow from the point A to the point B. Let the current beI. Then according to Ohm’s law,

Class 12 Physics Unit 2 Current Electricity Chapter 1 Electric Current and Ohm's Law Symbol Of a cell

The constant of proportionality R is called the resistance of the conductor.

Let VA – VB = V

∴ \(\frac{V}{I}\) = R

or, V = IR

or, I = \(\frac{V}{R}\)

Each of the Equations (1), (2), and (3) is the mathematical form of Ohm’s law.

Usually, any conductor that offers resistance to the flow of charge carriers in an electric circuit is called a resistor. A resistor is of ten simply called ‘a resistance’.

Electrical Resistance:

It is obvious from equation (3) that the potential difference ( V) being constant, the current (l) will decrease with the increase of resistance (R) and vice versa. So, physically it means that resistance is the capacity to change the current in the circuit.

Definition: The resistance of a conductor may be defined as the property of the conductor by virtue of which it opposeÿhe current flowing through it.

Resistances are measured by the ratio of the potential difference between the two ends of a conductor and the current flowing through it.

Unit Of resistance: The unit of resistance is ohm (Ω). So, according to Ohm’s law,

⇒ \(1 \Omega=\frac{1 \mathrm{~V}}{1 \mathrm{~A}}\)

i.e., if 1-ampere current flows through a conductor when subjected to a potential difference of 1 volt then the resistance of that conductor is said to be 1 ohm.

Current-voltage graph (linear and nonlinear):

The change of current (l) due to the change in potential difference (V) across a conductor. Here V is taken in the x-axis and I is taken in the y-axis. The graph OA, clearly a straight line one, is known as a current-voltage graph.

Note that, to change the potential difference across the conductor we practically proceed with the change in applied voltage.

The nature of the graph supports Ohm’s law i.e.,\(I=\frac{1}{R} \cdot V\).

Clearly, \(\frac{1}{R}\) is the slope of the graph. It means the reciprocal of the slope of the I-V graph is the resistance of the conductor. It is an experimental graph from which Ohm’s law can be justified. Hence the conductors that obey Ohm’s law are called ohmic conductors.

Class 12 Physics Unit 2 Current Electricity Chapter 1 Electric Current and Ohm's Law Current-voltage graph

On the other hand, the conductors which do not obey Ohm’s law are known as non-ohmic conductors. The 1-V graphs for these conductors will not be straight lines showing that the relation between current and voltage is non-linear. Some Examples of non-ohmic conduct are electrolytes, semiconductors, vacuum tubes etc.

Some non-linear relations between current and voltage. I-V curve for the good conductor. It is seen that up to a certain range of electric current, the curve is linear (OA), but the curve becomes non-linear at high electric current (AB). An I-V curve was drawn for a semiconductor diode. In this case, the current depends on the sign of the potential difference. On the other hand, the variation of current through gallium arsenide (GaAs) due to the varying potential difference between its ends as shown by the graph in shows that different values of V are possible for the same current

Ohm’s Law Effect Of Different Factors On Resistance

Dimension of the conductor: The resistance offered by a conductor to the flow of current though it is more if the conductor is thin, and if the conductor is long.

Material of the conductor: Resistance depends on the nature of the material of the conductor. Current flows easily through the metals like silver, copper, aluminum, etc. i.e., the cost of these metals is very low. These are called good conduct tors.

Almost all the metals are good conductors. Carbon, although a nonmetal, is a good conductor of electricity. Graphite, gas-carbon, and charcoal (under high pressure) which are the allotropes of carbon are also good conductors.

Air, wood, rubber, plastic, ebonite, cloth etc all nonmetallic substances are bad conductors of electricity and are called insulators, which allow almost no current to pass through them.

It is to be noted that there are a few types of substances other than metals which may be called good conductors of electricity; Examples—are electrolytes, gas under low pressure, semiconductors, etc. Ordinary water is an electrolyte and so it conducts electricity, but pure water is not.

Temperature of the conductor: The resistance of the metallic conductors increases with the rise of temperature. The platinum resistance thermometer is constructed depending on this principle.

On the other hand, when the temperature reaches close to 0K, the resistance of some metallic conductors vanishes. This phenomenon is called superconductivity.

For Example, the resistivity’ of mercury absolutely disappears below about 4.2 K. Currents created in a superconducting ting persist for several years without diminution. The phenomenon is of vast potential importance in technology. The explanation is given by the so-called BCS theory offered in 1957.

It is to be noted that the resistance of electrolytes, carbon, glass, gas maintained at low pressure, semiconductors etc. decreases with the increase in temp

Other factors: The following incidents happen in some special types of conductors.

1. If light is incident on selenium, its resistance decreases. Its resistance decreases further with the increase of the intensity of the incident light

2. If bismuth is placed in a magnetic field, its resistance increases. Its resistance increases further with the increase – of the intensity of the magnetic field.

3. Air bubbles exist in the pores of charcoal. As air is an insulator, the resistance of charcoal is high. If pressure is applied to charcoal, the bubbles within its pores will evaporate and the particles of charcoal will come in close contact with each other. So, the resistance of carbon decreases.

These effects have been utilized in many important practical applications.

Superconductors: There are some metals or compounds whose resistance becomes zero at certain low temperatures. These substances are called superconductors and that very temperature is called critical temperature.

Above critical temperature, the resistance-temperature graph of any superconductor is just like that of other common metals. But at critical temperature, the resistance of the superconductor suddenly comes to zero.

Dutch physicist Heike Kamerlingli Onnes (1911 AD) has discovered this phenomenon. He showed that below 4.2 K temperature, mercury is a superconductor.

Superconductivity can be found in different substances, e.g., tin, aluminum, various metallic alloys, highly doped semiconductors etc

Resistivity or Specific Resistance:

The resistance of a conductor (R) under constant temperature depends on the length (l), cross-sectional area (A) and material of the conductor

1. Rule of length:

The resistance of a conductor is directly proportional to its length if the area of the cross-section remains same, i.e.,

R oc l, when A is constant

2. Rule of cross-section: The resistance of a conductor is inversely proportional to its area of cross-section if the length of the conductor remains unchanged, i.e.,

R oc \(\frac{1}{A}\), whenl is constant

⇒ \(R \propto \frac{l}{A} \text { or, } R=\rho \cdot \frac{l}{A}\)….(1)

Here, p is the constant of proportionality, called resistivity or specific resistance of the material of the conductor. Its value depends on the material of the conductor.

If l = 1 and A = 1 then from equation (1) it is obtained

R = p.

i.e., the resistance of the conductor becomes numerically equal to its resistivity when both l and A are in unity. This leads to the definition of resistivity

Definition: The electrical resistance of a conductor of unit cross-sectional area and unit length is defined as the resistivity of the material of the conductor.

Unit of resistivity:

As \(\rho=\frac{R A}{l}\)

⇒ \(\text { unit of } \rho=\frac{\mathrm{ohm} \cdot \mathrm{cm}^2}{\mathrm{~cm}}=\mathrm{ohm} \cdot \mathrm{cm}(\Omega \cdot \mathrm{cm})\)

If length is expressed in meters, the SIunitis ohm-meter (Ωm).

1 Ω m = 1Ω.100 cm

= 100Ω.cm

For Example, the resistivity of copper at 20°C is 1.76 x 10-6 Ω.cm means that resistance across a copper conductor of cross-sectional area 1cm2 and length 1cm at 20°C is 1.76 x 10-6 Ω.

The resistivity of different substances: The resistivity of a few conductors and insulators at 20°C is given below.

Class 12 Physics Unit 2 Current Electricity Chapter 1 Electric Current and Ohm's Law Resistivity of different substances

From the above, it appears that the resistivity of silver is the lowest. This means silver is the best-conducting substance. However, since silver os costly, good conductors are mostly made of copper.

Resistivity and conductivity: If an amount of charge Q passes through a conductor In time t, then-current l \(\frac{Q}{t}\). If the length of the conductor Is l, the cross-sectional area is A, and, the potential difference between Us two ends is VA – VB, then

⇒ \(V_A-V_B=I R=\frac{Q}{t} \cdot \frac{\rho l}{A}\)

or; \(Q=\frac{1}{\rho} \cdot \frac{A\left(V_A-V_B\right) t}{l}\)

This relation is Identical to that of the conduction of heal,

⇒ \(Q=\frac{K A\left(T_2-T_1\right) t}{l}\)

So, If \(\frac{1}{\rho}\) is taken as \(\rho\) then \(\rho\)  may be analogically called electrical conductivity.

It Is obvious that the two physical quantities, conductivity and resistivity are the reverse of each other.

Hence another way of saying that silver is the best conductor is to say that silver has the least resistivity.

An analogous term conductance has been Introduced as a counter to resistance. The concepts obviously bear reciprocal relation.

The SI unit of conductance is Siemens (S) which was formerly called reciprocal ohm or mho (y). If Y is the conductance, then V = \(\frac{1}{R}\).

As conductance is opposite to resistance, the conductance of a body decreases for the factors for which its resistance increases.

Like resistance, tire conductance of a body depends on its temperature and other physical conditions.

An important comment regarding the magnitude of resistance: Consider a copper wire of length 1 m, cross-sectional area 1 mm² or 0.01 cm². The magnitude of its resistance is

⇒ \(R=\rho \frac{l}{A}=1.76 \times 10^{-6} \times \frac{100}{0.01}=0.0176 \Omega\)

So the resistance of the wire is negligible. If a wire of about 57 m in length having the same cross-section is taken, its resistance will be only 1Ω. So this type of wire cannot be regarded as a resistor. Such wires are used for connecting purposes in an electrical circuit. So it can be taken for granted that the wires that connect different electrical instruments in an electrical circuit have almost no resistance.

Short circuit: The circuit obtained by connecting the two electrodes of a source of electricity directly with the help of a wire having almost no resistance is called a short circuit.

For example, if the positive and the negative terminals of a battery are connected by a copper wire, the wire becomes very hot. As the resistance of such a wire U is very small, the current passing through It is high.

So, there Is a chance of damage to both the connecting wire and the source of electricity. To avoid short circuits, fuse wire Is used In domestic electrical lines.

Ohm’s Law Effect Of Different Factors On Resistance Numerical Examples

Example 1. The length, radius, and resistivity of two wires are each in the ratio 1:3. The resistance of the comparatively thin wire Is . 20 fi. Determine the. resistance of the other wire.
solution:

⇒ \(R=\rho \frac{l}{A}=\frac{\rho l}{\frac{\pi d^2}{4}}=\frac{4 \rho l}{\pi d^2}[d=\text { diameter of the wire }]\)

For the first wire, \(R_1=\frac{4 \rho_1 l_1}{\pi d_{1^{\prime}}^2}\)

For the second wire, \(R_2=\frac{4 \rho_2 l_2}{\pi d_2^2}\)

∴ \(\frac{R_1}{R_2}=\frac{\rho_1}{\rho_2} \times \frac{l_1}{l_2} \times\left(\frac{d_2}{d_1}\right)^2\)

= \(\frac{1}{3} \times \frac{1}{3} \times\left(\frac{3}{1}\right)^2\)

= 1

or R1 = R2

The diameter of the first wire is less. So it is thin

∴ \(R_1=20 \Omega \text {; so } R_2=20 \Omega\)

Example 2. If the length of a copper wire is increased by 0.1%, show that the resistance of the copper wire will increase by 0.2%.
Solution:

If the initial length be, then the final length will be

l2 = l1 x (100.1%)

= \(l_1 \times \frac{100.1}{100}\) = 1.001l1

If the temperature of the wire remains constant, its volume will remain constant.

Now, volume = length x cross-sectional area

So if the length increases from l1 to 1.001 l1 cross-sectional area

A1 decreases to A2, where \(A_2=\frac{A_1}{1.001}\)

Now, \(R_1=\rho \frac{l_1}{A_1} \text { and } R_2=\rho \cdot \frac{l_2}{A_2}\)

∴ \(\frac{R_2}{R_1}=\frac{l_2}{l_1} \times \frac{A_1}{A_2}\)

= 1.001 X 1.001

= 1.002

= 100.2%

i.e., increase of resistance = 0.2%

Alternative Method:

⇒ \(R=\rho \frac{l}{A}\)

By logarithmic differentiation, \(\frac{d R}{R}=\frac{d l}{l}-\frac{d A}{A} \quad(\rho=\text { constant })\)

Now, V = lA = constant

By logarithmic differentiation, \(\frac{d l}{l}=-\frac{d A}{A}\)

Hence, \(\frac{d R}{R}=\frac{2 d l}{l}=2 \times 0.1 \%=0.2 \%\)

Example 3. A lump of copper of mass 10 g and of density 9 g.cm-3 is given. What should be the length and cross-section of the wire made from it so that its resistance is 2cm ohm. (Given, specific resistance of copper 1.8 x 10-6 Ω.cm)
Solution:

⇒ \(\frac{mass}{density}\) = volume

= length x cross-sectional area

⇒ \(\frac{10}{9}=l A \text { or, } l A=\frac{10}{9}\)…(1)

Again, resistance, \(R=\rho \frac{l}{A} \text { or, } \cdot 2=1.8 \times 10^{-6} \times \frac{l}{A}\)

or, \(\frac{l}{A}=\frac{2}{1.8} \times 10^6=\frac{10}{9} \times 10^6\)…(2)

Now, by multiplying (1) and (2) we get,

⇒ \(l^2=\frac{10}{9} \times \frac{10}{9} \times 10^6 \quad \text { or, } l=\frac{10}{9} \times 10^3 \mathrm{~cm}=11.1 \mathrm{~m}\)

Again dividing (1) by (2) we get, A2 = 10-6

or, A = 10-3 cm2

= 10-1 mm2

= 0.1 mm2

Example 4. A wire of resistance 5Ω is stretched 20%. If the volume remains constant, find the new resistance.
Solution:

The volume of the wire = V = constant. Let the initial length of the wire = l

∴ Initial cross-sectional area, \(A_1=\frac{V}{l}\)

If the length of the wire is increased by 20%

⇒ \(\text { final length }=l \times \frac{120}{100}=1.2 l\)

∴ Final cross-sectional area, \(A_2=\frac{V}{1.2 l}\)

Now, \(R_1=\rho \cdot \frac{l_1}{A_1} \quad \text { and } R_2=\rho \cdot \frac{l_2}{A_2}\)

∴ \(\frac{R_2}{R_1}=\frac{l_2}{l_1} \times \frac{A_1}{A_2}\)

or, \(R_2=R_1 \times \frac{l_2}{l_1} \times \frac{A_1}{A_2}=5 \times \frac{1.2 l}{l} \times \frac{\frac{V}{l}}{\frac{V}{1.2 l}}\)

= 5 X 1.2 X 1.2

= 7.2 Ω

Example 5. A lump of copper Is stretched into a wire 5mm in diameter. Another wire of 1 cm diameter is made from another lump of copper of the same mass. Find the ratio of the resistances of the two wires.
Solution:

5mm = 0.5 cm.

Resistance, \(R=\rho \frac{l}{A}=\rho \frac{V}{A^2}=\frac{\rho m}{D\left(\pi \frac{d^2}{4}\right)^2}=\frac{16 \rho m}{\pi D d^4}\)

Here, \(\rho\) = resistivity, l = length, d = diameter

⇒ \(A=\frac{\pi d^2}{4}\) V = lA = volume,

m = mass, D = \(\frac{m}{V}\) = density

Both the lumps have the same D and p and according to the question, mass m is equal.

∴ \(\frac{R_1}{R_2}=\left(\frac{d_2}{d_1}\right)^4=\left(\frac{1}{0.5}\right)^4=(2)^4=16 \text { or, } R_1: R_2=16: 1\)

Example 6. The length of a wire of cylindrical cross-section is increased by 100%. Find out the percentage change in the resistance, taking into account the consequent decrease in the diameter of the wire.
Solution:

Initial length = l; final length, \(l^{\prime}=l+l \times \frac{100}{100}=2 l\)

If V be the volume, area of cross-section, A = \(\frac{V}{l}\)

∴ Final area of cross-section, \(A=\frac{V}{l}\)

From the formula \(R=\rho \frac{l}{A}\) we get

⇒ \(\frac{R}{R^{\prime}}=\frac{l}{l^{\prime}} \cdot \frac{A^{\prime}}{A}=\frac{l}{2 l} \cdot \frac{V / 2 l}{V / l}=\frac{1}{4}\)

∴ \(R^{\prime}=4 R=R+3 R=R+\frac{300}{100} R\)

i.e., the percentage change in resistance = 300%.

Example 7. What will be the distance of a semicircle between points A and B? Given that radial thickness = 3 cm, axial thickness = 4 cm, Inner radius = 6 cm, and resistivity = 4 x 10-6 Ωcm.

Electric Current and Ohm's Law the resistance of a semicircle

Solution:

Cross-sectional area, A = 4 cm  x 3cm = 12 cm2

Linear length,l = πr = π(6 + \(\frac{3}{2}\) = 7.5 7T cm .

∴ Resistance = \(\rho \frac{l}{A} \quad\left(\rho=\text { resistivity }=4 \times 10^{-6} \Omega \cdot \mathrm{m}\right)\)

⇒ \(\frac{4 \times 10^{-6} \times \pi \times 7.5}{12} \Omega\)

= \(7.85 \times 10^{-6} \Omega\)

Ohm’s Law Carbon Resistors

In electrical experiments, resistances starting from about 0.1 A to 1000 A are widely used. To manufacture these types of resistances, generally, copper or any other conducting metal or alloy are used.

On the other hand in the experiments of electronics, resistances less than 1000 A are seldom used. For these high resistances graphite and gas carbon are widely used nowadays. These are called carbon resistors.

If we observe the table of specific resistance of the conductors, it will be found that the specific resistances of the allotropes of carbon are over a thousand times greater than those of metals.

So carbon is very effective as an element of high resistance. Another advantage is that carbon resistors are cheaper than metal resistors.

Class 12 Physics Unit 2 Current Electricity Chapter 1 Electric Current and Ohm's Law carbon resistors

To manufacture a carbon resistor, a cylindrical shell made of bad conductors was taken. Its length is more or less 1 cm and its diameter is between 2 mm to 5 mm.

Conducting carbon power is introduced in a controlled manner into the shell. The value of the resistance depends on the length and the diameter of the shell and also on the amount of carbon powder.

Two conducting metal wires are taken out from either side of the shell along its axis through which the resistor is connected to the external circuit.

The shell has four different colored rings or bands 4, B, C, and D on its surface. The fourth band D is painted further off with respect to the bands A, B, and C. The resistance of the resistor is obtained from the color of these four bands according to an acknowledged code.

Colour Code of Carbon Resistors:

According to this code, ten single digits from 0 (zero) to 9 correspond to ten colors. The code is as follows:

The first two bands (A and B) represent the first two digits to ascertain their values in ohms. The third band (C) represents the multiplier and the fourth band (D) tolerance.

If A is absent, it will indicate that the tolerance of the resistor is 20%

In some resistors, a 5-band or 6-band coding is used. In these cases, the third band represents the third digit. In that case, the fourth and fifth builds represent the multiplier and tolerance respectively. For a six-banded resistor, the sixth band denotes the temperature coefficient.

The determining digits are shown against color codes in the following table:

Class 12 Physics Unit 2 Current Electricity Chapter 1 Electric Current and Ohm's Law The determining dibits are shown against colour codes

Mnemonics: The sequence of the colour code given above can be remembered by the following sentence, B B ROY of Great Britain has a Very Good Wife wearing.Gold Silver necklace.

Example: Suppose the color bands on a carbon resistor are in the sequence yellow, violet, red and silver.

A: Colouryellow; so from the table, the digit is 4,

B: Colourviolet; so from the table, the digit is 7.

C: Colour red; so from the table, the digit is 2 i.e., the multiplier is 10.

So, the value of the resistance is 47 x 102 or 4700 A.

D: Colour silver; so tolerance is 10%

Hence, the value of the resistance is 4700A± 10% i.e., lies between 4230 A and 5170 A. However in electronics no special importance is given on band D.

Ohm’s Law Carbon Resistors Numerical Examples

Example 1. The equivalent resistance of two cells connected in series and in parallel are \(12 \Omega \text { and } \frac{5}{3} \Omega\) respectively. Calculate the value of each resistance.
Solution:

If R1 and R2 be the two resistances, then equivalent resistance in series combination

R1 +R2 = 12 …..(1)

Equivalent resistance of parallel combination,

⇒ \(\frac{R_1 R_2}{R_1+R_2}=\frac{5}{3} \quad \text { or, } \frac{R_1 R_2}{12}=\frac{5}{3} \quad \text { or, } R_1 R_2=20\)

Now, \(\left(R_1-R_2\right)^2=\left(R_1+R_2\right)^2-4 R_1 R_2\)

= 122 – 4 X 20

= 144 – 80

= 64

or, R1 – R2 = 8 [Here we asuume R1 > R2]…(2)

Adding equations (1) and (2) we have,

2R1 = 20

or, R, = 10Ω

From equation (1) we have,

R2 = I2– R1

= 12- 10

= 2Ω

Example 2. A 5-ampere current Is distributed In branches. The ratio of the lengths of the wires in the three branches is 1: 2: 3. Determine the tire magnitude of current in each branch. The material and the cross-sectional area of each wire are the same.
Solution:

Resistance of the wire is proportional to Its length as the material and the cross-sectional area are the same. So we take the resistances as 2a and 3. v and the equivalent resistance as R.

We have

⇒ \(\frac{1}{R}=\frac{1}{x}+\frac{1}{2 x}+\frac{1}{3 x}=\frac{6+3+2}{6 x}=\frac{11}{6 x} \quad \text { or, } R=\frac{6 x}{11}\)

So, the potential difference between the two terminals,

⇒ \(V=I R=I \cdot \frac{6 x}{11}\)

As die potential difference across each wire is the same, the currentin the first branch

= \(\frac{V}{x}=I \cdot \frac{6}{11}=5 \times \frac{6}{11}=\frac{30}{11} \mathrm{~A}\)

current in the second branch

⇒ \(\frac{V}{2 x}=I \cdot \frac{3}{11}=5 \times \frac{3}{11}=\frac{15}{11} \mathrm{~A}\)

Currently the third branch

⇒ \(\frac{V}{3 x}=I \cdot \frac{2}{11}=5 \times \frac{2}{11}=\frac{10}{11} \mathrm{~A}\)

Example 3. Current is allowed to pass in a circuit formed by two wires of the same material connected In a parallel combination. The ratio of the lengths and the radii of the two wires are 4: 3 and 2: 3 respectively. Determine is the ratio of the currents flowing through the two wires.
Solution:

⇒ \(R=\rho \frac{l}{A}=\rho \frac{l}{\pi r^2}\)

So, \(\frac{R_1}{R_2}=\frac{l_1}{l_2} \cdot (\frac{r_1}{r_2})^2\)

= \(\frac{4}{3}\) x \((\frac{3}{2})^2\)

= 3

In a parallel combination, the current through a branch is inversely proportional to the resistance of the branch.

⇒ \(\frac{l_1}{l_2}\) = \(\frac{R_2}{R_1}\)

= \(\frac{1}{3}\)

∴ The ratio of current flowing through the wires

= l1: l2 = 1: 3.

Mixed Combination of Resistances:

Tho complicated circuits of radio, television, etc. Involve mixed combinations of resistances.

Let us discuss the mixed combination with three resistances as a simple Example.

Three resistances R1, R2, and R3, fig are to be connected between two points A and It In a circuit In mixed combination. The resistance can be connected In the following ways.

1. The parallel combination of R1, and R2 Is connected with
in series. The equivalent resistance of this combination,

⇒ \(R=\frac{R_1 R_2}{R_1+R_2}+R_3\)

Changing the relative positions of R1 and R3 and again of R2 and R3, two more similar mixed combinations are obtained. Equivalent resistance changes accordingly.

2. The series combination of it R1 and R2, Is connected in parallel. The equivalent resistance of the series combination of R1 and R2 is R’ = R1 + R2. Since R3 Is connected In parallel with R’, the equivalent resistance of this mixed combination Is

⇒ \({R}=\frac{R^{\prime} R_3}{R^{\prime}+R_3}=\frac{\left(R_1+R_2\right) R_3}{R_1+R_2+R_3}\)

Electric Current and Ohm's Law The series combination

In this case also, by changing the relative positions of R1 and R3 and again of R2 and R3, two more similar mixed combinations are obtained. Equivalent resistance will change accordingly.

Connection Of Electric Cell In A Circuit Internal Resistance Of A Cell And Lost Volt

An arrangement in which a resistance R is joined between points A and B of a conducting wire that terminates at the two poles of a cell of emf E.

The resistance of the connecting wire is negligible. A current passes through R and completes the circuit from the negative pole to the positive pole inside the cell.

When the current flows through the cell the elements of the cell oppose it to some extent indicating that inside the cell a resistance is developed. This is the internal resistance of the cell (r).

Electric Current and Ohm's Law connection of electric cell in a circute

The equivalent resistance of the circuit = external resistance + internal resistance of the cell = R +r.

Current, \(I=\frac{E}{R+r} \quad \text { or, } E=I R+I r\)

Now, the potential difference of the external circuit i.e., the potential difference across the resistance, R is V = IR.

∴ E = V + Ir

or, V = E – Ir …(1)

V is the potential difference of the cell. Obviously, the value of V is less than E.

It indicates that the whole emf of the cell is not obtained as a potential difference in the external circuit.

A potential of magnitude Ir is lost inside the cell for the internal resistance (r).

This is called an internal potential drop or lost volt of the cell. This is so called because this portion of the emf of the cell does not contribute to the current through the external resistance.

Discussion:

Connection Of voltmeter: If the two ends of a voltmeter are connected to the two poles of a cell, the reading it indicates is not the value of E (emf of the cell) but the value of V (potential difference of the cell) because the voltmeter acts here as the external circuit.

Negligible internal resistance: if the internal resistance of the cell is very small in comparison to the external resistance then r may be taken as zero. Then Ir = 0 and no potential is lost inside the cell. So, emf of the cell and potential differences are equal.

For Example: In a lead-acid accumulator the magnitude of the internal resistance ranges from 0.01 fl to 0.1 fl. So, this resistance may be neglected in most of the electrical circuits.

Definition of electromotive force:

In an open circuit, no current flows in the external circuit i.e., 1=0. Then lost volt, Ir = 0 and V = E. From this relation emf of the tire cell may be defined as follows.

The potential difference between the two poles of a cell In an open circuit (when it does not deliver current in an external circuit) is called the emf of the cell.

Limit of current: In the equation V = E-Ir if the magnitude of l or r is very high then the magnitude of V becomes very small.IfIr becomes equal to E, then V = 0.

Thus the cell cannot produce any potential difference in the external circuit. So, for every cell has a maximum limit.

For Example, the emf of a dry cell is 1.5 V and if its internal resistance is 3Ω, the maximum current it delivers in the external circuit is \(I=\frac{E}{r}=\frac{1.5}{3}=0.5 \mathrm{~A}\). Hence primary cells are not used where high currents are required.

Voltage source and current source: Consider a battery of emf E with an internal resistance r, to which a variable resistor (R) is connected. The current in the circuit will be

⇒ \(I=\frac{E}{R+r}\)….(1)

Class 12 Physics Unit 2 Current Electricity Chapter 1 Electric Current and Ohm's Law Voltage source and current source

The potential differences between the two poles of the battery i.e., the term potential differences of the resistance R is

⇒ \(V^{\prime}=I R=\frac{E R}{R+r}\)….(2)

1. When r << R; we get from equation (2) \(V=\frac{E}{1+\frac{r}{R}} \approx E=\text { constant }\) Hence, whatever be the value of R, we get a constant potential difference (or voltage) from the battery. Under this condition, the battery is taken as a voltage source. 2. When r >> R, we get from equation (1),

⇒ \(I=\frac{E}{r\left(1+\frac{R}{r}\right)} \approx \frac{E}{r}=\text { constant }\)

Therefore, whatever the value of R, we get a constant current in the external circuit Under this, condition, the battery acts as a current source.

Connection Of Electric Cell In A Circuit Internal Resistance Of A Cell And Lost Volt Numerical Examples

Example 1. A resistor is fabricated by connecting two wires of the same material. The radii of the two wires are 1 mm and 3 mm respectively and their lengths are 3 cm and 5 cm respectively. If the two ends of the resistor are connected to the two terminals of a battery of emf 16 V and of negligible internal resistance, what will be the potential difference between the two ends of the wire of shorter length?
Solution:

Resistance of wire, \(R=\rho \frac{l}{A}=\rho \cdot \frac{l}{\pi r^2}\)

So, for the two wires of the same material,

⇒ \(\frac{R_1}{R_2}=\frac{l_1}{l_2} \times \frac{r_2^2}{r_1^2}\)

= \(\frac{3}{5} \times\left(\frac{3}{1}\right)^2\)

= \(\frac{27}{5}\)

The two wires are connected in series. So, the ratio of the potential difference across their ends is

⇒ \(\frac{V_1}{V_2}=\frac{I R_1}{I R_2}\)

= \(\frac{R_1}{R_2}=\frac{27}{5}\)

or, \(\frac{V_1}{V_1+V_2}=\frac{27}{27+5}\)

= \(\frac{27}{32}\)

or, \(V_1=\frac{27}{32}\left(V_1+V_2\right)\)

In the circuit, there are only two resistances. So the electromotive force,

⇒ ⇒\(E=V_1+V_2=16 \quad ∴ V_1=\frac{27}{32} \times 16=13.5 \mathrm{~V}\)

So, the potential difference across the resistance of length 3 cm

V1 = 13.5 V

Example 2. A heater of resistance 140Ω capable of carrying a current of 1.2 A Rasputin a dc mains of 210 V. Find out the minimum value of an additional resistance to be added to run the heater
Solution:

Let the value of the minimum additional resistance to be added be x Ω

We know, that V = IR

or, 210 = 1.2(140 + x) [Here R = 140 + x]

or, 140 + x = \(\frac{210}{1.2}\)

or, x = 175-140

or, x = 35Ω

So, a minimum additional resistance of 35Ω is to be added to run the heater

Example 3. To the parallel combination of two resistances 3Ω and 1Ω, a series combination of resistances 2.15Ω and 1Ω and a battery are connected. The internal resistance of the battery is 0.1Ω and the emf is 2 V. Determine the values of currents flowing through the resistances. Draw the diagram of the circuit
Solution:

The equivalent resistance of the parallel combination of 3Ω and 1Ω is

⇒ \(R=\frac{3 \times 1}{3+1}=\frac{3}{4}=0.75 \Omega\)

Class 12 Physics Unit 2 Current Electricity Chapter 1 Electric Current and Ohm's Law Example 3 the parallel combination of two resistances

This equivalent resistance is in series with the other resistances.

So, the main current of the circuit is

⇒ \(I=\frac{2}{0.75+2.15+1+0.1}=\frac{2}{4}=0.5 \mathrm{~A}\)

This main current flows through the resistances 2.15Ω and 1Ω connected in series.

Now, current through 3Ω

⇒ \(I_1=I \times \frac{1}{3+1}=0.5 \times \frac{1}{4}=0.125 \mathrm{~A}\)

and current through 1Ω,

I2 = I – I1

= 0.5 – 0.125

= 0.375 A

Example 4. The electromotive force of a cell is 2 V. The potential difference becomes 1.5 V when a resistance of 15Ω is added to the two ends of the cell. Determine the internal resistance of the cell and the lost volt
Solution:

Lost volt = E – V

= 2-1.5

= 0.5 V

The current of the external circuit

⇒ \(=\frac{\text { potential difference of the external circuit }}{\text { resistance of the external circuit }}\)

⇒ \(\frac{1.5}{15}=0.1 \mathrm{~A}\)

This is the current flowing through the cell.

So, the internal resistance of the cell,

⇒ \(r=\frac{\text { lost volt }(I r)}{\text { current }(I)}=\frac{0.5}{0.1}=5 \Omega\)

Example 5. In the given circuit diagram, what is the current sent by the battery?

Electric Current and Ohm's Law current sent by the battery

Solution:

The equivalent circuit.

The equivalent resistance of the middle branch

⇒ \(1.5+\frac{2 \times 6}{2+6}\)

= \(3 \Omega\)

With this 3Ω resistance, the outer 3Ω resistance on the right side is in parallel combination. So the equivalent resistance of the circuit

⇒ \(\frac{3 \times 3}{3+3}=\frac{3}{2} \Omega\)

∴ The main current of the circuit,

⇒ \(I=\frac{6}{\frac{3}{2}}=4 \mathrm{~A}\)

Class 12 Physics Unit 2 Current Electricity Chapter 1 Electric Current and Ohm's Law Example 5 current sent by the battery.

Comparison of Electromotive Force and Potential Difference:

In a source of electricity, chemical or some other form of energy is converted into electrical energy. This produces the electromotive force. Again when a source of electricity is connected to an external circuit, then a potential difference is developed at the two ends of the circuit. As a result, the electrical energy of the source is converted into another form of energy in the external circuit.

1. Due to the existence of electromotive force in the source of electricity, potential difference develops in the external circuit i.e., if the electromotive force is called the cause the potential difference may be called the ‘effect’.

2. The unit of electromotive force and potential difference is the same and this unit is volt.

3. An amount of potential is lost inside the source of electricity due to its internal resistance. So, the entire electromotive force cannot be obtained as the potential difference in the external circuit i.e., potential difference can never be greater than the electromotive force.

4. When the source of electricity does not send current in the external circuit i.e., when the circuit is open, then no potential is lost due to the internal resistance of the source. In that case, the potential difference between the two ends of the source becomes equal to its electromotive force.

5. The electromotive force of an electric cell without any defect is a constant quantity. But from the equation V = E-Ir it can be said that if the current flowing through the circuit increases or decreases, the potential difference at the two ends of the circuit decreases or increases accordingly.

Electric Current and Ohm’s Law Conclusion

  • The amount of electric charge flowing through any cross-section of a conductor per second is called electric current strength.
  • Coulomb = ampere x second (coulomb—a unit of electric charge, ampere—a unit of electric strength)
  • In the external circuit electric current flows from the higher potential to the lower potential. In the internal circuit electric current flows from the lower potential to the higher potential.
  • During electric discharge for a long time, the electromotive force of a lead-acid accumulator remains at 2.0 V.
  • The total amount of charge that a secondary cell delivers to the external circuit while being discharged from the initial fully charged condition to the Anal fully discharged condition is called the capacity of that cell.
  • The unit of capacity of an accumulator is ampere-hour (Ah).
  • Energy efficiency of a cell or watt-hour efficiency

\(\eta=\frac{\text { average emf during discharging }}{\text { average emf during charging }}\) x ampere-hour efficiency

Ohm’s law: If temperature and other physical conditions remain the same, then die electric current flowing through a conductor is directly proportional to the potential difference between the two ends of the conductor.

  • The property by virtue of which a conductor opposes the flow of current through it is called the resistance of the conductor.
  • The electrical resistance of a conductor of unit cross-sectional area and unit length is defined as the resistivity of the material of the conductor,
  • If the two electrodes of a source of electricity are connected with a conducting wire having almost no resistance, then the thus obtained is known as a short circuit.
  • With the increase in temperature, the resistance of metals generally increases but that of electrolytes, carbon, glass, gas at low pressure, and semiconductors decreases.
  • If a single resistance can replace a combination of resistances so that the circuit current and the terminal potential difference remain the same then that single resistance is called the equivalent resistance of that combination.
  • In the case of a series combination of resistances, the value of the equivalent resistance is always greater than each of the component resistances.
  • In the case of a parallel combination of resistances, the value of the equivalent resistance is always less than each of the component resistances.
  • The entire electromotive force of a cell is not obtained as the potential difference in the external circuit. Due to the internal resistance of the cell some potential is used up by the interior of the cell. This is known as the internal potential drop or lost volt.
  • The potential difference across the two electrodes of a cell in an open circuit is called the electromotive force of a cell.
  • An ammeter is a current measuring device connected in series with the circuit and has very low resistance.
  • A voltmeter is a potential-difference measuring device connected in parallel with the circuit and has very high resistance.
  • In the case of the series combination of a number of cells: When the resistance of the external circuit is very much greater than the total internal resistance of the cells, then for n number of cells in series, the current Is n times that when a single cell is used.
  • In the case of the parallel combination of a number of cells: When the resistance of the external circuit is 4 much less than the total Internal resistance of the cells,- then for n number of cells in parallel, the current is n times of that when a single cell is used.

In the case of a mixed combination of a number of cells:

  • To get maximum current through the external circuit the cells should be arranged in series as well as in parallel in such a way that the equivalent resistance of the internal resistances of the cells becomes equal to the external resistance.
  • During the passage of electric current through a metallic conductor, the average velocity with which the free electrons move through the conductor is known as the drift velocity of the free electrons.
  • The amount of electric current flowing through the unit cross-sectional area of the conductor is called the current dencity through the conductor.
  • The velocity with which an electric field propagates through a conductor is called the velocity of electric current.
  • The mobility (μ) of a free electricity conductor is defined as the value of the drift velocity developed in it when the applied electric field is unit v
  • V = IR
    • [where, V= potential difference between the two ends of the conductor, I = electric current strength through any cross-section of the conductor, R = resistance]
  • \(R=\rho \frac{l}{A}\)
    • [p = specific resistance, l = length of the conductor, A = area of cross section of the conductor]
  • Rt = R0(l + aaat)
    • [R0 = resistance of the conductor at 0°C, Rt = resistance of the conductor at t°C, a = temperature coefficient of resistance, t = change in temperature]

The formula for the determination of equivalent resistance in series combination:

\(R=R_1+R_2+R_3+\cdots+R_n=\sum_{i=1}^n R_i\)

The formula for the determination of equivalent resistance In parallel combination:

\(\frac{1}{R}=\frac{1}{R_1}+\frac{1}{R_2}+\frac{1}{R_3}+\cdots+\frac{1}{R_n}=\sum_{i=1}^n \frac{1}{R_i}\)
  • E = Ir+V
    • [E = emf of the cell,I = current strength, r = internal resistance of the cell, V= potential difference]
  • \(\frac{I_G}{I_S}=\frac{S}{G}\)
    • [IG = current through the galvanometer, S = resistance of the shunt; Is current through the shunt, G = resistance of the galvanometer]
  • \(S=\frac{G}{n-1}[n=\text { power of the shunt }]\)
  • Current through the external resistance for a circuit containing n number of cells in series,
  • \(I=\frac{n e}{R+n r}\)
  • Current through the external resistance for a circuit containing n number of cells in parallel,
  • \(I=\frac{n e}{n R+r}\)
    • [e = emfof each cell, R = resistance ofexternal circuit, r = internal resistance of each cell]
  • The condition for maximum current in the circuit for a mixed combination of cells is mR = nr.
    • [where, m = number of rows connected in parallel combination, n =numberofcellsin eachrowconnectedIn series.]
  • Drift velocity of free electrons
  • \(v_d=\frac{I}{n e A}\)
    • [where I = current strength, n – number of free electrons in unit volume, e = charge of each electron, A – area of cross-section of the conductor]
  • Current density, \(j=\frac{I}{A}=n e v_d\)
  • If Rs and Rp, respectively be the equivalent resistances of n number of resistances connected in series and parallel then
  • \(\frac{R_s}{R_p}=n^2\)
  • If a wire of resistance R is divided into n number of equal parts and they are connected in parallel, then the equivalent resistance of the combination will be \(\frac{R}{n^2}[/ltaex]
  • If equivalent r∈ sistance of R1 and R2 in series and parallel be Rs and Rp respectively, then
  • [latex]R_1=\frac{1}{2}\left[R_s+\sqrt{R_s^2-4 R_s R_p}\right]\)
  • \(R_2= \frac{1}{2}\left[R_s-\sqrt{R_s^2-4 R_s R_p}\right]\)
  • In the given diagram, the equivalent resistance between the points A and B is
  • \(R_{A B}=\frac{1}{2}\left(R_1+R_2\right)+\frac{1}{2}\left[\left(R_1+R_2\right)^2+4 R_3\left(R_1+R_2\right)\right]^{\frac{1}{2}}\)

Class 12 Physics Unit 2 Current Electricity Chapter 1 Electric Current and Ohm's Law the equivalent resistance

  • If n number of identical cells are connected in a loop in order, then the terminal potential difference of any one cell is zero.
  • In a parallel combination of two resistances, the current gets divided into two branches.
  • Branch current = main current x \(\frac{\text { resistance of other branch }}{\text { sum of the two resistances }}\)
  • n cells each of emf E and internal resistance r are connected in series and by mistake, m cells are wrongly connected.If this series combination of cells is connected to an external resistance R, then
    • Effective emf of the combination, E’ = (n-2m)E,
    • Total internal resistance = nr
    • The total resistance of the circuit = nr+ R
    • Current through the circuit, \(I=\frac{(n-2 m) E}{n r+R}\)
  • If two cells of emf Ej and E2 and internal resistances Tj and r2 respectively are connected in parallel to an external resistance R, then

Electric Current and Ohm's Law connected in parallel to an external resistance

  • Effective emf = \(\frac{E_1 r_2+E_2 r_1}{r_1+r_2}\)
  • Effective internal resistance = \(\frac{r_1 r_2}{r_1+r_2}\)
  • Current through the circuit, \(I=\frac{E_1 r_2+E_2 r_1}{r_1 r_2+R\left(r_1+r_2\right)}\)
  • Percentage change in resistance due to change in size of wire:
  • \(\frac{\Delta R}{R} \times 100 \%=2 \frac{\Delta l}{l} \times 100 \%\)
  • \(\frac{\Delta R}{R} \times 100 \%=-2 \frac{\Delta A}{A} \times 100 \%\)
  • \(\frac{\Delta R}{R} \times 100 \%=-4 \frac{\Delta r}{r} \times 100 \%\)
  • [where l = length of the conductor, r = radius of the conductor and A = cross-sectional area of the conductor.]
  • \(\frac{R_1}{R_2}=\frac{l_1 A_2}{l_2 A_1}=\frac{l_1 \pi r_2^2}{l_2 \pi r_1^2}=\frac{l_1 r_2^2}{l_2 r_1^2} \text { and } \frac{A_1}{A_2}=\frac{l_2}{l_1}\)
  • When different cells are connected in different patterns:

Class 12 Physics Unit 2 Current Electricity Chapter 1 Electric Current and Ohm's Law different cells are connectedin different patterns

Class 12 Physics Unit 2 Current Electricity Chapter 1 Electric Current and Ohm's Law different cells are connectedin different patterns..

Ohm’s Law Very Short Questions And Answers

Question 1. For what property of conductors, current will flow through a wire connecting them?
Answer: Potential Difference

Question 2. Does the emf of a standard electric cell depend on the volume of the cell?
Answer: No

Question 3. What kind of cell should be preferred to get a high current?
Answer: Secondary cell

Question 4. Which active electrolyte is used in a lead-acid accumulator
Answer: Sulphuric acid

Question 5. If a current of 1 mA flows through a conductor having a potential difference of 1 V between its two ends, what will be the resistance of the conductor?
Answer: 1000Ω

Question 6. For a metallic conductor, what is the nature of the graph of current strength vs. potential difference?
Answer: Straight line

Question 7. The resistance of a conductor is 200Ω and the current through it is 10 mA. What is the potential difference across the two ends of the conductor?
Answer: 2V

Question 8. The resistivity of copper is 1.76 x 10-6 Ω. cm. Express it in
Answer: 1.76 x 10-8Ω.m

Question 9. The resistivity of copper is 1.76 x 10-6Ω.cm. Determine the resistance of a copper rod having a length of 10 cm and cross
Answer: 1.76 x 10-5

Question 10. Two conducting wires of lengths l and 2l have the same cross-sectional area. Compare their resistances.
Answer: Ratio = 1:2

Question 11. Two wires A and B are of the same metal and of the same length. Their areas of cross section are in the ratio of 2: 1. If the same potential difference is applied across each wire in turn, what will be the ratio of the currents flowing in A and B?
Answer: 2: 1

Question 12. What will be the change in the resistance of a Eureka wire, when its radius is halved and length is reduced to one-fourth of its original length?
Answer: Resistance Is Unchanged

Question 13. Two wires A and B of the same metal have the same cross-sectional area and have their lengths in the ratio 2:1. What will be the ratio of currents flowing through them respectively, when the same potential difference is applied across each of them?
Answer: 2:1

Question 14. Name a substance whose resistance decreases with the
Answer: Semiconductor

Question 15. What is the unit of temperature coefficient of resistance?
Answer: ºC-1

Question 16. The temperature coefficient of resistance for the material of a conductor is 38 x 10-4 ºC-1. What will be its value in ºF-1? The range of rise in temperature can be assumed small.
Answer: 21.1 x 10-4ºF-1

Question 17. A carbon resistor is coloured with four different bands brown, black, orange, and silver respectively. Find the range of its probable resistance.
Answer: 9000-11000Ω

Question 18. The resistance of a carbon resistor is 6.8kfl. What is the first
Answer: Blue, Grey, Red

Question 19. Of metals and alloys, which has a greater value of temperature coefficient of resistance?
Answer: Metals

Question 20. How are different electrical appliances connected in domestic electric connection?
Answer: In parallel

Question 21. Two resistances 1Ω and 2Ω are connected in series and a potential difference of 6Ω is applied across the ends of this combination. What will be the terminal potential difference across the second resistance?
Answer: 4V

Question 22. Two resistances 1Ω and 2Ω are connected in parallel and a potential difference of 6 V is applied across the ends of this combination. Calculate the current through the second conductor.
Answer: 3A

Question 23. In the circuit given, what is the potential difference between the two points A and B?

Class 12 Physics Unit 2 Current Electricity Chapter 1 Electric Current and Ohm's Law Question 26 the potential difference

Answer: 6V

Question 24. What is the value of I in the circuit?
Answer: 3A

Question 25. Two resistances of 6Ω and 3Ω are connected in parallel. When current is sent through this combination, compare the currents through the resistances.
Answer: Ratio = 1:2

Question 26. A metallic wire of resistance R is folded into two equal parts and then wound well. What will be new resistance then?
Answer: \(\frac{R}{4}\)

Question 27. The resistance of an electrical appliance is 200Ω and it can withstand a maximum current of 1 A. To operate the appliance on a dc source of 220 V what minimum resistance should be connected in series with it?
Answer: 20Ω

Question 28. The equivalent resistance of two resistances in a series is four times the equivalent resistance when they are in parallel. If one of the resistances is R, then what would be the resistance of the other?
Answer: R

Question 29. Three resistances, each of 4Ω, are connected in the form of an equilateral triangle. Find the effective resistance between its two corners.
Answer: 2.67Ω

Question 30. Name the quantity for which the potential difference of a cell becomes less than its emf due to its internal resistance.
Answer: Lost Volt

Question 31. Emf of a cell is 1.5 V and its internal resistance is 1Ω. When the cell sends current in an external circuit having resistance 2Ω, then what will be the value of the lost volt?
Answer: 0.5 V

Question 32. When a shunt of 1Ω is connected in parallel with a galvanometer, 1% of the main current flows through the galvanometer. Determine the resistance of the galvanometer.
Answer: 99Ω

Question 33. If the current through a galvanometer of resistance G is to be reduced n times, what should be the shunt resistance?
Answer: \(\frac{G}{n-1}\)

Question 34. A shunt of 1Ω is connected in parallel with a galvanometer of resistance 99Ω. If the main current of the circuit be 1 A, then what will be the galvanometer current?
Answer: 0.01 A

Question 35. n electric cells having emf e and internal resistance r each are connected in parallel. What is the emf of this combination?
Answer: e

Question 36. n electric cells of emf e and internal resistance r each are connected in series. What is the emf of this combination?
Answer: ne

Question 37. The potential difference across a given copper wire is increased. What happens to the drift velocity of the charge carriers?
Answer: It Increases

Ohm’s Law Fill in The Blanks

Question 1. Lead oxide is used as a positive electrode in a lead-acid accumulator as an active component

Question 2. Spongy lead is used as a negative electrode in a lead-acid accumulator as an active component

Question 3. The internal resistance of a secondary cell is less than that of a primary cell

Question 4. Equivalent resistance in a parallel combination is less than each of the component resistances

Question 5. If the current through a circuit or the internal resistance of a cell is zero, then the value of the lost volt becomes zero

Question 6. In metallic conductor, the direction of conventional current is opposite to the direction of the flow of free electrons

Question 7. The velocity of electric current is much greater than the drift velocity of free electrons in a metallic conductor

Ohm’s Law Assertion Reason Type

Direction: These questions have Statement 1 and Statement n. Of the four choices given below, choose the one that best describes the two statements.

  1. Statement 1 is true, Statement 2 is true; Statement 2 is a correct explanation for Statement 1.
  2. Statement 1 is true, Statement 2 is true; Statement 2 is not a correct explanation for Statement 1.
  3. Statement 1 is true, Statement 2 is false.
  4. Statement 1 is false, and Statement 2 is true.

Question 1.

Statement 1: If a conducting wire is stretched to twice its length, the resistance becomes twice.

Statement 2: For a fixed wire, the resistance is proportional to its length.

Answer: 4. Statement 1 is false, Statement 2 is true.

Question 2.

Statement 1: The temperature (t) dependence of the resistance (R) of a metallic conductor.

Class 12 Physics Unit 2 Current Electricity Chapter 1 Electric Current and Ohm's Law Question 2 temperature dependence of the resistance

Statement 2: The resistance R of a metallic conductor is related to its temperature t as R = R0(1 + αt).

Answer: 1. Statement 1 is true, Statement 2 is true; Statement 2 is a correct explanation for Statement 1.

Question 3.

Statement 1: The drift velocity of free electrons is vd when a current passes through a copper wire. The drift velocity is halved when the same current passes through another copper wire of double the diameter.

Statement 2: For the same current, the drift velocity of free electrons in a metal wire is inversely proportional to the area of the cross-section of the wire.

Answer: 4. Statement 1 is false, Statement 2 is true.

Question 4.

Statement 1: In the circuit of no current passes through the 2Ω resistor.

Statement 2: The current through a resistor is proportional to its terminal potential difference.

Class 12 Physics Unit 2 Current Electricity Chapter 1 Electric Current and Ohm's Law The current through a resistor is proportinal to its teminal potential difference

Answer: 1. Statement 1 is true, Statement 2 is true, and Statement 2 is a correct explanation for Statement 1.

Question 5.

Statement 1: In the circuit of IG = 10 mA.

Class 12 Physics Unit 2 Current Electricity Chapter 1 Electric Current and Ohm's Law In the circuit

Statement 2: If a shunt of resistance \(\frac{G}{n}\) is connected parallel to a galvanometer, \(\frac{1}{n}\) times the main circuit current passes through the galvanometer.

Answer: 3. Statement 1 is true, Statement 2 is false.

Question 6.

Statement 1: The voltmeter reading does not denote the correct emf of an electric cell when the terminals of the cell are directly connected to the voltmeter.

Statement 2: As every electric cell has some internal resistance, the current in the external circuit is reduced to some extent.

Answer: 2. Statement 1 is true, Statement 2 is true; Statement 2 is not a correct explanation for Statement 1.

Question 7.

Statement 1: In the circuit of the internal potential drop of the cell decreases with the increase of the external resistance R

Class 12 Physics Unit 2 Current Electricity Chapter 1 Electric Current and Ohm's Law the internal potential

Statement 2: The potential difference available from an electric cell in the external circuit is proportional to the resistance of that external circuit.

Answer: 3. Statement 1 is true, Statement 2 is false.

Question 8.

Statement 1: If a current flows through a wire of a non-uniform cross-section, the potential difference per unit length of the wire in the direction of the current is the same at different points.

Statement 2: V = IR and the current in the wire is the same throughout.

Answer: 4. Statement 1 is false, Statement 2 is true.

Question 9.

Statement 1: Out of the galvanometer, ammeter, and voltmeter, the resistance of the ammeter is the lowest and the resistance of the voltmeter is the highest.

Statement 2: An ammeter is connected in series and a voltmeter is connected in parallel, in a circuit.

Answer: 2. Statement 1 is true, Statement 2 is true; Statement 2 is not a correct explanation for Statement 1.

Question 10.

Statement 1: A current flows in a conductor only when there is an electric field within the conductor.

Statement 2: The drift velocity of electrons decreases in the presence of the electric field.

Answer: 3. Statement 1 is true, and Statement 2 is false.

Ohm’s Law Match The Columns

Question 1. Three uniform conducting wires of the same material have lengths in the ratio 2:2:1 and radii in the ratio 2:1:1. The resistance of the first wire is R.

Class 12 Physics Unit 2 Current Electricity Chapter 1 Electric Current and Ohm's Law Match the columns 1

Answer: 1-D, 2-B, 3-A, 4-C

Question 2. Silver has the highest electrical conductivity among different metals. Some materials are given in Column A and their resistivities (in Ω.cm) are in Column B.

Class 12 Physics Unit 2 Current Electricity Chapter 1 Electric Current and Ohm's Law Match the columns 2

Answer: 1-B, 2-C, 3-D, 4-A

Question 3. Match the following two columns in the context of an electric current passing through a metal conductor.

Class 12 Physics Unit 2 Current Electricity Chapter 1 Electric Current and Ohm's Law Match the columns 3

Answer: 1-D, 2-A, 3-B, 4-C

Question 4. In the circuit, the ammeter A and the voltmeter V are both ideal. Each of the cells E1 and E2 has an emf of 2 V and an internal resistance of 2Ω

Class 12 Physics Unit 2 Current Electricity Chapter 1 Electric Current and Ohm's Law Match the columns 4.

Class 12 Physics Unit 2 Current Electricity Chapter 1 Electric Current and Ohm's Law Match the columns 4

Answer: 1-B, 2-D, 3-A, 4-C

Question 5. In the circuit of the galvanometer resistance is G = 100Ω.

Class 12 Physics Unit 2 Current Electricity Chapter 1 Electric Current and Ohm's Law Match the columns 5.

Class 12 Physics Unit 2 Current Electricity Chapter 1 Electric Current and Ohm's Law Match the columns 5

Answer: 1-C, 2-A, 3-D, 4-B

Question 6. The first three characteristic colors of carbon resistors are given in column A and their resistances in column B.

Class 12 Physics Unit 2 Current Electricity Chapter 1 Electric Current and Ohm's Law Match the columns 6

Answer: 1-B, 2-D, 3-A, 4-C

Question 7. Match the columns from the circuit.

Class 12 Physics Unit 2 Current Electricity Chapter 1 Electric Current and Ohm's Law Match the columns 7

Class 12 Physics Unit 2 Current Electricity Chapter 1 Electric Current and Ohm's Law Match the columns 7.

Answer: 1-B, 2-C, 3-A, 4-D

WBCHSE Class 12 Physics Notes For Electric Potential

Electrostatics

Electric Potential

When two positively charged insulated conductors are connected by a metallic wire, charge flows from one conductor to the other. The direction of flow of charge does not depend on the amount of charge on them but depends on a specific electric condition of the two conductors.

This electric property of a charged body is called electric potential. So the electric potential of a body is the electric property that determines whether charge will flow from this body to any other body connected to ft or from any other body to itself.

1. Analogy between electric potential and hydrostatic level or height of water column: Two interconnected vessels A and B of different sizes contain water of different amounts held at different heights.

When the connection is established by opening the stop-cock D, water flows from the vessel with a higher level of water to the other, until the levels are equal.

Read and Learn More Class 12 Physics Notes

So the flow of water does not depend on the quantity of water but depends on the height of the water column or pressure of water. Similarly, the flow of charge or electricity between two bodies depends on the difference in their potential but not on the quantity of charge they have.

Class 12 Physics Unit 1 Electrostatics Chapter 3 Electric Potential Analogy between electric potential and hydrostatic level or height of water column

Water flows from higher to lower levels, i.e., from higher to lower pressure. Similarly, electric charge flows from one body at a higher potential to another at a lower potential.

So electric potential may be compared with the height of water as well as the pressure of water and the amount of charge with the quantity of water.

2. Analogy between electric potential and temperature: Heat flows from a hotter body to a colder one in contact till their temperatures are equalized. This occurs even if the heat content of the colder body is more than that of the hot one.

Thus flow of heat depends only on the temperature difference of the bodies in contact, but not on their heat contents. In this context, the electric potential may be compared with temperature and the amount of charge with heat.

It may be noted that while the flow of water or heat is indirect, the flow of charge depends on its nature. According to the sign convention, positive charge flows from higher to lower potential and negative charge from lower to higher potential. No charge however flows from one body to another if they are at the same potential

Potential at a Point in an Electric Field:

Consider an isolated charge in a region where no electric field is present. Now, due to the presence of the charge, an electric field develops in that region. When a second charge is brought into that region, an electric force acts on it.

To displace the second charge within this region, work has to be done, either by some external agent or by the field itself, depending on the nature of the charge.

Hence it can be concluded that a medium containing a charge acquires some property for which some work has to be done to displace another charge in that medium. This property is known as electric potential.

To define electric potential we need an infinitesimal test charge (q) that does not disturb the priorly existing charge (Q) which causes an electric field.

In this context, we shall name this test charge as a unit charge, i.e., q = 1. Also, we assume though there is no boundary of the electric field, the potential beyond the field i.e., at infinity is zero. The initial position of the test charge is assumed to be at infinity which is its reference point.

From this reference point if a unit positive charge is to be taken in an electric field one has to apply a minimum force (just equal but opposite to that electric field force) to take the test charge from infinity to a specific point in the electric field and hence work is to be done. As In such work done no net force is applied to the test charge it has no acceleration.

Definition: The potential at any point in an electric field is defined as the work done by external force in bringing a unit positive charge without acceleration from infinity to that point.

Suppose, the potential at any point in an electric field is V. By definition, work done to bring a unit positive charge from infinity to that point = V.

Therefore, work done to bring a charge q from infinity to that point, W = V.q

work done = potential x charge

This work done is stored up as electric potential energy of the system consisting of the charge and the external electric field. So, electric potential energy = potential x charge i.e., electric potential energy of a unit charge placed at a point in an electric field is the electric potential at that point.

In equation (1), since work done and charge are both scalar quantities, so, potential as well as potential energy are scalar quantities.

The potential of a positively charged body is said to be positive because work has to be done by some external agency to bring a unit positive charge from infinity to any point in the electric field.

This is to overcome the force of repulsion between the positively charged body and the unit positive charge (test charge). So energy is stored in the system consisting of the positively charged body and test charge. Therefore both W and V are positive.

On the other hand, the potential of a negatively charged body is said to be negative, because a unit positive charge moves itself closer to the charged body due to attraction, i.e., work is done by the attractive force. So both W and V are negative.

The potential difference between two points: The potential difference between two points in an electric field is defined as the amount of work done by an external force to bring a unit positive charge without acceleration from one point to the other.

Let VA and VB be the electric potentials at the points A and B, respectively in an electric field. If WAB is the work done in bringing the charge q from A to B, the physical difference between the two points is given by,

⇒ \(V_B-V_A=\frac{W_{A B}}{q}\)

If WAB is positive, VB>VA.

If WAB is negative, VB <  VA.

If WAB is zero’ VB = VA.

The potential difference between two points Is independent of the path connecting the points:

The potential difference between two points in an electric field does not depend on the path followed from one point to another. The path may be straight or curved but the amount of work done, i.e., potential difference remains the same.

Proof: Supposed and B are two points in an electric field and an external agent performs an amount of work Wy to bring a unit positive charge from point A to point B along the path ACB. Now this unit positive charge is brought back to A from B along the path BDA. In this case, let us suppose that the work done by the electrical force = W2, where W2 is not W1.

Let W2 > W1.

Class 12 Physics Unit 1 Electrostatics Chapter 3 Electric Potential Potential difference between two points is independent

So the amount of work done in the whole process is (W2– W1); i.e., this net amount of energy will be surplus. If the unit positive charge is taken repeatedly along the closed path ACBDA, energy would evolve continuously.

However, according to the principle of conservation of energy, this is not possible. So (W2– W1) should be equal to zero, i.e., W2 = W1.

So in an electrostatic field, O’s total work done in moving a charge around a closed path is zero and the potential difference between two points is independent of the path along which a charge may be brought from one point to the other. So, like gravitational force, electrical force is also conservative.

Units of Potential:

1. GO In CGS system: The CGS unit of potential is esu of potential or statvolt (statV). The potential at a point in an electric field is said to be 1 esu if 1 erg of work is done in bringing 1 esu of positive charge from infinity to that point.

2. In SI: The SI unit of potential is volt (V). The potential at a point in an electric field is said to be 1 volt if 1 joule of work brings 1 coulomb of positive charge from infinity to that point.

\(1 \mathrm{~V}=\frac{1 \mathrm{~J}}{1 \mathrm{C}}=\frac{10^7 \mathrm{erg}}{3 \times 10^9 \text { esu of charge }}\)

⇒ \(\frac{1}{300} \times \frac{1 \text { erg }}{1 \text { esu of charge }}\)

= \(\frac{1}{300}\) esu of potential

∴ 1 esu of potential = 300 volt

The unit of potential difference is the same as that of potential.

Dimension of Potential:

⇒ \([V]=\frac{[W]}{[q]}=\frac{\mathrm{ML}^2 \mathrm{~T}^{-2}}{I \mathrm{~T}}=\mathrm{ML}^2 \mathrm{~T}^{-3} \mathrm{I}^{-1}\)

The potential of the Earth:

The potential of a body is measured concerning the potential of the earth which is taken as the standard and conventionally assigned the value of zero. The earth is so large that its potential does not change appreciably due to small gain or loss of charge. It effectively maintains its constant potential.

A body is said to be at a positive potential if its potential is above that of the Earth and at a negative potential if its potential is below that of the Earth. This is analogous to the situation in that the sea level is taken as the standard zero level to measure the altitudes and depths of different places on the Earth.

Potential of a Charged Body:

A body is said to be at a positive potential if there is a flow of electrons from the earth to it when the body comes in contact with the earth. If electrons flow from the body to the earth when electrical contact is established between them, the body is said to be at a negative potential.

In both cases, the flow of electrons will continue until the potential of the body becomes zero, i.e., equal to the potential of the earth. Any earthed (or grounded) conductor is effectively at zero potential

Class 12 Physics Unit 1 Electrostatics Chapter 3 Electric Potential Potential of a Charged Body

The measure of the potential of a charged body: The work done by an external force to bring a unit positive charge without acceleration very near to a charged body from infinity is the measure of the potential of that body

Potential at a Point in the Field of a Point Charge:

Consider a charge +q at a point A in vacuum or air. Let P be a point at a distance r from A, where the electric potential due to the charge at A is to be determined.

The intensity at P, in vacuum or air, due to the charge +q at

⇒ \(A=\frac{1}{4 \pi \epsilon_0} \frac{q}{r^2} ; \text { along } \overrightarrow{A P}\)

Class 12 Physics Unit 1 Electrostatics Chapter 3 Electric Potential Potential at a Point in the Field of a Point charge

If PP1 = dr be a very small distance beyond P where the intensity effectively remains the same, then work done by the external force (equal but opposite of the electric force) in bringing a unit positive charge from Py to P is,

dW = external force acting on the unit positive charge x its displacement

⇒ \(\frac{1}{4 \pi \epsilon_0} \frac{q}{r^2} \cdot(-d r)\) [The negative sign is taken because intensity and displacement are oppositely directed]

Therefore, the work done in bringing the unit positive charge from infinity to P is given by,

⇒ \(W=\frac{1}{4 \pi \epsilon_0} \int_{\infty}^r \frac{-q}{r^2} d r=-\frac{q}{4 \pi \epsilon_0} \int_{\infty}^r \frac{d r}{r^2}\)

⇒ \(-\frac{q}{4 \pi \epsilon_0}\left[-\frac{1}{r}\right]_{\infty}^r=\frac{1}{4 \pi \epsilon_0} \frac{q}{r}\)

So, the potential at P due to the charge +q at A is,

⇒ \(V=\frac{1}{4 \pi \epsilon_0} \frac{q}{r}\)…..(1)

The potential of a point at a distance r from the charge q in a medium of permittivity e is,

⇒ \(V=\frac{1}{4 \pi \epsilon} \frac{q}{r}=\frac{1}{4 \pi \kappa \epsilon_0} \frac{q}{r}\)

In CGS system, for vacuum or in air, V = \(\frac{q}{r}\)

The potential difference between the two points:

Points are collinear with the charge: Suppose, B and A are two points at a distance r2 and r1 respectively from a point charge +q at O. The points 0, B, and A lie on the same straight line

Class 12 Physics Unit 1 Electrostatics Chapter 3 Electric Potential Potential difference between two points

According to equation (1), the potential difference between B arid A is,

⇒ \(V_B-V_A=\frac{1}{4 \pi \epsilon_0} \int_{r_1}^{r_2} \frac{-q}{r^2} d \dot{r}\)

⇒ \(\frac{-q}{4 \pi \epsilon_0} \int_{r_1}^{r_2} \frac{d r}{r^2}=\frac{-q}{4 \pi \epsilon_0}\left[-\frac{1}{r}\right]_{r_1}^{r_2}\)

or, \(V_B-V_A=\frac{q}{4 \pi \epsilon_0}\left(\frac{1}{r_2}-\frac{1}{r_1}\right)\)….(2)

It may be noted that the potentials of the points equidistant from a point charge are equal.

For this reason, the potential difference between a point situated in a sphere of radius r2 and a point situated in a sphere of radius r1 may be obtained from equation (2).

2. For non-collinear points: Let the position vector of a point charge q be

⇒ \(\vec{r}\).

According to the \(\vec{r}_2 \text { and } \vec{r}_1\) be the position vectors of B and A respectively then equation (2) can be expressed as

VB – VA = \(\frac{q}{4 \pi \epsilon_0}\left(\frac{1}{\left|\vec{r}_2-\vec{r}\right|}-\frac{1}{\left|\vec{r}_1-\vec{r}\right|}\right)\)

Class 12 Physics Unit 1 Electrostatics Chapter 3 Electric Potential For non-collinear points

Potential due to a system of charges:

Consider the point charges q1, q2, q3 situated at distances r1, r2, r3,…. from a point P. Since the electric potential is a scalar quantity, the algebraic sum of the potentials at P due to individual charges is the potential at P. If the dielectric constant of a medium is K, then the potential at P due to multiple charges,

⇒ \(V=\frac{1}{4 \pi \kappa \epsilon_0}\left(\frac{q_1}{r_1}+\frac{q_2}{r_2}+\cdots+\frac{q_n}{r_n}\right)=\frac{1}{4 \pi \kappa \epsilon_0} \sum \frac{q}{r}\)

Potential due to an Electric Dipole:

1. Potential at a point on the axis of a dipole (end-on position):

Let AB be an electric dipole formed by the charges +q and -q separated by a small distance of 2l.

Obviously AB = 2l and dipole moment p = 2lq.

The dipole is placed in a vacuum or air. For this dipole, the potential at point P situated on the dipole axis at a distance r from the mid-point O of the dipole is to be determined. The distance of P from +q charge = (r – l) and that from -q charge = (r + l).

Class 12 Physics Unit 1 Electrostatics Chapter 3 Electric Potential Potential due to an Electric Dipole

Therefore, the potential at P due to the charge +q at B of the dipole is

⇒ \(V_1=\frac{1}{4 \pi \epsilon_0} \frac{q}{(r-l)} \quad(\text { in SI })\)

Potential at P due to the charge -q at A of the dipole is

⇒ \(V_2=\frac{1}{4 \pi \epsilon_0} \frac{-q}{(r+l)} \quad \text { (in SI) }\)

Electric potential is a scalar quantity. So the resultant potential at P is

⇒ \(V=V_1+V_2=\frac{1}{4 \pi \epsilon_0}\left[\frac{q}{(r-l)}+\frac{-q}{(r+l)}\right]\)

⇒ \(\frac{q}{4 \pi \epsilon_0}\left[\frac{1}{(r-l)}-\frac{1}{(r+l)}\right]\)

⇒ \(\frac{q}{4 \pi \epsilon_0}\left[\frac{r+l-r+l}{\left(r^2-l^2\right)}\right]=\frac{q}{4 \pi \epsilon_0} \cdot \frac{2 l}{r^2-l^2}\)

⇒ \(\frac{1}{4 \pi \epsilon_0} \cdot \frac{p}{r^2-l^2}\) [∵ p = 2ip]….(1)

If r >> l, l2 can be neglected in comparison to r2.

Then the electric potential at P due to the electric dipole is

⇒ \(V=\frac{1}{4 \pi \epsilon_0} \frac{p}{r^2} \quad(\text { In SI) }\)….(2)

In the CGS system equations, (I) and (2) are respectively given by,

⇒ \(V=\frac{p}{r^2-l^2}\)…(3)

⇒ \(V=\frac{p}{r^2}\)…(4)

2. Potential at a point on the perpendicular bisector of a dipole (broadside-on position):

Let AB be an electric dipole formed by the charges +q and -q separated by a small distance 21. The dipole is situated in a vacuum or air. Obviously AB = 21 and dipole moment’ p = 2Iq.

For this dipole, the potential at P situated on the perpendicular bisector of the dipole at a distance r from the mid-point O of the dipole is to be determined.

Class 12 Physics Unit 1 Electrostatics Chapter 3 Electric Potential Potential at a point on the perpendicular bisector of a dipole

Potential at P due to the charge +q at B of the dipole is

⇒ \(V_1=\frac{1}{4 \pi \epsilon_0} \cdot \frac{q}{B P}\)

Potential at P due to the charge -q at A of the dipole is

⇒ \(V_2=\frac{1}{4 \pi \epsilon_0} \cdot \frac{-q}{A P}\)

Therefore, the net potential at P is

V = V1 + V2

⇒ \(\frac{1}{4 \pi \epsilon_0} \cdot \frac{q}{B P}+\frac{1}{4 \pi \epsilon_0} \cdot \frac{-q}{A P}\)

⇒ \(\frac{q}{4 \pi \epsilon_0}\left(\frac{1}{B P}-\frac{1}{A P}\right)=0\) [∵ BP = AP]…..(5)

So the electric potential is zero everywhere on the equatorial line of an electric dipole (but the electric intensity is not zero).

Hence this perpendicular bisector is an equipotential line. No work is done to move a charge along this line.

In the CGS system equation (5) remains the same, i.e., V = 0.

3. Potential at any point due to an electric dipole: Let AB be an electric dipole formed by the charges +q and -R separated by a small distance 21. AB = 2l and dipole moment p = 2lq. Let Pÿbe a point at a distance r from the mid-point O of the dipole and the line OP make an angle θ with the axis of the dipole i.e., the polar coordinates of P are (r, θ). Potential at P due to the dipole is to be calculated.

Class 12 Physics Unit 1 Electrostatics Chapter 3 Electric Potential Question 3 The length of the dipole

Now join PA and PB. Then draw a perpendicular from A which meets the extended OP at D. Also draw BC perpendicular to OP.

If r>>l, we can write BP ≈ CP = OP- OC = r – lcosθ and Ap ≈ DP = OP + OD = r + lcosθ

Now, the potential at P due to the charge +q at B of the dipole is

⇒ \(V_1=\frac{1}{4 \pi \epsilon_0} \cdot \frac{q}{B P}=\frac{1}{4 \pi \epsilon_0} \cdot \frac{q}{(r-l \cos \theta)}\)

Potential at P due to the charge -q at A of the dipole is-

⇒ \(V_i=\frac{1}{4 \pi \epsilon_0} \cdot \frac{-q}{A P}=-\frac{1}{4 \pi \epsilon_0} \cdot \frac{q}{(r+l \cos \theta)}\)

So, the net potential at P is

⇒ \(V^{\prime}=V_1+V_2=\frac{q}{4 \pi \epsilon_0}\left[\frac{1}{(r-l \cos \theta)}-\frac{1}{(r+l \cos \theta)}\right]\)

⇒ \(\frac{q}{4 \pi \epsilon_0}\left[\frac{r+l \cos \theta-r+l \cos \theta}{r^2-l^2 \cos ^2 \theta}\right]\)

⇒ \(\frac{q}{4 \pi \epsilon_0} \cdot \frac{2 l \cos \theta}{r^2-l^2 \cos ^2 \theta}\)

= \(\frac{1}{4 \pi \epsilon_0} \cdot \frac{p_{\cos \theta}}{r^2-l^2 \cos ^2 \theta}\)

⇒ \(\text { As } r \gg l \text {, we get } r^2-l^2 \cos ^2 \theta=r^2\)

∴ \(V=\frac{-1}{4 \pi \epsilon_0} \cdot \frac{p \cos \theta}{r^2}\)….(6)

Special Case:

CO P is on the axis of the dipole, then θ = 0 or, cosθ = 1

From equation (6) we get, \(V=\frac{1}{4 \pi \epsilon_0} \cdot \frac{p}{r^2}\)

This is equation (2).

2. If P is on the perpendicular bisector, then

θ = 90º

or, cos90º = 0

So from equation (6) we have, V = 0.

This is equation (5).

The vector form of equation (6) is

⇒ \(V=\frac{1}{4 \pi \epsilon_0}\).\(\frac{\vec{p} \cdot \hat{r}}{r^2}\)….(7)

where \(\hat{r}\) = unit vector along \(\hat{r}\)

In the CGS system equations (6) and (7) are respectively given by,

⇒ \(V=\frac{p \cos \theta}{r^2}\)….(8)

⇒ \(V=\frac{\vec{p} \cdot \hat{r}}{r^2}\)….(9)

Electrostatics

Electric Potential Relation Between Electric Field Intensity And Electric Potential

In uniform electric field: Suppose, A and B are two points on a field line in a uniform electric field of intensity E.

Let d be the distance between the two points and VA and VB be the electric potentials at A and B, respectively. The direction of electric intensity is from A to B, then VA > VB, and if a free positive charge is placed in this field the charge will move from A to B. Potential difference between A and

B = VA-VB.

Class 12 Physics Unit 1 Electrostatics Chapter 3 Electric Potential In uniform electric field

Work done to bring a unit positive charge from B to A

= -E.d [negative sign indicates that intensity and displacement are oppositely directed]

According to the definition of potential difference, this work done is equal to the potential difference between the two points.

∴ VA – VB = -E x d

or, \(E=-\frac{V_A-V_B}{d}\)….(1)

This is the relation between intensity and potential difference in a uniform electric field.

In A non-uniform electric field: Let us consider two points nA land £ very close to each other in a non-uniform electric field- and the distance between the two points = dx. Let the electric field intensity E be directed from A to B.

Since dx is very small, E is practically constant between A and B. As the direction of Intensity is from A to B, VA > VB.

Class 12 Physics Unit 1 Electrostatics Chapter 3 Electric Potential In a non-uniform electric field

Let the potential at A be V+dV and that at B be V. The work done to bring a unit positive charge from B to A = -E.dx. [negative sign Indicates that intensity and displacement are oppositely directed]

According to the definition of potential difference, this work done is equal to the potential difference between the two points.

∴ (V+dV)-V = -Edx

or, \(E=-\frac{d V}{d x}\)…(2)

This is the relation between intensity and potential difference in a non-uniform electric field.

An alternative unit of electric field intensity: The relation E = – \(\frac{dV}{dx}\) suggests that, in the CGS system, a unit of electric field intensity is statV.cm-1 and in SI, it is V.m-1;

Electric potential gradient: In equation (2), \(\frac{dV}{dx}\) is known as a potential gradient. Potential gradient is the rate of change of electric potential concerning displacement. The negative sign indicates the value of potential gradually decreases as we proceed in the direction of the electric field.

Electric Field Intensity and Electric Potential due to a Uniformly Charged Sphere:

It is ten difficult to calculate electric field intensity and potential due to an irregularly shaped charged conductor. But for some regular shaped conductors, intensity and potential can be calculated easily. To calculate intensity and potential due to a uniformly charged sphere at an external point or on its surface, it can be assumed that the whole charge of the sphere is concentrated at its center.

Suppose a spherical conductor of radius r has q Unit of charge. For the calculation of electric field intensity and potential at P at a distance x from its center O, it is assumed that the charge q is concentrated at the center of the sphere.

The electric field intensity at \(p=\frac{1}{4 \pi \epsilon_0} \frac{q}{x^2}\)

[It is assumed that the sphere and the point P are In the air]

Electric potential at \(P=\frac{1}{4 \pi \epsilon_0} \frac{q}{x},(x>r)\)

Class 12 Physics Unit 1 Electrostatics Chapter 3 Electric Potential Electric Field Intensity and Electric potential due to a uniformly charged sphere

If point P is situated on the surface of the sphere, then x = r.

So electric field intensity on the surface of the sphere

⇒ \(\frac{1}{4 \pi \epsilon_0} \frac{q}{r^2} ; \text { and potential }=\frac{1}{4 \pi \epsilon_0} \frac{q}{r}\)

It can also be proved that the potential everywhere inside a charged sphere is equal, and the value of this potential is equal to that on its surface, i.e, \(\frac{1}{4 \pi \epsilon_0} \frac{q}{r}\)

Since the potential inside a sphere is a constant, it is obvious from the relation E = –\({dV}{dx}\), that the electric field intensity at every point inside a sphere is zero

Electrostatics

Electric Potential Relation Between Electric Field Intensity And Electric Potential Numerical Examples

Example 1. A region is specified by the potential function V = 2x2 + 3y3– 5z2. Calculate the electric field intensity at a point (2, 4, 5) in this region.
Solution:

We know, \(\vec{E}=E_x \hat{i}+E_y \hat{j}+E_z \hat{k}=-\frac{d V}{d x} \hat{i}-\frac{d V}{d y} \hat{j}-\frac{d V}{d z} \hat{k}\)

Now, V = 2x2 + 3y3– 5z2

∴ \(\frac{d V}{d x}=4 x ; \frac{d V}{d y}=9 y^2 ; \frac{d V}{d z}=-10 z\)

[here, x, y, and z are mutually independent variables]

At the point (2, 4, 5)

⇒ \(\frac{d V}{d x}=4 \times 2=8 ; \frac{d V}{d y}=9 \times(4)^2\)

= 144

⇒ \(\frac{d V}{d x}\)

= -10 x 5

= -50

∴ At the point (2, 4, 5), \(\vec{E}=-8 \hat{i}-144 \hat{j}+50 \hat{k}\)

∴ \(E=\sqrt{E_x^2+E_y^2+E_z^2}\)

⇒ \(=\sqrt{(8)^2+(144)^2+(50)^2}\)

= 152.64 units

Example 2. Two points A and B are situated at distances lm and 2 m from the source of an electrostatic field. The field at a distance x from the source is E = \(B=\frac{5}{x^2}\). What is the potential difference between A and B?
Solution:

We know that, E = –\(\frac{dV}{dx}\)

Here, \(B=\frac{5}{x^2}\)

∴ \(\frac{5}{x^2}=-\frac{d V}{d x} \quad \text { or, } d V=-\frac{5}{x^2} d x\)

∴ \(V_A-V_B=\int_2^1-\frac{5}{x^2} d x^{\prime}=-5\left[-\frac{1}{x}\right]_2^1\)

⇒ \(5\left(1-\frac{1}{2}\right)\)

= \(\frac{5}{2}\)

= 2.5 units

Example 3. In an electric field, the potential V(x) depending only on the x-coordinate, is given by V(x) = ax- bx3, where a and b are constants. Find out the points on the x-axis where the electric field intensity would vanish.
Solution:

⇒ \(E=-\frac{d V}{d x}=-\frac{d}{d x}\left(a x-b x^3\right)=-a+3 b x^2\)

E would vanish, i.e., E = 0, under this condition that,

⇒ \(-a+3 b x^2=0 \quad\)

or, \(x^2=\frac{a}{3 b} \quad\)

or, \(x= \pm \sqrt{\frac{a}{3 b}}\)

∴ The electric field intensity vanishes at the points

⇒ \(x=\sqrt{\frac{a}{3 b}} \text { and } x=-\sqrt{\frac{a}{3 b}} \text {, on the } x \text {-axis }\)

Electrostatics

Electric Potential Electrical Potential Energy

The electrical potential energy of a system of charges is equal to the total work done by an external agent to bring the charges, one by one, from infinite separation to the desired positions to form the system.

Let q1 and q2 be two charges placed at points A and B respectively and AB = r. To determine the potential energy of the system of the two charges q1 and q2, let us suppose that in the region only the charge q1 existed initially but q2 was absent.

Class 12 Physics Unit 1 Electrostatics Chapter 3 Electric Potential electrical potential energy

The potential at B due to the charge q1 is,

⇒ \(V=\frac{1}{4 \pi \epsilon_0} \frac{q_1}{r}\)

Now to bring the charge q2 from infinity to point B, work done,

⇒ \(W=V q_2\)

This work done gets stored in the system of charges q1 and q2 and is called the potential energy of the system (U)

So, \(U=V q_2=\frac{1}{4 \pi \epsilon_0} \frac{q_1 q_2}{r}\)…(1)

In the CGS system, ∈0 is to be replaced by \(\frac{1}{4 \pi}\) in equation (1).

The units of U in SI is joule (J) and in CGS is erg.

To determine the potential energy of a system of multiple charges, the above procedure has to be repeated step by step.

For a system of charges q1, q2, q3, q4……. we have to continue the calculations after equation (1) as follows :

1. Work done to bring the charge q3 from infinity to a point near the already formed system of q1 and q2;

2. Work done to bring the charge q3 from infinity to a point near the already formed system of q1, q2, and q3; and so on.

The algebraic sum of all the above values of work done would then be equal to the potential energy stored in the system of charges.

The procedure is often extremely complicated. However, the calculations turn out to be fairly easy for some systems with specific symmetries.

Calculation of electrostatic potential energy for a system ofiÿree point charges:

The potential; energy of a system of three charges q1, q2, and q3 is equal to the total work done to bring these charges one by one from infinity to the positions \(\vec{r}_1, \vec{r}_2 \text { and } \vec{r}_3\) respectively.

To bring the charge q1 at the position \(\vec{r}_1\), no work is done, as all other charges are still at infinity, i.e., there is no field.

So, W1 = 0

Class 12 Physics Unit 1 Electrostatics Chapter 3 Electric Potential Calculation of electrostatic potential energy for a syatem of three point charges

To bring the charge q2 from infinity to the position \(\vec{r}_2\) at a distance r12 from q1 , work is done

⇒ \(W_2=\left[\text { potential for } q_1\right] \times q_2=\frac{q_1}{4 \pi \epsilon_0 \kappa r_{12}} \times q_2=\frac{q_1 q_2}{4 \pi \epsilon_0 \kappa r_{12}}\)

Similarly, to bring the charge q3 from infinity to the position r3; work has to be done against the electrostatic forces of both q1 and q2,

W3 = [potential for q1 and q2] x q3

⇒ \(\frac{1}{4 \pi \epsilon_0 k}\left(\frac{q_1}{r_{13}}+\frac{q_2}{r_{23}}\right) \times q_3=\frac{1}{4 \pi \epsilon_0 \kappa}\left(\frac{q_1 q_3}{r_{13}}+\frac{q_2 q_3}{r_{23}}\right)\)

∴ The potential energy of a system of three charges,

U = W1 + W2 + W3

⇒ \(0+\frac{1}{4 \pi \epsilon_0 \kappa}\left(\frac{q_1 q_2}{r_{12}}\right)+\frac{1}{4 \pi \epsilon_0 \kappa}\left(\frac{q_1 q_3}{r_{13}}+\frac{q_2 q_3}{r_{23}}\right)\)

⇒ \(\frac{1}{4 \pi \epsilon_0 \times}\left[\frac{q_1 q_2}{r_{12}}+\frac{q_1 q_3}{r_{13}}+\frac{q_2 q_3}{r_{23}}\right]\)

Work Done in Deflecting a Dipole in a Uniform Electric Field and Potential Energy of the Dipole:

Consider an electric dipole placed at an angle θ with a uniform electric field of strength E. The dipole moment dipole is p. We know that the torque acting on is,

\(\tau_{\text {ext }}=p E \sin \theta\)

This torque acting on the dipole brings it along the direction of the electric field. Obviously when θ = 0°, the dipole is in equilibrium position. To deflect it from its equilibrium position work has to be done on the dipole. This work is stored up as potential energy in the dipole at its deflected position.

Suppose, the dipole is in an equilibrium position. An external torque acts on it and deflects it through an angle of θ, In this position the magnitude of the torque applied by the external agent is,

⇒ \(\tau_{\text {ext }}=p E \sin \theta\)

Work done by the external agent to deflect it from angle θ to θ + dθ is,

⇒ \(d W=\tau_{\text {ext }} d \theta\)

Class 12 Physics Unit 1 Electrostatics Chapter 3 Electric Potential potential energy in the dipole at its deflected position

So, work done to deflect the dipole from its equilibrium position to an angle of θ is

⇒ \(W=\int_0^\theta \tau \tau_{\mathrm{ext}} d \theta\)

⇒ \(\int_0^\theta p E \sin \theta d \theta=p E(1-\cos \theta)\)

Therefore, in this position potential energy of the electric dipole
is given by,

U = pE(1-cosθ)

From equation (1) we can say that to deflect the dipole

1. From the equilibrium position through an angle of 90°, work done, W = pE

2. From the equilibrium position through an angle of 180°, work done, W = 2pE

3. From angle θ1 to angle θ2, work done, W = pE(cosθ1 – cosθ2)

Again, let the dipole at first be at right angles to the electric field.

Then to bring the dipole from that position to an angle of θ, the change in potential energy is given by,

⇒ \(U(\theta)-U\left(\frac{\pi}{2}\right)=\int_{\pi / 2}^\theta p E \sin \theta d \theta\)

⇒ \(p E[-\cos \theta]_{\pi / 2}^\theta=-p E \cos \theta=-\vec{p} \cdot \vec{E}\)

The work done on a dipole is equal to the difference in potential energies between its two orientations.

As the initial value of potential energy is not physically significant, it can be taken as zero for any standard orientation of this dipole.

If we take the potential energy of the dipole to be zero when It remains at right angles to the electric field, then \(U\left(\ \ \frac {\pi}{2}\right)=0\)

Therefore in that case,

⇒ \(U(\theta)=-\vec{p} \cdot \vec{E}\)…(3)

Kinetic Energy of a Charged Body in an Electric Field:

We know that, when a body falls freely from a height under the action of gravity, it loses its potential energy but gains an equal amount of kinetic energy.

Similarly, when a charged body moves freely in an electric field, it loses a certain amount of potential energy and gains an equal amount of kinetic energy. Suppose a particle of charge q is moving from one point to another in an electric field.

If the potential difference between the two points is V, the particle loses potential energy of amount q V. So the increase of its kinetic energy will also be,

Ek = qV….(1)

If m is the mass of the particle and v1, v2 is its velocities at the first and the second points, respectively, then

⇒ \(E_k=\frac{1}{2} m\left(v_2^2-v_1^2\right)\)…(2)

∴ \(\frac{1}{2} m\left(v_2^2-v_1^2\right)=q V\)….(3)

If the particle starts from rest and acquires a velocity v at the second point then,

⇒ \(\frac{1}{2} m v^2=q V \quad\)

or, \(v=\sqrt{\frac{2 q V}{m}}\)…..(4)

With the help of this equation, velocities of charged particles like electrons, protons, etc., are determined in different atomic and nuclear experiments.

Electronvolt: It is a special unit of energy. The energies of particles like electrons, protons, etc., in atomic and nuclear physics are measured in this unit.

Definition: The amount of kinetic energy acquired by an acquired by electron, when it accelerates through a potential difference of 1 volt, is called an electronvolt (eV).

1 electronvolt (leV)

= charge of an electron x IV

= 4.8 x 10-10 esu of charge x \(\frac{1}{100}\) esu 0f potential

= 1.6 x 10-12 erg

= 1.6 X 10-19J [∵ 1 erg = 10-7 J ]

Bigger units like electron volt (keV), mega electron volt (MeV), etc. are also used.

IkeV = 103 eV = 1.6 X 10-9 erg = 1.6 X 10-16 J

IMeV = 106 eV = 1.6 X 10-6 erg = 1.6 x 10-13 J

Electrostatics

Electric Potential Electrical Potential Energy Numerical Examples

Example 1. Two point charges of +49 esu and +81 esu are placed at a separation of 100 cm In the air. Determine the position of the neutral point in the electric fields of the two charges. What Is the electric potential at the neutral point?
Solution:

The neutral point must be in between the two charges because both charges are positive. Let +49 esu and +81 esu of charges be placed respectively at A and B.

Class 12 Physics Unit 1 Electrostatics Chapter 3 Electric Potential Example 1 The neutral point must be in between the two charges

Let the neutral point P be at a distance x cm from point A. So the distance of P from the point B is (100- x) cm.

According to the question,

⇒ \(\frac{49}{x^2}=\frac{81}{(100-x)^2} \quad \text { or, } \frac{7}{x}=\frac{9}{100-x}\)

or, x = 43.75 cm.

So the neutral point is on line AB at a distance of 43.75 cm from the charge +49 esu.

If V is the potential at the neutral point, then

⇒ \(V=\frac{49}{43.75}+\frac{81}{56.25}\)

= 1.12 + 1.44

= 2.56 esu of potential

Example 2. An electron–Is subjected to a potential difference- of 180 V. The Mass and charge of an electron are 9 x 10-31 kg and 1.6 X 10-19 C, respectively. Find the velocity Definition: The amount of kinetic energy acquired by an acquired by electron.
Solution:

Here, mass of the electron, m = 9 x 10-31 kg

Charge of the electron, e = 1.6 x 10-19 C

Potential difference, V = 180 V

∴ Velocity acquired by the electron, v is given by

⇒ \(\frac{1}{2} m v^2=\mathrm{eV}\)

or, \(v=\sqrt{\frac{2 e V}{m}}=\sqrt{\frac{2 \times 1.6 \times 10^{-19} \times 180}{9 \times 10^{-31}}}\)

= 8 x 106 m.s-1

Example 3. At each of the four vertices of a square of side 10 cm, four positive charges each of 20 esu are placed. Find the potential at the point of Intersection of the two diagonals.
Solution:

Amount of charge at each comer, q – 20 esu charge

Length of each side of the square =10 cm

So, length of the diagonal = \(\sqrt{10^2+10^2}=10 \sqrt{2} \mathrm{~cm}\)

The distance of a vertex from the point of intersection of tire diagonals is,

∴ \(x=\frac{1}{2} \times \text { length of the diagonal }=\frac{1}{2} \times 10 \sqrt{2}=5 \sqrt{2} \mathrm{~cm}\)

∴ A Potential at the point of intersection of the two diagonals

⇒ \(\frac{20}{5 \sqrt{2}}+\frac{20}{5 \sqrt{2}}+\frac{20}{5 \sqrt{2}}+\frac{20}{5 \sqrt{2}}\)

⇒ \(\frac{80}{5 \sqrt{2}}=8 \sqrt{2}\)

= 11.31 statV

Example 4. The distance between two points A and B in vacuum is 2d. At each of these two points, a +Q charge is placed. P is the midpoint of AB. Find the intensity and potential at P due to the electric field. How will the value of these quantities change if the charge at B is replaced by a charge -Q?
Solution:

AB = 2d; midpoint of AB is P, i.e., AP = BP = d

Class 12 Physics Unit 1 Electrostatics Chapter 3 Electric Potential example 4 The distance between two points A and B in vacuum

Intensity at P due to the charge + Q at A

⇒ \(\frac{Q}{d^2} \text {; along } \overrightarrow{P B}\)…(1)

Intensity at P due to the charge +Q at B

⇒ \(\frac{Q}{d^2} \text {; along } \overrightarrow{P A}\)….(2)

So, intensities at P due to the charges at A and B are equal and opposite.

Therefore, the resultant intensity at P is zero.

Potential at P due to the charge at A = \(\frac{Q}{d}\)…(3)

Potential at P due to the charge at B = \(\frac{Q}{d}\)….(4)

∴ ‍Total potential at \(P=\frac{Q}{d}+\frac{Q}{d}=\frac{2 Q}{d}\)

Now if-Q charge is placed at point B, intensity in equation (2) will be

⇒ \(\frac{Q}{d^2} \text { along } \overrightarrow{P B}\)

∴ Resultant intensity at \(P=\frac{Q}{d^2}+\frac{Q}{d^2}=\frac{2 Q}{d^2} ; \text { along } \overrightarrow{P B}\)

Again the value of potential in equation (4) will be –\(\frac{Q}{d}\).

∴ Total potential at \(P=\frac{Q}{d}-\frac{Q}{d}=0\)

Example 5. At each of the four vertices of a square of side 10 cm, four positive charges each of 20 esu are placed. Find the potential at the point of Intersection of the two diagonals.
Solution:

Length of each side of the square

⇒ \(\sqrt{10^2+10^2}=10 \sqrt{2} \mathrm{~cm}\)

The point of intersection of the two diagonals is die point O

Class 12 Physics Unit 1 Electrostatics Chapter 3 Electric Potential Example 5 four charges each equal

∴ AO = BO = CO = DO cm = \(\frac{10 \sqrt{2}}{2}=5 \sqrt{2} \mathrm{~cm}\)

Intensity acts along \(\vec{OC}\) due to the charge at A and it acts along \(\vec{OC}\) due to the charge at C.

So resultant intensity at O due to the charges at A and C is given by

⇒ \(E_1=\frac{100}{(5 \sqrt{2})^2}-\frac{20}{(5 \sqrt{2})^2}\)

⇒ \(\left(2-\frac{2}{5}\right)=\frac{8}{5} \text { dyn } \cdot \text { statC }^{-1} ; \text { along } \overrightarrow{O C}\)

Similarly, the intensity at O due to the charges at B and D acts along

⇒ \(\vec{OB}\).

So, the resultant intensity at O due to the charges at B and D is given by,

⇒ \(E_2=\frac{30}{(5 \sqrt{2})^2}+\frac{50}{(5 \sqrt{2})^2}\)

⇒ \(\left(\frac{3}{5}+1\right)=\frac{8}{5} \mathrm{dyn} \cdot \text {statC }^{-1} \text {, along } \overrightarrow{O B}\)

Here E1 and E2 act perpendicular to each other and Ey = E2

Resultant intensity at O,

⇒ \(E=\sqrt{E_1^2+E_2^2}=\sqrt{\left(\frac{8}{5}\right)^2+\left(\frac{8}{5}\right)^2=\frac{8}{5} \sqrt{2}}\)

= 2.263 dyn.statC-1

The direction of E is along the bisector \(\vec{OP}\) of the angle COB i.e., parallel to the side AB or DC.

Again, potential at O = \(\frac{100}{5 \sqrt{2}}-\frac{50}{5 \sqrt{2}}+\frac{20}{5 \sqrt{2}}+\frac{30}{5 \sqrt{2}}\)

⇒ \(\frac{100}{5 \sqrt{2}}=10 \sqrt{2}\)

= 14.14 state

Example 6. Electrons starting from rest and passing through a potential difference of 60 kV are found to acquire a velocity of 1.46 x 1010cm.s-1. Calculate the ratio of charge to the mass of an electron.
Solution:

Here, the velocity of an electron,

v = 1.46 X 1010 cm.s-1

= 1.46 X 108 m.s-1

Potential difference, V = 60 kV = 60000 V

If e is the charge m is the mass of an electron and v is the velocity acquired by it in passing through a potential difference V, we have,

⇒ \(\frac{1}{2}\)mv2 = eV

or, \(\frac{e}{m}=\frac{v^2}{2 V}=\frac{\left(1.46 \times 10^8\right)^2}{2 \times 60000}\)

= 1.776 x 1011 C.kg-1

Example 7. A particle charged with 1.6 X 10-19 C is in motion. It enters the space between two parallel metal plates, parallel along the midway between them. The plates are 10 cm long and the separation between them is 2 cm. A potential difference of 300 V exists between the plates. Find out the maximum velocity of the charged particle at the point of entry, for which it would be unable to emerge from the space between the plates. Given, the mass of the particle = 12 x 10-24 kg.
Solution:

Length of the plates A and B is

l = 10 cm

= 0.1 m

Half of the distance between the plates,

d = \(\frac{1}{2}\) x 2 cm

= 1 cm

= 0.01 m

Class 12 Physics Unit 1 Electrostatics Chapter 3 Electric Potential example 7 two parallel metal plates

Uniform electric field in the intermediate space,

⇒ \(E=\frac{300 \mathrm{~V}}{2 \mathrm{~cm}}\)

= \(\frac{300 \mathrm{~V}}{0.02 \mathrm{~m}}\)

= \(15 \times 10^3 \mathrm{~V} \cdot \mathrm{m}^{-1}\)

Let the charge q = 1.6 X 10-19 be positive.

So, a downward force acts on it due to the electric field E given by

F = qE

∴ Downward acceleration, \(a:=\frac{q E}{m}\)

At the point of entry, let v be the velocity of the charged particle. In tire axial direction, it experiences no force; so the time taken to travel the distance l is t = \(\frac{1}{v}\).

Again, at the point of entry, the downward component of velocity = 0; Thus the downward displacement in time t is

⇒ \(x=\frac{1}{2} a t^2=\frac{1}{2} \frac{q E}{m}\left(\frac{l}{v}\right)^2\)

The condition, that the particle will not emerge from the space between the plates, is

⇒ \(x ≥ d\)

or, \(\frac{1}{2} \frac{q E}{m}\left(\frac{l}{v}\right)^2 ≥ d\)

or, \(\frac{1}{2} \frac{q E l^2}{m d} ≥ v^2 \)

or, \(v≤ l \sqrt{\frac{q E}{2 m d}}\)

Therefore, the maximum permitted velocity is

⇒ \(v_{\max }=l \sqrt{\frac{q E}{2 m d}}\)

= \(0.1 \times \sqrt{\frac{\left(1.6 \times 10^{-19}\right) \times\left(15 \times 10^3\right)}{2 \times\left(12 \times 10^{-24}\right) \times 0.01}}\)

= 104 m.s-1

Example 8. An infinite number of charges, each of value q, are placed on the x-axis at the points x = 1, x = 2, x = 4, x = 8,… Find the potential and intensity due to these charges at x = 0. If the charges are alternately positive and negative what will be the potential and intensity at the same point?
Solution:

If V is the potential at x = 0, then

⇒ \(V=\frac{q}{1}+\frac{q}{2}+\frac{q}{4}+\frac{q}{8}+\cdots \infty\)

⇒ \(q\left(1+\frac{1}{2}+\frac{1}{4}+\frac{1}{8}+\cdots \infty\right)=q\left(\frac{1}{1-\frac{1}{2}}\right)\)

= 2q

If E is the intensity at x = 0, then

⇒ \(E=\frac{q}{1^2}+\frac{q}{2^2}+\frac{q}{4^2}+\frac{q}{8^2}+\cdots \infty\)

⇒ \(q\left(1+\frac{1}{4}+\frac{1}{16}+\frac{1}{64}+\cdots \infty\right)\)

⇒ \(q\left(\frac{1}{1-\frac{1}{4}}\right)=\frac{4 q}{3}\) in the direction of negative x -axis

In the second case, when the consecutive charges are of opposite site signs, let us assume that the first charge is positive. Then potential,

⇒ \(V=\frac{q}{1}-\frac{q}{2}+\frac{q}{4}-\frac{q}{8}+\cdots \infty\)

⇒ \(q\left(1-\frac{1}{2}+\frac{1}{4}-\frac{1}{8}+\cdots \infty\right)=q\left(\frac{1}{1+\frac{1}{2}}\right)=\frac{2 q}{3}\)

Intensity, \(E=\frac{q}{1^2}-\frac{q}{2^2}+\frac{q}{4^2}-\frac{q}{8^2}+\cdots \infty\)

⇒ \(q\left(1-\frac{1}{4}+\frac{1}{16}-\frac{1}{64}+\cdots \infty\right)\)

⇒ \(q\left(\frac{1}{1+\frac{1}{4}}\right)=\frac{4 q}{5}\); in the direction of negative x-axis

Example 9. Two soap bubbles of equal volume are joined together to form a larger bubble. If each bubble had a potential V, find the potential of the resultant bubble.
Solution:

Let the radius of the smaller bubble be r and that of 47ce0r the larger bubble be R . Charge of each bubble- q.

According to the question,

⇒ \(\frac{4}{3} \pi R^3=2 \cdot \frac{4}{3} \pi r^3 \quad \text { or, } R=2^{1 / 3} \cdot r\)

The potential of each soap bubble,

⇒ \(V=\frac{q}{r} \quad \text { or, } q=V r\)

∴ The potential of the larger bubble

⇒ \(=\frac{\text { total charge }}{\text { radius }}=\frac{2 q}{R}=\frac{2 V r}{R}=\frac{2 V r}{2^{1 / 3} r}=.2^{\frac{2}{3}} V\)

Example 10. An electric dipole of moment 5 x 10-8 C In an electric field of magnitude 4 x 105 N.C-1. What amount of work is to be done to deflect it through an angle of 60°?
Solution:

We know, work done, AO = BO = CO = DO = 1 m

W = pE(1- cos0)

= 5 x 10-8 x 4 x 105(1- cos60°)

Here, p = 5 x 10-8C.m,

E = 4 x 105 N.C-1,

B = 60°

W = 2 x 10-2(1 -0.5)

= 10-2 J

Example 11. Find out the maximum charge on an unearthed hollow metal sphere of radius 3.0 m for which it would not discharge into the air. What would be the potential of the sphere in that condition? Assume that, electric discharge into air initiates at a field Intensity of 3 X 106 Vm-1.
Solution:

The electric field on the surface of a metal sphere of radius r has a charge q,

⇒ \(E=\frac{1}{4 \pi \epsilon_0} \frac{q}{r^2}\)

The maximum charge qm on the sphere, without any discharge into air, corresponds to E = 3 x 106 V.m-1.

∴ \(\frac{1}{4 \pi \epsilon_0} \frac{q_m}{r^2}=3 \times 10^6\)

or, \(\dot{q}_m=\frac{\left(3 \times 10^6\right) r^2}{1 /\left(4 \pi \epsilon_0\right)}\)

= \(\frac{3 \times 10^6 \times(3.0)^2}{9 \times 10^9}\)

= \(3 \times 10^{-3} \mathrm{C}\)

Under this condition, the potential of the sphere,

⇒ \(V=\frac{1}{4 \pi \epsilon_0 r} q=9 \times 10^9 \times \frac{3 \times 10^{-3}}{3.0}=9 \times 10^6 \mathrm{~V}\)

Example 12. Find the potential at the center of a square of side V2 m which carries at its four corners charges +2 x 10-9C, +1 x 10-9C, -2 x 10-9C, and +3 X 10-9C.
Solution:

AB = √2m = BC

Class 12 Physics Unit 1 Electrostatics Chapter 3 Electric Potential example 12 potential at the centre of a square

∴ \(A C=\sqrt{A B^2+B C^2}\)

= \(\sqrt{(\sqrt{2})^2+(\sqrt{2})^2}\)

=2m

= BD

∴ AO = BO = CO = DO = 1m

∴ \(V_O=\frac{1}{4 \pi \epsilon_0}\left(\frac{q_1}{A O}+\frac{q_2}{B O}+\frac{q_3}{C O}+\frac{q_4}{D O}\right)\)

⇒ \(9 \times 10^9 \times\left(\frac{2 \times 10^{-9}}{1}+\frac{1 \times 10^{-9}}{1}-\frac{2 \times 10^{-9}}{1}+\frac{3 \times 10^{-9}}{1}\right)\)

= 9 x (2 + 1 – 2 + 3)

= 36V

Example 13. Two charges q1 and q2 are placed 30 cm apart. A third charge q3 is moving along the arc of a circle of radius 40 cm from C to D. The change in the potential energy of the system is \(\frac{q_3}{4 \pi \epsilon_0} k\) What is the value of k?

Class 12 Physics Unit 1 Electrostatics Chapter 3 Electric Potential example 13 The change in the potential

Solution:

The initial potential energy of the system,

⇒ \(U_i=\frac{1}{4 \pi \epsilon_0}\left(\frac{q_1 q_3}{0.4}+\frac{q_2 q_3}{0.5}+\frac{q_1 q_2}{0.3}\right)\) (∵ BC= 50 cm)

Similarly, the final potential energy of the system,

⇒ \(U_f=\frac{1}{4 \pi \epsilon_0}\left(\frac{q_1 q_3}{0.4}+\frac{q_2 q_3}{0.1}+\frac{q_1 q_2}{0.3}\right)\)

Then, change in potential energy,

⇒ \(\Delta U=U_f-U_i=\frac{1}{4 \pi \epsilon_0}\left(\frac{q_2 q_3}{0.1}-\frac{q_2 q_3}{0.5}\right)\)

Accordingly,

⇒ \(\frac{1}{4 \pi \epsilon_0}\left(\frac{q_2 q_3}{0.1}-\frac{q_2 q_3}{0.5}\right)=\frac{q_3}{4 \pi \epsilon_0} k\)

or, \(q_2\left(\frac{1}{0.1}-\frac{1}{0.5}\right)=k \quad\)

or, \(k=q_2(10-2)=8 q_2\)

k = 8q2

Example 14. Two charges, each of +103 esu, are placed at two points A and B separated by a distance of 200 cm. From the middle point of AB, along its perpendicular bisector, a particle having -103 esu of charge is thrown upwards with energy 104 erg. Determine the maximum height attained by the particle. The effect of gravitation can be neglected.
Solution:

Let us assume that the charged particle turns back from D.

Class 12 Physics Unit 1 Electrostatics Chapter 3 Electric Potential example 14 The effect of gravitation

Let the distance between C and D be x cm. For moving the charged particle from C to D, we may write from the conservation of mechanical energy, initial kinetic energy = increase in electrical potential energy.

Potential at C due to charges placed at A and B,

⇒ \(V_C=\frac{1000}{A C}+\frac{1000}{B C}\)

But AC = BC = 100 cm

∴ \(V_C=\frac{1000}{100}+\frac{1000}{100}\)

= 20 esu

When -1000 esu of charge is at C then the potential energy of the system,

⇒ \(U_C=V_C \times(-1000)=-20 \times 10^3 \mathrm{erg}\)

Again, potential at D due to charges placed at A and B,

⇒ \(V_D=2 \times \frac{1000}{\sqrt{x^2+100^2}} \text { esu }\)

Thus, when -1000 esu of. charge is at D, the potential energy of the system,

⇒ \(U_D=V_D \times(-1000)=2 \times \frac{1000}{\sqrt{x^2+100^2}}(-1000) \mathrm{erg}\)

Accordingly,

UD-UC = initial kinetic energy of the particle at C

or, \(\left[20-\frac{2 \times 10^3}{\sqrt{x^2+10^4}}\right] \times 10^3=10^4\)

or, \(\frac{-2 \times 10^3}{\sqrt{x^2+10^4}}=10-20\)

or, \(\sqrt{x^2+10^4}=200\)

x = 173.2 cm

Therefore, the particle will turn back after covering a distance of 173.2 cm.

Example 15. Two point charges of values -20 esu and +20 esu are placed on the x-axis at x = -10 cm and x = +10cm respectively. Calculate

1. The potential and

2. The electric fields at the points P(0, 10) and Q(20, 0).

3. Find the work done in carrying a positive charge of value + 6 esu from P to Q along a straight line joining them,

4. Is there any path along which the work done is less than the above value? Why?

Solution:

1. Two charges + 20 esu and -20 esu are placed at A and B respectively.

Coordinates of P are (0. 10),

Thus, AP = BP = \(\sqrt{10^2+10^2}\)

= \(10 \sqrt{2} \mathrm{~cm}\)

Class 12 Physics Unit 1 Electrostatics Chapter 3 Electric Potential example 15 calculate potentials and electric fields at the points

Potential at \(P, V_P=-\frac{20}{A P}+\frac{20}{B P}=0\)

The coordinates of Q are (20, 0).

So, AQ = 30 cm and BQ = 10 cm

Thus potential at Q,

⇒ \(V_Q=\frac{20}{B Q}-\frac{20}{A Q}\)

= \(\frac{20}{10}-\frac{20}{30}\)

= \(\frac{4}{3}\)

= 1.33 esu

2. Electric field intensity at P due to the charge -20 esu at A is

⇒ \(E_1=\frac{20}{A P^2} \text {; along } \overrightarrow{P A}\)

Again, electric field intensity at P due to the charge + 20 esu at B is

⇒ \(E_2=\frac{20}{B P^2} ; \text { along extended } \overrightarrow{B P}\)

Along the y-axis, a component of E1,

⇒ \(\left(E_1\right)_y=\frac{20}{A P^2} \sin \theta \text {; along } \overrightarrow{P O}\) [Here θ = ∠PAO = ∠PBO]

Along the y-axis, a component of E2,

Potential at the center of the loop A due to charge on it,

⇒ \(\left(E_2\right)_y=\frac{20}{B P^2} \sin \theta ; \text { along } \overrightarrow{P Y}\)

∴ AP = BP

⇒ \(\left(E_1\right)_y=-\left(E_2\right)_y\)

Hence along y -y-axis, the resultant of the component of E1 and E2 becomes zero.

Again along x -the axis, the component of E1 is

⇒ \(\left(E_1\right)_x=\frac{20}{A P^2} \cos \theta ; \text { along } \overrightarrow{P E}\)

⇒ \(\left(E_2\right)_x=\frac{20}{B P^2} \cos \theta ; \text { along } \overrightarrow{P E}\)

⇒ \(\left(E_1\right)_x \text { and }\left(E_2\right)_x\) are equal and acting in the same direction.

∴ Resultant of \(\left(E_1\right)_x \text { and }\left(E_2\right)_x\)

⇒ \(E_x=\frac{20}{A P^2} \cos \theta+\frac{20}{B P^2} \cos \theta=2 \times \frac{20}{A P^2} \cos \theta\)

Thus resultant of the y component of electric field Intensity is zero, so

⇒ \(E_P=E_x\)

= \(\frac{2 \times 20}{A P^2} \times \frac{O A}{A P}\)

= \(\frac{2 \times 20 \times 10}{\left(10^2+10^2\right)^{3 / 2}}\)

= 0.14 dyne/statcoulomb; along \(\vec{PE}\)

And Intensity at Q,

⇒ \(E_Q=\frac{20}{10^2}-\frac{20}{30^2}=\frac{20}{100}-\frac{20}{900}=\frac{8}{45}\)

= 0.178 = 0.18 dyne/statcoulomb; along \(\vec{QX}\)

2. Work done to bring a positive charge of value 6esu from P to Q,

⇒ \(W=6\left(V_Q-V_P\right)=6 \times \frac{4}{3}=8 \mathrm{erg}\)

3. As the electrostatic field is conservative, it does not depend on the path and the work done remains the same.

Example 16. Two circular loops of radii 0.05 m and 0.09 m are placed such that their axes coincide and their centres are 0.12m apart. A charge of 10-6C is spread uniformly on each loop. Find the potential difference between the centres of loops.
Solution:

Distance between the two centres, x = 0.12 m

Class 12 Physics Unit 1 Electrostatics Chapter 3 Electric Potential example 16 Two circular loops of radii

Potential at the center of the loop A due to charge on it,

⇒ \(V_A=\frac{1}{4 \pi \epsilon_0} \frac{q_1}{r_1}\)

= \(\frac{9 \times 10^9 \times 10^{-6}}{0.05}\)

⇒ \(=1.8 \times 10^5 \mathrm{~V}\)

Potential at the center of the loop A due to charge on loop B,

⇒ \(V_A^{\prime}=\frac{1}{4 \pi \epsilon_0} \frac{q_1}{\sqrt{r_2^2+x^2}}\)

= \(\frac{9 \times 10^9 \times 10^{-6}}{\sqrt{(0.09)^2+(0.12)^2}}\)

⇒ \(\frac{9 \times 10^3}{15 \times 10^{-2}}=0.6 \times 10^5 \mathrm{~V}\)

Therefore total potential at the centre of loop A,

⇒ \(\left(V_A\right)_T=V_A+V_A^{\prime}=(1.8+0.6) \times 10^5 \mathrm{~V}\)

=2.4 X 105 V

Now potential at the center of the loop H due to charge on it,

⇒ \(V_B=\frac{1}{4 \pi \epsilon_0} \frac{q_2}{r_2}=\frac{9 \times 10^9 \times 10^{-6}}{0.09}=1 \times 10^5 \mathrm{~V}\)

Potential at the center of the loop B due to charge on loop A,

⇒ \(V_B^{\prime}=\frac{1}{4 \pi \epsilon_0} \frac{q_2}{\sqrt{r_2^2+x^2}}\)

= \(\frac{9 \times 10^9 \times 10^{-6}}{\sqrt{(0.05)^2+(0.12)^2}}\)

⇒ \(\frac{9 \times 10^3}{13 \times 10^{-2}}=0.69 \times 10^5 \mathrm{~V}\)

Therefore total potential at the centre of loop B,

⇒ \(\left(V_B\right)_T=\left(V_B+V_B^{\prime}\right)=\left(1 \times 10^5+0.69 \times 10^5\right)\)

= 1.69 x 105 V

Thus potential differences,

⇒ \(\left(V_A\right)_T-\left(V_B\right)_T=(2.4-1.69) \times 10^5 \mathrm{~V}\)

= (2.4 – 1.69) X 105V

= 7.1 x 104 V ≈ 72kV

Example 17. Three-point charges 1C, 2C, and 3C are placed at the corners of an equilateral triangle of side 1 m. Calculate the work required to move these charges to the comers of a smaller equilateral triangle of side 0.5 m as shown.

Class 12 Physics Unit 1 Electrostatics Chapter 3 Electric Potential example 17 equilateral triangle

Solution:

The initial potential energy of the system i.e., for ΔABC = Ui = algebraic sum of potential energy of 3 pairs of charges.

So,\(U_i=\frac{1}{4 \pi \epsilon_0}\left(\frac{1 \times 2}{1}\right)+\frac{1}{4 \pi \epsilon_0}\left(\frac{1 \times 3}{1}\right)+\frac{1}{4 \pi \epsilon_0}\left(\frac{2 \times 3}{1}\right)\)

⇒ \(\frac{1}{4 \pi \epsilon_0}[2+3+6]=9 \times 10^9 \times 11\)

= 99 x 109J

Class 12 Physics Unit 1 Electrostatics Chapter 3 Electric Potential example 17 Initial potential energy

Again, the final potential energy of the system, Le., for ΔA’B’C’ = Uf = algebraic sum of the potential energy of 3 pairs of charges.

So, \(U_f=\frac{1}{4 \pi \epsilon_0}\left(\frac{1 \times 2}{0.5}\right)+\frac{1}{4 \pi \epsilon_0}\left(\frac{1 \times 3}{0.5}\right)+\frac{1}{4 \pi \epsilon_0}\left(\frac{2 \times 3}{0.5}\right)\)

⇒ \(\frac{1}{4 \pi \epsilon_0}[4+6+12]=9 \times 10^9 \times 22=198 \times 10^9 \mathrm{~J}\)

Hence, work required to move the charges,

W =Uf-Ui

= (198- 99) X 109 J

=9.9 x 1010 J

Example 18. A non-conducting disc of radius a with uniform positive surface charge density cr is placed on the ground with its axis vertical. A particle of mass m and positive charge q is dropped along the axis of the disc from a height H with zero initial velocity. Charge per unit of the particle is \(\frac{q}{m}=\frac{4 e_0 g}{\sigma}\).

  1. Find the value of H If the particle just reaches the disc,
  2. Find the height for its equilibrium position.

Solution:

Let us assume, the amount of charge in the ring of thickness dr is dq.

Potential at P due to this ring,

⇒ \(d V=\frac{1}{4 \pi \epsilon_0} \frac{d q}{x}, \text { where } x=\sqrt{H^2+r^2}\)

∴ \(d V=\frac{1}{4 \pi \epsilon_0} \frac{(2 \pi r d r) \sigma}{\sqrt{H^2+r^2}}\)  ∵ dq = 2 πrdrσ

⇒ \(\frac{\sigma}{2 \epsilon_0} \frac{r d r}{\sqrt{H^2+r^2}}\)

Class 12 Physics Unit 1 Electrostatics Chapter 3 Electric Potential example 18 A non-conducting disc of radius

Now, the potential at P due to the complete disc,

⇒ \(V_P=\int_{r=0}^{r=a} d V=\frac{\sigma}{2 \epsilon_0} \int_{r=0}^{r=a} \frac{r d r}{\sqrt{H^2+r^2}}\)

⇒ \(V_P=\frac{\sigma}{2 \epsilon_0}\left[\sqrt{a^2+H^2}-H\right]\)

So potential at the centre (O) , \(V_o=\frac{\sigma a}{2 \epsilon_0}\)[∵ H = 0 ]

1. Particle is released from P and it just reaches O. Since initial kinetic energy = final kinetic energy = 0 Hence increase in kinetic energy = 0

From the conservation of mechanical energy, we may write, a decrease in gravitational potential energy = an increase in electrostatic potential energy.

mgH = q[VO – Vp]

or, \(m g H=(q)\left(\frac{\sigma}{2 \epsilon_0}\right)\left[a-\sqrt{a^2+H^2}+H\right]\)

or, \(g H=\left(\frac{q}{m}\right)\left(\frac{\sigma}{2 \epsilon_0}\right)\left[a-\sqrt{a^2+H^2}+H\right]\)…(1)

Given, \(\frac{q}{m}=\frac{4 \epsilon_0 g}{\sigma} \quad \text { or, } \frac{q \sigma}{2 \epsilon_0 m}=2 g\)

Substituting this value in equation (1), we get

⇒ \(g H=2 g\left[a+H-\sqrt{a^2+H^2}\right]\)

or, \(\frac{H}{2}=(a+H)-\sqrt{a^2+H^2}\)

or, \(a^2+H^2=a^2+\frac{H^2}{4}+a H \quad\)

or, \(\frac{3}{4} H^2=a H\)

H = \(\frac{4}{3}\)a [.. H not = 0]

2. Potential energy of the particle at height H = electrostatic potential energy + gravitational potential energy

or, U = qVp+ mgH [ Vp = electric potential at high H]

or, \(U=\frac{\sigma q}{2 \epsilon_0}\left[\sqrt{a^2+H^2}-H\right]+m g H\)…(2)

At equilibrium, \(F=-\frac{d U}{d H}=0\)

Differentiating equation, (2) with respect to H,

⇒ \(m g+\sigma \frac{q}{2 \epsilon_0}\left[\left(\frac{1}{2}\right)(2 H) \frac{1}{\sqrt{a^2+H^2}}-1\right]=0\)

or, \(m g+2 m g\left[\frac{H}{\sqrt{a^2+H^2}}-1\right]=0\)

or, \(1+\frac{2 H}{\sqrt{a^2+H^2}}-2=0 \quad\)

or, \(4 H^2-H^2=a^2\)

∴ \(H=\frac{a}{\sqrt{3}}\)

Example 19. Positive charges of magnitude 6nC, 12nC, and 24 nC are placed at the vertices A, B, and C of a square ABCD of side 20 cm. What is the amount of work done to place a charge of 1C at vertex D?
Solution:

The length of diagonal \(\sqrt{20^2+20^2}=20 \sqrt{2} \mathrm{~cm}\)

The electric potential at D,

⇒ \(V_D=\frac{1}{4 \pi \epsilon_0}\left[\frac{q_A}{A D}+\frac{q_C}{C D}+\frac{q_B}{B D}\right]\)

⇒ \(\frac{1}{4 \pi \epsilon_0}\left[\frac{6 \times 10^{-9}}{20 \times 10^{-2}}+\frac{24 \times 10^{-9}}{20 \times 10^{-2}}+\frac{12 \times 10^{-9}}{20 \sqrt{2} \times 10^{-2}}\right]\)

⇒ \(9 \times 10^9 \times \frac{10^{-9}}{10^{-2}}\left[\frac{6}{20}+\frac{24}{20}+\frac{12 \sqrt{2}}{40}\right]\)

⇒ \(900\left[\frac{12+48+12 \times 1.414}{40}\right]\)

= \(\frac{45}{2} \times 76.968\)

= 1731.78 V

Class 12 Physics Unit 1 Electrostatics Chapter 3 Electric Potential example 19 Positive charges of magnitude

Hence the work done to place a charge of 1C at D = 1731.78J

Example 20. A proton is fired with a velocity of 7.45 X 105 m/s towards another free proton at rest. Calculate the minimum distance of approach between the protons. The mass of a proton = 1.66 X 10-22 kg
Solution:

Let us consider the moving proton to be the first proton and the free proton at rest to be the second proton.

The initial velocity of the first proton, v = 7.45 x 105 m/s. As the protons repel each other, the second proton starts moving away from the first proton.

The velocity of the first proton gradually decreases and that of the second proton gradually increases.

When the velocities of the two protons become equal, the distance between them will be minimal.

Let this velocity = v’

According to the law of conservation of momentum

mv + 0 = mv’ + mv’

or, v’ = \(\frac{v}{z}\)

Let the minimum distance of approach between the protons be r.

The electric potential energy of the protons when they are at a distance r apart = \(\frac{1}{4 \pi \epsilon_0} \cdot \frac{e}{r}\)

According to the law of conservation of energy,

⇒ \(\frac{1}{2} m v^2+0=\frac{1}{4 \pi \epsilon_0} \cdot \frac{e^2}{r}+\frac{1}{2} m v^{\prime 2}+\frac{1}{2} m v^{\prime 2}\)

⇒ \(\frac{1}{4 \pi \epsilon_0} \cdot \frac{e^2}{r}+m\left(\frac{\nu}{2}\right)^2\)

or, \(\frac{1}{4 \pi \epsilon_0} \cdot \frac{e^2}{r}=\frac{m v^2}{4}\)

or, \(r=\frac{4}{m v^2} \times \frac{e^2}{4 \pi \epsilon_0}=\frac{4 \times\left(1.6 \times 10^{-19}\right)^2 \times 9 \times 10^9}{1.66 \times 10^{-27} \times\left(7.45 \times 10^5\right)^2}\)

=1.0 x 10-12 m

Electrostatics

Electric Potential Equipotential Surface

Definition: A surface containing points at the same potential in an electric field is called an equipotential surface.

Equipotential surfaces due to an isolated point charge: Few equipotential surfaces due to an isolated positive point charge have been shown. We know that, for

Class 12 Physics Unit 1 Electrostatics Chapter 3 Electric Potential Equipotential surfaces due to an isolated point charge

An isolated point charge q at a point O in a vacuum, potential at a distance r is \(\frac{1}{4 \pi \epsilon_0} \frac{q}{r}\) [in SI].

So if a hollow sphere of radius r is drawn taking O as the centre, the outer surface of the sphere will be an equipotential surface, because potential at all points on the surface is equal to \(\frac{1}{4 \pi \epsilon_0} \frac{q}{r}\).

The outer surfaces of all centric spheres so drawn will be equipotential surfaces. With O as a centre, the larger the radius of the sphere drawn, the smaller will be the surface potential.

Equipotential surfaces due to a uniform electric field:

A uniform electric field is represented by parallel and equispaced field lines. Equipotential surfaces in such a field would be normal to these field lines at every point in the field. Hence, each of these equipotential surfaces A, B, and C would be plane and parallel to each other. The surfaces correspond to different potentials V1, V2, V3 respectively; but the potential at every point on any particular surface is the same.

Class 12 Physics Unit 1 Electrostatics Chapter 3 Electric Potential Equipotential surfaces due to a uniform electric field

In a strong electric field, the potential changes quickly along the direction of the field lines; in a weaker field, this change is slower. As a result, the spacing between successive equipotential surfaces, for a given fixed potential difference, is less for a stronger field than that for a weaker field.

Class 12 Physics Unit 1 Electrostatics Chapter 3 Electric Potential Equipotential surfaces due to a uniform electric field.

Equipotential surfaces for a pair of point charges: In this case, the shape of the equipotential surfaces depends on the algebraic sum, at any point in the electric field, of the potentials generated by the two-point charges separately. The equipotential surfaces O for two like charges of the same magnitude and θ for two unlike charges of the same magnitude, respectively. In the blue and red lines represent the equipotential surface and field lines, respectively.

Class 12 Physics Unit 1 Electrostatics Chapter 3 Electric Potential Equipotential surfaces for a pair of point charges

Properties of equipotential surfaces:

1. No work has to be done to move a charge from one point to another on an equipotential surface. Work done to move a unit positive charge from one point to another is equal to the potential difference between the two points.

Since the potential at all points on an equipotential surface is equal, to move a charge from one point to another on an equipotential surface, no work has tube done.

2. Field lines intersect the equipotential surface perpendicularly. Let us consider two points A and B very close to each other on an equipotential surface S.

Class 12 Physics Unit 1 Electrostatics Chapter 3 Electric Potential Properties of equipotential surfaces

Let the electric field intensity E, in the region AB, make an angle 6 with the equipotential surface. Since A and B are very close to each other, AB may be taken as a straight line.

Component of electric intensity E along AB = Ecosθ. So work was done to move a unit positive charge from A to

B = Ecosθ x AB

From the property of an equipotential surface, we know that no work is done in moving a charge from one point to another on an equipotential surface.

∴ Ecosθ x AB = 0 or, cosθ = 0 [∵ AB ≠ 0; 13 ≠ 0]

or, \(\theta=\frac{\pi}{2}\)

So the direction of field Intensity Is perpendicular to the equipotential surface. We know that the direction of electric field intensity and that of a field line through any point is the same. So it may be said that the field lines intersect equipotential surfaces normally.

By calculus: Let us consider two points A and If very close to each other on surface S. VA and VB are the potentials at the points A and B respectively. So, the potential difference between A and B Is

Class 12 Physics Unit 1 Electrostatics Chapter 3 Electric Potential By calculus

⇒ \(\Delta V=V_B-V_A=-\int_A^B \vec{E} \cdot d \vec{r}\)

By definition, on an equipotential surface, AV = 0 i.e., \(\vec{E} \cdot d \vec{r}=0\)

So, \(\vec{E}\) is normal to d\(\vec{r}\). The latter being tangential to the curve, E is normal to the surface represented by the curve.

3. If there is a difference of potential between two different points on the surface of a conductor, charges will begin to flow from one point to the other until the potential becomes the same at the two points. So top surface of a charged conductor is an equipotential surface and charges on such a surface, remain at rest.

4. No two equipotential surfaces intersect each other. Any intersection would mean that the point of intersection corresponds to two different potentials; also, there are two electric field intensities at that point in two normal directions. These are impractical.

The surface of a charged conductor is an equipotential surface—experimental demonstration:

We have already come to know that charge of a conductor resides on its surface. The surface charge density of a charged conductor depends on the shape of the conductor and is maximum at the region where its curvature is greatest.

But the potential at every point on the surface of a charged conductor is the same and is quite independent of the shape of the conductor.

To verify it experimentally, an insulated pear-shaped conductor A is charged positively, The disc of an uncharged gold leaf electroscope is connected by a wire to a proof-plane.

The electroscope is placed at such a large distance from the conductor that no electrical induction takes place in it. The proof-plane is held by its insulating handle and brought in contact with the pear-shaped conductor.

The proof-plane is moved to different parts of the surface and it is found that the divergence of leaves remain constant. This proves that the potential, on a charged conductor is uniform, i.e., the surface of a charged conductor is an equipotential surface.

Class 12 Physics Unit 1 Electrostatics Chapter 3 Electric Potential The surface of a charged conductor is an equipotential surface experimental demonstration

Electrostatics

Electric Potential Potential Inside A Hollow Conductor

Though no charge resides inside a hollow charged conductor, the potential at any point on the inner surface is equal to that at the outer surface of the conductor. This can be proved by the following experiment.

An insulated deep hollow metallic vessel C is charged positively and it is placed at a sufficient distance from a gold leaf electroscope. A proof-plane P is connected by a wire to the disc of the electroscope and is dipped gradually inside the vessel.

The leaves of the gold-leaf electroscope would diverge; this divergence would be maximum when the depth of the proof-plane becomes sufficiently larger than the dimensions of the top opening.

This divergence would not increase any more if the proof-plane is dipped farther, or is moved sideways. As the divergence of the leaves indicates the magnitude of potential, it can be said that the potential inside the vessel is constant.

Class 12 Physics Unit 1 Electrostatics Chapter 3 Electric Potential Potential inside A hollow conductor

Now if the proof plane is made to touch the outer surface of the vessel, the magnitude divergence of the leaves does not change. This proves that the potential inside a charged hollow conductor is uniform, having the same value as that on its surface. The potential inside a charged hollow conductor, V = constant.

Hence, the electric field intensity,

⇒ \(E=-\frac{d V}{d x}=0\)

This means that the interior of a hollow conductor is associated with no electric field and no field line.

E = 0 in a region corresponds to V=constant; it does not necessarily mean that V = 0 in that region.

Electrostatics

Electric Potential Potential Inside A Hollow Conductor Numerical Examples

Example 1. Charge Q is distributed between two concentric hollow spheres placed in a vacuum in such a way that their surface densities of charge are equal. If the radii of the two spheres are r and R (R> r), calculate the potential at their centre.
Solution:

Let the charge on the smaller and the bigger spheres be Q1 and Q2, respectively

∴ Q = Q1 + Q2

As the surface densities of charge of the two spheres are equal,

∴ \(\frac{Q_1}{4 \pi r^2}=\frac{Q_2}{4 \pi \cdot R^2} \text { or, } \frac{Q_1}{Q_2}=\frac{r^2}{R^2}\)

or, \(\frac{Q_1+Q_2}{Q_2}=\frac{r^2+R^2}{R^2}\)

or, \(\frac{Q_2}{Q}=\frac{R^2}{r^2+R^2}\)

or, \(Q_2=\frac{Q \cdot R^2}{r^2+R^2}\)

Similarly, \(Q_1=\frac{Q \cdot r^2}{r^2+R^2}\)

Potential at the centre,

⇒ \(V=\frac{Q_1}{r}+\frac{Q_2}{R}\)

= \(\frac{Q \cdot r^2}{r\left(r^2+R^2\right)}+\frac{Q \cdot R^2}{R\left(r^2+R^2\right)}\)

= \(\frac{Q(r+R)}{r^2+R^2}\)

Example 2. Charges +2 x 10-7C, -4 x 10-7C and +8 x 10-7C are placed at the vertices of an equilateral triangle of side 10 cm in air. Determine the electrical potential energy of this system of charges.
Solution:

Distance between any two of the charges,

r = 10 cm = 0.1m.

The electrical potential energy’ of the system of charges is the algebraic sum of the potential energies of each pair of charges.

For air, permittivity is ∈0.

The potential energy of the system of charges

⇒ \(\frac{1}{4 \pi \epsilon_0}\left(\frac{q_1 q_2}{r}+\frac{q_2 q_3}{r}+\frac{q_3 q_1}{r}\right)\)

⇒ \(\frac{1}{4 \pi \epsilon_0 r}\left(q_1 q_2+q_2 q_3+q_3 q_1\right)\)

⇒ \(=\frac{1}{4 \pi \epsilon_0 \times 0.1}[2 \times(-4)+(-4) \times 8+8 \times 2] \times 10^{-7}\)

⇒ \(\frac{9 \times 10^9}{0.1} \times\left[-8 \times 10^{-14}-32 \times 10^{-14}+16 \times 10^{-14}\right]\)

⇒ \(-\frac{9 \times 24 \times 10^9 \times 10^{-14}}{0.1}\)

= -0.0216J

The negative sign indicates that 0.0216J work is to be done to transfer the charges from their positions to infinity.

Example 3. In a vacuum, three small spheres are placed on the circumference of a circle of radius r in such a way that an equilateral triangle is formed. If q is the charge on each sphere, determine the intensity of the electric field and potential at the centre of the circle.
Answer:

Let O be the centre of the circle. Three spheres are placed at the points A, B and C on the circumference of the circle. ABC is an equilateral triangle.

Intensity at O due to the charge at A,

⇒ \(E_A=\frac{q}{4 \pi \epsilon_0 r^2} \text {; along } \overrightarrow{A O}\)

Intensity at O due to the charge at B,

⇒ \(E_B=\frac{q}{4 \pi \epsilon_0 r^2} ; \text { along } \overrightarrow{B O}\)

Intensity at O due to the charge at C,

⇒ \(E_C=\frac{q}{4 \pi \epsilon_0 r^2} \text {; along } \overrightarrow{C O}\)

Resolving the above intensities along AO and perpendicular to AO we have,

intensity along AO = EA – EBcos60°-ECcos60º

⇒ \(\frac{1}{4 \pi \epsilon_0}\left[\frac{q}{r^2}-\frac{q}{r^2} \times \frac{1}{2}-\frac{q}{r^2} \times \frac{1}{2}\right]\)

= 0

intensity perpendicular to AO = EAcos90° + EBsin60°- ECsin60°

⇒ \(\frac{1}{4 \pi \epsilon_0}\left[0+\frac{q}{r^2} \sin 60^{\circ}-\frac{q}{r^2} \sin 60^{\circ}\right]\)

= 0

∴ Resultant intensity at O = 0 and electric potential at \(\frac{1}{4 \pi \epsilon_0}\left[\frac{q}{r}+\frac{q}{r}+\frac{q}{r}\right]=\frac{3 q}{4 \pi \epsilon_0 r}\) units

Class 12 Physics Unit 1 Electrostatics Chapter 3 Electric Potential example 3 three small spheres

Example 4. A charged particle q Is thrown with a velocity v towards another charged particle Q at rest. It approaches Q up to the closest distance r and then returns. If q is thrown with a velocity of 2v, what should be the closest distance of approach?
Solution:

Let the closest distance of approach be r’.

From the principle of conservation of energy we have, kinetic energy = electrostatic potential energy.

In the first case,

⇒ \(\frac{1}{2} m v^2=\frac{1}{4 \pi \epsilon_0} \cdot \frac{q Q}{r}\)…(1)

In the second case,

⇒ \(\frac{1}{2} m(2 v)^2=\frac{1}{4 \pi \epsilon_0} \cdot \frac{q Q}{r^{\prime}}\)…(2)

Dividing equation (1) by equation (2) we get,

⇒ \(\frac{1}{4}=\frac{r^{\prime}}{r} \quad \text { or, } r^{\prime}=\frac{r}{4}\)

Example 5. In a vacuum, four charges each equal to q are placed at each of the four vertices of a square. Find the intensity and potential of the electric field at the point of intersection of the two diagonals.
Solution:

Let the length of each side of the square be a.

⇒ \(A C=\sqrt{A B^2+B C^2}=\sqrt{a^2+a^2}\)

= V2a

Now, AC = BD

and AO = BO = CO = DO = \(\frac{1}{2}\) AC

Class 12 Physics Unit 1 Electrostatics Chapter 3 Electric Potential Example 5 four charges each equal

Therefore, intensity at O due to the charge q at A is given by

⇒ \(\frac{1}{2} \cdot \sqrt{2} a\)

= \(\frac{a}{\sqrt{2}}\)

Intensity at O due to the charge at C is given by

⇒ \(E_1=\frac{1}{4 \pi \epsilon_0} \cdot \frac{q}{\left(\frac{a}{\sqrt{2}}\right)^2} ; \text { along } \overrightarrow{O C}\)

These two intensities being equal and opposite balance each other.

Similarly, intensities at 0 due to the charges at B and D being equal and opposite balance each other. So the intensity at the point of intersection of the two diagonals is zero.

Potential at \(O=\frac{1}{4 \pi \epsilon_0} \cdot\left[\frac{q}{\frac{a}{\sqrt{2}}}+\frac{q}{\frac{a}{\sqrt{2}}}+\frac{q}{\frac{a}{\sqrt{2}}}+\frac{q}{\frac{a}{\sqrt{2}}}\right]\)

= \(\frac{4 \sqrt{2} q}{4 \pi \epsilon_0 a}\)

Example 6. Three-point charges q, 2q and 8q are to be placed–on a 0.09 m long straight line. Find the positions of the charges so that the potential energy of this system becomes minimal. In this situation, find the Intensity at the position of the charge q due to the other two charges.
Solution:

Let the charge 2q be placed in between the charges q and 8q and the distance between q and 2q be x metre.

So, the distance between 2q and 8q = (0.09 JC) m

Class 12 Physics Unit 1 Electrostatics Chapter 3 Electric Potential Example 6 potential energy of this system

The potential energy of the system,

⇒ \(U=\frac{1}{4 \pi \epsilon_0}\left[\frac{q \cdot 2 q}{x}+\frac{2 q \cdot 8 q}{(0.09-x)}+\frac{q \cdot 8 q}{0.09}\right]\)

⇒ \(9 \times 10^9 \times 2 q^2\left[\frac{1}{x}+\frac{8}{(0.09-x)}+\frac{4}{0.09}\right]\)

For minimum value of U, \(\frac{dU}{dx}\) = 0

i,e., \(9 \times 10^9 \times 2 q^2\left[-\frac{1}{x^2}+\frac{8}{(0.09-x)^2}\right]\)

= 0

or, \(\frac{1}{x}=\frac{2 \sqrt{2}}{0.09-x}=\frac{2.83}{0.09-x}\)

or, x = 0.0235

So, the charge 2q is to be placed at a distance of 0.0235 m, i.e., 2.35 cm from the charge q.

Electric field intensity at the position of the charge q due to the
other charges,

⇒ \(E=9 \times 10^9 \times \frac{2 q}{x^2}+9 \times 10^9 \times \frac{8 q}{(0.09)^2}\)

⇒ \(9 \times 10^9 \times 2 q\left[\frac{1}{(0.0235)^2}+\frac{4}{(0.09)^2}\right]\)

4.148 X 1013q N.C-1

Electrostatics – Electric Potential Conclusion

  • The potential at any point In an electric field is defined as the amount of work done in bringing a unit positive charge from infinity to that point.
  • The potential of a positively charged body is called positive potential and that of a negatively charged body is called negative potential.
  • In a static electric field,
  • The total work done in carrying a charge taken around a closed path is zero
  • The potential difference between two points does not depend on the path followed by any charge between the two points.

Unit of electric potential:

Class 12 Physics Unit 1 Electrostatics Chapter 3 Electric Potential Unit of electric potential

  • 1 esu of potential = 300 V.m-1 The potential of the earth Is taken us zero. The potential of other bodies Is expressed with reference to this zero potential of the earth.
  • The potential of any charged body connected to the earth becomes zero.
  • The electrostatic potential energy of a system of charges is defined as the work done to bring the charges from infinity to their positions to constitute the system.
  • An electronvolt (eV) is a unit of work, defined as the work done when an electron crosses a potential difference of 1 V. leV = 1.6 x 10-19J
  • An equipotential surface is a surface containing the points having the same potential in an electric field

Potential at a point at distance x from a point charge q placed in air or in a vacuum:

Class 12 Physics Unit 1 Electrostatics Chapter 3 Electric Potential Potential at a point at distance x from a point charge q placed in air

Potential due to a system of charges:

Class 12 Physics Unit 1 Electrostatics Chapter 3 Electric Potential Potential due to a system of charge

In a uniform electric field, VA and VB are the electric potentials at the points A and B respectively. The separation between these points is d. If VA > VB, the relation between intensity and the potential difference between these two points is,

⇒ \(E=-\frac{V_A-V_B}{d}\)

For a non-uniform electric field, the intensity (E) and potential (V) are related as E = –\(\frac{dV}{dx}\)

Potential at any internal point of a uniformly charged sphere of radius r is the same and equal to the potential on the surface of the sphere,

i.e, \(V=\frac{1}{4 \pi \epsilon_0} \cdot \frac{q}{r}(\text { in } \mathrm{SI})\)

V = \(\frac{q}{r}\)

The electrical potential energy of a system of charges ql and q2 separated by a distance r is given by,

⇒ \(U=\frac{q_1 q_2}{4 \pi \epsilon_0 r} \quad \text { (in SI) }\)

⇒ \(U=\frac{q_1 q_2}{r} \text { (in CGS system) }\)

Work done in deflecting a dipole through an angle 6 from its equilibrium position in a uniform electric field.

In CGS system In SI (in SI)

W = pE(1- cosθ)

The potential energy of the dipole in this position,

V = pE(1- cosθ) [p = dipole moment of the electric dipole]

When a particle of mass m and charge q moves from rest in an electric field between two points due to a potential difference y existing between them, the velocity acquired by the particle is,

⇒ \(v=\sqrt{\frac{2 q V}{m}}\)

Electrostatics

Electric Potential Very ShortAnswer Type Questions

Question 1. What is the relation between statvolt and volt?
Answer: 1V = \(\frac{1}{300}\) statV

Question 2. A free electron moves from a higher to a lower potential. Is the statement correct?
Answer: No

Question 3. Write down the name of the physical quantity whose unit is J C-1
Answer: Potential

Question 4. What is the electric field intensity inside a charged conductor
Answer: 0

Question 5. what are the shapes of the equipotential surfaces in the field of a point charge?
Answer: Spherical surfaces with different radii

Question 6. What will be the shape of the equipotential surface siturated at infinity due to a point charge?
Answer: Plane

Question 7. Which physical quantity has the unit eV?
Answer: Electric energy

Question 8. Is the electric field conservative or non-conservative?
Answer: Conservative

Question 9. If a charge q moves through a potential difference V, what will be the kinetic energy of the charge?
Answer: Vp

Question 10. A positive charge +Q is placed at a point. A circle of radius r is drawn with the point as the centre. Another charge q is carried once in that circular path. What will be the work done?
Answer: Zero

Question 11. To transfer a charge of 20 C through a distance of 2 cm, 2 J work is performed. What is the potential difference between the endpoints of that distance?
Answer: 0.1V

Question 12. If the potential is constant around a point, what will be the electric field intensity at that point?
Answer: Zero

Electrostatics

Electric Potential Fill In The Blanks

1. l esu of potential = 300 V.

2. The electric field lines pass through an equipotential surface Perpendicularly

3. The surface of a charged conductor is an equipotential surface

4. The electrical potential energy of a unit charge placed at a point in an electrical field is the electric potential at that point.

5. In an electric field, if a charged body loses its potential energy, it gains an equal amount of kinetic energy

 Electrostatics

Electric Potential Assertion-reason type

Direction: These questions have statement 1 and statement 2. Of the four choices given below, choose the one that best describes the two statements.

  1. Statement 1 is true, statement 2 is true, and statement 2 is a correct explanation for statement 1.
  2. Statement 1 is true, and statement 2 is true; statement 2 is not a correct explanation for statement 1.
  3. Statement 1 is true, and statement 2 is false.
  4. Statement 1 is false, and statement 1 is true.

Question 1.

Statement 1: For practical purposes, the potential of the earth is used as a reference, and is assumed to be the zero of potential in electrical circuits.

Statement 2: The electrical potential of a sphere of radius R, with charge Q uniformly distributed on its surface, is given by,

⇒ \(\frac{Q}{4 \pi \epsilon_0 R}\)

Answer: 1. Statement 1 is hue, statement 2 is true; statement n is a correct explanation for statement 1.

Question 2.

Statement 1: A non-zero electric potential may exist at a point where electric field strength is zero.

Statement 2: Both electric potential and electric field strength depend on the source charge and not on the test charge

Answer: 2. Statement 1 is true, statement 2 is true; statement 2 is not a correct explanation for statement 1.

Question 3.

Statement 1: If two source charges produce potentials Vj and V2 at a point, then the total potential at that point is

V1 + V2

Statement 2: Electric potential is a scalar quantity

Answer: 1. Statement 1 is hue, statement 2 is true; statement 2 is a correct explanation for statement 1.

Question 4.

Statement 1: A and B are two conducting spheres of the same radius, A being solid and B; being hollow. Both are charged to the same potential. Then, charge on A = charge on B.

Statement 2: Potentials on both are the same.

Answer: 1. Statement 1 is hue, statement 2 is true; statement 2 is a correct explanation for statement 1.

Question 5.

Statement 1: For a charged particle moving from point P to point Q , the net work done by an electrostatic field on the particle is independent of the path connecting points P and Q.

Statement 2: The net work done by a conservative force on an object moving along a closed loop is zero.

Answer: 1. Statement 1 is hue, statement 2 is true; statement 2 is a correct explanation for statement 1.

Question 6.

Statement 1: Due to an infinitely long linear charge distribution, the potential at any point at a distance r from the line is proportional to the log.

Statement 2: \(E \propto \frac{1}{r} \text { and } E=-\frac{d V}{d r}\)

Answer: 1. Statement 1 is hue, statement 2 is true; statement 2 is a correct explanation for statement 1.

Question 7.

Statement 1: Though electric potential is scalar, electric potential gradient is a vector quantity.

Statement 2: Potential gradient is the rate of change of potential with distance.

Answer: 2. Statement 1 is true, statement 2 is true; statement 2 is not a correct explanation for statement 1.

Question 8.

Statement 1: A system of three positive charges, each having a charge q and placed at equal distances from each other along a straight line cannot be in equilibrium.

Statement 2: The charge in the middle experiences zero net force, but the force acting on the charges at the extreme ends is not zero.

Answer: 1. Statement 1 is hue, statement 2 is true; statement 2 is a correct explanation for statement 1.

 Electrostatics

Electric Potential Match The Columns

Question 1. The shape of equipotential surfaces and their charge distribution are given in column I and column n, respectively.

Class 12 Physics Unit 1 Electrostatics Chapter 3 Electric Potential Match the column 1

Answer: 1-D,E, 2-C, 3-A,B

Question 2. Match the nature of point charges in column I with the variation of their potentials in column n.

Class 12 Physics Unit 1 Electrostatics Chapter 3 Electric Potential Match the column 2

Answer: 1-B,D 2-A,C

Question 3. Potentialÿ at different points duo to the charged conductors of different shapes are given in the columns. Suppose, each of the given charged conductors has a radius r and x is the distance of a certain point from their centers.

Class 12 Physics Unit 1 Electrostatics Chapter 3 Electric Potential Match the column 3

Answer: 1-B, 2-C, 3-A, 4-A

Question 4. Two spherical shells. Suppose r is the distance of a point from their common centre, then

Class 12 Physics Unit 1 Electrostatics Chapter 3 Electric Potential Match the column 4.

Class 12 Physics Unit 1 Electrostatics Chapter 3 Electric Potential Match the column 4

Answer: 1-D, 2-C, 3-A, 4-B

WBCHSE Class 12 Physics Notes For Chapter 2 Electric Field

Electrostatics

Electric Field Force Of Attraction Or Repulsion Between Two Charges Couloms’s Law

We know that two like charges repel each other and unlike charges attract each other. Coulomb’s law gives the force between two charges.

This law is valid:

  1. Only for point charges, i.e., when the sizes of the charged bodies are negligible in comparison to their distance of separation,
  2. Only for distances greater than 10-15 m (nuclear distance)
  3. For charges at rest with respect to the observer.

Coulomb’s law: The force of attraction or repulsion between two point charges at rest is directly proportional to the product of the charges and inversely proportional to the square of the distance between them.

The force acts along the line joining the two charges. It depends on the nature of the intervening medium.

Read and Learn More Class 12 Physics Notes

Suppose two point charges q1 and q2 are at rest at a distance r from each other. If F is the magnitude of the electrostatic force acting between them, then according to Coulomb’s law

Class 12 Physics Unit 1 Electrostatics Chapter 2 Electric Field coulombs law

F ∝ q1q2, when r is constant

and \(P \propto \frac{1}{r^2}\), when q1 and q2 are constant

∴ \(F \propto \frac{q_1 q_2}{r^2}\), when q1, q2 and r variables

or, \(F=k^{\prime} \frac{q_1 q_2}{r^2}\)

where k’ is a constant of proportionality. Its value depends on the nature of the intervening medium and also on the units. In which the quantities F, q1,q2 and r are measured. Sometimes the constant K’ is referred to as the electrostatic constant or Coulomb constant.

WBCHSE Class 12 Physics Notes For Chapter 2 Electric Field

1. In CGS system:

⇒ \(F=\frac{q_1 q_2}{r^2}\) ( for vacuum or air)….(2)

2. In SI, For vacuum or air:

⇒ \(k^{\prime}=\frac{1}{4 \pi \epsilon_0}\)

where ∈0 is the permittivity of vacuum or air or permittivity of free space.

Hence, \(F=\frac{1}{4 \pi \epsilon_0} \cdot \frac{q_1 q_2}{r^2}\) (for vacuum or air)…..(3)

i.e., \(\epsilon_0=\frac{1}{4 \pi} \cdot \frac{q_1 q_2}{F r^2}\)……(4)

In this system, the charge is expressed in coulomb (C), force in newton (N) and distance in metre (m).

In this case, ∈0 = 8.854 x 10-12 C2.N-1.m-2

∴ \(\frac{1}{4 \pi \epsilon_0}=9 \times 10^9 \mathrm{~N} \cdot \mathrm{m}^2 \cdot \mathrm{C}^{-2}\)

Hence, from equation (3) \(F=9 \times 10^9 \frac{q_1 q_2}{r^2}\)….(5)

For any other intervening medium,

⇒ \(\dot{F}=\frac{1}{4 \pi \epsilon} \cdot \frac{q_1 q_2}{r^2}\)….(6)

where e is the permittivity of the medium

Again, e = k∈0, where K is the relative permittivity or dielectric constant of the medium

In the CGS system, ∈0 = l, so, ∈ = K, i.e., the permittivity of a medium is numerically equal to its dielectric constant. But permittivity has the unit statC² dyn-1 cm-2.

For Example, the dielectric constant of mica is 5.4 and the permittivity of mica is 5.4 statC². dyn-1.cm-1.

But in SI, the value of ∈0 is 8.854 x 10-12 C2.N-1.m-2 and Comparison between electrostatic force and grav- hence the magnitudes of the dielectric constant and the permit-rotational force: Both electrostatic and gravitational forces activity of a medium are not the same.

So, in SI, the permittivity of mica,

∈ = K ∈0

= 5.4 x 8.854 x 10-12 C2. N-1.m-2

= 4.78 x 10-11 C2. N-1.m-2

Note that, the permittivity of any medium is greater than that of vacuum. So the force of attraction or repulsion between two charges in any medium is less than that in a vacuum.

Only dry air has experimentally been observed to have almost the same permittivity (∈0) as that of vacuum.

Thus, the general formula of Coulomb’s law for any medium is given by

⇒ \(F=\frac{1}{4 \pi \kappa \epsilon_0} \cdot \frac{q_1 q_2}{r^2}\)…….(7)

From equation (4) we come to know the dimension of ∈0,

⇒ \(\left[\epsilon_0\right]=\frac{\left[q_1 q_2\right]}{[F]\left[r^2\right]}\)

= \(\frac{(\mathrm{IT})^2}{\mathrm{MLT}^{-2} \cdot \mathrm{L}^2}\)

= \(\left.\mathrm{M}^{-1} \mathrm{~L}^{-3} \mathrm{~T}^4\right|^2\)

It may be noted that in static electricity Coulomb’s law is an analogue of Newton’s law of gravitation. Of course, in the case of gravitation, this force is always attractive in nature and the proportionality constant (G) is universal.

Vector form of Coulomb’s law: Suppose, \(\vec{r}_1\) are the position vectors of charges\(\vec{r}_2\), respectively. So the force acting on q2 due to q1,

⇒ \(\vec{F}_{12}=\frac{1}{4 \pi \epsilon_0} \cdot \frac{q_1 q_2}{r_{21}^2} \cdot \hat{r}_{21}\)

= \(\frac{1}{4 \pi \epsilon_0} \cdot \frac{q_1 q_2}{r_{21}^3} \cdot \vec{r}_{21}\)……(8)

where \(\vec{r}_12\) is the position vector of 42 with respect to q1,

i.e., \(\vec{r}_{21}=\vec{r}_2-\vec{r}_1=r_{21} \hat{r}_{21}\)

Similarly, the force acting on q1 due to q2,

⇒ \(\vec{F}_{21}=\frac{1}{4 \pi \epsilon_0} \cdot \frac{q_1 q_2}{r_{12}^2} \cdot \hat{r}_{12}\)

= \(\frac{1}{4 \pi \epsilon_0} \cdot \frac{q_1 q_2}{r_{12}^3} \cdot \vec{r}_{12}\)

Class 12 Physics Unit 1 Electrostatics Chapter 2 Electric Field Vector form of Coulomb's law

Each of the equations (8) and (9) represents the vector form of Coulomb’s law.

As, \(\vec{r}_{12}=-\vec{r}_{21} \text {, and } \hat{r}_{12}=-\hat{r}_{21} \text {, we have } \vec{F}_{12}=-\vec{F}_{21}\)

Comparison between electrostatic force and gravitational force:

Both electrostatic and gravitational forces act between two charged bodies. Similarities and dissimilarities between these two forces are given below.

Similarities:

  1. Both forces are inversely proportional to the square of the distance between the two bodies.
  2. Both the forces are conservative, Le., work done by these forces is independent of path.
  3. Both the forces act in a vacuum as well.
  4. Both the forces are central forces where the force is expressed as \(\vec{F}=-F(r) \hat{r}\)
  5. Both the forces are mutually interactive, i.e., \(\vec{F}_{21}=-\vec{F}_{12}\)
  6. Both the forces are spherically symmetric [function of r only].

Dissimilarities:

Class 12 Physics Unit 1 Electrostatics Chapter 2 Electric Field gravitational force and electrostatic force

Unit of Charge:

CGS unit: Coulomb’s law, in the CGS system, is expressed as, \(F=\frac{q_1 q_2}{r^2}\) If we put F = 1 dyne, r = 1 cm, and q1 = q2 = q, we get from equation (2),

q2=1  or, q = ±1

Definition: If an electric repulsive force of 1 dyne arises between two similar point charges kept 1 cm apart in air, then each point charge is called a unit charge. This unit is known as I esu or 1 statcoulomb (state).

SI: Coulomb’s law In SI for vacuum or air is expressed as

⇒ \(F=\frac{1}{4 \pi \epsilon_0} \cdot \frac{q_1 q_2}{r^2}\)

Now, putting F = 9 x 109 N, r = 1 m,

q1= q2 = q and \(\frac{1}{4 \pi \epsilon_0}\) = 9 X 109 N.m2.C-2 we get,

⇒ \(9 \times 10^9=9 \times 10^9 \times \frac{q \times q}{(1)^2}\)

or, q2 = 1 or, q = ±1

Definition: If an electric repulsive force of 9 x 109 N arises between two similar point charges kept 1 m apart in vacuum or air, the amount of each charge is 1 C.

Relation between the units of charge:

1 C = 3 X 109 state (precisely, 2.99792458 x 109 esu)

An electromagnetic unit or emu is another unit of charge, defined in the discussions of electromagnetism. Its relations with the other units are,

1 emu of charge = 10 C = 3 X 10-10 statC

In atomic physics, the charge of an electron or a proton is taken as the unit of measurement of charge. This is called an electronic charge unit (e).

1 electronic unit of charge is equal to the charge of an electron or a proton.

1e = 4.8 x 10-10 statC = 1.6 X 10-19 C

±e is the least amount of charge that can exist in nature. No charge exists which is not an integral multiple of ±e.

The force between Multiple Charges Superposition Principle:

The principle of superposition: The total force acting on a charge due to a number of interacting charges around it is the vector sum of the individual forces acting on that charge due to each of the other charges.

Individual forces are calculated on Coulomb’s law. The force with which two charges interact does not change by the presence of other charges. Coulomb’s law gives the electrostatic force between two point charges.

In this case, it is assumed that other charges are not present near the two charges. If there are a number of interacting charges, the force on a particular charge may be obtained by the principle of superposition.

Suppose a system contains n number of point charges q1, q2, q3,….qn placed in vacuum or in air. So by the principle of superposition, the total force acting on qx exerted by all the other charges is given by

⇒ \(\vec{F}_1=\vec{F}_{21}+\vec{F}_{31}+\vec{F}_{41}+\cdots+\vec{F}_{n 1}\)

Here, \(\vec{F}_{21}\) = force acting on q1 due to q2

\(\vec{F}_{31}\) = force acting on q1 due to q3

………………………………………………..

\(\vec{F}_{n1}\) = force acting on q1 due to qn

Class 12 Physics Unit 1 Electrostatics Chapter 2 Electric Field The principle of superposition

According to Coulomb’s law,

⇒ \(\vec{F}_{21}=\frac{1}{4 \pi \epsilon_0} \frac{q_1 q_2}{r_{12}^2} \hat{r}_{12}\)

⇒ \(\hat{r}_{12}\) is the position vector of charge q1 with respect to q2

Similarly,

\(\vec{F}_{31}=\frac{1}{4 \pi \epsilon_0} \frac{q_1 q_3}{r_{13}^2} \hat{r}_{13} \text {; }\) \(\vec{F}_{41}=\frac{1}{4 \pi \epsilon_0} \frac{q_1 q_4}{r_{14}^2} \hat{r}_{14}\)

……………………………………………………..

\(\vec{F}_{n 1}=\frac{1}{4 \pi \epsilon_0} \frac{q_1 q_n}{r_{1 n}^2} \hat{r}_{1 n}\)

∴ \(\vec{F}_1=\frac{1}{4 \pi \epsilon_0}\left[\frac{q_1 q_2}{r_{12}^2} \hat{r}_{12}+\frac{q_1 q_3}{r_{13}^2} \hat{r}_{13}+\cdots+\frac{q_1 q_n}{r_{1 n}^2} \hat{r}_{1 n}\right]\)….(1)

Similarly, the total force acting on q2 exerted by all the other charges is given by,

⇒ \(\vec{F}_2=\vec{F}_{12}+\vec{F}_{32}+\vec{F}_{42}+\cdots+\vec{F}_{n 2}\)

⇒ \(\begin{aligned}
\frac{1}{4 \pi \epsilon_0}\left[\frac{q_1 q_2}{r_{21}^2} \hat{r}_{21}+\frac{q_2 q_3}{r_{23}^2} \hat{r}_{23}+\frac{q_2 q_4}{r_{24}^2} \hat{r}_{24}\right. & \\
& \left.+\cdots+\frac{q_2 q_n}{r_{2 n}^2} \hat{r}_{2 n}\right]
\end{aligned}\)……(2)

In general, if \(\vec{F}_i\). be the total force on the i th charge of the configuration, then,

⇒ \(\vec{F}_i=\frac{1}{4 \pi \epsilon_0} \sum_{\substack{j=1 \\ j \neq i}}^n \frac{q_i q_j}{r_{j l}^2} \hat{r}_{j l}\)……(3)

Here, \(\hat{r}_{j i}\). is the position vector of the charge with respect to aj.

In the CGS system, the above equation is

⇒ \(\vec{F}_i=\sum_{\substack{j=1 \\ j \neq i}}^n \frac{q_i q_j}{r_{j i}^2} \hat{r}_{j i}\)….(4)

Force due to Continuous Charge Distribution:

Although charges are quantised we can consider a large collection of charges in a region as a continuous distribution.

The continuous distribution of charges may be of three types:

  1. One dimensional,
  2. Two-dimensional
  3. Three-dimensional.

These distributions of charges are called linear charge distribution, surface charge distribution and volume charge distribution, respectively.

1. Linear charge distribution: Let there be a continuous distribution of charge along a straight line or a curved line placed in a vacuum or in air. The linear charge density = \(\left(\frac{\text { charge }}{\text { length }}\right)=\lambda\). Suppose, dl is an infinitesimally small element of this line charge distribution. Charge .on the element dl is dq = μdl.

Suppose, a test point charge q1 is at a distance r from the small element dl.

Class 12 Physics Unit 1 Electrostatics Chapter 2 Electric Field Linear charge distribution

Now, the force acting on q0 due to the charge dq is given by,

⇒ \(d \vec{F}=\frac{1}{4 \pi \epsilon_0} \frac{q_0 d q}{r^2} \hat{r} ; \text { where } \hat{r}=\frac{\vec{r}}{r}=\text { unit vector along } \vec{r}\)

⇒ \(\frac{1}{4 \pi \epsilon_0} \frac{q_0 \lambda d l}{r^2} \hat{r}\)

∴ The total force acting on q0 due to the whole line charge is given by

⇒ \(\vec{F}=\frac{q_0}{4 \pi \epsilon_0} \int_l \frac{\lambda}{r^2} \hat{r} d l\)…..(1)

l indicates the line along which the Integration extends.

In the CGS system, the above equation Is (replacing \(\epsilon_0 \text { by } \frac{1}{4 \pi}\))

⇒ \(\vec{F}=q_0 \int_l \frac{\lambda}{r^2} \hat{r} d l\)……(2)

Til Surface charge distribution: Let there be a continuous distribution of charge over a surface placed in a vacuum or in air. The surface charge density = \(\left(\frac{\text { charge }}{\text { area }}\right)=\sigma\) Suppose, ds be an infinitesimally small surface element of this surface distribution of charge. Charge on the surface element ds is dq = σds

Class 12 Physics Unit 1 Electrostatics Chapter 2 Electric Field Surface charge distribution

Suppose, a test point charge q0 is at a distance r from the small surface element ds.

Now, the force acting on qQ due to the charge dq is given by

⇒ \(d \vec{F}=\frac{1}{4 \pi \epsilon_0} \frac{q_0 d q}{r^2} \hat{r}=\frac{1}{4 \pi \epsilon_0} \frac{q_0 \sigma d s}{r^2} \hat{r}\)

∴ The total force acting on q0 due to the whole surface the charge distribution is given by,

⇒ \(\vec{F}=q_0 \int_s \frac{\sigma}{r^2} \hat{r} d s\)….(3)

s indicates the surface over which the integration extends.

In the CGS system, the above equation is,

⇒ \(\vec{F}=q_0 \int_s \frac{\sigma}{r^2} \hat{r} d s\)….(4)

3. Volume or spatial charge distribution: Let there be a continuous distribution of charge over a volume (such cube or a sphere) placed in a vacuum or in air. The volume density of charge \(\left(\frac{\text { charge }}{\text { volume }}\right)=\rho\)

Suppose, dv is an infinitesimally small volume element of till volume distribution of charge. Charge on the volume element dv is dq = pdv.

Class 12 Physics Unit 1 Electrostatics Chapter 2 Electric Field Volume or spatial charge distribution

Suppose, a test point charge q0 is at a distance r from the small volume element dv.

Now, the force acting on q0 due to the charge dq is
given by,

⇒ \(d \vec{F}=\frac{1}{4 \pi \epsilon_0} \frac{q_0 d q^2}{r^2} \hat{r}=\frac{1}{4 \pi \epsilon_0} \frac{q_0 \rho d v}{r^2} \hat{r}\)

∴ The total force acting on q0 due to the whole volume of the charge distribution is given by

⇒ \(\vec{F}=\frac{q_0}{4 \pi \epsilon_0} \int_v \frac{\rho}{r^2} \hat{r} d v\)….(5)

v indicates the volume to which the integration extends.

In the CGS system, the above equation is,

⇒ \(\vec{F}=q_0 \int_v \frac{\rho}{r^2} \hat{r} d v\)……(6)

Electrostatics

Electric Field Numerical Examples

Example 1. A 196 dyn force acts between two point charges separated by a distance of 6 cm. If the magnitude of one charge is 4 times that of the other, calculate the charges on each.
Solution:

From Coulomb’s law, \(F=\frac{q_1 q_2}{r^2}\)

Suppose, q1 = q; so, q2 = 4q

Given, F = 196 dyn; r = 6cm

∴ \(196=\frac{q \times 4 q}{6^2} \text { or, } q^2=49 \times 6^2\)

or, q = 42 statC

∴ The other charge

4 X 42 = 168 statC.

Example 2. In free space, the force between two charges separated by a constant distance Is 9 dyn. When the charges are placed In a medium of dielectric constant K, the force between them becomes 4 dyn. Then the value of k.
Solution:

Let the two charges are q1 and q2, and they are separated by a distance r.

Prom Coulomb’s law, \(F_1=\frac{q_1 q_2}{4 \pi \epsilon r^2}\) [e = permittivity of the medium]

and \(F_2=\frac{q_1 q_2}{4 \pi \epsilon_0 r^2}\) [∈0 = permittivity of free space]

∴ \(\frac{F_2}{F_1}=\frac{\epsilon}{\epsilon_0}=\kappa\)

or, k = \(\frac{9}{4}\)

= 2.25

Example 3. A pith ball having charge -20 esu and mass 0.1 g remains suspended at rest In air 2 cm below an Insulated charged sphere. What Is the amount of charge on the sphere and what is the nature of that charge? [Given, g = 980 cm s-2 ]
Solution:

Suppose, the charge on the sphere = q

Since the pith ball is at rest, the weight of the pith ball = upward attractive force on the pith ball

or, \(0.1 \times 980=\frac{20 \times q}{(2)^2}\)

or, q = 19.6

Since the pith ball is at rest in space, the charge on the sphere is of the opposite nature to that of the pith ball. The nature of the charge on the pith ball is negative. So the charge on the sphere is positive.

∴ Charge on the sphere = +19.6 esu.

Example 4. The weight of each of the two small metal spheres having the same amount of charge is 3g. The two spheres are suspended with two threads of length 13 cm each from a point. Due to mutual repulsion, they are at rest at a distance of 10 cm. Determine the charge on each sphere and tension in the threads.
Solution:

Let the two spheres at rest be at A and B.

Here, AB = 10 cm. The length of each thread

= OA = OB

= 13 cm, 6 = angle made by each thread with the vertical,

T = tension in each thread and q = charge on each sphere

In equilibrium, three forces an on each ball These are

1. Weight of the ball = mg,

2. Tension in thread = T

3. Mutual force of electrostatic repulsion = F.

Class 12 Physics Unit 1 Electrostatics Chapter 2 Electric Field example 4 In equilibrium three forces an on each ball

In the equilibrium of any sphere, the vertical component Tcosθ of the tension balances the weight mg of the sphere.

Tcosθ = mg

or, T x \(\frac{12}{13}\) = 3 x 980 [∵ \(\sin \theta=\frac{5}{13}, \cos \theta=\sqrt{1-\frac{25}{169}}=\frac{12}{13}\)]

or, \(T=\frac{3 \times 980 \times 13}{12}\) = 3185 dyn.

Again, the repulsive force F is balanced by the horizontal component Tsinθ of tension in the thread.

∴ F = Tsinθ

or, \(\frac{q \times q}{(10)^2}=3185 \times \frac{5}{13}\)

or, q2 = 245 x 500

or, q = 350 esu

Example 5. Two similar balls are suspended from a point by two silk threads, each of length l. Each hall of mass m contains q amount of charge. If the angle between the two threads is very small, show that the distance between the centres of the two balls at equilibrium is \(x=\left(\frac{2 q^2 l}{m g}\right)^{1 / 3}\).
Solution:

Let A and B be the equilibrium positions of the two balls, OA = OB = l and AB = x. Suppose, the angle of inclination of the two threads with the vertical = 0.

Class 12 Physics Unit 1 Electrostatics Chapter 2 Electric Field example 5 inclination of the two threads

At equilibrium, three forces act on each ball:

1. Weight of the ball mg;

2. Tension in the string T

3. Multual repulsive force between the charged balls, F.

Here, \(T \sin \theta=F=\frac{q^2}{x^2}\); T cosθ = mg

∴ \(\tan \theta=\frac{q^2}{m g x^2}\)

or, \(\sin \theta=\frac{q^2}{m g x^2}\) [∵ θ is small, tan θ ≈ sinθ]

or, \(\frac{x / 2}{l}=\frac{q^2}{m g x^2} \quad \text { or, } x=\left(\frac{2 q^2 l}{m g}\right)^{1 / 3}\)

Example 6. A+250 esu of charge P. is on the line joining the other two charges Q (+50 esu of charge) and R (-300 esu of charge), in between them. The distance of P from Q is 5 cm and from R, it is 10 cm. What is the resultant force acting on P?

Class 12 Physics Unit 1 Electrostatics Chapter 2 Electric Field example 6 The force acting on P due to the charge Q

Solution:

The force acting on P due to the charge Q

⇒ \(\frac{50 \times 250}{(5)^2}=500 \text { dyn; along } \overrightarrow{P R}\)

The force acting on P due to the charge R

⇒ \(\frac{250 \times 300}{(10)^2}=750 \mathrm{dyn} ; \text { along } \overrightarrow{P R}\)

∴ The resultant force acting on the charge P

= 500 + 750 = 1250 dyn; along \(\overrightarrow{P R}\)

Example 7. Three positive charges 3esu, 4 esu and 5 esu are placed at the vertice of an equilateral triangle of side 10cm. What is the force acting on the largest charge?
Solution:

Three charges are placed at the comers of the equilateral triangle ABC. The largest charge, i.e., 5 esu is situated at point C.

Class 12 Physics Unit 1 Electrostatics Chapter 2 Electric Field example 7 three charges are placed

Force acting on the charge at C due to the charge at A,

⇒ \(F_1=\frac{3 \times 5}{(10)^2}=0.15 \text { dyn; along } \overrightarrow{A C}\)

Force acting on the charge at C due to the charge at B,

⇒ \(F_2=\frac{4 \times 5}{(10)^2}=0.2 \mathrm{dyn} \text {; along } \overrightarrow{B C}\)

∴ The resultant force acting on the charge at C,

⇒ \(F=\sqrt{F_1^2+F_2^2+2 F_1 F_2 \cos 60^{\circ}}\)

⇒ \(\sqrt{(0.15)^2+(0.2)^2+2 \times 0.15 \times 0.2 \times 0.5}\)

⇒ \(\sqrt{0.0925}\)

= 0.304 dyn

Example 8. Two particles are placed in the air at a distance of 10 cm. How will you distribute 20 esu charge between the two particles so that the mutual force of repulsion between them will be maximum? What is the magnitude of this maximum force?
Solution:

Suppose, the charges on the particles are q esu and (20- q ) esu.

The force of repulsion between the two particles,

⇒ \(F=\frac{q \times(20-q)}{(10)^2} \mathrm{dyn}\)

For maximum value of \(F, \frac{d F}{d q}=0\)

∴ \(\frac{d F}{d q}=\frac{1}{100} \frac{d}{d q}\left(20 q-q^2\right)\)

= 0

or, 20- 2q = 0 or, q = 10

So one particle contains 10 esu of charge and the other (20- 10) or 10 esu of charge, i.e., the total charge is equally shared between them.

Maximum force, \(F_{\max }=\frac{10 \times 10}{(10)^2}=1 \mathrm{dyn}\)

= 1 dyn

Example 9. Three identical small spheres each of mass 0.1 suspended by three silk threads each of length 20 cm from a certain point How much charge should be given to each sphere so that each thread will make an angle of 30° with the vertical? You may suppose that each sphere has an equal charge.
Solution:

The spheres are identical and each sphere has equal charge. They will repel each other with equal force.

So in the position of equilibrium, they will form an equilateral triangle ABC in the horizontal plane.

Here, the length of the thread OA = OB = OC = 20 cm, mass of each sphere,

m = 0.1 g, angle of inclination of the thread with the vertical, 6 = 30°.

Class 12 Physics Unit 1 Electrostatics Chapter 2 Electric Field example 9 angle of inclination of the thread with the vertical

Suppose, the charge of each sphere is q.

Clearly, the vertical line OG passes through the centre of gravity of the triangle ABC. Suppose, the length of each side of the triangle = a.

Median, \(A D=\sqrt{a^2-\frac{a^2}{4}}=\frac{\sqrt{3}}{2} a\)

Now, \(A G=\frac{2}{3} A D=\frac{2}{3} \cdot \frac{\sqrt{3}}{2} a=\frac{a}{\sqrt{3}}\)

Again, \(\sin 30^{\circ}=\frac{A G}{O A} \quad\)

or, \(A G=O A \sin 30^{\circ}\)

= \(20 \times \frac{1}{2}\)

= 10 cm

∴ \(\frac{a}{\sqrt{3}}=10\)

or, \(a=10 \sqrt{3}\)cm

Force of repulsion between any two balls, \(F=\frac{q^2}{a^2}\)

On sphere A two equal forces of repulsion F act due to the spheres B and C. Suppose, the resultant of these two equal forces is R.

∴ R² =F² + F² + 2F.F cos60° [∵ ABAC = 60° , .. internal angle between F and F = 60°

= F² + F² + F² = 3F²

or, \(R=\sqrt{3} F=\sqrt{3} \cdot \frac{q^2}{a^2}\)

Let the tension along AO = T.

∴ At equilibrium, T sin30° = R , and T cos30° = mg

∴ \(\tan 30^{\circ}=\frac{R}{m g}=\frac{\sqrt{3} q^2}{a^2 \cdot m g}\)

or, \(\frac{1}{\sqrt{3}}=\frac{\sqrt{3} q^2}{a^2 m g} \quad \text { or, } \quad q^2=\frac{a^2 m g}{3}\)

or, \(q=a \sqrt{\frac{m g}{3}}=10 \sqrt{3} \cdot \sqrt{\frac{0,1 \times 980}{3}}\)

= 99 esu (approx.)

Example 10. Three-point charges are lying along the x-axis. If two charges q1 = -2C and q2 = -3C are placed at x1 = 2 m and x2 = -1 m, respectively and the third positive charge is so located between the first two charges that the resultant force on it is zero, find the position of the third charge.
Solution:

In, two charges q1 and q2 are placed at points A and B along the x-axis. Suppose, the third charge q is placed in between q1 and q2 at C at a distance x m from the origin O so that the resultant force on it is zero.

Class 12 Physics Unit 1 Electrostatics Chapter 2 Electric Field example 10 Three point charges are lying along the x -axis

∴ \(\frac{1}{4 \pi \epsilon_0} \cdot \frac{q q_1}{(2-x)^2}=\frac{1}{4 \pi \epsilon_0} \cdot \frac{q q_2}{(1+x)^2}\)

or, \(\frac{-2}{(2-x)^2}=\frac{-3}{(1+x)^2}\)

or, 2(1 +x)² = 3(2-x)²

or, \(\sqrt{2}(1+x)=\sqrt{3}(2-x) \quad\)

or, \((\sqrt{2}+\sqrt{3}) x=2 \sqrt{3}-\sqrt{2}\)

∴\(x=\frac{2 \sqrt{3}-\sqrt{2}}{\sqrt{2}+\sqrt{3}}\)

= \(\frac{(2 \sqrt{3}-\sqrt{2})(\sqrt{3}-\sqrt{2})}{3-2}\)

= \(8+-3 \sqrt{6} \approx 0.65\)

Therefore, the third charge is to be placed at a distance of +0.65 m from the origin O.

Example 11. Two negative point charges each of magnitude 2 esu and another point charge q are lying on a straight line. Each of the charges are in equilibrium. Determine the position, value and the nature of the charge q.
Solution:

Since the two negative charges and the other charge q are in a straight line and in equilibrium, then the charge q must be positive. Also, the charge q should be placed in between the two negative charges such that the resultant force acting on q due to the two negative charges becomes zero.

The two 2 esu charges are placed at points A and B respectively and charge q is placed at point C.

Class 12 Physics Unit 1 Electrostatics Chapter 2 Electric Field example 11 Two negative point charges

Let AB = r and AC = x

Now, the attractive force acting on +q due to the negative charge at

⇒ \(A=\frac{q \times 2}{x^2}\)

and the attractive force acting on +q due to the negative charge at

⇒ \(B=\frac{q \times 2}{(r-x)^2}\)

Now, from the condition of equilibrium of charge q,

⇒ \(\frac{q \times 2}{x^2}=\frac{q \times 2}{(r-x)^2} \quad \text { or, } x=\frac{r}{2}\)

Again, the repulsive force between the charges at A and B will try to push them apart, but the positive charge at C will attract them and maintain the equilibrium. Then the attractive force acting on any negative charge due to the positive charge q must be equal to the force of repulsion between two negative charges.

∴ \(\frac{q \times 2}{\left(\frac{r}{2}\right)^2}=\frac{2 \times 2}{r^2} \quad\)

or, q = \(\frac{1}{2}\)

= 0.5 esu

Example 12. A shower of protons from outer space deposits equal amounts of charge +q on the Earth and the moon. The electrostaticrepulsionthenexactlycounterbalancesthe counterbalanc the gravitational attraction. Find the amount of charge q.
Solution:

Let the mass of the moon be Mm and that of the earth be Mg and the distance between them be r.

The electrostatic repulsion between the moon and the earth

= \(\frac{q \times q}{r^2}=\frac{q^2}{r^2}\)

Gravitational attraction = \(\frac{G M_e M_m}{r^2}\)

According to the question,

⇒ \(\frac{q^2}{r^2}=\frac{G M_e M_m}{r^2}[/laetx]

or, [latex]q=\sqrt{G M_e M_m}\)

Example 13. The electrostatic force between two point charges and the gravitational attractive force between two point masses are inversely proportional to the square of the distance between the charges and the masses, respectively.

1.

  1. Compare the electrostatic force (Fc) and the gravitational force (FG) between an electron and a proton,
  2. Compare Fc and FG between two protons.

2. Find out the accelerations of an electron and of a proton due to the electrostatic force between them when they are placed at a distance of 1 A.
Solution:

1.

1. An electron and a proton have the same amount of charge e.

∴ Electrostatic force, \(F_C=\frac{1}{4 \pi \epsilon_0} \cdot \frac{e^2}{r^2}\)

Gravitational force, \(F_G=G \cdot \frac{m_e m_p}{r^2}\)

∴ \(\frac{F_C}{F_G}=\frac{1}{4 \pi \epsilon_0} \frac{e^2}{r^2} \cdot \frac{r^2}{G m_{e^2 m_p}}=\frac{1}{4 \pi \epsilon_0} \cdot \frac{e^2}{G m_{e^{m_p}}}\)

⇒ \(\begin{aligned}
& \frac{\left(9 \times 10^9\right) \times\left(1.6 \times 10^{-19}\right)^2}{\left(6.67 \times 10^{-11}\right) \times\left(9.1 \times 10^{-31}\right)} \\
& \times\left(1.67 \times 10^{-27}\right) \\
&
\end{aligned}\)

= 2.27 x 1039

2. The electrostatic force between two protons,

⇒ \(F_C=\frac{1}{4 \pi \epsilon_0} \cdot \frac{e^2}{r^2}\)

The gravitational force between them,

⇒ \(F_G=\frac{G m_p^2}{r^2}\)

∴ \(\frac{F_C}{F_G}=\frac{1}{4 \pi \epsilon_0} \frac{e^2}{r^2} \cdot \frac{r^2}{G m_p^2}=\frac{1}{4 \pi \epsilon_0} \cdot \frac{e^2}{G m_p^2}\)

⇒ \(\left(9 \times 10^9\right) \times \frac{\left(1.6 \times 10^{-19}\right)^2}{\left(6.67 \times 10^{-11}\right) \times\left(1.67 \times 10^{-27}\right)^2}\)

= 1.24 x 1039

2. The electrostatic force between an electron and a proton,

⇒ \(F_C=\frac{1}{4 \pi \epsilon_0} \cdot \frac{e^2}{r^2}=\left(9 \times 10^9\right) \times \frac{\left(1.6 \times 10^{-19}\right)^2}{\left(10^{-10}\right)^2}\) [∵ r = 1 A = 10-10m]

= 2.3 X 10-8 N

∴ Acceleration of the electron,

⇒ \(a_e=\frac{F_C}{m_e}=\frac{2.3 \times 10^{-8}}{9.1 \times 10^{-31}}=2.5 \times 10^{22} \mathrm{~m} \cdot \mathrm{s}^{-2}\)

Acceleration of the proton,

⇒ \(a_p=\frac{F_C}{m_p}=\frac{2.3 \times 10^{-8}}{1.67 \times 10^{-27}}=1.4 \times 10^{19} \mathrm{~m} \cdot \mathrm{s}^{-2}\)

Example 14. Three charges q, q and -q are kept at the three vertices of an equilateral triangle of side l. Find out the resultant force on each of the charges.
Solution:

Due to symmetry, the magnitude of the force between each pair of charges would be the same, given by

⇒ \(F=\frac{1}{4 \pi \epsilon_0} \frac{q^2}{l^2}\)

Class 12 Physics Unit 1 Electrostatics Chapter 2 Electric Field example 14 Three charges q, q and -q

Force acting on charge q at A due to the charges at points. B and C,

⇒ \(F_A=\sqrt{F_{A B}^2+F_{A C}^2+2 F_{A B} \cdot F_{A C} \cos 120^{\circ}}\)

⇒ \(\sqrt{F_{A B}^2+F_{A C}^2-F_{A B} \cdot F_{A C}}=F_{A B}\) [∵ FAB = FAC]

⇒ \(\frac{1}{4 \pi \epsilon_0} \cdot \frac{q^2}{l^2} ; \text { along } \overrightarrow{A D}\)

Force acting on charge q at B due to the charges at points A and C,

⇒ \(F_B=\sqrt{F_{B A}^2+F_{B C}^2+2 F_{B A} \cdot F_{B C} \cos 120^{\circ}} .\)

⇒ \(\sqrt{F_{B A}^2+F_{B C}^2-F_{B A} \cdot F_{B C}}\)

= FBA [∵ FBA = FBC]

⇒ \(\frac{1}{4 \pi \epsilon_0} \cdot \frac{q^2}{l^2} \text {; along } \overrightarrow{B E}\)

The force acting on charge -q at C due to the charges at points A and B,

⇒ \(F_C=\sqrt{F_{C A}^2+F_{C B}^2+2 F_{C A} \cdot F_{C B} \cos 60^{\circ}}\)

⇒ \(\sqrt{3} F_{C A}\) [∵ FCA = FCB]

⇒ \(\sqrt{3} \cdot \frac{1}{4 \pi \epsilon_0} \cdot \frac{q^2}{l^2} ; \text { along } \overrightarrow{C F}\)

The forces FA, FB and Fc are shown.

Let [latec]\vec{F}[/latex] be the resultant of the two forces \(\vec{F}_A\) and \(\vec{F}_B\)

Class 12 Physics Unit 1 Electrostatics Chapter 2 Electric Field example 14 Force acting on charge -q

∴ \(F=\sqrt{F_A^2+F_B^2+2 F_A F_B \cos 60^{\circ}}\)

= \(\sqrt{3} F_A=\sqrt{3} \cdot \frac{1}{4 \pi \epsilon_0} \cdot \frac{q^2}{l^2}\)

Therefore, F and Fc have the same magnitude, and they are oppositely directed. So they cancel each other

∴ \(\vec{F}_A+\vec{F}_B+\vec{F}_C=\vec{F}+\vec{F}_C=-\vec{F}_C+\vec{F}_C=0\)

Example 15. Three charges are placed at the vertices of an equilateral triangle of side l. Each of the charges Is q. Find out the force on a charge Q placed at the centre of mass of the triangle.
Solution:

The centre of mass O is equidistant from the vertices A, B, and C of the equilateral triangle

Class 12 Physics Unit 1 Electrostatics Chapter 2 Electric Field example 15 Three charges are placed at the vertices of an equllateral triangle of side l

So, each of the charge q will exert equal force F on the charge Q at O, i.e., FA = FB = FC = F

Now, two equal forces FB and FC act at O along with OE and OF, respectively. So their resultant Ff will act along OA, which is inclined at an angle of 60° with either OE or OF.

∴ F’ = FBcos60° + FCcos60° = \(F \cdot \frac{1}{2}+F \cdot \frac{1}{2}=F\)

As the force, FA of magnitude F acts along OD, i.e., opposite to OA, the resultant of F’ and FA will be zero.

∴ \(\vec{F}_A+\vec{F}_B+\vec{F}_C=\vec{F}_A+\vec{F}^{\prime}=-\vec{F}+\vec{F}=0\)

So, any charge Q placed at the centre of mass of a triangle will experience no force

Example 16. Two small metal spheres of equal volume are oppositely charged and are kept 0.5 m apart in a vacuum. The attractive force between them Is 0.144 N. But a repulsive force of 0.081 N acts between them after they are momentarily connected with a conducting wire. Find out the initial charges on the two spheres.
Solution:

Let the two initial charges be +q1 and -q2.

According to the question, the attractive force between the spheres = 0.144 N.

∴ \(\frac{1}{4 \pi \epsilon_0} \cdot \frac{q_1 q_2}{(0.5)^2}=0.144\)

or, \(q_1 q_2=\frac{0.144 \times 0.25}{9 \times 10^9}=4 \times 10^{-12}\)

The momentary connection brings the spheres to the same potential [See the chapter ‘Electric Potential’]. As the spheres have the same volume, their charges will also be the same. So, the charge on each sphere = \(\frac{1}{2}\left(q_1-q_2\right)\)

According to the question, the repulsive force between the spheres = 0.081 N

∴ \(\frac{1}{4 \pi \epsilon_0} \cdot \frac{\left(q_1-q_2\right)^2}{4 \times(0.5)^2}=0.081\)

or, \(\left(q_1-q_2\right)^2=\frac{0.081 \times 0.25 \times 4}{9 \times 10^9}\)

= \(9 \times 10^{-12}\)

or, q1-q2 = 3 X 10-6 ….(1)

∴ \(q_1+q_2=\sqrt{\left(q_1-q_2\right)^2+4 q_1 q_2}\)

⇒ \(\sqrt{9 \times 10^{-12}+4 \times\left(4 \times 10^{-12}\right)}\)

= 5 X 10-6 …(2)

From equations (1) and (2),

q1 = 4 x 10-6 C and q2 = 1 x 10-6 C

So, the initial charges on the spheres are +4 x 10-6 C and -10-6 C

Example 17. Two charges of equal magnitude 2 X 10-8C separated by a silk thread of length lm are placed on an insulated, smooth, horizontal surface. Calculate the tension on the string.
Solution:

Tension on the string = force acting between the two charges

⇒ \(\frac{1}{4 \pi \epsilon_0} \cdot \frac{q_1 q_2}{r^2}\)

⇒\(9 \times 10^9 \times \frac{\left(2 \times 10^{-8}\right)^2}{(1)^2}\) [∵ q1 = q2 = 2 X 10-8C , r = lm]

= 9 X 109 x 4 x 1-16

= 36 X 10-7N

Example 18. Three charges -q1, +q2 and -q3 are kept in vacuum. Show that the horizontal component of the resultant force in the charge -q1 would be proportional to \(\frac{q_2}{b^2}+\frac{q_3}{a^2} \sin \theta\)

Class 12 Physics Unit 1 Electrostatics Chapter 2 Electric Field example 18 horizontal component

Solution:

The force on -q1 due to +q2 is

⇒ \(F_{12}=\frac{1}{4 \pi \epsilon_0} \cdot \frac{q_1 q_2}{b^2} ; \text { along } \overrightarrow{O X}\)

The force on -q1 due to -q3 is

⇒ \(F_{13}=\frac{1}{4 \pi \epsilon_0} \cdot \frac{q_1 q_3}{a^2} ; \text { along } \overrightarrow{P O}\)

The component of F13 along \(\overrightarrow{O X}\)

⇒ \(\left(F_{13}\right)_X=\frac{1}{4 \pi \epsilon_0} \cdot \frac{q_1 q_3}{a^2} \sin \theta\)

∴ The horizontal component of the resultant force on -q1,

⇒ \(F_X=\frac{1}{4 \pi \epsilon_0} \cdot \frac{q_1 q_2}{b^2}+\frac{1}{4 \pi \epsilon_0} \cdot \frac{q_1 q_3}{a^2} \sin \theta\)

⇒ \(\frac{q_1}{4 \pi \epsilon_0}\left(\frac{q_2}{b^2}+\frac{q_3}{a^2} \sin \theta\right)\)

∴ \(F_X \propto\left(\frac{q_2}{b^2}+\frac{q_3}{a^2} \sin \theta\right)\)

Example 19. Four charges, each of -Q, are placed at the four corners of a square. Find out the value of a charge q placed at the centre of the square, such that all the charges would be at equilibrium.
Solution:

Let each side of the square = a

Class 12 Physics Unit 1 Electrostatics Chapter 2 Electric Field example 19 four corners of a square

∴ \(A C=\sqrt{a^2+a^2}=\sqrt{2} a \text { and } A O=\frac{A C}{2}=\frac{\sqrt{2} a}{2}=\frac{a}{\sqrt{2}}\)

Force on the charge -Q at A due to the charge -Q at D,

⇒ \(F=\frac{1}{4 \pi \epsilon_0} \cdot \frac{Q^2}{a^2} ; \text { along } \overrightarrow{B A}\)

Similarly, the force on -Q at A due to -Q at D will also be F, but along \(\overrightarrow{D A}\)

The resultant of these two forces will be,

⇒ \(F_1=\sqrt{F^2+F^2}=\sqrt{2} F=\frac{1}{4 \pi \epsilon_0} \cdot \frac{\sqrt{2} Q^2}{a^2} ; \text { along } \overrightarrow{C A}\)

Again, force on -Q at A due to the charge -Q at C,

⇒ \(F_2=\frac{1}{4 \pi \epsilon_0} \cdot \frac{Q^2}{(\sqrt{2} a)^2}=\frac{1}{4 \pi \epsilon_0} \cdot \frac{Q^2}{2 a^2} ; \text { along } \overrightarrow{C A}\)

Now, force on -Q at A due to the charge q at O,

⇒ \(F_3=-\frac{1}{4 \pi \epsilon_0} \cdot \frac{q Q}{\left(\frac{a}{\sqrt{2}}\right)^2}=-\frac{1}{4 \pi \epsilon_0} \cdot \frac{2 q Q}{a^2} ; \text { along } \overrightarrow{C A}\)

At equilibrium, the resultant of these three forces along \(\) must be zero, i.e.,

⇒ \(F_1+F_2+F_3=0 \quad \text { or, } \frac{1}{4 \pi \epsilon_0}\left[\frac{\sqrt{2} Q^2}{a^2}+\frac{Q^2}{2 a^2}-\frac{2 q Q}{a^2}\right]=0\)

or, \(\frac{1}{4 \pi \epsilon_0} \cdot \frac{Q}{a^2}\left[Q\left(\sqrt{2}+\frac{1}{2}\right)-2 q\right]=0\)

or, \(Q\left(\sqrt{2}+\frac{1}{2}\right)-2 q=0\)

or, \(2 q=Q \frac{2 \sqrt{2}+1}{2} \quad\)

or, \(q=\frac{Q}{4}(2 \sqrt{2}+1)\)

Example 20. Two identically charged spheres are suspended by strings of equal length. The strings make an angle of 30° with each other. When suspended in a liquid of density 0.8 g.cm-3, the angle remains the same. What is the dielectric constant of the liquid? The density of the material of the sphere is 1.6 g.cm-3.
Solution:

Let A and B be the equilibrium positions of the two spheres in both cases. Suppose, AB = r, the charge of each sphere = q, the mass of each sphere = m.

When the system is in air, the tension in the string = Tx and repulsive force between the spheres,

⇒ \(F_1=\frac{1}{4 \pi \epsilon_0} \frac{q^2}{r^2}\)

At equilibrium,

F1 = T1sin15° …(1)

and mg = T1 cos15°

From equations (1) and (2),

⇒ \(\frac{F_1}{m g}=\frac{T_1 \sin 15^{\circ}}{T_1 \cos 15^{\circ}}=\tan 15^{\circ}\)

or, F1 = mgtan15°

⇒ \(\frac{1}{4 \pi \epsilon_0} \frac{q^2}{r^2}=m g \tan 15^{\circ}\)

Class 12 Physics Unit 1 Electrostatics Chapter 2 Electric Field example 20 in air and in liquid

When the spheres are suspended in the liquid, tension in the string = T2, upthrust by the liquid on each sphere = u and repulsive force between the two spheres, \(F_2=\frac{1}{4 \pi \epsilon_0 \kappa} \frac{q^2}{r^2}\) [where k = dielectric constant of the medium]

Now at equilibrium,

F2 = T2sin15° …..(4)

and mg- u = T2cos15° ….(5)

From equations (4) and (5),

⇒ \(\frac{F_2}{m g-u}=\tan 15^{\circ}\)

or, F2 = (mg – u) tan15°

or, \(\frac{1}{4 \pi \epsilon_0 \kappa} \frac{q^2}{r^2}=(m g-u) \tan 15^{\circ}\)….(6)

From equations (3) and (6), we get

⇒ \(\kappa=\frac{m g}{m g-u}\)…..(7)

Let the volume of the sphere = V, the density of the material of the sphere = d and the density of the liquid = dl

∴ m = Vd and u – Vdl g

Hence from equation (7), we get \(\kappa=\frac{V d g}{V d g-V d_l g}=\frac{d}{d-d_l}\)

or, \(\kappa=\frac{1.6}{1.6-0.8}\) [∵ d = 1.6 g/cm³ and d1 – 0.8 g/cm³]

or, k = 2

Hence, the required dielectric constant is 2.

Example 21. An infinite number of charges, each equal to 4μC are placed along the x-axis at x = 1m, 2m, 4m, 8m and so on. Find the total force on a charge of 1 C placed at the origin.
Solution:

Here, a charge of 1 C is placed at x = 0

Let, the total force on the charge at x = 0 due to an infinite number of charges of 4μC placed at distances x = lm, x = 2m, x = 4m, x = 8 m … up to x = ∞ be F.

∴ \(\begin{array}{r}
F=\frac{1}{4 \pi \epsilon_0}\left[\frac{1 \times 4 \times 10^{-6}}{1^2}+\frac{1 \times 4 \times 10^{-6}}{2^2}+\frac{1 \times 4 \times 10^{-6}}{4^2}\right. \\
\left.+\frac{1 \times 4 \times 10^{-6}}{8^2}+\cdots+\infty\right]
\end{array}\)

⇒ \(=9 \times 10^9 \times 4 \times 10^{-6}\left[\frac{1}{1^2}+\frac{1}{2^2}+\frac{1}{4^2}+\frac{1}{8^2}+\cdots+\infty\right]\)

⇒ \(36 \times 10^3\left[\frac{1}{1-\frac{1}{4}}\right]=36 \times 10^3 \times \frac{4}{3}\)

= 48 x 10³N

Hence, the total force acting on the charge at x = 0 is 48 x 10³ N.

Example 22. A small ball of mass 2 x 10-3 kg having a charge of 1μC Is suspended by a string of length 0.8 m. Another identical ball having the same charge is kept at the point of suspension. Determine the minimum horizontal velocity which should be imparted to the lower ball, so that it can make a complete revolution.
Solution:

Let the charge of each ball – q, the radius of the vertical circular path = l and the velocity of the rotating ball at the highest and lowest point of that circular path be v and u respectively.

At the highest point A, three forces act simultaneously which are

1. Upward electrostatic force of repulsion, \(F_e=\frac{q^2}{l^2}\)[in CGS system],

2. Weight of the rotating ball, W = mg (downwards),

3. Tension in the string = T (downwards).

Class 12 Physics Unit 1 Electrostatics Chapter 2 Electric Field example 22 horizontal velocity

To move in a vertical circular path, the ball should have a minimum velocity at A. In that case, T = 0 and the necessary centripetal force is provided by (W – Fe).

∴ \(W-F_e=\frac{m v^2}{l}\)

∴ \(v^2=\frac{l}{m}\left[m g-\frac{q^2}{l^2}\right]\)

Applying the law of conservation of total mechanical energy at A and B we can write,

⇒ \(\frac{1}{2} m v^2+m g \times 2 l=\frac{1}{2} m u^2\)

or, \(u^2=v^2+4 g l=\frac{l}{m}\left[m g-\frac{q^2}{l^2}\right]+4 g l=5 g l-\frac{q^2}{m l}\)

∴ \(u=\sqrt{5 g l-\frac{q^2}{m l}}\)

Hence, the minimum horizontal velocity at the lowest point is

⇒ \(\sqrt{5 g l-\frac{q^2}{m l}}\)

Example 23. A rigid insulated wireframe in the die form of a right-angled triangle ABC is set in a vertical plane as shown in the figure. Two beads, each of mass m and carrying charges q1 and q2 are connected by a cord of length l and kept between the arms AB and AC of the triangle In such a way that they can slide without friction along the arms. When the beads arc stationary, determine

  1. The angle a,
  2. The tension in the cord and
  3. The normal reaction on the beads.

Class 12 Physics Unit 1 Electrostatics Chapter 2 Electric Field example 23 A rigid insulate wire frame in die form 1

If the cord is now cut what are the values of charge for which the beads continue to remain stationary?
Solution:

Let us consider the forces acting on bead P. The forces are O weight mg acting vertically downward, Q tension T in the cord, G electric force between P and Q given by, \(F=\frac{1}{4 \pi \epsilon_0} \cdot \frac{q_1 q_2}{l^2}\), normal reaction \(N_{R_1}\)

Class 12 Physics Unit 1 Electrostatics Chapter 2 Electric Field example 23 A rigid insulated wire frame in die form.

When the bead P is at equilibrium,

mgcos60° = (7’- F)cosα…..(1)

and \(N_{R_1}=m g \cos 30^{\circ}+(T-F) \sin \alpha\)…(2)

For die bead Q, we have

mgsin60° = (T-F)sinα….(3)

and \(N_{R_2}=m g \cos 60^{\circ}+(T-F) \cos \alpha\)….(4)

1. Dividing (3) by (1), tanα = tan60°

∴ a = 60°

2. Again putting a = 60° in equation (3),

mgsin60° = (T-F)sin60°

or, \(T=F+m g=\frac{1}{4 \pi \epsilon_0} \cdot \frac{q_1 q_2}{l^2}+m g\)……(5)

3. From equation (4), T-F = mg

∴ \(N_{R_2}=m g \cos 60^{\circ}+m g \cos 60^{\circ}=m g\)

From equation (2),

⇒ \(N_{R_1}=m g \cos 30^{\circ}+m g \sin 60^{\circ}=\sqrt{3} m g\)

When the cord is cut, T=0

From equation (5) we get,

⇒ \(m g=-\frac{1}{4 \pi \epsilon_0} \cdot \frac{q_1 q_2}{l^2}\)

The right-hand side of the equation should be positive which is possible only if q1 and q2 have opposite signs. Thus for equilibrium, die beads must have unlike charges. The magnitude of the product of the charges is \(\left|q_1 q_2\right|=4 \pi \epsilon_0 m g l^2\)

Example 24. A particle A having a charge of 5.0 X 10~7 C Is fixed In a vertical wall. A second particle B of mass 100 g and having equal charge Is suspended by a silk thread of length 30 cm from the wall. The point of suspension Is located 30 cm vertically above the first particle. Find the angle of the thread with the vertical when It stays In equilibrium.
Solution:

The situation lets the point of suspension be O, where the tire thread makes an angle of θ with the vertical. Forces on the particle B are

  1. Weight mg acting downward,
  2. Tension T along the thread,
  3. The electric force of repulsion F along AB.

At equilibrium, the sum of all these forces becomes zero.

From the given, \(O A=O B, \angle O B A=\angle O A B=\left(90^{\circ}-\frac{\theta}{2}\right)\)

Considering the components along BX, we get

⇒ \(F \cos \frac{\theta}{2}=m g \cos \left(90^{\circ}-\theta\right)=m g \sin \theta=2 m g \sin \frac{\theta}{2} \cos \frac{\theta}{2}\)

or, \(\sin \frac{\theta}{2}=\frac{F}{2 m g}\)

Now, \(F=\frac{1}{4 \pi \epsilon_0} \cdot \frac{q_1 q_2}{A B^2} \text { and } A B=2(O A) \sin \frac{\theta}{2}\)

∴ \(\sin \frac{\theta}{2}=\frac{1}{4 \pi \epsilon_0} \cdot \frac{q_1 q_2}{4(O A)^2 \sin ^2 \frac{\theta}{2}} \cdot \frac{1}{2 m g}\)

Class 12 Physics Unit 1 Electrostatics Chapter 2 Electric Field example 24 A particle A having a charge

or, \(\sin ^3 \frac{\theta}{2}=\frac{1}{4 \pi \epsilon_0} \cdot \frac{q_1 q_2}{4(O A)^2 \sin ^2 \frac{\theta}{2}} \cdot \frac{1}{2 m g}\)

⇒ \(\begin{aligned}
& \left(9 \times 10^9\right) \times \frac{\left(5 \times 10^{-7}\right)^2}{4 \times\left(30 \times 10^{-2}\right)^2} \\
& \quad \times \frac{1}{2 \times\left(100 \times 10^{-3}\right) \times 9.8}
\end{aligned}\)

= 0.0032

or, \(\sin \frac{\theta}{2}=0.15\)

∴ 0 = 17°

Electrostatics

Electric Field Electric Field Intensity Of An Electric Field At A Point

Electric field: The space around any electric charge in which its influence can be felt, so that any other charge in that region is acted upon by a force, is known as the electric field of that charge.

Theoretically, the field extends up to infinity, but the limited sensitivity of detecting instruments predicts that charge exerts its influence (force of attraction or repulsion) only over a limited region.

Intensity of an electric field: The intensity or strength of an electric field at any point in the field is the force experienced by a unit positive charge placed at that point.

Obviously, the intensity of an electric field is a vector quantity. The direction of intensity is given by the direction of force acting on the positive charge. An electric field may be generated due to more than one charge.

Intensity at a point in such an electric field is determined by calculating the field at that point due to all the charges individually and then the resultant of all the field strengths are found out by vector addition.

The intensity or strength of the electric field at a point is generally denoted by the letter \(\vec{E}\). Often it is simply called an ‘electric field \(\vec{E}\)

Suppose, a test charge q0 experiences a force of F when it is placed at a point in an electric field. Then, the electric field at that point is \(\vec{E}=\frac{\vec{F}}{q_0}\)

Here, the test charge q0 should be very small so that it cannot alter the original electric field. So the correct definition of \(\vec{E}\) should be,

⇒ \(\vec{E}=\lim _{q_0 \rightarrow 0} \frac{\vec{F}}{q_0}\)

1. There is no problem as such in using the definition of \(\vec{E}\), except that we cannot make q0 arbitrarily small, because in the real world, we have never observed a charge smaller than the electronic charge.

2. No problem arises however, if we take the definition of \(\vec{E}\) of a charge distribution at any point (x, y, z) , where the position vector is \(\vec{r}(=\hat{i} x+\hat{j} y+\hat{k} z)\) as

⇒ \(\vec{E}=\frac{1}{4 \pi \epsilon_0} \sum_{j=1}^n \frac{q_j \hat{r}_j}{r_j^2}\)

where \(\hat{r}_j\) is the unit vector from the jth charge to the point (\(\vec{r}\)) (x, y, z).

Unit: The unit of electric field intensity in the CGS system is dyn/esu of charge or dyn/statcoulomb and in SI the unit is newton/coulomb (N.C-1).

An alternative unit of electric intensity in SI :

⇒ \(\frac{\mathrm{N}}{\mathrm{C}}=\frac{\mathrm{N} \times \mathrm{m}}{\mathrm{C} \times \mathrm{m}}=\frac{\mathrm{J}}{\mathrm{C} \times \mathrm{m}}=\frac{\mathrm{V}}{\mathrm{m}}=\mathrm{V} \cdot \mathrm{m}^{-1}\)

Dimension: \([E]=\frac{[F]}{[q]}=\frac{\mathrm{MLT}^{-2}}{\mathrm{IT}}=\mathrm{MLT}^{-3} \mathrm{I}^{-1}\)

Relation between CGS and SI units of intensity:

⇒ \(1 \mathrm{~N} \cdot \mathrm{C}^{-1}=\frac{10^5 \mathrm{dyn}}{3 \times 10^9 \text { esu of charge }}=\frac{1}{3 \times 10^4} \mathrm{dyn} \cdot \mathrm{statC}^{-1}\)

So, 1 esu electric intensity = 3 x 104 N.C-1

Electric field due to a point charge:

Let a point charge q be placed at a point in a medium of permittivity k. This charge will create an electric field around it.

To determine the electric field at a point at a distance r from the charge, a unit positive charge is imagined to be placed at that point.

The force experienced by the unit positive charge is the magnitude of the electric field at that point.

If charge q is placed at a point in a medium of permittivity E, the electric field at a distance r from the charge is given by,

⇒ \(E=\frac{1}{4 \pi \epsilon} \cdot \frac{q}{r^2}=\frac{1}{4 \pi \kappa \epsilon_0} \cdot \frac{q}{r^2}\)…..(1)

[K = dielectric constant of the medium]

For vacuum or air, K = 1

∴ \(E=\frac{1}{4 \pi \epsilon_0} \cdot \frac{q}{r^2}\) ….(2)

Therefore, an electric field due to a point charge is directly proportional; to the charge (q) and inversely proportional to the square of the distance (r) from it.

It is obvious that the electric field is different at different points of a field. Since we have assumed for definition that the test charge is positive, the electric field due to this will be directed away from the charge.

In case the source charge is negative, the field will be directed towards the charge

If E be the electric field at a point, the force acting on a charge q placed at that point, by definition is given by, F = qE i.e., force = charge x field intensity

The force acting on a charge in an electric field does not depend ‘ on its mass.

In the CGS system, for air or vacuum,

⇒ \(F=\frac{q}{r^2}\)

It should be noted that if the magnitude and direction of the field are the same at all points in the field, it is called a uniform electric field.

Electrostatics Electric Field Numerical Examples

Example 1. A negative charge of 20 units is placed at a distance 50 cm away from a positive charge of 80 units. Where will the electric field be zero on the line joining the two charges?
Solution:

Suppose, +80 unit and -20 unit of charges are placed at A and B respectively. The point where the electric field will be zero cannot lie in between A and B, because in that case intensity could be along the same direction, i.e., along \(\vec{AB}\) for both the charges.

As the charge at A is greater, the point where the resultant inequality is zero will be situated on the right side of B, say at P.

Class 12 Physics Unit 1 Electrostatics Chapter 2 Electric Field example 1 A negative charge

Suppose, BP = x

The electric field at P due to the charge at \(B=\frac{20}{x^2} \text {; along } \overrightarrow{P B}\)

The electric field at P due to the charge at \(A=\frac{80}{(50+x)^2}; \text { along } \overrightarrow{A P}\)

Since, the resultant intensity at P = 0

∴ \(\frac{20}{x^2}=\frac{80}{(50+x)^2} \quad \text { or, } x=50,-\frac{50}{3}\)

Now, \(x \neq-\frac{50}{3} \mathrm{~cm}\), because the point in that case would be situated in between A and B.

So, x = 50 cm; the point where the field Is zero at a distance of 50 cm from the -20 unit charge on its right side.

Example 2. At each of the four vertices of a square of side 10 cm, there is a + 20 esu of charge. Find the intensity of the electric field at the point of intersection of the two diagonals.
Solution:

The point of intersection O of the diagonals is equidistant from four equal charges. So the electric field at O due to each charge is the same in magnitude (=E, say).

Class 12 Physics Unit 1 Electrostatics Chapter 2 Electric Field example 2 The point ofintefsection

Now, the fields E along \(\overrightarrow{O C}\) and E along \(\overrightarrow{O A}\) cancel each other.

Similarly, E along \(\overrightarrow{O D}\) and E along \(\overrightarrow{O B}\) also canceL

Therefore, the intensity of the electric field at the point of intersection of the two diagonals of the square is zero.

Example 3. AB and CD are two perpendicular diameters of a circle of circumference 20πcm. There are +10 esu, +10esu, -10esu and -10esu of charges at A, B, C and D respectively. What is the intensity of the electric field at the centre O of the circle? What is the direction of the field?
Solution:

Suppose, O is the centre of the circle

Class 12 Physics Unit 1 Electrostatics Chapter 2 Electric Field example 3 two perpendicular diameters

The radius of the circle = r

Here, 2πr = 20π

or, r = 10 cm

Intensities of the electric field at O due to the charges at A and B cancel each other because the two fields are equal and opposite.

Intensity at O due to the charge at C

⇒ \(=\frac{10}{(10)^2}=\frac{1}{10} \text { dyn } \cdot \operatorname{statC}^{-1} \text {; along } \overrightarrow{O D}\)

Intensity at O due to the charge at D

⇒ \(\frac{10}{(10)^2}=\frac{1}{10} \mathrm{dyn} \cdot \operatorname{statC}^{-1} \text {; along } \overrightarrow{O D}\)

∴ Resultant intensity at O

⇒ \(\left(\frac{1}{10}+\frac{1}{10}\right)=0.2 \mathrm{dyn} \cdot \operatorname{statC}^{-1} \text {; along } \overrightarrow{O D}\)

Example 4. The bob of a pendulum of weight 80 mg carries a charge of 2 x 10-8 C. The bob is at rest in a horizontal electric field of magnitude 2 x 104 V.m-1. Determine the tension in the string and the angle of the string with the vertical. Given, g = 9.8 m.s-2
Solution:

Suppose, the string of the pendulum is inclined at an angle of θ with the vertical

Class 12 Physics Unit 1 Electrostatics Chapter 2 Electric Field example 4 he bob of a pendulum of weight

Let the horizontal electric field = E; tension in the string = T; charge on the bob = q. The forces acting on the bob.

At equilibrium,

Tsinθ = Eq; Tcosθ = mg

∴ \(\tan \theta=\frac{E q}{m g}\)

= \(\frac{2 \times 10^4 \times 2 \times 10^{-8}}{80 \times 10^{-6} \times 9.8}\)

= \(\frac{25}{49}\)

or, \(\theta=\tan ^{-1}\left(\frac{25}{49}\right) \approx 27^{\circ}\)

Now, cosθ = cos27° = 0.89

∴ \(T=\frac{m g}{\cos \theta}=\frac{80 \times 10^{-6} \times 9.8}{0.89}\)

= 8.8 x 10-4 N

Example 5. A circular copper ring of radius r, placed in a vacuum, has a charge q on it. Find out the electric fields

  1. At the center of the ring, and
  2. On the axis of the ring at a distance x from its center,
  3. For what value of x would the electric field be maximum?

Solution:

The charge q would be uniformly distributed along the circumference of the circular ring.

So, the linear density of charge along the ring,

⇒ \(\lambda=\frac{q}{2 \pi r}\)

1. The electric field at the centre of the ring due to the charge on the small element of the ring would be cancelled due to the charge on the diametrically opposite small element.

The whole ring is effectively an assembly of a large number. of such diametrically opposite pairs of elements. As a result, the resultant electric field at the die centre would be zero.

2. Let us take a small element of length dl on the ring at the position A. Charge on the element \(d l=\lambda d l=\frac{q}{2 \pi r} d l\). The electric field at the axial point P due to the charge on the element dl,

Class 12 Physics Unit 1 Electrostatics Chapter 2 Electric Field example 5 A circular copper ring of radius

⇒ \(d E=\frac{1}{4 \pi \epsilon_0} \cdot \frac{\dot{\lambda} d l}{(A P)^2} ; \text { along } \overrightarrow{A P}\)

Its axial component is

⇒ \(d E \cos \theta=\frac{1}{4 \pi \epsilon_0} \cdot \frac{\lambda d l}{(A P)^2} \frac{x}{(A P)}=\frac{1}{4 \pi \epsilon_0} \cdot \frac{\lambda x d l}{(A P)^3}\)

The component of dE perpendicular to the axis = dEsind.

For charges distributed throughout the entire ring, \(\sum d E \sin \theta=0\), due to symmetry.

Therefore, the resultant axial electric field at the point P,

⇒ \(E=\sum d E \cos \theta=\sum \frac{1}{4 \pi \epsilon_0} \cdot \frac{\lambda x d l}{(A P)^3}\)

⇒ \(\frac{1}{4 \pi \epsilon_0} \cdot \frac{q}{2 \pi r} \cdot \frac{x}{\left(r^2+x^2\right)^{3 / 2}} \cdot \sum d l\) [∵ \(\lambda=\frac{q}{2 \pi r} \text { and } A P=\left(r^2+x^2\right)^{1 / 2}\)]

⇒ \(\frac{1}{4 \pi \epsilon_0} \cdot \frac{q}{2 \pi r} \cdot \frac{x}{\left(r^2+x^2\right)^{3 / 2}} \cdot 2 \pi r\)

⇒ \(\frac{1}{4 \pi \epsilon_0} \cdot \frac{q x}{\left(r^2+x^2\right)^{3 / 2}}\)

2. For E to be maximum, \(\frac{dE}{dx}\) = 0

Now, \(\frac{d E}{d x}=\frac{q}{4 \pi \epsilon_0} \cdot \frac{d}{d x}\left[x\left(r^2+x^2\right)^{-3 / 2}\right]\)

⇒ \(\begin{aligned}
& \frac{q}{4 \pi \epsilon_0}\left[1 \cdot\left(r^2+x^2\right)^{-3 / 2}\right. \\
& \left.\quad+x \cdot\left(-\frac{3}{2}\right) \cdot\left(r^2+x^2\right)^{-5 / 2} \cdot 2 x\right]
\end{aligned}\)

⇒ \(\frac{q}{4 \pi \epsilon_0}\left(r^2+x^2\right)^{-5 / 2}\left[\left(r^2+x^2\right)-3 x^2\right]\)

⇒ \(\frac{q}{4 \pi \epsilon_0}\left(r^2+x^2\right)^{-5 / 2}\left(r^2-2 x^2\right)\)

For maximum E,

⇒ \(\frac{d E}{d x}=0 \text { or, } r^2-2 x^2=0 \text { or, } x= \pm \frac{r}{\sqrt{2}}\)

The ‘±’ sign implies that the electric field on the die axis will be maximum at a distance \(\frac{r}{\sqrt{2}}\) on either side of the ring.

Example 6. Starting from rest, an electron of mass me and a proton of mass mp travel through a certain distance in a uniform electric field in times t1 and t2, respectively. Find out the ratio t2/t1, neglecting the influence of gravity.
Solution:

In a uniform electric field of intensity E, the force acting on a charge q is F = qE and its acceleration, a = \(\frac{qE}{m}\).

The distance travelled from rest in time t,

⇒ \(s:=\frac{1}{2} a t^2=\frac{1}{2}\left(\frac{q E}{m}\right) t^2\)

For electron, \(s_e=\frac{1}{2}\left(\frac{q E}{m_e}\right) t_1^2 ; \text { for proton, } s_p=\frac{1}{2}\left(\frac{q E}{m_p}\right) t_2^2\)

Given se = sp; so we have,

⇒ \(\frac{1}{2}\left(\frac{q E}{m_e}\right) t_1^2=\frac{1}{2}\left(\frac{q E}{m_p}\right) t_2^2 \quad \text { or, } \frac{1}{m_e} t_1^2=\frac{1}{m_p} t_2^2\)

or, \(\frac{t_2^2}{t_1^2}=\frac{m_p}{m_e} \text { or; } \frac{t_2}{t_1}=\sqrt{\frac{m_p}{m_e}}\)

Example 7. Calculate the radius of a charged water drop which remains just suspended in equilibrium in the earth’s electric field. The charge in the water drop is equal to that of an electron. The magnitude of the earth’s electric field is 10-2statC.cm-1. [e = 4.805 x 10-10 esu of charge; g = 980 cm.s-2]
Solution:

Let the radius of the water drop be r cm and mass mg.

Here the intensity of the electric field,

E = 10-2 statC.cm-1

In equilibrium, electrical force on the charged water drop = weight of the water drop.

or, eE = mg

or, \(e E=\frac{4}{3} \pi r^3 \rho g \text { [denstiy of water, } \rho=1 \mathrm{~g} \cdot \mathrm{cm}^{-3} \text { ] }\)

or, \(r^3=\frac{3 e E}{4 \pi g}\)

= \(\frac{3 \times 4.805 \times 10^{-10} \times 10^{-2}}{4 \times \pi \times 980}\)

= 1.171 x 10-15

∴ r = 1.054 x 10-5 cm

Example 8. A simple pendulum consists of a small sphere of mass suspended by a thread of length l. The sphere carries a charge q. The pendulum Is placed in a uniform electric field of strength E directed vertically upwards. With what period will the pendulum oscillate if the electrostatic force acting on the sphere is less than the gravitational force?
Solution:

Let the pendulum be at an angle of θ with the vertical Net force acting on the bob = mg-qE Torque experienced by the bob about the point of suspension,

⇒ \(\tau\) = -(mg-qE)lsinθ (where the negative sign indicates that the E torque experienced is restoring torque)

As the amplitude of oscillation is small,

sinθ ≈ θ

Then, \(\tau=-(m g-q E) l \theta\)

Again \(\tau=\text { moment of inertia }(I) \times \text { angular acceleration }(\alpha)\)

∴ \(\alpha=\frac{\tau}{I}=\frac{-(m g-q E) l \theta}{I}=\frac{-(m g-q E) l \theta}{m l^2}\) [∵ I = ml²]

⇒ \(\frac{-\left(g-\frac{q E}{m}\right) \theta}{l}\)

∴ \(\alpha \propto-\theta\) [∵ \frac{\left(g-\frac{q E}{m}\right)}{l}=\text { constant }\right][/latex]

∴ The pendulum executes an SHM.

∴ Period of the pendulum

⇒ \(T=2 \pi \sqrt{\frac{\text { angular displacement }}{\text { angular acceleration }}}\)

= \(2 \pi \sqrt{\frac{\theta}{\alpha}}\)

∴ \(T=2 \pi \sqrt{\frac{l}{g-\frac{q E}{m}}}\)

Class 12 Physics Unit 1 Electrostatics Chapter 2 Electric Field example 8 A simple pendulum

Example 9. Find the electric field intensity at the centre of a semicircular arc of radius r, uniformly charged with a charge q.
Solution:

The length of the semicircular wire is πr.

Linear charge density, \(\lambda=\frac{q}{\pi r}\)

Let the charge of a small part dl of the wire be dq.

∴ \(d q=\lambda \cdot d l=\frac{q}{\pi r} \cdot d l\)

Field intensity at the centre O due to the small part,

⇒ \(d E=\frac{1}{4 \pi \epsilon_0} \cdot \frac{d q}{r^2}=\frac{1}{4 \pi \epsilon_0} \cdot \frac{q d l}{\pi r \cdot r^2}\)

⇒ \(\frac{1}{4 \pi \epsilon_0} \cdot \frac{q d \theta}{\pi r^2}\) [∵ dl = rdθ]

Class 12 Physics Unit 1 Electrostatics Chapter 2 Electric Field example 9 the electric field intensity

Now the field intensity dE is resolved into two components, one along the radius which is dEx = dEcosd and another particular dicular to the radius which is dEy = dEsind. Considering the whole wire, it is seen that all the dEx components get cancelled. Only the dEy components get to remain.

∴ Total field intensity at O,

⇒ \(E=\int_0^\pi d E \sin \theta=\frac{1}{4 \pi \epsilon_0} \cdot \frac{q}{\pi r^2} \int_0^\pi \sin \theta \cdot d \theta\)

⇒ \(\frac{1}{4 \pi \epsilon_0} \cdot \frac{q}{\pi r^2}(1-\cos \pi)\)

= \(\frac{1}{4 \pi \epsilon_0} \cdot \frac{2 q}{\pi r^2}\)

Example 10. An electron of charge 1.6 x 10-19C and 9.1 X 10-31 kg, travelling along the X-axis with a uniform velocity of 106 m s-1, enters in a uniform elecCrisfield of 103 V.m-1 acting perpendicular to the X-axis. If the electric field extends over a length of 2 cm along the X-axis, what will be the deflection of the electron along the direction of the field when it emerges from it?
Solution:

Here, the electric field E = 103 V/m is directed along the Yaxis and effective up to a length L = 2 cm along the X-axis.

An electron is projected from O with an initial speed of vx = 106 m/s along + X-axis. Charge of the electron, e = 1.6 x 10-19 C and mass of the electron, m = 9.1 x 10-31 kg.

Class 12 Physics Unit 1 Electrostatics Chapter 2 Electric Field example 10 An electron of charge

If we consider the motion of the electron along the Y-axis, the force on the electron due to the electric field, Fy = eE.

∴ Acceleration of the electron along 7-axis, ay = \(\frac{eE}{m}\)

Let the deflection of the electron when it is emitted from the electric field be y and the time taken to cover that vertical distance be t.

∴ \(y=\frac{1}{2} \cdot a_y t^2\)

or, \(y=\frac{1}{2} \cdot \frac{e E}{m} t^2\)….(1)

Along the X-axis, force on the electron, Fx = 0

∴ Acceleration, ax = 0

∴ Velocity, vx = 106 m/s (constant)

Now distance covered along the X-axis in time t is L = vxt

∴ \(t=\frac{L}{v_x}\)

From equation (1) and (2),

⇒ \(y=\frac{1}{2} \cdot \frac{e E}{m} \cdot \frac{L^2}{v_x^2}\)

⇒ \(\frac{1}{2} \times \frac{1.6 \times 10^{-19} \times 10^3 \times(.02)^2}{9.1 \times 10^{-31} \times\left(10^6\right)^2}\)

= 0.0351 m

= 3.51 cm

Thus, the required deflection is 3.51 cm.

Example 11. A charged cork ball of mass m is suspended on a light Y string in the adjacent figure. T When \(\vec{E}=\left(E_x \hat{i}+E_y \hat{j}\right) \hat{N} / \mathrm{C}\) the ball is in equilibrium at the angle θ. Find the charge of the ball and the tension in the string.

Class-12 Physics Unit 1 Electrostatics Chapter 2 Electric Field example 10 a charge cork ball

Solution:

In equilibrium, the net force acting on the ball = 0.

Class 12 Physics Unit 1 Electrostatics Chapter 2 Electric Field example 11 Two negative point charges.

⇒ \(T \cos \theta \hat{j}-T \sin \theta \hat{i}+q E_x \hat{i}+q E_y \hat{j}-m g \hat{j}=0\)

∴ Tsinθ = qEx….(1)

and Tcosθ = mg – qEy….(2)

From equations (1) and (2) we get,

⇒ \(\tan \theta=\frac{q E_x}{m g-q E_y}\)….(3)

or, \(m g \tan \theta-q E_y \tan \theta=q E_x\)

∴ \(q=\frac{m g \tan \theta}{E_x+E_y \tan \theta}\)

Putting the value of q in equation (1) we get,

⇒ \(T \sin \theta=\frac{m g \tan \theta}{E_x+E_y \tan \theta} E_x\)

∴ \(T=\frac{m g E_x \sec \hat{\theta}}{E_x+E_y \tan \theta}\)

Example 12. Two point charges of 10 esu and 40 esu are located at points A and B separated by a distance of 4 cm. Find the electric field intensity at a point halfway between the charges.
Solution:

Since the charges are 4 cm apart, each of them is 2 cm away from the midpoint (O) joining them Electric field intensity at O due to the charge 10 esu,

⇒ \(E_1=\frac{10}{(2)^2}=2.5 \mathrm{dyn} / \mathrm{esu} ; \text { along } \overrightarrow{O B}\)

Class 12 Physics Unit 1 Electrostatics Chapter 2 Electric Field example 12 electric field intensity at a point halfway

Electric field intensity at O due to the charge 40 esu,

⇒ \(E_2=\frac{40}{(2)^2}=10 \mathrm{dyn} / \mathrm{esu} ; \text { along } \overrightarrow{O A}\)

Since \(\overrightarrow{E_1} \text { and } \overrightarrow{E_2}\) are oppositely directed, the resultant field intensity at O,

E = E2-E1

= 10- 2.5

= 7.5 dyn/esu; along \(\vec{OA}\)

Example 13. Two negative charges each of 1 unit and a positive charge q are placed along a straight line. Find the magnitude and position of q for which all the charges will remain at equilibrium. Is the equilibrium stable, unstable or neutral in this case?
Solution:

The positive charge q has to be placed somewhere between the negative charges so that the attractive forces due to the negative charges may balance each other. Let the negative charges be placed at A and B and the charge +q be placed at C

Class 12 Physics Unit 1 Electrostatics Chapter 2 Electric Field example 13 equilibrium stable

Let AB = r; AC = x

The attractive force on +q due to the negative charge at \(A=\frac{q}{x^2}\)

The attractive force on +q is due to the negative charge at

⇒ \(B=\frac{q}{(r-x)^2}\)

The charge q is at equilibrium, thus

⇒ \(\frac{q}{r^2}=\frac{q}{(r-x)^2}\)

or, \(x=\frac{r}{2}\)

The negative charges at A and B tend to move away from each other due to repulsion but are held at those points by the attractive forces of q.

At equilibrium, the attractive force between q and any negative charge = the repulsive force between two negative charges.

∴ \(\frac{q \times 1}{\left(\frac{r}{1}\right)^2}=\frac{1 \times 1}{r^2}\)

or, q = \(\frac{1}{4}\)

= 0.25 unit

This is an unstable equilibrium because a slight change in the position of any of the charges disturbs the equilibrium.

Example 14. Two identical balls each of mass m are hung from a point by two silk threads of length l. Each of them has a charge q. The angle between the two threads is negligibly small, showing that at equilibrium distance between the centres of the balls will be, \(x=\left(\frac{2 q^2 l}{m g}\right)^{1 / 3}\)
Solution:

Let at equilibrium, the two balls will be at A and B OA = OB = l and AB = x

Also, let the angle made by each thread with the vertical be d.

Three forces act on each ball at equilibrium

1. weight of the ball mg,

2. tension on thread T

3. repulsive force F acting between the balls.

Class 12 Physics Unit 1 Electrostatics Chapter 2 Electric Field example 14 Two identical balls

Here, Tsinθ = F = \(\frac{q^2}{x^2}\); Tcosd = mg

∴ \(\tan \theta=\frac{q^2}{m g x^2} \quad\)

or, \(\sin \theta=\frac{q^2}{m g x^2}\) [∵ d is small, and = sinθ]

or, \(\frac{x}{2 l}=\frac{q^2}{m g x^2}\)

∴ \(x=\left(\frac{2 q^2 l}{m g}\right)^{1 / 3}\)

Example 15. Due to cosmic shower same amount of cations +q are stored in the atmosphere of the earth and the moon. If the gravitational attraction between the Earth and the moon is exactly equal to the electric repulsion between them. Then find the value of charge q stored in their atmosphere.
Solution:

Let the mass of the moon and the earth be Mm and Me respectively and the distance between them be r.

The repulsive force between them = \(\frac{q \times q}{r^2}=\frac{q^2}{r^2}\) and gravitational attractive force = \(=\frac{G M_e M_m}{r^2}\)

According to the problem,

⇒ \(\frac{q^2}{r^2}=\frac{G M_e M_m}{r^2}\)

or, \(q=\sqrt{G M_e M_m} \text { esu }\)

Example 16. The value of the electric field at a region of space is given by, E = Ar where A = 100 V.m-2 and r = distance (in m) from the origin inside the electric field. Find the amount of charge enclosed in a sphere of radius 20 cm centred at the origin. Given, \(\frac{1}{4 \pi \epsilon_0}=9 \times 10\) \(\frac{1}{4 \pi \epsilon_0}=9 \times 10^9 \mathrm{~N} \cdot \mathrm{m}^2 \cdot \mathrm{c}^{-2}\)
Solution:

We know, \(E=\frac{1}{4 \pi \epsilon_0} \cdot \frac{q}{r^2}\)

Again according to the problem,

E = Ar = 100 r V.m-1

∴ \(100 r=9 \times 10^9 \cdot \frac{q}{r^2} \quad \text { or, } 100 r^3=9 \times 10^9 \times q\)

or, \(q=\frac{100 \times\left(20 \times 10^{-2}\right)^3}{9 \times 10^9}\)

= 888.8 x 10-13 C

Electric Field at a Point Near a Charged Conductor:

Suppose, a spherical conductor of radius r placed in a medium of permittivity e has +Q amount of charge. The electric field is to be calculated at a point P close to the sphere.

If we place a unit positive charge at P, the force acting on it gives the electric field at P. It can be proved that to determine the electric field at an external point due to a charged spherical conductor, we can assume that the whole charge of the sphere is concentrated at the centre of the sphere.

So the distance between the charge concentrated at the centre O of the sphere and the unit positive charge at P may be taken to be equal to the radius of the sphere.

The electric field at P

Class 12 Physics Unit 1 Electrostatics Chapter 2 Electric Field Spherical conductor of radious

⇒ \(E=\frac{Q}{4 \pi \epsilon r^2}=\frac{Q}{4 \pi \kappa \epsilon_0 r^2}\) [K = dielectric constant of the medium]

⇒ \(\frac{4 \pi r^2 \sigma}{4 \pi \kappa \epsilon_0 r^2}\) [.. Q = 4πr2σ]

∴ \(E=\frac{\sigma}{\kappa \epsilon_0}\)

In vector form, \(\vec{E}=\frac{\sigma}{\kappa \epsilon_0} \hat{n}\)

where \(\hat{n}\) is the unit vector normal in the direction of \(\vec{E}\).

It may be noted that the electric field E does not depend on the radius of the sphere.

So the above equation applies not only to a spherical conductor but also to a conductor of any shape.

Electrostatics

Electric Field Electric Field Lines Or Electric Lines Of Force

Electric field intensity acts on every point of an electric field. This intensity has a definite magnitude and direction. But the magnitude and direction is different at different points.

If a free isolated unit positive charge is placed at a point in an electric field, it will experience a force along the tangent of the electric field line at that point and will move under its influence.

The path described by this unit’s positive charge is called the electric field line.

The direction of the force on it at that point gives the direction of the line. The direction is indicated by an arrowhead on the field line.

The tangent at any point on a field line gives the direction of the electric field intensity at that point.

Remember that the lines have no real existence. Scientist FaraDay introduced these imaginary lines to explain attraction and repulsion between two charged bodies and mentioned some definite properties of these lines

Properties of electric field fines:

  1. Electric field lines start from a positive charge and end on a negative charge. There is no electric field line inside a conductor For an isolated positive charge, the lines start from it and diverge to infinity and for an isolated negative charge, the lines of force come from infinity and converge on the charge.
  2. Two field lines can never intersect Because if they do so, then two tangents could be drawn at the point of intersection, which would indicate two directions of the electric field intensity at a single point But this is not possible.
  3. Electric field lines are always normal to the surface of a charged conductor.
  4. The electric field lines tend to contract longitudinally This phenomenon explains the attraction between two unit charges.
  5. The lines tend to repel one another laterally. This explains the repulsion between two like charges.
  6. At the two ends of each field line, there should be equal and opposite charges.
  7. No line starts from a conductor and ends on it From this fact it may be concluded that there are no lines inside a low conductor
  8. A region of closely spaced field lines indicates a strong electric field; sparsely spaced lines indicate a region of weak electric field.

Maps Of field lines: Maps of field lines are shown below for a few special cases.

An isolated positive charge: In this case, the field lines are directed away from the charge and are arranged uniformly. If they are drawn backwards, they meet at the centre of the charge

Class 12 Physics Unit 1 Electrostatics Chapter 2 Electric Field An isolated positive charge

An isolated negative charge: In this case, the field lines are directed towards the charge and arranged uniformly. If the lines are extended, they meet at the centre of the charge. The lines are similar to those but are oppositely directed.

Class 12 Physics Unit 1 Electrostatics Chapter 2 Electric Field An isolated negative charge

Two equal but opposite charges: Here the lines start from the positive charge and some of them end at the negative charge. Due to the longitudinal contractive tendency of the lines, two opposite charges attract each other. This mapping of the lines is similar to that of a bar magnet.

Class 12 Physics Unit 1 Electrostatics Chapter 2 Electric Field Two equal but opposite charges

Two equal and similar charges: Here the field lines starting from the charges repel one another and travel off to an infinite distance. At the neutral point, denoted by the x sign, the resultant intensity is zero. At this point, due to both charges, the intensities are equal and opposite. As the lines repel laterally (i.e., sidewise), repulsion takes place between the two charges.

Class 12 Physics Unit 1 Electrostatics Chapter 2 Electric Field Two equal and similar charges

Uniform electric field: The field lines of a form electric field are represented by parallel equidistant straight lines.

It may be noted that all the electric fields are non-uniform.

Class 12 Physics Unit 1 Electrostatics Chapter 2 Electric Field Uniform electric field

Electric intensity in terms of field lines: According to scientist Maxwell, the number of lines emanating from a charge q placed in a medium of permittivity

⇒ \(\epsilon \text { is } \frac{q}{\epsilon} \text { (in CGS system } \frac{4 \pi q}{k} \text { ) }\).

Now imagine a sphere of radius r having a charge q at its centre. L field lines will cross the surface area of the sphere normally. Since the surface area of the sphere is 4πr2, the number of field lines passing normally per unit area of the sphere = \(\frac{q / \epsilon}{4 \pi r^2}\)

= \(\frac{q}{4 \pi \epsilon r^2}\)

But due to the charge q, the electric field at any point on the surface of the sphere = \(\frac{q}{4 \pi \epsilon r^2}\). So we can say that the electric field at a point is equal, to the number of field lines passing mally through the unit area surrounding that point.

Class 12 Physics Unit 1 Electrostatics Chapter 2 Electric Field Electric intensity in terms of field lines

Electrostatics

Electric Field Electric Dipole

Definition:

An electric dipole is a combination of two equal but opposite point charges separated by a small distance.

Example: The molecules of water (H2O), ammonia (NH3), hydrochloric acid (HCl), carbon dioxide (CO2) etc. are electric dipoles. A water molecule and a hydrochloric acid molecule

Class 12 Physics Unit 1 Electrostatics Chapter 2 Electric Field electric dipole molecule of water and molecule of hydrochloric acid

The total charge of the electric dipole is zero (as, +q- q = 0 ), but the field of the electric dipole is not zero.

Dipole moment: The moment of an electric dipole is the product of the magnitude of either charge (q) and the distance (21) between them.

According to, dipole moment, p = q.21…..(1)

The electric dipole moment is a vector quantity. It is directed along the axis of the dipole (the line joining the two charges) from the negative to the positive charge.

Class 12 Physics Unit 1 Electrostatics Chapter 2 Electric Field Electric dipole moment is a vector quantity

The vector form of dipole moment is \(\vec{p}=2 q \vec{l}\)

Unit and dimension: in the CGS system the unit of dipole moment is state. cm and in SI it is C.m . Its dimension is [p] = LTI

Field Intensity at a Point on the Axis of a Dipole:

Let the two charges -q and +q separated by a distance of 2l form an electric dipole AB. AB = 2l and dipole moment, p = 2lq. Electric field intensity at a point P situated on the axis of the dipole is to be calculated. This position of P concerning the dipole is called the end-on position or axial position.

Class 12 Physics Unit 1 Electrostatics Chapter 2 Electric Field end-on position or axial position

Let the distance of the point P from the centre 0 of the dipole be r.

Suppose, the dipole is placed in a medium of dielectric constant k. The permittivity of air medium or vacuum is ∈0.

Now, field intensity at P due to the charge +q

⇒ \(E_1=\frac{q}{4 \pi \kappa \epsilon_0(r-l)^2} ; \text { along } \overrightarrow{O P}\)

Again, field intensity at P due to the charge -q,

⇒ \(E_2=\frac{q}{4 \pi \kappa \epsilon_0(r+l)^2} ; \text { along } \underset{;}{\overrightarrow{P O}}\)

As and E2 acting in opposite directions and E1> E2, therefore, resultant field intensity at P,

E = E1-E2

⇒ \(\frac{q}{4 \pi \kappa \epsilon_0}\left[\frac{1}{(r-l)^2}-\frac{1}{(r+l)^2}\right]\)

⇒ \(\frac{q}{4 \pi \kappa \epsilon_0} \cdot \frac{4 r l}{\left(r^2-l^2\right)^2}\)

⇒ \(\frac{2 p r}{4 \pi \kappa \epsilon_0\left(r^2-l^2\right)^2}\) [… p = 2lp] ….(1)

The direction of E is along \(\vec{OP}\)

For air or vacuum, K = 1

∴ \(E=\frac{2 p r}{4 \pi \epsilon_0\left(r^2-l^2\right)^2}\)…(2)

The vector form of equation (1) is

⇒ \(\vec{E}=\frac{2 \vec{p} r}{4 \pi \kappa \epsilon_0\left(r^2-l^2\right)^2}\)

The corresponding CGS expression of equation (2) is

⇒ \(E=\frac{2 p r}{\left(r^2-l^2\right)^2}\)

Special case:

If the point P is far away from the centre O of the dipole i.e., if r >> l, then neglecting l1 in comparison with r2, we get from equations (1) and (2) respectively,

⇒ \(E=\frac{1}{4 \pi \kappa \epsilon_0} \cdot \frac{2 p}{r^3}\)….(3)

and \(E_v=\frac{1}{4 \pi \epsilon_0} \cdot \frac{2 p}{r^3}\) …(4)

For this end-on position, \(\vec{p}\) and \(\vec{E}\) are along the same direction.

Field Intensity at a Point on the Perpendicular Bisector of a Dipole:

Any line passing through the centre of the dipole and perpendicular to its axis is called its perpendicular bisector.

Let the two charges -q and +q separated by a distance of 21 form an electric dipole AB. AB = 21 and dipole moment, p = 2lq. Electric field intensity at a point P situated on the perpendicular bisector of the dipole is to be calculated.

This position of P concerning the dipole is called broadside on or equatorial position

Let the distance of the point P from the centre of the tire dipole be r.

Suppose, the dipole is placed in a medium of dielectric constant K.

The permittivity of air medium or vacuum is ∈0.

Now, field intensity at P due to the charge +q,

Class 12 Physics Unit 1 Electrostatics Chapter 2 Electric Field broadsideon or equatorial position

⇒ \(E_1=\frac{1}{4 \pi \kappa \epsilon_0} \cdot \frac{q}{(B P)^2} ; \text { along } \overrightarrow{B P}\)

Again, field intensity at P due to the charge -q,

⇒ \(E_2=\frac{1}{4 \pi \kappa \epsilon_0} \cdot \frac{q}{(A P)^2} \text {; along } \overrightarrow{P A}\)

Since BP = AP

∴ \(E_1=E_2 \text { and } \angle P A B=\angle P B A=\theta \text { (say) }\)

If PR be parallel to BA, then

∠QPR = ∠PBA = 0 and ∠RPA = ∠PAB = 0

Now, the field intensity E1 is resolved into two perpendicular components along PR and PT (extended OP). The components are respectively E1cosθ and E1sinθ.

Similarly, the field intensity E2 is resolved into two perpendicular components along PR and PO. The components are respectively E2cosθ and E2sinθ.

The sine components of PT and PO being equal and opposite balance each other. So the resultant field intensity at P is

E = E1cosθ + E2cosθ

= 2E1 cosθ [∵ E1 = E2]

⇒ \(\frac{2}{4 \pi \kappa \epsilon_0} \cdot \frac{q}{(BP)^2} \cos \theta\)

⇒ \(\frac{2}{4 \pi \kappa \epsilon_0} \cdot \frac{q}{\left(r^2+l^2\right)} \cdot \frac{l}{\sqrt{r^2+l^2}}\) [∵ OP = r and OB = OA = l]

⇒ \(E=\frac{l}{4 \pi \kappa \epsilon_0} \cdot \frac{p}{\left(r^2+l^2\right)^{3 / 2}}\) [∵ p = 2lp]…(1)

The direction of E is along \(\vec{PR}\)

In air or vacuum, K = 1

∴ \(E=\frac{1}{4 \pi \epsilon_0} \cdot \frac{p}{\left(r^2+l^2\right)^{3 / 2}}\) ….(2)

The vector form of equation (1) is

⇒ \(\vec{E}=-\frac{1}{4 \pi \kappa \epsilon_0} \cdot \frac{\vec{p}}{\left(r^2+l^2\right)^{3 / 2}}\)

The corresponding CGS expression of equation (2) is

⇒ \(E=\frac{p}{\left(r^2+l^2\right)^{3 / 2}}\)

Special case:

If the point P is far away from the centre O of the dipole i.e., if r >> l, then neglecting l2 in comparison with r2, we get from equations (1) and (2) respectively,

⇒ \(E=\frac{1}{4 \pi \kappa \epsilon_0}: \frac{p}{r^3}\)….(3)

and \(E=\frac{1}{4 \pi \epsilon_0} \cdot \frac{p}{r^3}\)…(3)

For this broadside-on position, \(\vec{p}\) and \(\vec{E}\) are oppositely directed.

It is to be noted that, when point P is far away from the centre of the dipole, the electric field intensity at a point in the axial position is double the electric field intensity at a point in the equatorial position.

Field Intensity at any Point due to an Electric Dipole:

Suppose, the point P is situated at a distance r from the centre O of the dipole and the line OP makes an angle 0 with the axis of the dipole. The dipole moment \(\vec{p}\) acts along \(\vec{AB}\) and the length of the dipole is negligible in comparison to r.

Class 12 Physics Unit 1 Electrostatics Chapter 2 Electric Field Field Intensity at any Point due to an electric dipole

⇒ \(\vec{p}\) is resolved into two perpendicular components:

Pcosθ; along \(\vec{OP}\)

psinθ; along perpendicular to \(\vec{OP}\)

Hence, point P is situated on the axial line of a dipole of moment pcosθ, and the equatorial line of a dipole of moment psinθ.

Let \(\vec{E}_1 \text { and } \vec{E}_2\) be the Intensities at P due to the components pcosθ and psinθ respectively, and the resultant intensity at P be \(\vec{E}\)

So, \(\vec{E}=\vec{E}_1+\vec{E}_2\)

∴ \(E^2=E_1^2+E_2^2 \quad\)

or, \(E=\sqrt{E_1^2+E_2^2}\)

Suppose, the dipole is placed in a medium of dielectric constant K. The permittivity of air medium or vacuum is ∈0.

From equation (3)

⇒ \(E_1=\frac{1}{4 \pi \kappa \epsilon_0} \cdot \frac{2 p \cos \theta}{r^3} \text {; along } \overrightarrow{P T}\)

and \(E_2=\frac{1}{4 \pi \kappa \epsilon_0} \cdot \frac{p \sin \theta}{r^3} \text {; along } \overrightarrow{P S}\)

∴ Resultant intensity at P,

⇒ \(E=\sqrt{\left(\frac{1}{4 \pi \kappa \epsilon_0}\right)^2 \cdot\left(\frac{p}{r^3}\right)^2\left[4 \cos ^2 \theta+\sin ^2 \theta\right]}\)

or, \(E=\frac{1}{4 \pi \kappa \epsilon_0} \cdot \frac{p}{r^3} \sqrt{3 \cos ^2 \theta+1}\)…(1)

The direction of E is along \(\vec{PR}\).

Suppose, \(\angle T P R=\phi\)

∴ \(\tan \phi=\frac{E_2}{E_1}=\frac{1}{2} \tan \theta\)…(2)

For air or vacuum, k = 1

∴ \(E=\frac{1}{4 \pi \epsilon_0} \cdot \frac{p}{r^3} \sqrt{3 \cos ^2 \theta+1}\)….(3)

The corresponding CGS expression of equation (3) is

⇒ \(E=\frac{p}{r^3} \sqrt{3 \cos ^2 \theta+1}\)

Special cases:

1. End-on position: If the point P is on the end-on position, i.e., on the axis of the dipole, θ = 0° or cosθ = 1.

Therefore, from equation (1) we get

⇒ \(E=\frac{1}{4 \pi \kappa \epsilon_0} \cdot \frac{p}{r^3} \sqrt{3+1}=\frac{1}{4 \pi \kappa \epsilon_0} \cdot \frac{2 p}{r^3}\)

2. Broadside-on position or equational position: If the point P is on the broadside-on position, i.e., on the perpendicular bisector of the dipole, 0 = 90° or, cosθ = 0.

Therefore, from equation (1) we get,

⇒ \(E=\frac{1}{4 \pi \kappa \epsilon_0} \cdot \frac{p}{r^3} \cdot \sqrt{0+1}=\frac{1}{4 \pi \kappa \epsilon_0} \cdot \frac{p}{r^3}\)

Torque Acting on an Electric Dipole in a Uniform Electric Field:

Consider an electric dipole consisting of charges -q at A and +q at B separated by a distance of 2l. So dipole moment, p = q 21. It is a vector quantity and its direction is from negative to positive charge along the axis of the dipole.

Class 12 Physics Unit 1 Electrostatics Chapter 2 Electric Field Torque Acting on an Electric Dipole in a uniform electric field

Suppose, the dipole makes an angle 6 with a uniform electric field of strength \(\vec{E}\). The force on charge +q is qE, along the direction of \(\vec{E}\) and the force on charge -q is qE, opposite to the direction of \(\vec{E}\).

These forces are equal in magnitude but opposite in direction.

Therefore, the net force on the dipole, \(\vec{F}_{\text {net }}=q \vec{E}-q \vec{E}=0\);

i.e., the net force on an electric dipole in a form electric field is zero.

Since these two forces are equal in magnitude, opposite in direction and act at different points, they constitute a couple. If the magnitude of the torque is \(\tau\), then

⇒ \(\tau\) = either force x perpendicular distance between the lines of action of the two forces

= qE x AC

= qE x 2lsinθ

= q x 2l x Esinθ

or, \(\tau\) = pEsinθ….(1)

The torque acting on the dipole tends to bring it along the direction of the electric field.

Special cases:

1. When θ = 90°, the magnitude of the torque becomes maximum, i.e., \(\tau\) = pE. In this case, the electric dipole is in stable equilibrium.

Now, if E = 1, \(\tau\) = p = dipole moment It provides an alternative definition of electric dipole moment

The moment of an electric dipole is defined as the torque acting on it when it is placed at right angles to a uniform electric field of unit strength.

2. When θ = 0° or 180° , the magnitude ofthe torque is minimum, i.e., r = 0. In this case, the electric dipole is in an unstable equilibrium.

Torque in vector form: Torque \(\vec{\tau}\) acting on a dipole of moment \(\vec{p}\) when placed in a uniform electric field \(\vec{E}\) is given in vector form as,

⇒ \(\vec{\tau}=\vec{p} \times \vec{E}\)….(2)

According to the rule of the cross product, \(\vec{\tau}\) is perpendicular to the plane containing \(\vec{p}\) and \(\vec{E}\)

Class 12 Physics Unit 1 Electrostatics Chapter 2 Electric Field Torque in vector form

Electrostatics

Electric Field Numerical Examples

Example 1. How many field lines would emanate from a 1 C positive charge placed in a vacuum?
Solution:

The number of lines emanated

⇒ \(\frac{1}{\epsilon_0}=\frac{1}{8.854 \times 10^{-12}}=1.129 \times 10^{11}\)

= 1.129 X 1011

Example 2. A straight rod of length Z placed in a vacuum is charged uniformly with an amount q of charge. Calculate the electric field intensity in SI at a point on the axis of the rod at a distance x from Its nearer end. Write the result in the CGS system also.
Solution:

The length of the rod AB is Z. Let P be a point at a distance x from the end B. Since the rod is uniformly charged with q, the linear charge density of the rod is

⇒ \(\lambda=\frac{q}{l}\).

Class 12 Physics Unit 1 Electrostatics Chapter 2 Electric Field A straight rod of length

Consider a small length dr of the rod at a distance r from P.

Charge of this portion of length dr,

⇒ \(d q=\lambda d r=\frac{q}{l} d r\)

Electric field intensity at P due to this charge dq,

⇒ \(d E=\frac{1}{4 \pi \epsilon_0} \cdot \frac{\frac{q}{l}}{r^2} d r=\frac{1}{4 \pi \epsilon_0} \cdot \frac{q}{l} \frac{d r}{r^2}\)

Therefore, field intensity at P due to the whole charged rod,

⇒ \(E=\int d E=\frac{q}{4 \pi \epsilon_0 l} \int_x^{l+x} \frac{1}{r^2} d r=\frac{q}{4 \pi \epsilon_0 l} \cdot\left[-\frac{1}{r}\right]_x^{l+x}\)

⇒ \(\frac{q}{4 \pi \epsilon_0 l} \cdot\left[\frac{1}{x}-\frac{1}{l+x}\right]\)

⇒ \(\frac{q}{4 \pi \epsilon_0 l} \cdot \frac{l}{x(l+x)}=\frac{1}{4 \pi \epsilon_0} \cdot \frac{q}{x(l+x)} \text {; along } \overrightarrow{B P}\)

In CGS system, replacing \(\epsilon_0 \text { by } \frac{1}{4 \pi}\), the electric field intensity becomes

⇒ \(E=\frac{q}{x(l+x)} ; \text { along } \overrightarrow{B P}\)

Example 3. Two point charges are placed on the y-axis at y = +a and y = -a in vacuum. The magnitude of each charge is q. Determine the electric field intensity at P(x, 0) on the x-axis.
Solution:

Suppose, electric field intensity at the point P due to the charges at A and B are E1 and E2, respectively.

Class 12 Physics Unit 1 Electrostatics Chapter 2 Electric Field example 3 the electric field Intensity

∴ \(E_1=\frac{1}{4 \pi \epsilon_0} \cdot \frac{q}{A P^2} ; \text { along } \overrightarrow{P C}\)

⇒ \(\frac{1}{4 \pi \epsilon_0} \cdot \frac{q}{a^2+x^2} \text {; along } \overrightarrow{P C}\)

⇒ \(E_2=\frac{1}{4 \pi \epsilon_0} \cdot \frac{q}{B P^2} ; \text { along } \overrightarrow{P D}\)

⇒ \(\frac{1}{4 \pi \epsilon_0} \cdot \frac{q}{a^2+x^2} \text {; along } \overrightarrow{P D}\)

∴ \(E_1=E_2 \text { (in magnitude) }=E \text { (say) }\)

Both E1 and E2 are resolved into two perpendicular components. It is seen that the sine components are equal and opposite, and balance each other. But the cosine components are added up and they act In the same direction.

∴ The field strength at P

⇒ \(2 E \cos \theta=2 \times \frac{1}{4 \pi \epsilon_0} \cdot \frac{q}{\left(a^2+x^2\right)} \cdot \frac{x}{\sqrt{a^2+x^2}}\)

⇒ \(\frac{1}{4 \pi \epsilon_0} \cdot \frac{2 q_x}{\left(a^2+x^2\right)^{3 / 2}}\)

Example 4. An electric dipole placed in a vacuum is formed by two equal but opposite charges each of magnitude 1μC separated by a distance of 2 cm. Calculate the electric field intensities in the following cases:

  1. At a point on the axis of the dipole situated at a distance 60 cm away from its centre.
  2. At a point on the perpendicular bisector of the dipole situated at a distance 60 cm away from its centre.

Solution:

Moment of the electric dipole,

p = q 21 = 1 x 10-6 X 2 X 10-6

= 2 x 10-8 C.m

1. Electric field intensity on the axis of the dipole,

⇒ \(E_1=\frac{1}{4 \pi \epsilon_0} \cdot \frac{2 p}{r^3}=\frac{9 \times 10^9 \times 2 \times 2 \times 10^{-8}}{(0.6)^3}\)

= 1666.6N.C-1; along the axis

2. Electric field intensity on the perpendicular bisector of the dipole

⇒ \(E_2=\frac{1}{4 \pi \epsilon_0} \cdot \frac{p}{r^3}=\frac{1}{2} E_1\)

= 833.3 N.C-1; parallel to the axis of the dipole

Example 5. Three charges q, -2q and q are placed at three vertices of an equilateral triangle. Determine the equivalent dipole moment of the system
Solution:

The given system is a combination of two dipoles at an angle 60° with each other.

Class 12 Physics Unit 1 Electrostatics Chapter 2 Electric Field example 5 Three charges q, -2q and q

Let the dipole moment of one dipole is p1 and that of another dipole is p2, the length of each side of the equilateral triangle is a.

∴ Equivalent dipole moment,

p = p1cos30° + p2cos30° [∵ sine components cancel each other]

= 2qacos30° [∵ p1 = p2 = qa]

⇒ \(2 q a \times \frac{\sqrt{3}}{2}=\sqrt{3} q a\)

Example 6. Electric field intensities at two axial points of an electric dipole at distances 5 cm and 10cm from its centre are 2.5 x 104 N.C-1 and 2 X 103 N.C-1 respectively. The dipole is placed In the air. Determine the length of the dipole.
Solution:

Suppose, the length of the electric dipole = 21.

We know,

⇒ \(E=\frac{1}{4 \pi \epsilon_0} \cdot \frac{2 p r}{\left(r^2-l^2\right)^2}\)

According to the question

⇒ \(2.5 \times 10^4=9 \times 10^9 \cdot \frac{2 p \times 0.05}{\left[(0.05)^2-l^2\right]^2}\)…(1)

and \(2 \times 10^3=9 \times 10^9 \cdot \frac{2 p \times 0.1}{\left[(0.1)^2-l^2\right]^2}\)….(2)

Dividing (1) by (2) we get,

⇒ \(1.25 \times 10=\frac{\left[(0.1)^2-l^2\right]^2 \times 0.05}{\left[(0.05)^2-l^2\right]^2 \times 0.1}\)

or, \(25=\frac{\left[(0.1)^2-l^2\right]^2}{\left[(0.05)^2-l^2\right]^2} \quad\)

or, \(5=\frac{(0.1)^2-l^2}{(0.05)^2-l^2}\)

or, 2l = 0.05m

∴ The length of the electric dipole = 0.05 m.

Example 7. An electric dipole is formed by two equal but opposite charges, each of magnitude 1μC, separated by a distance of 5 cm. What is the magnitude of the torque required to place the dipole at right angles to an electric field of Intensity 3 x 10s N C-1?
Solution:

Here,

p = q X 2l = 1 x 10-6 x 5 x 10-2

= 5 x 10-8 C.m

E = 3 X 105 N – C-1 and θ = 90°

∴ The required torque,

⇒ \(\tau\) = pEsinθ

= 5 x 10-8 x 3 x 105 x sin90°

= 0.015 N.m

Example 8. An electric dipole is formed by placing charges ±20 x 10-6 C at a distance of 2 min. Calculate the electric field at a point on the perpendicular bisector of the axis of the dipole situated at a distance of 10 cm from the midpoint of the dipole.
Solution:

Electric field, \(E=\frac{1}{4 \pi c_0} \cdot \frac{p}{\left(r^2+l^2\right)^{3 / 2}}\)

[here, l = 1 x 10-3m , p = q x 21 = 20 X 10-6 X 2 X 10-3 =40 X 10-9 cm]

∴ \(E=\frac{9 \times 10^9 \times 40 \times 10^{-9}}{\left[(10 \times 10)^{-2}+\left(1 \times 10^{-3}\right)^2\right]^{3 / 2}}\)

= \(\frac{360}{\left(10^{-2}+10^{-6}\right)^{3 / 2}}\)

= 36 X 10-4 N.C-1

Electrostatics

Electric Field Area vector Electric flux And Gauss Theorem

Area vectors: In different cases of physics, the surface area is treated as a vector. The magnitude of the area is represented by the length of the vector and the direction of the area vector is given by the outward drawn normal to the closed surface.

Suppose, dS is a small surface area on the surface S. A normal is drawn in the outward direction on the surface at dS. So \(d \vec{s}\) is an area vector.

Class 12 Physics Unit 1 Electrostatics Chapter 2 Electric Field Area vector

Electric flux: Electric flux linked with a surface gives an idea of the number of field lines passing through the surface. The number of electric field lines passing normally through a surface Is called the electric flux through the surface. It is denoted by the symbol Ω. It is a scalar quantity.

Let us consider a small area element \(d \vec{S}\) on a surface S in an electric field of inequality \(\vec{E}\).

Let θ be the angle between E and \(d \vec{S}\).

Class 12 Physics Unit 1 Electrostatics Chapter 2 Electric Field Electric flux

Therefore, electric flux passes through the surface \(d \vec{S}\).

⇒ \(d \phi=\vec{E} \cdot d \vec{S}=E d S \cos \theta\)

= (Ecosθ)dS

= EndS….(1)

Here, En = Ecosθ = normal component of electric field intensity.

As the whole surface is the sum of a large number of such small-area elements, the electric flux passing through the surface S is given by

⇒ \(\phi=\int_S \vec{E} \cdot d \vec{S}=\int_S E_n d S=\int_S E d S \cos \theta\)….(2)

where \(\int_S\) represents the integral taken over the whole surface.

Special cases:

1. If an electric field \(\vec{E}\) is normal to a surface, then 0 = 0° . So the electric flux linked with the surface,

⇒ \(\phi=\int_S E d S \cos 0^{\circ}=E S \text { (maximum) }\)

2. If an electric field \(\vec{E}\) Is parallel to a surface, then 0 = 90°.

So the electric flux linked with the surface,

⇒ \(\phi=\int_S E d S \cos 90^{\circ}=0\)

Closed surface:

Now, let us consider a definite volume enclosed by a dosed surface S. Naturally, the electric flux linked with this surface S will be,

Class 12 Physics Unit 1 Electrostatics Chapter 2 Electric Field Closed surface

⇒ \(\phi=\oint_S \vec{E} \cdot d \vec{S}=\oint E d S \cos \theta\)….(3)

Here, the symbol \(\oint_S\) stands for integration across all the surface elements d\(\vec{S}\) which has different directions around the entire closed surface S.

Moreover, \(\vec{E}\) may be different at different points on the surface, So, none of 000 and E are constant when the entire surface is taken into account.

As a result, the integral in equation (3) is of ten very hard to compute. However, specific symmetries in connection with some special systems help us to tackle the integral effectively.

Positive electric flux:

The electric flux linked with a closed surface is taken as positive when the electric field vector is outwardly directed. Here, the volume acts as a source of field lines.

Class 12 Physics Unit 1 Electrostatics Chapter 2 Electric Field Positive electric flux

Negative electric flux:

The electric flux linked with a closed surface is taken as negative when the electric field vector is Inwardly directed. Here, the volume acts as a sink of field lines.

Unit and dimension of electric flux: In SI, a unit of electric flux is N.m2.C-1 or V m

Dimension of electric flux

= dimension of electric intensity x dimension of area

= MLT-3l-1 x L2

= ML3T-3T-1

Solid Angle:

If a surface is placed at any distance in front of a point, then it subtends a solid angle at that point. In this surface of area, S subtends a solid angle H at the point O.

Class 12 Physics Unit 1 Electrostatics Chapter 2 Electric Field Solid Angle

Unit solid angle:

The solid angle subtended by an area r2 on a sphere of radius r, at the centre of the sphere, is called a unit solid angle. This unit of solid angle is called steradian (sr).

The solid angle subtended by a surface of area S on a sphere of radius r, at the centre of the sphere, is given by,

⇒ \(\Omega=\frac{S}{r^2}=\frac{\text { area of this spherical surface }}{\text { (radius) }^2}\)

The dimension of Ω

⇒ \([\Omega]=\frac{\text { dimension of area }}{(\text { dimension of radius })^2}=\frac{\mathrm{L}^2}{\mathrm{~L}^2}\)

= 1

So, the solid angle is a dimensionless quantity. Steradian is a dimensionless unit.

The solid angle at the centre of a sphere of radius r due to the entire surface of the sphere

⇒ \(\frac{\text { area of the entire surface }}{r^2}\)

⇒ \(\frac{4 \pi r^2}{r^2}=4 \pi\)

If the surface is a closed surface (not necessarily a sphere), then it also subtends a solid angle of 4π at any of its internal points.

Now, let us take a surface dS at a distance r from point O. In general, dS is not normal to the radius vector \(\vec{r}\). If 000 be the angle between d\(\vec{S}\) and \(\vec{r}\), then the component of dS along the normal to r = dScosθ.

So, the solid angle subtended at O by the surface dS is,

⇒ \(d \omega=\frac{d S \cos \theta}{r^2}\)

Obviously, if a closed surface of any shape surrounds the point O, then the solid angle subtended at O by that closed surface,

⇒ \(\oint d \omega=\oint \frac{d S \cos \theta}{r^2}=4 \pi\)

Statement and Proof of Gauss’ Theorem:

Statement: The net electric flux linked with a closed surface is \(\frac{1}{∈}\) times the net charge within the surface.

Mathematically, \(\phi=\oint_S \vec{E} \cdot d \vec{S}:=\frac{q}{\epsilon}\)….(1)

Here q is the total charge enclosed by the surface and e is the permittivity of the medium.

For vacuum, \(\phi=\oint_S \vec{E} \cdot d \vec{S}=\frac{q}{\epsilon_0}\)

Proof:

Let S be a closed surface enclosing +q charge situated at O. P is a point situated on the surface and dS is a small area on the surface surrounding P. Suppose, OP = r.

Electric field intensity at P due to the charge +q,

⇒ \(E=\frac{1}{4 \pi \epsilon} \cdot \frac{q}{r^2}\)

Class 12 Physics Unit 1 Electrostatics Chapter 2 Electric Field Statement and Proof of Gauss' Theorem

Let \(\vec{PN}\) be the normal at P on dS and 0 be the angle between \(\vec{E}\) and \(\vec{PN}\).

∴ The electric flux through the area dS

⇒ \(\vec{E} \cdot d \vec{S}=E d S \cos \theta\)

⇒ \(\frac{1}{4 \pi \epsilon} \cdot \frac{q}{r^2} d S \cos \theta\)

= \(\frac{q}{4 \pi \epsilon} \cdot \frac{d S \cos \theta}{r^2}\)

= \(\frac{q}{4 \pi \epsilon} \cdot d \omega\)

How, \(d \omega=\frac{d S \cos \theta}{r^2}\) = solid angle subtended at O by the area dS.

Therefore, net flux linked with the entire closed surface,

⇒ \(\phi=\oint_S \frac{q}{4 \pi \epsilon} d \omega\)

= \(\frac{q}{4 \pi \epsilon} \oint_S d \omega\)

= \(\frac{q}{4 \pi \epsilon} \cdot 4 \pi\)

= \(\frac{q}{\epsilon}\) [because a closed surface subtends a solid angle An at any point
within the surface]

If q1, q2 ….. qn be the charges lying within the closed surface and E1, E2,…, En be the corresponding electric fields, the net flux linked with the closed surface will be,

⇒ \(\phi=\oint_S \vec{E} \cdot d \vec{S}=\oint_S\left(\vec{E}_1+\vec{E}_2+\cdots+\vec{E}_n\right) \cdot d \vec{S}\)

⇒ \(\frac{1}{\epsilon}\left(q_1+q_2+\cdots+q_n\right)\)

= \(\frac{Q}{\epsilon}\)

where Q is the net charge inside the closed surface.

According to the nature of the charges (positive and negative), positive or negative signs are to be used in the sum of the charges.

Electric flux due to a charge lying outside a closed surface:

Let a charge of +q be placed at O outside the closed surface S. A cone of solid angle da) having its vertex at O intercepts small areas dS1 and dS2 on the closed surface. Let P be a point on dS1 and Q be another point on dS2. E1 and E2 are the electric field intensities at P and Q, respectively

Class 12 Physics Unit 1 Electrostatics Chapter 2 Electric Field Electric flux due to a charge lying outside a closed surface

Suppose, OP = r1 and OQ = r2.

Component of E1 along the normal to dS1 = E1cosθ1 and a component of E2 along the normal to dS2 = E2cosθ2. Since electric flux through dS1 directed inwards is negative, the flux through dS2 directed outwards is positive.

Therefore, electric flux through dS1 = -E1cosθ1dS1 and electric flux through dS2 = E2cosθ2dS2

So, net electric flux through dS1 and dS2

= -E1cosθ1dS1+E2cosθ2dS2

⇒ \(-\frac{1}{4 \pi \epsilon} \cdot \frac{q}{r_1^2} \cos \theta_1 d S_1+\frac{1}{4 \pi \epsilon} \cdot \frac{q}{r_2^2} \cos \theta_2 d S_2\)

⇒ \(-\frac{q}{4 \pi \epsilon} \cdot \frac{d S_1 \cos \theta_1}{r_1^2}+\frac{q}{4 \pi \epsilon} \cdot \frac{d S_2 \cos \theta_2}{r_2^2}\)

⇒ \(-\frac{q}{4 \pi \epsilon} \cdot d \omega+\frac{q}{4 \pi \epsilon} \cdot d \omega\)

= 0

Clearly, this result is in accordance with equation (1), because in this case, q = charge within the closed surface = 0.

Discussions:

  1. Only the charges inside a volume contribute to the electric flux linked with the surface enclosing that volume.
  2. The net charge q is the algebric sum of all the charges, positive or negative, inside the closed surface.
  3. If the net charge q is positive, the outward flux across the closed surface is greater than the inward flux; the enclosed volume acts as a source.
  4. If the net charge q is negative, the inward flux exceeds the outward flux; the enclosed volume acts as a sink.
  5. If the closed surface encloses equal amounts of positive and negative charges, then the net charge q is zero; consequently, the net flux linked with the surface is also zero.
  6. For Example, if a closed surface encloses an electric dipole, the net electric flux through it is zero.
  7. Charges outside the closed surface have no net contribution towards the electric flux linked with

Applications of Gauss’ Theorem:

Field intensity at a point due to a point charge: We have to calculate the electric field intensity at a point situated at a distance r from a point charge q.

Let us consider a spherical Gaussian surface of radius r having the point charge q at its centre. We can say from symmetry that electric field intensity due to a positive point charge is directed radially outwards.

So intensity at any point on the Gaussian surface is perpendicular to it Therefore, \(\vec{E}\) and d\(\vec{S}\) are parallel at each point.

Class 12 Physics Unit 1 Electrostatics Chapter 2 Electric Field Field intensity at a point due to a point charge

∴ \(\vec{E} \cdot d \vec{S}=E d S \cos 0^{\circ}=E d S\)

According to Gauss’ theorem,

⇒ \(\phi=\oint_S \vec{E} \cdot d \vec{S}=\oint_S E d S=\frac{q}{\epsilon}\) [eee = permittivity of the medium]

By symmetry, E is constant over the entire Gaussian surface.

⇒ \(\oint_S E d S=E \oint_S d S=E \cdot 4 \pi r^2\) [4πr² = surface area of the sphere]

∴ \(E \cdot 4 \pi r^2=\frac{q}{\epsilon}\)

or, \(E=\frac{q}{4 \pi \epsilon r^2}=\frac{q}{4 \pi \kappa \epsilon_0 r^2}\)….(1)

where K and eee0 are respectively the dielectric constant of the medium and the permittivity of the vacuum.

In vector form, \(\vec{E}=\frac{q}{4 \pi \kappa \epsilon_0 r^2} \hat{r}\)

where \(\hat{r}\) is a unit vector along the outward drawn normal.

Note that if the point charge is negative, q is replaced by -q.

Then E would be negative, meaning that it would be radially inwards.

Field intensity at a point due to a uniformly charged thin spherical shell:

Point outside the spherical shell:

Suppose, a thin spherical shell of radius R is charged uniformly with +q

Class 12 Physics Unit 1 Electrostatics Chapter 2 Electric Field In case offield outside a uniformly charged sphere

Electric field intensity at any external point P is to be calculated. Suppose, the distance of P from the centre O of the shell is r. Imagine a Gaussian spherical shell of radius r with centre at O.

We can say from symmetry that electric field intensity at each point on the surface of the spherical shell is equal and is directed along the outward drawn normal.

Suppose, the intensity at P is E. Electric flux linked with the imaginary surface of the spherical shell = E.4πr².

Since the Gaussian surface encloses the charge +q, the net flux according to Gauss’ theorem is \(\frac{q}{\epsilon}\)

∴ \(E \cdot 4 \pi r^2=\frac{q}{\epsilon}\)

or, \(E=\frac{1}{4 \pi \epsilon} \cdot \frac{q}{r^2}\)…(2)

In vector form, \(\vec{E}=\frac{1}{4 \pi \epsilon} \cdot \frac{q}{r^2} \hat{r}\)

where \(\hat{r}\) is a unit vector along the outward drawn normal.

Equation (2) shows that for an external point, a charged spherical shell behaves as though its total charge were concentrated at its centre.

Point inside the spherical shell:

The point P is situated inside the charged spherical shell. Imagine a Gaussian spherical shell of radius r with centre at O. Since the Gaussian surface does not enclose any charge (because the charge lies on the surface of the spherical shell), the flux linked with the imaginary surface of the spherical shell is given by,

Class 12 Physics Unit 1 Electrostatics Chapter 2 Electric Field Point inside the spherical shell

E.4πr² = 0 [by Gauss’ theorem]

or, E = 0

i.e., there exists no field intensity inside a charged spherical shell.

Field intensity at a point due to an infinitely long Straight charged conducting Wire:

Consider an infinitely long thin straight wire charged uniformly.

Class 12 Physics Unit 1 Electrostatics Chapter 2 Electric Field Field intensity at a point due to an infinitely long Straight charged conducting Wire

Let +A be the linear charge density, i.e., a charge per unit length of the wire. Electric field intensity is to be calculated at P at a distance r from the wire.

Imagine a cylinder of radius r and length Z with the wire as its axis. Point P lies on the surface of such a cylinder. The surface of this cylinder will act as the Gaussian surface. All the points on the curved surface of this cylinder are equidistant from the wire.

From symmetry, it can be said that the electric field intensity at each point on the curved surface of the imaginary cylinder is equal and normally outwards.

No electric flux is linked with the two flat circular faces of the cylinder as the direction of electric field E is parallel to these two faces (i.e., the component of the electric field along the normal to the two flat circular faces is zero).

So, electric flux linked with the curved surface of the Gaussian cylinder = E 2πrl.

Now, since the cylinder encloses an amount of charge λl, by Gauss’ theorem, electric flux linked with the Gaussian cylinder = \(\frac{\lambda l}{\epsilon}\)

∴ \(E \cdot 2 \pi r l=\frac{\lambda l}{\epsilon} \text { or, } E=\frac{1}{4 \pi \epsilon} \cdot \frac{2 \lambda}{r}\)….(3)

In vector form, \(\vec{E}=\frac{1}{4 \pi \epsilon} \cdot \frac{2 \lambda}{r} \hat{r}\)

where \(\hat{r}\) is a unit vector along the outward drawn normal

Field intensity at a point due to an infinite nonconducting uniformly charged plane lamina:

Consider infinite nonconducting uniformly a surface charged plane lamina having face charge density +σ. A point P is taken in front of the plane lamina, where the electric field intensity is to be determined.

A small cylinder of cross-sectional area dS, normal to the lamina and having equal lengths on either side of it, is imagined such that P and p’ lie on its flat circular faces. The surface of this cylinder will act as the Gaussian surface.

Since the lamina is infinite, the electric field intensity at each point of the two flat circular faces of the Gaussian cylinder is equal and normally outwards due to symmetry.

The electric flux through the curved surface of the Gaussian cylinder is zero, since the direction of E is parallel to the curved surface (i.e., the component of E along the normal to the curved surface at every point is zero).

Now, electric flux through the two flat circular faces of the Gaussian cylinder = E.2dS

Class 12 Physics Unit 1 Electrostatics Chapter 2 Electric Field Field intensity at a point due to an infinite nonconducting uniform charged plane lamina

Now, since the cylinder encloses a number of charge cards, by Gauss’ theorem, electric flux linked with the cylinder = \(\frac{\sigma d S}{\epsilon}\)

∴ \(E \cdot 2 d S=\frac{\sigma d S}{\epsilon} \quad \text { or, } E=\frac{\sigma}{2 \epsilon}=\frac{\sigma}{2 \kappa \epsilon_0}\)….(4)

In vector form, \(\vec{E}=\frac{\sigma}{2 \epsilon} \hat{n}=\frac{\sigma}{2 \kappa \epsilon_0} \hat{n}\)

where \(\hat{n}\) is the unit vector directed normally away from the plane.

Field intensity at a point due to a charged conductor:

Let C be a charged conductor. The electric field intensity, developed due to it, at an external point P is to be determined. The distance of P from the conductor is much less than the dimensions of the conducting body.

Now, we consider a right circular cylinder of cross-sectional area dS, normal to the surface of the conductor, one of whose flat circular faces contains the point P.

The surface of this cylinder serves as the Gaussian surface in this case. We assume that the cross-sectional area dS is small enough that E is constant over the flat circular face. From symmetry, tire electric field intensity Is normally outwards with respect to the surface dS.

Thus, electric flux across the flat circular face around P is Eds.

Class 12 Physics Unit 1 Electrostatics Chapter 2 Electric Field Field intensity at a point due to a charged conductor

The curved surface of the cylinder does not contribute to any electric flux, since the direction of E is parallel to the curved surface at every point (i.e., the component of E along the normal to the curved surface at every point is zero). Also, the flat circular face inside the conducting body has no contribution, because the electric field inside a conductor is zero.

The entire charge resides on the outer surface of the conductor. In general, the curvature is not uniform around the surface. As a result, the charge would be non-uniformly distributed.

Let +cr be the surface charge density on a small area of dS on the surface just in front of P.

Now, since the cylinder encloses a number of charge σ dS, by Gauss’ theorem, electric flux linked with the Gaussian cylinder

= \(\frac{\sigma d S}{\epsilon}\)

∴ \(E d S=\frac{\sigma d S}{\epsilon} \quad \text { or, } E=\frac{\sigma}{\epsilon}\)

In vector form, \(\vec{E}=\frac{\sigma}{\epsilon_0} \hat{n}\)

where \(\hat{n}\) is the unit vector normal to the surface in the outward direction.

Electrostatics

Electric Field Numerical Example

Example 1. An electric field \((2 \hat{i}+3 \hat{j}) \mathrm{N} \cdot \mathrm{C}^{-1}\) exists In a region. Calculate the electric flux linked with a square plate of side 0.5 m held parallel to

  1. yz -plane
  2. xy -plane.

Solution:

Electric field, \(\vec{E}=2 \hat{i}+3 \hat{j}\); so it lies on the xy -plane. Area of the square plate, S = (0.5)² = 0.25 m²

1. The vector representation of the plate parallel to the yz -plane is, \(\vec{S}=S \hat{i}=0.25 \hat{i}\)

∴ Electric flux linked with the square plate

⇒ \(\vec{E} \cdot \vec{S}=(2 \hat{i}+3 \hat{j}) \cdot(0.25 \hat{i})\)

= 0.5 N.m2.C-1.

2. The vector representation of the plate parallel to the xy -plane is, \(\vec{S}=S \hat{k}=0.25 \hat{k}\)

∴ Electric flux linked with the square plate

⇒ \(\vec{E} \cdot \vec{S}=(2 \hat{i}+3 \hat{j}) \cdot(0.25 \hat{k})\)

= 0.

Example 2. In vacuum,

  1. Find out the electric flux across, an area \(\vec{s}=10 \hat{j}\) placed in an electric flux \(\vec{E}=2 \hat{i}+4 \hat{j}+7 \hat{k}\)
  2. How much electricity? charges are to be placed at a point such that 4400 electric lines of force will emerge from that point.
  3. What will be the electric flux through any one of the faces of a cube of 10 cm if a charge of 1 eus is placed at the center of the cube?

Solution:

1. Electric flux, \(\phi=\vec{E} \cdot \vec{S}=(2 \hat{i}+4 \hat{j}+7 \hat{k}) \cdot 10 j\)

= 40 unit

2. In the CGS system, according to Gauss’ theorem, in a vacuum the net electric flux linked with a closed surface is 4 TC times the net charge within the surface.

∴ 4400 = 4πq

or, \(q=\frac{4400}{4 \pi}=\frac{4400 \times 7}{4 \times 22}\)

= 350 esu

3. Net electric flux = 4πq

= 4π.1

= 4n esu

As a cube has 6 faces, the electric flux linked with each face of the cube = \(\frac{1}{6} \times 4 \pi=\frac{2}{3} \pi \text { esu }\)

Example 3. An electric flux of 6.5 x 103 N.m2.C-2 is linked with a sphere due to some charge placed In a vacuum Inside the sphere. Calculate the magnitude of the charge
Solution:

In this case, total flux linked = 6.5 x 103 N.m2.C-1

According to Gauss’ theorem,

total flux linked with the sphere = \(\frac{1}{\epsilon_0} \cdot q\)

∴ \(\frac{1}{\epsilon_0} \cdot q=6.5 \times 10^3\)

or, q = 6.5 x 103 x ∈0

= 6.5 x 103 x 8.85 x 10-12

= 5.75 X 10-8 C.

Example 4. A spherical shell of radius 20cm has a 20μC charge placed in a vacuum. Calculate the electric field Intensity

  1. At a distance of 15 cm
  2. At a distance of 40 cm from the centre of the spherical shell

Solution:

1. Electric field intensity at a distance of 15 cm from the centre of the spherical shell = 0; because charge does not reside Inside a charged spherical shell.

2. Electric field Intensity at a distance of 40 cm or 0.4 m from the centre of the spherical shell

⇒ \(E=\frac{1}{4 \pi \epsilon_0} \cdot \frac{q}{x^2}=\frac{9 \times 10^9 \times 20 \times 10^{-6}}{(0.4)^2}\)

= 1.125 X 106 N.C-1

Example 5. A thin straight wire of length 40 cm placed in a vacuum has a 20μC charge. Calculate the electric field intensity at a distance of 15 cm from the wire.
Solution:

⇒ \(E=\frac{1}{4 \pi \epsilon_0} \cdot \frac{2 \lambda}{x}\) [ λ = charge per unit length of the wire = \(\frac{20 \times 10^{-6}}{40 \times 10^{-2}} \mathrm{C} \cdot \mathrm{m}^{-1}\) ]

= \(\frac{9 \times 10^9 \times 2 \times 20 \times 10^{-6}}{15 \times 10^{-2} \times 40 \times 10^{-2}}\)

= 6 x 106 N C-1

Example 6. A cube placed in a vacuum contains a charge of 9 x 10-9 C. What will be the electric flux linked with each face of the cube?
Answer:

Electric flux linked with the 6 faces of the cube = \(\frac{q}{\epsilon_0}\) Due to symmetry, this flux is equally shared among the 6 faces. So, the electric flux linked with each face of the cube

⇒ \(\frac{1}{6} \cdot \frac{q}{\epsilon_0}=\frac{1}{6} \times \frac{9 \times 10^{-9}}{8.854 \times 10^{-12}}\)

= 169.41V.m

Example 7. An electric field is expressed as \(\vec{E}=(5 \hat{i}+3 \hat{j}+2 \hat{k})\) unit. Find out the electric flux across an area of 200 units on the yz-plane in that field.
Solution:

The yz-plane is perpendicular to the x-axis.

∴ The given area vector is \(\vec{S}=200 \hat{i} \text { unit. }\)

Then, the electric flux across that area,

⇒ \(\phi=\vec{E} \cdot \vec{S}=(5 \hat{i}+3 \hat{j}+2 \hat{k}) \cdot 200 \hat{i}\)

= 1000 units.

Example 8. The surface density of charge on a large, vertical positively charged plate Is σ C.m-2. A string attaches a metal ball of mass M and charges +q with the plate. Find out the angle between the string and the plate In equilibrium
Solution:

Let the angle between the string and the plate in equilibrium = θ

Tension in the string = T;

The electric field at the position of the ball = E

= \(\frac{\sigma}{2 \epsilon_0}\)

In equilibrium,

⇒ \(T \sin \theta=q E=\frac{q \sigma}{2 \epsilon_0} ; T \cos \theta=M g\)

∴ \(\tan \theta=\frac{q \sigma}{2 \epsilon_0 M g}\)

or, \(\theta=\tan ^{-1}\left(\frac{q \sigma}{2 \epsilon_0 M g}\right)\)

Class 12 Physics Unit 1 Electrostatics Chapter 2 Electric Field The surface density of charge

Example 9. The classical concept of atomic structure- is that negative charges are uniformly distributed inside a sphere of radius R keeping the nucleus of positive charge Ze at the centre of that sphere. An atom as a whole is electrically neutral. Find out the electric field at a distance r from the nucleus, according to this atomic model.
Solution:

Charge of the nucleus = +Ze.

∴ Amount of charge outside the nucleus = -Ze, as an atom is electrically neutral.

Volume density of negative charge inside the sphere,

⇒ \(\rho=\frac{-Z e}{\frac{4}{3} \pi R^3}=-\frac{3 Z e}{4 \pi R^3}\)

The electric field at external points (r > R):

Let A be any such point outside the sphere. A concentric sphere of radius r, passing through A, is a convenient Gaussian surface. The charge enclosed by this Gaussian surface, q = charge of the nucleus + charge distributed inside the sphere of radius R

= +Ze +(-Ze)

= 0

∴ From Gauss’ theorem, \(\oint_S \vec{E} \cdot d \vec{S}=\frac{q}{\epsilon_0}=0\)

∴ E = 0, at all external points.

The electric field at Internal points (r<R): Let B be any such point inside the sphere. A concentric sphere of radius r, passing through B is a convenient Gauislan surface.

It is to be noted that this surface encloses the entire positive charge +Ze of the nucleus, but a part of the negative charge -Ze remains outside it The amount of negative charge inside this Gaussian surface,

Class 12 Physics Unit 1 Electrostatics Chapter 2 Electric Field electric field at internal points

q’ = volume of this endorsed sphere x volume density p of negative charge

⇒ \(\frac{4}{3} \pi r^3\left(\frac{3 Z e}{4 \pi R^3}\right)=-Z e \frac{r^3}{R^3}\)

∴ Net charge enclosed by the sphere,

⇒ \(q=+Z e+q^{\prime}=Z e-Z e \frac{r^3}{R^3}=Z e\left(1-\frac{r^3}{R^3}\right)\)

For any surface element d\(\vec{S}\) on the Gaussian surface, the electric field \(\vec{E}\) and the area vector d\(\vec{S}\) are parallel to each other.

∴ \(\vec{E} \cdot d \vec{S}=E d S \cos 0^{\circ}=E d S\)

Again, from symmetry, £ is uniform in magnitude over the Gaussian surface. Then, from Gauss’ theorem,

⇒ \(\oint_S \vec{E} \cdot d \vec{S}=\frac{q}{\epsilon_0}\)

or, \(\oint_S E d S=\frac{q}{\epsilon_0} \text { or, } E \oint_S d S=\frac{q}{\epsilon_0} \quad \text { or, } E S=\frac{q}{\epsilon_0}\)

or, \(E \cdot 4 \pi r^2=\frac{1}{\epsilon_0} Z e\left(1-\frac{r^3}{R^3}\right)\)

or, \(E=\frac{Z e}{4 \pi \epsilon_0} \cdot \frac{1}{r^2}\left(1-\frac{r^3}{R^3}\right)=\frac{Z e}{4 \pi \epsilon_0}\left(\frac{1}{r^2}-\frac{r}{R^3}\right)\)

Example 10. A uniform electric field along the x-axis is given as,

⇒ \(\begin{aligned}
\vec{E} & =(200 \hat{i}) \mathrm{N} \cdot \mathrm{C}^{-1}, \text { for } x>0 \\
& =(-200 \hat{l}) \mathrm{N} \cdot \mathrm{C}^{-1}, \text { for } x<0
\end{aligned}\)

A cylinder of length 20 cm and radius 5 cm has Its center at the origin and axis along the x-axis and is placed in a vacuum. Find out

  1. The electric flux across each of its circular faces,
  2. The flux across hs curved surface,
  3. The flux across its entire outer surface and
  4. The net charge enclosed by it,

Solution:

Radius of the cylinder, r = 5 cm = 0.05 m

∴ Area of each circular face,

S = πr²

= 3.14 x (0.05)² m²

Class 12 Physics Unit 1 Electrostatics Chapter 2 Electric Field example 10 radius of the cylinder

As the length of the cylinder is 20 cm or 0.2 m, the two circular faces are at x = +0.1 m and x = -0.1 m.

The area vectors representing the right and the left circular faces \(\vec{S}_1=\hat{i}\left(\pi r^2\right) \text { and } \vec{S}_2=-\hat{i}\left(\pi r^2\right)\) respectively.

The electric fields at the positions of these circular faces are, \(\) respectively

1. The electric flux across the right circular face,

⇒ \(\vec{E}_1=(200 \hat{i}) \mathrm{N} \cdot \mathrm{C}^{-1} \text { and } \vec{E}_2=(-200 \hat{i}) \mathrm{N} \cdot \mathrm{C}^{-1}\)

= 200 x 3.14 x (0.05)²

= 1.57 N.m2.C-1

Similarly, for the left circular face,

⇒ \(\phi_1=\vec{E}_1 \cdot \vec{S}_1=(200 \hat{i}) \cdot \hat{i}\left(\pi r^2\right)\)

= 200 x 3.14 X (0.05)²

= 1.57 N.m2 – C-1

∴ \(\phi_1=\phi_2\) i.e., equal flux passes across each of the two faces.

2. The curved surface is everywhere parallel to the electric field vector.

∴ The electric flux linked with the curved surface = 0

3. The entire outer surface consists of two circular faces and a curved surface. So, the flux linked with the entire surface,

∴ \(\phi=\phi_1+\phi_2+0=1.57+1.57+0\)

= 3.14 N.m2.C-1

4. FromchargeGauss’is, theorem, net flux, \(\phi=\frac{q}{\epsilon_0}\). So the endorsed closed is,

q = \(\phi \epsilon_0\) = 3.14 x 8.854 x 10-12

= 2.78×10-11C

Electrostatics

Electric Field Synopsis

  • Coulomb’s law: The force of attraction or repulsion between two point charges is proportional to the product of the charges and inversely proportional to the square of the distance between them.
  • SI unit of charge: If an electric repulsive force of 9 x 109 N arises between two similar electric charges lm apart in air (or vacuum), the amount of each charge is 1C.

Units of charge in different unit systems:

Class 12 Physics Unit 1 Electrostatics Chapter 2 Electric Field Units of charge in different unit systems

1 coulomb = 3 x 109 esu of charge.

  • The dielectric constant of a medium (k) is defined as the ratio of the permittivity of the medium (∈) to that of vacuum (∈0). It is also known as relative permittivity.
    So, \(\kappa=\frac{\epsilon}{\epsilon_0}\)
  • The electric field due to a charge distribution is the region in which its effect is felt. Any other charge, placed at any point in that region, experiences a force due to the original charge distribution.
  • The intensity or strength of an electric field at any point in the field is the force experienced by a unit of positive charge placed at that point.

Unit of intensity of electric field:

Class 12 Physics Unit 1 Electrostatics Chapter 2 Electric Field Unit of intensity of electric field

1 dyn statC-1 = 3 x 104 N.C-1

  • If the resultant intensity of the electric field at any point due to two or more charges is zero, then that point is called a neutral point.
  • An electric dipole consists of a combination of two equal and opposite point charges separated by a very small distance.

Unit of dipole moment:

Class 12 Physics Unit 1 Electrostatics Chapter 2 Electric Field Unit of dipole moment

1 C.m = 3 x 10º esu.cm

  • The electric dipole moment of an electric dipole is defined as the torque acting on it when it is placed at right angles to a uniform electric field of unit strength.
  • The substances that have no free electrons cannot conduct electricity. They are called insulators or dielectrics.

According to the position of charge in the molecule of a dielectric, it is classified into two groups:

1. Nonpolar substance

2. Polar substance.

  • The phenomenon of alignment of the molecules, which behave as electric dipoles, under the influence of an external electric field, is called electric polarisation.
  • An electric field line is an imaginary line in an electric field along which a free and isolated positive charge would tend to move. A tangent at any point on the line gives the direction of the electric field at that point.
  • The magnitude of the electric field intensity at any point is equal to the number of field lines passing normally through a unit surface area around that point.
  • Electric flux is the number of field lines passing normally through a surface.

Unit of electric flux:

  • SI: N.m2.C-1 or V m

Gauss’ theorem:

  • The net electric flux linked with a closed surface is \(\frac{1}{\epsilon}\) times the total charge within the surface.

Electrostatics

Electric Field Useful Relations for Solving Numerical Problems

  • The electrostatic force between two point charges,
    ⇒ \(F=\frac{q_1 q_2}{r^2}\) (In CGS system);
    ⇒ \(F=\frac{q_1 q_2}{4 \pi \epsilon_0 r^2}\)
    where e0 = permittivity of air or vacuum.
  • Relation between K and e:
    ⇒ \(\kappa=\frac{\epsilon}{\epsilon_0}\) [where e0 = permittivity of vacuum (or air)
    = 8.854 x 10-12 C2.N-1.m-2.
  • The electric intensity of a point at distance r from o point charge q,
    ⇒ \(E=\frac{1}{4 \pi \alpha^{\prime} C_0} \cdot \frac{q}{r^2}(\ln \mathrm{SI})\)
  • Relation between electrostatic force and electric intensity,
    F = qE
  • Electric dipole moment, p = 2lq, where, 2l = length of the dipole, q = charge of dipole It is a vector quantity defined as \(\), where \(\vec{l}\) is directed from the negative to the positive charge of the dipole.
  • Electric field intensity at a point on the axis of a dipole,
    ⇒ \(E=\frac{2 p r}{4 \pi k \epsilon_0\left(r^2-l^2\right)^2}(\text { in SI })\)
    When \(r \gg l, E=\frac{1}{4 \pi \kappa \epsilon_0} \cdot \frac{2 p}{r^3}\);
  • Electric field intensity at a point on the perpendicular bisector of a dipole
    ⇒ \(E=\frac{p}{4 \pi \kappa \epsilon_0\left(r^2+l^2\right)^{3 / 2}} \text { (in SI) }\)
    When \(r \gg l, E=\frac{1}{4 \pi \kappa \epsilon_0} \cdot \frac{p}{r^3} \text { (in SI) }\)
  • Electric field intensity at any point due to an electric dipole,
    ⇒ \(E=\frac{1}{4 \pi \kappa \epsilon_0} \cdot \frac{p}{r^3} \sqrt{3 \cos ^2 \theta+1}(\text { in SI })\)
  • The torque acting on an electric dipole inclined at an angle 0 with a uniform electric field, \(\tau=p E \sin \theta\)
  • Electric flux through a surface d\(\vec{S}\) in an electric field of intensity \(\vec{E}\) is,
    ⇒ \(\phi=\int_S \vec{E} \cdot d \vec{S}\)
  • According lo Gauss’ theorem, total electric flux linked with a closed surface is given by,
    ⇒ \(\oint_S \vec{E} \cdot d \vec{S}=\frac{q}{\epsilon} \text { (in SI) }\)
    where q is the charge enclosed by the closed surface and e is the permittivity of the medium.
  • The electric field intensity at a point due to a point charge.
    ⇒ \(E=\frac{q}{4 \pi \epsilon r^2}(\text { in SI })\)
  • The electric field intensity due to a uniformly charged thin spherical shell at a point outside the shell,
    ⇒ \(E=\frac{1}{4 \pi \epsilon} \cdot \frac{2 \lambda}{r}(\text { in SI })\)
  • The electric field intensity at a point due to an infinitely long straight charged conducting wire,
    ⇒ \(E=\frac{\sigma}{2 \epsilon} \text { (in SI) }\) [where, λ = linear charge density of the wire]
  • The electric field intensity at a point due to an infinite non-conducting uniformly charged plane lamina,
    ⇒ \(E=\frac{\sigma}{2 \epsilon} \text { (in SI) }\)
  • The intensity of the electric field at a point very close to a charged conductor having a surface density of charge cr
    ⇒ \(E=\frac{\sigma}{\epsilon}(\text { in SI })\)

Electrostatics

Electric Field Very Short Answer Type Questions

Question 1. The force of interaction between two charges placed in vacuum is F. What will be the force between the charges placed at the same distance in a medium of dielectric constant k?
Answer: F/k

Question 2. What is the relation of esu of charge with coulomb?
Answer: 1C = 2 x 109 esu of charge

Question 3. If 1μC = x esu of charge, what is the value of x?
Answer: 3 x 10-3

Question 4. What is the order of the ratio of the gravitational force and the electrical force acting between two electrons?
Answer: 1043

Question 5. What is the electric field intensity at a distance r from a charge q placed in a vacuum?
Answer: q/r2

Question 6. What is the unit of the intensity of the electric field in SI?
Answer: N.C-1

Question 7. Which physical quantity has the unit V.m-1 ? placed in a uniform electric field?
Answer: Intensity of the electric field

Question 8. Two point charges e1 and e2 are placed at a distance d from each other. In between them, there is no point where the electric field is zero. From this what conclusion can you
Answer: e1 and e2 are oppositely charged

Question 9. What is the intensity of the electric field at the centre of a
Answer: Zero

Question 10. The direction of electrostatic field intensity at a point on the surface of a charged conductor is along the tangent drawn on the surface at that point. Is the statement true or false?
Answer: False

Question 11. If a conductor is placed in an electric field, what change of the intensity of the electric field takes place inside the
Answer: Electric field intensity becomes zero

Question 12. what is the ratio of the intensity of the electric field at a particular distance on the axis to that on the perpendicular bisector of a very small dipole?
Answer: 2:1

Question 13. How should an electric dipole be kept in an electric field so that no torque acts on it?
Answer: If it remains parallel to the electric field

Question 14. is the dipole moment a scalar or a vector quantity?
Answer: Vector quantity

Question 15. What is the unit of dipole moment in SI?
Answer: C.m

Question 16. An electric dipole is inclined with a uniform electric field. State whether a force or a torque acts on the dipole. charge q placed in a vacuum?
Answer: Torque

Question 17. What is the resultant electric force on an electric dipole placed in a uniform electric field?
Answer: 0

Question 18. What is the dipole moment of a non-polar molecule?
Answer: 0

Question 19. If electric flux liked with a surface is maximum, what will be the angle of the field lines with the normal to the surface?
Answer: 0

Question 20. A square plate of side a is placed perpendicular to a uniform electric field of intensity E. What will be the flux linked with the plate?
Answer: Ea²

Question 21. What is the intensity of the electric field inside a uniformly charged spherical shell?
Answer: 0

Electrostatics

Electric Field Fill In The Blanks

1. Two point charges each equal to 1C exert a force 9 x 109N mutually when they are placed 1 m apart in air

2. The electric field intensity at all points in a uniform electric field is equal

3. The field lines in a uniform electric field are equidistant parallel lines.

4. Three small spheres are placed on the circumference of a circle of radius r in such a way that an equilateral triangle is formed. If the charge of each sphere is q, the electric field intensity at the center of the circle will be zero

5. A few field lines moving from the left converge on the right side. The electric field intensity will be greater at the right side

6. The field fines of an isolated negative charge are radially inwards

7. The torque acting on an electric dipole placed in a uniform electric field is maximum when the dipole is perpendicular to the direction of the electric field.

8. An electric dipole of moment p is placed in an equilibrium position in a uniform electric field of intensity E. To rotate the dipole through an angle 6 from its initial position, the required moment of the torque is pE sinθ

9. If a charge exists outside a closed surface, the net electric flux linked with the closed surface will be zero

10. The direction of the area vector is perpendicular to the area

11. The flux linked with a surface perpendicular to the field lines in an electric field will be the maximum

12. According to Gauss’ theorem in the CGS system, the total electric flux linked with a closed surface is equal to 4π/k times the total charge lying within the surface.

13. Electric flux is a scalar quantity.

Electrostatics

Electric Field Assertion-Reason Type

Direction: These questions have statement 1 and statement 2. Of the four choices given below, choose the one that best describes the two statements.

  1. Statement 1 is true, statement 2 is true; statement 2 is a correct explanation for statement 1.
  2. Statement 1 is true, and statement 2 is true; statement 2 is not a correct explanation for statement 1.
  3. Statement 1 is true, statement 2 is false.
  4. Statement 1 is false, and statement 2 is true.

Question 1.

Statement 1: If a proton and an electron are placed in the same uniform electric field, they experience different accelerations.

Statement 2: Electric force on a test charge is independent of its mass.

Answer: 2. Statement 1 is true, and statement 2 is true; statement 2 is not a correct explanation for statement 1.

Question 2.

Statement 1: Electric dipole moment is a vector quantity.

Statement 2: The direction of electric dipole moment is from negative charge to the positive charge.

Answer: 1. Statement 1 is true, statement 2 is true; statement 2 is a correct explanation for statement 1.

Question 3.

Statement I: When an electric dipole is perpendicular to an electric field, maximum torque acts on it

Statement II : If 6 = 90°, then torque, r = pEsin90° = pE.

Answer: 1. Statement 1 is true, statement 2 is true; statement 2 is a correct explanation for statement 1.

Question 4.

Statement 1: A free electron suffers the same acceleration as a free proton when placed in a uniform field.

Statement 2: An electron and a proton have the same quantity of charge.

Answer: 4. Statement 1 is false, and statement 2 is true.

Question 5.

Statement 1: The electric field due to a dipole on its axial line at a distance r is E. The electric field due to the same dipole on the equatorial line at the same distance will be \(\frac{E}{2}\)

Statement 2: Electric field due to a dipole at a point varies inversely with distance.

Answer: 3. Statement 1 is true, statement 2 is false.

Electrostatics

Electric Field Match The Columns

Question 1. Some physical quantities are given in column I and their corresponding dimensions are given in column II

Class 12 Physics Unit 1 Electrostatics Chapter 2 Electric Field match the columns 1

Answer: 1-C, 2-D, 3-A, 4-B

Question 2.

Class 12 Physics Unit 1 Electrostatics Chapter 2 Electric Field match the columns 2

Answer: 1-C, 2-A, 3-D, 4-B

Question 3. Some physical quantities are given in column I and their corresponding dimensions are given in column II

Class 12 Physics Unit 1 Electrostatics Chapter 2 Electric Field match the columns 3

Answer: 1-B, 2-D, 3-A, 4-C

Question 4. Charge inside the different types of closed surfaces are
given in column I and their corresponding values of flux
are given in column II.

Class 12 Physics Unit 1 Electrostatics Chapter 2 Electric Field match the columns 4

Class 12 Physics Unit 1 Electrostatics Chapter 2 Electric Field match the columns 4.

Answer: 1-D, 2-A, 3-B, 4-C, 5-D

WBCHSE Class 12 Physics Notes For Elementary Phenomena Of Electrostatics

Electrostatics Elementary Phenomena Of Electrostatics Introduction

  • In about 600 BC, Greek philosopher Thales of Miletus observed that if amber (hard resin from pine tree) is rubbed with flannel acquires the property of attracting small pieces of paper or light bodies towards it.
  • In 1600 AD, William Gilbert, a physician to Queen Elizabeth, observed that many other substances behave in the same way. For example, a glass rod rubbed with silk exhibits the same phenomenon. Even when we comb our hair, the comb will show the same property.
  • An object which gets such an ability to attract others due to rubbing is called an electrified object and the process is called electrification. Practically, this electrification is due to the transfer of electric charge from one object to another.
  • The electricity produced by rubbing is called frictional electricity. This type of electricity remains confined within the body where it originates and cannot move from one place to another. Hence it is also called statical electricity.
  • The word ‘electricity’ is derived probably from electron which is the Greek name of amber.

Electric charge:

Gravitational attraction exists between any two particles inside a material. The mass of each particle is the property that is responsible for this attraction.

Read and Learn More Class 12 Physics Notes

On the other hand, the repulsion between two elements or the attraction between an electron and a proton is due to a force known as an electric force. This force originates from a property of the particles known as electric charge. This is a measurable physical quantity.

  1. Electric charge is a scalar quantity.
  2. Its unit in the CGS system is esu or statcoulomb (state).
  3. The unit of charge in SI is coulomb (C).
    1C = 3 x 109 statC
  4. The dimension of electric charge is IT

Electrostatics Elementary Phenomena Of Electrostatics Electrification By Rubbing

Experiment: In winter, after combing our hair if we bring the comb near some small pieces of paper, it will attract them. Here, the comb is electrified due to rubbing with hair and it acquires the property of attracting the pieces of paper. Sometimes the comb and the hair are so much electrified during rubbing that we can hear a crackling sound due to tiny sparks.

Example: A metallic chain is often hung from the fuel tank of a motor vehicle and it touches the road below. When the vehicle is in motion, static electricity is developed due to rubbing between the fuel and the inner wall of the tank. The hanging chain helps to move the electric charges to the ground instantly. In the absence of such a chain, the accumulated charges may inflame the fuel in the tank.

WBCHSE Class 12 Physics Notes For Elementary Phenomena Of Electrostatics

Electrostatics

Elementary Phenomena Of Electrostatics Two Kinds Of Electric Charge Positive And Negative

With the help of a simple experiment, it can be proved that electric charges are of two types — positive charge and negative charge.

Experiment: A glass rod rubbed with silk is suspended by a silk thread from a support. An ebonite rod rubbed with fur is suspended from the same support close to the glass rod. The two rods will attract each other.

So two bodies being electrified differently may attract each other. The ebonite rod is now replaced with another glass rod, rubbed with silk, and is suspended in its position. The two rods will repel each other. So two bodies being electrified similarly repel each other.

Class 12 Physics Unit 1 Electrostatics Chapter 1 Elementary Phenomena Of Electrostatics A glass rod rubber

Inference:

From the above experiment, we can infer that similar electric charges are produced in two similar rods if they are rubbed with similar substances. Since the two rods repel each other, it is proved that similar charges repel each other.

On the other hand, opposite charges may be produced in two different rods if they are rubbed with different substances. Then they attract each other. Here, we can infer that opposite charges attract each other.

The fundamental law of statical electricity:

Opposite or unlike charges attract each other and similar or like charges repel each other. Note that both the charges attract an uncharged body.

American scientist Benjamin Franklin called one of them a positive charge and the other one a negative charge. The reason is that if we give equal amounts of two opposite charges to a body, it will remain unchanged (like 5-5 = 0 ).

The convention is that the charge produced in a glass rod rubbed with silk is positive and the charge produced in an ebonite rod rubbed with flannel is negative.

Electrostatic series:

This is a list where the substances have been arranged in such a way that if any two of them are rubbed together, the one preceding the other in the list acquires a positive charge and the latter acquires a negative charge. This is known as the electrostatic series.

  1. Fur
  2. Flannel
  3. Sealing wax
  4. Glass
  5. Paper
  6. Silk
  7. Human body
  8. Wood
  9. Metals
  10. Rubber
  11. Resin
  12. Amber
  13. Sulphur
  14. Ebonite

From the table, it is clear that a particular substance may be either positively charged or negatively charged if it is rubbed with two different substances. For example, a glass rod becomes positively charged if It is rubbed with silk and negatively charged if it is rubbed with flannel

Repulsion is the Conclusive Test of Electrification:

A charged body attracts an uncharged body as well as an oppositely charged body. But if repulsion takes place between the two bodies, it can be concluded that the experimental body must be charged because repulsion is possible only between two bodies with the same kind of charge. Hence, repulsion is the conclusive test of electrification.

Electrostatics

Elementary Phenomena Of Electrostatics Electronic Theory Of Electricity

Electronic theory accounts for the source of the charge. This is based on the electronic structure of matter. We know that atoms are made up of a positively charged central core called the nucleus, around which negatively charged electrons rotate in various closed orbits. The nucleus contains positively charged protons. The number of electrons in an atom is equal to the number of protons in the nucleus. As a whole, an atom is electrically neutral.

Class 12 Physics Unit 1 Electrostatics Chapter 1 Elementary Phenomena Of Electrostatics A glass rod rubber

The nucleus of an atom attracts the electrons towards it. Electrons lying in the outermost orbit are rather loosely bound and hence are easy to detach. If one or more electrons are somehow removed from the atom, the atom becomes positively charged. Conversely, if an atom is given one or more electrons, it becomes negatively charged.

Therefore, a positively charged body has a deficiency of electrons in its atoms and a negatively charged body has excess electrons in its This is the electronic theory. Note that electrons alone are responsible for the electrification of a body and not the protons because protons are firmly held in the nucleus.

An atom is composed of three fundamental particles—electron, proton, and neutron. The number of these fundamental particles obviously varies from element to element. The mass of an electron is 9 x 10-31 kg and the amount of charge in it is 1.6 x 10-19C. This is the smallest possible amount of charge.

  1. The mass of a positively charged body is less than the mass of the same electrically neutral body. In this case, lost mass of the body = number of electrons lost by the body x mass of an electron.
  2. The mass of a negatively charged body Is greater than the mass of the same electrically neutral body. In Till’s case, gained mass of the body = number of electrons gained by the body x mass of an electron,

Quantization of charge: The charge carried by a body Is always an Integral multiple of the smallest unit of charge which is the charge of an electron. This is known as the quantization of charge.

That’s why, charge q = ±ne, where, e = charge of an electron and n = 1, 2, 3,… a natural number. Electric charge is a scalar quantity. The net charge of a body is the algebraic sum of all the charges present.

Invariance relative to tiro frame of reference:

The amount of a charge is independent of its state of rest or of motion; and also independent of the state of rest or of motion of the observer. This is expressed by saying that an amount of charge is invariant with respect to the frame of reference. So we can say, qrest = qmoving

Explanation of Frictional Electricity on the Basis of Electronic Theory:

The electrons are bound to an atom due to the attraction of the positively charged nucleus. This attraction is obviously not equal for the atoms of different substances.

So during rubbing between two bodies, some electrons are transferred from one to the other, producing equal and opposite charges simultaneously. For example, when a glass rod is rubbed with silk, some electrons from the glass migrate to silk.

The force of attraction between the electrons and the nucleus in glass is less than that in silk. Hence the glass rod becomes positively charged with a deficit of electrons and the silk becomes negatively charged with an excess of electrons.

Principle of conservation of electric charge:

It states that the algebraic sum of the positive and the negative charges in an isolated system Is constant. The electronic theory clearly indicates that electric charge can neither be generated nor destroyed. It is only redistributed when electrons migrate from one body to another.

It should be mentioned that the principle had been known, long before the invention of electrons and protons. To date,’ no deviation has been observed for this principle. The principle conservation of charge is a universal law—no physical phenomenon exists for which this law is violated.

Electrostatics

Elementary Phenomena Of Electrostatics Conductor And Insulators

Electric charge cannot flow through all substances with equal ease. According to the ability of charge to move through the materials, they are classified into two major groups

  1. Conductor
  2. Insulator or non-conductor.

Conductor:

Materials through which charge can move easily are called conductors. If some charge is given to any part of a conductor:

If some charge is given to any part of a conductor, it will spread all over the body of the conductor

Generally, all metals are conductors of varying degrees. Among them silver, copper, and aluminum rank higher. Eaiui, the human body, gas carhop, graphite, mercury, etc., are it conductors. Acid, alkali, and aqueous solutions of salts also conduct electricity. Generally, no material is a perfect conductor.

Insulator or non-conductor:

Materials through which charge cannot move are called insulators or non-conductors:

If any part of an insulator is charged, charges remain confined to that part of the insulator and do not spread all over the body. Dry air, glass, rubber, ebonite, mica, silk, paraffin, bakelite, etc., are insulators. Remember that generally no material can be said to be a perfect insulator.

Besides conductors and insulators, there is a third kind of material called semiconductors, which are neither good conductors nor good insulators. Selenium, germanium, silicon, etc., are semiconductors. They are widely used in electronic circuit elements like transistors, Integrated circuits, etc.

Nowadays a few substances can be made to behave as almost perfect conductors at very low temperatures. These are called superconductors. Aluminum acts as a superconductor at -272°C.

Dry air is a good insulator. But the charge can flow through moist air. Hence experiments on statical electricity cannot be conducted satisfactorily in the rainy season. In the electricity supply system, the transmission wires are joined through porcelain pots, which are non-conductors.

But if the wires were directly connected to the electric post, electricity would have been grounded instantly and there would be a huge transmission loss, and if someone touched the post, there, would be a chance of getting shocked. Rubber, silk, or cotton to reduce the chance of short circuits.

These are called insulated wires. Pure water is a non-conductor, but various types of salts, bases, and acids are dissolved in natural water which makes it a fairly good conductor.

Conductors and Insulators on the basis of electronic theory:

The electrons of the outermost orbits of the atoms of a conductor are very loosely bound to the nucleus and move freely from one atom to another These electrons are called free electrons.

These free electrons carry electricity from one place to another throughout the conductor Metals are good conductors due to the presence of free electrons. In insulators or nonconductors, electrons are tightly bound to the nucleus—they are not free electrons. So electricity cannot flow through them

Electrostatics Elementary Phenomena Of Electrostatics Charging By Conduction

A conductor with a non-conducting handle is called an insulated conductor. Charges given to this conductor cannot go elsewhere and it becomes a charged conductor.

If an uncharged insulated conductor is brought in contact with such a charged insulated conductor, some charges cross over to the uncharged conductor, which becomes charged.

This process of charging a conductor by direct contact with a charged body is known as charging by conduction. It can be explained on the basis of electronic theory.

Explanation of charging by conduction on the basis of free electronic theory:

Suppose a body is negatively charged. So it has excess electrons. When this charged body comes in contact with an uncharged conductor, a few electrons from the charged body move into the conductor. As the conductor now has excess electrons, it becomes negatively charged.

If the body is positively charged initially and is touched with an uncharged conductor, a few electrons of the uncharged conductor move to the charged body due to attraction by the positive charge. As the conductor has a deficit of electrons, it becomes positively charged.

Note that, in both cases, the initial amount of charge in the charged hotly is shared between two bodies due to conduction. As a result, the strength of charge in the first body is reduced to some extent.

 Electrostatics

Elementary Phenomena Of Electrostatics Gold-Leaf Electroscope

An electroscope is an instrument used for detecting the presence of charge and its nature. A gold-leaf electroscope is a widely used instrument.

Description:

A metal rod P passes through an insulating stopper into a vessel fitted with glass panes. Two thin foils of gold (L, L) are hung at the lower end of the metal rod. The upper end of the rod is capped with a metal disc D.

The glass vessel protects the leaves from air current Some fused calcium chloride is placed inside the vessel as a hygroscopic substance. Two tin foils (f, t) are attached to the inner face of the glass walls. These foils which are in contact with the metallic base of the vessel are earthed.

Class 12 Physics Unit 1 Electrostatics Chapter 1 Elementary Phenomena Of Electrostatics A gold-leaf electroscope

Charging the electroscope by conduction: To charge the gold-leaf electroscope by conduction, a charged body is touched on the disc of the electroscope. A glass rod rubbed with silk becomes positively charged.

If this glass rod is made to touch the disc of the electroscope, a part of the positive charge spreads in the instrument. As a result, the leaves, being positively charged, diverge due to repulsion, and remain so even after the removal of the glass rod.

Class 12 Physics Unit 1 Electrostatics Chapter 1 Elementary Phenomena Of Electrostatics Charging the electroscope by conduction

If the disc is now touched, all the charges of the electroscope will flow’ to die earth through the body of the experimenter, and the leaves will collapse. This is called earthing of a gold-leaf electroscope.

Charging by conduction is however not a good process at all. If the charged both- carries a large amount of charge, then as soon as it is brought in contact with the disc, the divergence of the leaves will be so great that they may get detached from the rod.

Uses:

Detection of charge: The body to be tested is brought slowly near or in contact with the disc of an uncharged electroscope. If the leaves are deflected, both’ are charged otherwise, uncharged.

Determination of the nature of charge: To test the nature of charge on a body, we have to start with a charged electroscope. Suppose, the electroscope is positively charged with its leaves divergent.

The body to be tested is brought slowly to touch the disc. If the divergence of the leaves increases, the body is positively charged. If the divergence decreases, the body is negatively charged. The nature of the charges would be just the opposite if we started initially with a negatively charged electroscope.

An uncharged body may also diminish the divergence to some extent, so it cannot definitely indicate that the charge of the test body is opposite in nature to that on the electroscope. Thus, an increase in the divergence of the leaves of the electroscope provides the surer test for the nature of a charge on a body.

The experimental results are given in the following table:

Class 12 Physics Unit 1 Electrostatics Chapter 1 Elementary Phenomena Of Electrostatics The experimental results are given in the following table

Obviously, an insulator will have no effect on the divergence of the leaves.

Identification of conductor and Insulator:

The body to be tested is brought slowly to touch the disk of a charged gold leaf electroscope. In this case, the body must be grounded with a conductive wire.

If the test body is a conductor then all the charges of the electroscope will flow to the earth through the conductive wire and the leaves will collapse. If the die body is an insulator then the charge of the electroscope will not be able to flow to the earth.

So there will be no change in the divergence of die leaves of the electroscope. This way, the identification of the conductor and insulator can be done using a gold-leaf electroscope.

Proof Plane:

It is of ten difficult to bring up a strongly charged test body near the electroscope. A portion of the charge from the body can be picked and taken to an electroscope for testing by using what is called a proof plane. A proof plane consists of a small metallic disc mounted with an art-insulating handle.

It is held by the insulating handle and the metal disc is momentarily placed in contact with the charged test body. The disc gets charged by conduction. It is now brought to a charged electroscope for testing the nature of the charge. A proof plane can also be used to detect the presence of charge or the nature of distribution of charge on a body

Class 12 Physics Unit 1 Electrostatics Chapter 1 Elementary Phenomena Of Electrostatics proof plane

Electrostatics

Elementary Phenomena Of Electrostatics Electrostatic Induction

When a charged body is brought near, but not in contact with an insulator or insulated uncharged conductor, an opposite charge is produced at the near end and a similar charge at the far end of the insulator or conductor.

The charges so produced disappear as soon as the charged body is removed. This phenomenon is called electrostatic Induction which Is defined as the porary charging of the body by the influence of nearby charges.

Experiment: A positively charged glass rod A brought near the end R of an insulated uncharged conductor RC. Now the following operations are performed.

The disc of a proof plane Is made to touch the end B. Next, the proof plane is brought near an uncharged gold-leaf electroscope without actually touching it.

It is found that the leaves diverge. Hence the end B of the conductor must be charged. The proof plane is now discharged by touching with a hand. If the other end C of the conductor is examined, it will be found that this end is also charged.

But the middle of the conductor BC will be found to have practically no charge. So it may be concluded that induction produces charges only at both ends of a conductor.

Class 12 Physics Unit 1 Electrostatics Chapter 1 Elementary Phenomena Of Electrostatics Electrostatic induction

The nature of the charges induced can be tested with a charged gold-leaf electroscope. Suppose we take a negatively charged electroscope. Keeping the glass rod A the end B, a proof plane is made to touch the end B.

Now it is taken near the negatively charged gold-leaf electroscope. It is observed that the divergence of the leaves increases which shows that the end B of the conductor is negatively charged.

The gold-leaf electroscope is now charged positively. The proof plane is discharged by touching with a hand. The end C of the conductor is touched with the proof plane. It is brought near the gold-leaf electroscope and it is found that the leaves diverge further. So the end C is positively charged, i.e., it has a charge similar to that of the glass rod.

Class 12 Physics Unit 1 Electrostatics Chapter 1 Elementary Phenomena Of Electrostatics The gold-leaf electroscope

In this experiment if, instead of the glass rod A, an ebonite rod rubbed with flannel be taken, it will be found that the end B has a positive charge and the end C has a negative charge. In the middle of BC, there is no charge.

Now if the glass rod or the ebonite rod is removed from the vicinity of the conductor BC, it will be found that no charge exists either at the end B or at the end C. So if the inducing body is removed, the charges of the conductor disappear. It is proved that under the influence of a charged body an uncharged conductor

  1. Becomes temporarily charged,
  2. Developsunlike charge at the near end and like charge at the far end, and
  3. The induced charges disappear when the charged body is moved away from the conductor.

Explanation of Electrostatic Induction from Electronic Theory:

Electrostatic Induction In Conductor:

Electrostatic induction in a conductor can be easily explained by the electronic theory. Every conductor has a large number of free electrons which can flow from one atom to another within the conductor.

In the first experiment, due to the presence of the positive charge in rod A, some free electrons in the conductor BC are attracted to end B and cause an abundance of electrons at that end.

Hence the end B becomes negatively charged. On the other hand, a deficit of electrons by the same number has occurred at the end C and so it becomes equally positively charged.

In the second experiment, the negative charges of the ebonite rod repel the free electrons from the end B to the end C causing an excess of electrons at C and a deficiency of electrons at the end B. Thus the end C becomes negatively charged and the end B positively charged.

It may be noted that induction only changes the arrangement of the free electrons in the conductor. The total number of electrons in the conductor remains the same. So when the charged body is removed from the vicinity of the conductor, the electrons are redistributed uniformly and the conductor becomes uncharged.

Electrostatic Induction In insulator: Insulators have no free electrons. Electrostatic induction takes place in the insulators due to the polarisation of atoms placed in an electric field.

If a positively charged body is placed near an insulator, and the closest atom of the insulator finds itself in the field of the positive charge. This attracts the electrons of the atom and repels the nucleus.

So a deformation takes place in the structure of the atom. As a result, the nucleus of the atom is no longer symmetrical with respect to the electrons.

A slight relative displacement occurs between the positive and the negative charges inside the atom. This happens for all the atoms of the insulator. This is called the polarisation of atoms.

When a positively charged body is brought near an insulator, the negative charge of each atom is pulled toward the body while the positive charge is repelled in the opposite direction.

Since both types of charges exist in equal amounts inside an insulator, they neutralize each other. Hence, the inside of an insulator is electrically neutral. Just the opposite phenomenon will happen if a negatively charged body is brought near an insulator.

Class 12 Physics Unit 1 Electrostatics Chapter 1 Elementary Phenomena Of Electrostatics Electrostatic induction In insulator

Class 12 Physics Unit 1 Electrostatics Chapter 1 Elementary Phenomena Of Electrostatics Electrostatic induction In insulator 1

Inducing Charge and Induced Charge Free Charge and Bound Charge:

Inducing and Induced charge: The charge responsible for creating induction, is called an inducing charge and the charge created due to induction is called an induced charge.

In section 1.8, the positive charge on the glass rod A or the negative charge on the ebonite rod is inducing charge and the charge developed in the conductor BC is induced charge.

From and bound charges: The induced charge at the end of a conductor near the inducing charge is opposite in nature to that of the inducing charge.

Hence the charges induced at the near end of the conductor remain immobile due to the electrostatic force of attraction and cannot escape by conduction. These fixed charges at the near end of the induced conductor are called bound charges.

The charges induced at the far end of the conductor are of the same kind as the inducing charge and hence a force of repulsion exists between these two.

So these charges can immediately flow to the earth by conduction if the conductor is touched by hand. Hence these charges at the far end of the induced conductor are called free charges.

In section 1.8, charges developed due to induction at the end B of the conductor BC are called bound charges and those developed at the tend C are called free charges.

A few facts about induction:

  • From the discussions in the previous sections regarding electrostatic induction the following conclusions can be made
  • Two kinds of charges, positive and negative are produced simultaneously due to induction.
  • Unlike charge is induced at the near end of the conductor and like charge at the far end.
  • Positive and negative charges are induced in equal amounts.
  • It is a temporary phenomenon. The induced opposite charges neutralize, each other as soon as the inducing body is removed.

Induction Precedes Attraction:

  • When a charged body is brought near an uncharged body, the uncharged body is attracted towards the charged body. This is due to electrostatic induction.
  • When achargedbodyisbroughtnearan an uncharged body, unlike charges are induced at the near end and like charges at the far end of the uncharged body. The force of attraction or repulsion between two charges varies inversely to the square of the distance between them.
  • As the unlike-induced charge is nearer to the inducing charge than the like-induced charge, the attraction between the unlike charges predominates over the repulsion between the like charges.
  • So a resultant force of attraction acts on the uncharged body and the body as a whole is attracted towards the charged body. Soinductionprecedes attraction. Effectively, induction is the cause and attraction is its effect.

Electrostatics

Elementary Phenomena Of Electrostatics Charge Always Resides On The Outer Surface Of A Conductor

  • If one end of an insulated rod is charged, the charge is confined to that end only. But when any part of a conductor is charged, the charge distributes itself over the whole surface.
  • No charge is found to exist on the inside of a solid conductor or on the inner surface of a hollow conductor. According to the property of a conductor, a charge may flow easily through it.
  • Like charges repel each other and try to move away from each other as far as possible. So, they distribute themselves on the outer surface of the conductor, where their mutual distance becomes maximum.
  • Under special circumstances, charges may reside on the inner surface of a hollow conductor. A charged body is kept inside a hollow sphere in such a way that it does not touch the hollow sphere.
  • In this case, unlike charges are induced on the inner surface of the hollow sphere and like charges on the outer surface. Without removal, the charged body of the hollow sphere is touched by hand.
  • The free(charges on the outer surface of the hollow sphere move to the earth but the bound charges on the inner charge surface of the sphere exist there.
  • As long as the inducing charge remains inside the hollow sphere, the bound charge also remains on the inner surface. When the charged body is removed, charges shift to the outer surface of the sphere.
  • Electric Screen: Any arrangement, that can keep any space free from external electrical influences, is called an electric screen, Charge always resides on the outer surface of a conductor. Electric screens are based on this property.

Class 12 Physics Unit 1 Electrostatics Chapter 1 Elementary Phenomena Of Electrostatics Electric Screen

A gold-leaf electroscope enclosed in a wire-gauge cage C is placed on an insulated base A. Now if a charged body is brought near to or in contact with the cage, no effect is produced on the electroscope because the charge resides on the outer surface of the cage, not inside. So the region enclosed by the cage is free from external electrical influences.

The space inside a closed metallic box is free from electrical influences for the same reason. The valves of a radio are placed in metallic cases to shield it from external electrical influences. Sensitive electrical instruments are always kept within electric screens.

Electrostatics

Elementary Phenomena Of Electrostatics Distribution Of Charge On The Surface Of A Conductor Surface Density Of Charge

Distribution of charge on a conducting surface: Although the charge on a conductor distributes itself all over the surface, it should not however be concluded that the distribution is always uniform all over the surface.

The distribution of charge depends on the shape of the conductor. The greater the curvature at any point, the greater will be the accumulation of charge at that point. Distributions of charge on charged conductors of different shapes are shown by dotted lines B.

The boundary of each conductor is shown by the line A. The density of charge in each case is roughly represented by the distance of the dotted line B from the boundary line A of each conductor.

Class 12 Physics Unit 1 Electrostatics Chapter 1 Elementary Phenomena Of Electrostatics Distribution of charge on a conducting surface

Surface density of charge: The surface density of charge at a point on a charged conductor is the amount of charge per unit area of the surface of the conductor surrounding the point. The surface density of charge is generally denoted by the symbol cr.

If Q is the charge distributed uniformly over the surface of area A of a spherical conductor having radius r, the surface density of charge is given by,

⇒ \(\sigma=\frac{Q}{A}=\frac{Q}{4 \pi r^2} \text { or, } \sigma \propto \frac{1}{r^2}\)

So, the surface density of charge reduces with the increase of the radius of the object concerned and vice versa. Hence, at sharp bends or pointed portions of a conductor, the surface density of charge will be greatest. So a conductor having different curvatures at different points has different surface densities of charge at those points.

Unit of σ:

Class 12 Physics Unit 1 Electrostatics Chapter 1 Elementary Phenomena Of Electrostatics Unit of sigma

Dimension of σ: [σ] = L-2TI

Electrostatics

Elementary Phenomena Of Electrostatics Numerical Examples

Example 1. A hollow spherical conductor of radius 2 cm is charged with 62.8 states. Determine the surface density of charge on the inner and outer surfaces of the conductor. If the sphere is solid, what will be the values of the above quantities?
Solution:

No charge resides on the inner surface of a hollow conductor. So surface density of charge on the internal surface of the hollow sphere is zero.

⇒ \(\sigma=\frac{Q}{4 \pi r^2}\) [Q = 62.8 statC; r = 2 cm]

⇒ \(\frac{62.8}{4 \pi(2)^2}=\frac{62.8}{16 \pi}\)

= 1.249 StaC cm-2

If the sphere is a solid one, it has no interned surface. The surface area of a hollow sphere and that of a solid sphere of the same radius are equal. So the surface density of charge on the external surface of the solid sphere will be the same as that of the hollow sphere.

Example 2. 27 drops of water, each of radius 3 mm and having equal charge are combined to form a large drop. Find the ratio of the surface density of charge on the large drop to that on each small drop.
Solution:

Suppose, the charge on each small drop of water is q. So the charge in the combined drop will be 27q. In the first case, the surface density of charge,

⇒ \(\sigma_1=\frac{q}{4 \pi r^2}=\frac{q}{4 \pi(0.3)^2}\) [here, r = 3mm = 0.3 cm]

If R is the radius of the large drop, we have,

⇒ \(\frac{4}{3} \pi R^3=27 \times \frac{4}{3} \pi(0.3)^3\)

or, R = 0.9 cm.

In the second case,

⇒ \(\sigma_2=\frac{27 q}{4 \pi R^2}=\frac{27 q}{4 \pi(0.9)^2}\)

∴ \(\frac{\sigma_2}{\sigma_1}=\frac{27 q}{4 \pi(0.9)^2} \times \frac{4 \pi(0.3)^2}{q}\)

= \(\frac{27 \times 0.09}{0.81}\)

= \(\frac{3}{1}\)

σ2 :  σ1 = 3:1

Example 3. A hollow spherical conductor of radius 2 cm is electrified with 20 states. Determine the surface density of charge on the external surface of the conductor
Solution:

Surface density of charge of a spherical conductor,

⇒ \(\sigma=\frac{Q}{4 \pi r^2}\) [Q – charge on the surface of the sphere; r = radius of the sphere]

Here, Q = 20 statC; r = 2 cm

∴ \(\sigma=\frac{20}{4 \pi(2)^2}\)

= 0.398 statC.cm-2

 Electrostatics

Elementary Phenomena Of Electrostatics Action Of A Pointed Conductor

We know that if a conductor with a sharp point is charged, the surface density of charge at the pointed part becomes very high. It will then induce unlike charges of the air molecules and dust particles in the vicinity which will be attracted towards it.

Due to this attraction, they come in contact with the sharp end, and their unlike charges get neutralized.

Only the particles having charges similar to the sharp end retain their charge and are therefore repelled strongly by the pointed end.

As a result of both these processes (attraction and repulsion), the conductor gradually loses its charge through the pointed end. This is known as the discharging action of points. So a conductor should be round and without any sharp end to retain its charge for a long time.

Lightning Conductor or Lightning Arrester:

  • In 1752 AD, Benjamin Franklin experimentally proved the existence of charges in the clouds and in the atmosphere.
  • Scientists found that cosmic rays, ultraviolet rays, and the rays emitted from radioactive substances on earth, charge the air particles and water drops of the clouds.
  • Moreover, due to mutual friction of the clouds, the water drops in it become charged. Both types of charges, positive and negative, are produced.
  • A flash of lightning is nothing but a discharge of electricity along an air-tracking the sky.
  • Such discharges are possible because of the enormous difference in potential that may exist between a charged cloud and the earth or between two oppositely charged clouds.
  • The air, in the path of the lightning, is heated up due to the discharge and expands suddenly.
  • This sudden expansion highly reduces the pressure in this area and as a result, the surrounding air rushes there with a tremendous force.
  • The report of the thunder is due to these sudden expansions and contractions of the air.
  • A highly charged cloud usually causes electrostatic induction on Earth.
  • So the potential difference between cloud and ground may become high enough to start an electric discharge.
  • This is known as lightning. The sound heard just after lightning is called a thunderclap.
  • During lightning, a high current is set up from the cloud to the ground.
  • This current follows the least resistive path. For this, thunder generally strikes on trill buildings or trees.
  • To protect highrise buildings from thunderbolts, lightning conductor is used. It consists of a long and thick copper strip.
  • The upper end of it is designed with shaip points and the lower end is fixed to a metal plate buried deep m the ground.
  • The lightning conductor protects the building using the discharging action of sharp points of the conductor.
  • When a charged cloud passes above the points of the lightning conductor, it induces, unlike charge on the sharp points.
  • The like charge induced on the other end passes to the earth. The discharging action of the sharp points partially neutralizes the charge of the cloud.
  • Hence the possibility of lightning is reduced markedly.
  • For this reason, the lightning conductor is also called a copper plate lightning arrester.
  • Sometimes even after the disground charging action of the sharp points, a discharge may take place between the charged cloud and the building.
  • Then the thick copper strip provides the path of least resistance for the charge to flow to the earth without damaging the building.
  • For this reason, copper strips with sharp points are used as lightning conductors.

Class 12 Physics Unit 1 Electrostatics Chapter 1 Elementary Phenomena Of Electrostatics Lightning Conductor or Lightning Arrester

Necessary qualities of a good lightning conductor:

A good lightning conductor should have the following properties:

  • The copper strip should not melt due to heat evolved during lightning discharge.
  • The upper part of the copper strip should be provided with a sufficient number of sharp points.
  • No discontinuity should exist in the copper strip. The lower end of it should be buried deep in the ground.

Safe shelters and Unsafe places during lightning:

Buildings on metal frames and houses fitted with lightning conductors are the safest places during lightning. Staying in a car with a metal frame with windows closed also offers adequate protection, if it is connected to the earth.

On the other hand, stray tall trees, telegraph and telephone posts, high fences of mud, etc. are unsafe places at the time of lightning.

It may be noted that, although lightning and thunder take place simultaneously, the sound of thunder reaches us much later, because the velocity of sound is much less than that of light. So, if a man hears the sound of thunder, there will be no chance of his being struck by lightning.

Electrostatics

Elementary Phenomena Of Electrostatics Ncert Text Book Questions With Answer Hint

Question 1. Why can one ignore the quantization of electric charge when dealing with macroscopic i.e., large-scale charges?
Answer:

The charge at the macroscopic level is so large compared to the charge of an electron that quantization of charge has no practical importance at this level. The charge of an electron is 1.6 x 10-19C which means that a small charge of 1μC has about 1013 electrons presentinit. For such a large number of electrons, there is no significance of quantization, and should be treated as continuous.

Question 2. A polythene piece rubbed with wool is found to have a negative charge of 3 x 10-7 C.

1. Estimate the number of electrons transferred (from which to which?)
Answer:

q = ne

∴ N = \(\frac{q}{e}\)

or, \(n=\frac{-3 \times 10^{-7}}{-1.6 \times 10^{-19}}\)

= 1.875 x 1012

Here, q = -3 x 10-7C

e = -1.6 x 10-19C

So, 1.875 x 1012 electrons are transferred from wool to polythene.

2. Is there a transfer of mass from wool to polythene?
Answer:

Yes, some electrons are transferred from wool to polythene but the mass of electrons transferred is infinitesimally small, the transfer of mass may be neglected.

Electrostatics

Elementary Phenomena Of Electrostatics Conclusion

  • Due to rubbing, an object which gets the ability to attract others is called an electrified object and the process is known as electrification.
  • The principle of conservation of electric charge states that the total charge in an isolated system remains constant.
  • If electrons are added to anatomy, it will be negatively charged and if electrons are removed from an atom, it becomes positively charged.
  • This is the electronic theory of electricity.
  • According to the ability of charge to move through the materials, they are classified into two major groups:
    1. Conductor
    2. Non-conductor orinsulator.
  • Materials through which charge flows easily are called conductors. Generally, all metals are conductors.
  • Materials through which charge is not found easily are called non-conductors or insulators.
  • Dry air, glass, rubber, etc., are examples of conductors.
  • Any conductor with a non-conducting base is called an insulated conductor.
  • Friction produces simultaneously equal and opposite charges in two bodies.
  • Electrostatic induction is defined as the charging of a body temporarily by the influence of nearby charges.
  • The charge of a body that induces a charge on a conductor Is known as an inducing charge and the charge on the conductor is called Induced charge.
  • The induced charge disappears as soon as the charged body is removed.
  • The tire opposite charge Induced at the nearer end of the conductor due to electrostatic induction is called the bound charge while the similar charge induced at the far end is called the free charge.
  • Equal and opposite charges are developed simultaneously in the same body due to induction.
  • Induction precedes attraction. Charge always resides on the outer surface of a charged conductor.
  • The arrangement that shields or screens a space from external electrical Influences is called electrical shielding or screening.
  • The surface density of charge at a given point on a conductor is the amount of charge per unit area surrounding the point on the tire surface of the conductor.
  • The greater the die curvature at any point of a conductor, the greater will be die accumulation of charge at that point.
  • A lightning conductor or lightning arrester is a device used to protect a tall building from a thunderbolt.

 Electrostatics  Elementary Phenomena Of Electrostatics Useful Relations For Solving Numerical Problems

  • If Q is the charge distributed uniformly on a surface of area A, then the surface density of charge is given by,
    ⇒ \(\sigma=\frac{Q}{A}\)
  • For a spherical conductor,
    ⇒ \(\sigma=\frac{Q}{4 \pi r^2}\) (where r is the radius of the spherical conductor)

Simple Harmonic Motion Multiple Choice Questions And Answers

Simple Harmonic Motion

Question 1. A particle executing an SHM has a maximum acceleration of a and a maximum velocity of β. Then, its time period of oscillations will be

  1. \(\frac{\beta^2}{\alpha^2}\)
  2. \(\frac{\beta^2}{\alpha}\)
  3. \(\frac{\alpha}{\beta}\)
  4. \(\frac{2 \pi \beta}{\alpha}\)

Answer: 4. \(\frac{2 \pi \beta}{\alpha}\)

In an SHM, maximum acceleration = ω²A = α

and maximum velocity = ωA = β, where A = amplitude.

∴ \(\frac{\alpha}{\beta}=\omega=\frac{2 \pi}{T}\)

⇒ \(T=\frac{2 \pi \beta}{\alpha}\)

Question 2. Out of the following four functions, each representing the motion of a particle, which represents an SHM?

1. y = sin ω -cos ωt

2. y = 1 + ωt+ ω²t²

3. y = sin³ωt

4. \(y=A \cos \left(\frac{3 \pi}{4}-3 \omega t\right)\)

  1. All except (2)
  2. Both (1) and (4)
  3. Both (1) and (3)
  4. Only (1).

Answer: 2. Both (1) and (4)

For an SHM, \(\frac{d^2 y}{d t^2}=-k y\)

This is satisfied only by the given functions (1) and (4).

In anSHM, displacement = y = Asin(ω ± Φ).

(1) can also be expressed as

⇒ \(y=\sqrt{2} \sin \left(\omega t-\frac{\pi}{4}\right)\)

which is an SHM, and so is (4).

Question 3. A simple pendulum performs an SHM about x = 0 with an amplitude of a and a time period of T. The speed of its bob at x = \(\frac{a}{2}\) will be

  1. \(\frac{\sqrt{3} \pi a}{2 T}\)
  2. \(\frac{\pi a}{T}\)
  3. \(\frac{3 \pi^2 a}{T}\)
  4. \(\frac{\sqrt{3} \pi a}{T}\)

Answer: 4. \(\frac{\sqrt{3} \pi a}{T}\)

The speed at a distance x = \(\frac{a}{2}\) from the mean position (x = 0) is

⇒ \(v=\omega \sqrt{a^2-x^2}=\omega \sqrt{a^2-\frac{a^2}{4}}\)

=\(\frac{\sqrt{3}}{2} a \omega\)

=\(\frac{\sqrt{3}}{2}\left(\frac{2 \pi}{T}\right) a\)

=\(\frac{\sqrt{3} \pi a}{T}\)

Question 4. A particle executes an SHM with a time period of T. Its motion is represented by \(x=a \sin \left(\omega t+\frac{\pi}{6}\right)\) After the elapse of what fraction of time period, the velocity of the particle will be equal to half its maximum velocity?

  1. \(\frac{T}{8}\)
  2. \(\frac{T}{6}\)
  3. \(\frac{T}{3}\)
  4. \(\frac{T}{12}\)

Answer: 2. \(\frac{T}{6}\)

The instantaneous velocity is given by

⇒ \(v=\frac{d x}{d t}=a \omega \cos \left(\omega t+\frac{\pi}{6}\right)\)

Now, \(v=\frac{1}{2} v_{\max }=\frac{a \omega}{2}\)

⇒ \(\frac{a \omega}{2}=a \omega \cos \left(\omega t+\frac{\pi}{6}\right)\)

⇒ \(\frac{1}{2}=\cos \left(\omega t+\frac{\pi}{6}\right)=\cos \frac{\pi}{3}\)

⇒ \(\omega t=\frac{\pi}{3}-\frac{\pi}{6}=\frac{\pi}{6} .\)

∴ \(t=\frac{\pi}{6 \omega}\)

=\(\frac{\pi}{6\left(\frac{2 \pi}{T}\right)}\)

=\(\frac{T}{12}\)

Question 5. A particle executes an SHM with an amplitude of A and a time period of T. The minimum time taken by the particle to travel half its amplitude from the equilibrium position is

  1. \(\frac{T}{4}\)
  2. \(\frac{T}{8}\)
  3. \(\frac{T}{12}\)
  4. \(\frac{T}{2}\)

Answer: 3. \(\frac{T}{12}\)

In an SHM, x = \(A \sin \frac{2 \pi t}{T}\)

For \(x=\frac{A}{2}, \frac{A}{2}\)

=\(A \sin \frac{2 \pi t}{T}\)

⇒ \(\frac{1}{2}=\sin \frac{\pi}{6}\)

=\(\sin \frac{2 \pi t}{T}\)

∴ t = \(\frac{T}{12}\)

Question 6. Which of the following statements is true for the velocity (v) and the acceleration of a particle executing an SHM?

  1. When v is maximum, a is maximum.
  2. The value of a is zero for all values of u.
  3. When v is zero, a is zero.
  4. When v is maximum, a is zero.

Answer: 4. When v is maximum, a is zero.

In an SHM,| acceleration | = ω²x

and velocity = \(\omega \sqrt{A^2-x^2}\)

At the extreme position, x = ±A.

Now, \(|a|_{\max }=\omega^2 A \text { at } x= \pm A\)

⇒ \(|a|_{\min }=0 \quad \text { at } x=0 \text {, }\)

⇒ \(|v|_{\max }=\omega A \text { at } x=0\)

and \(|v|_{\min }=0 \quad \text { at } x= \pm A\)

Thus, v is maximum with acceleration = 0 at the equilibrium position
(x = 0).

Question 7. A simple harmonic oscillator has a displacement amplitude of a and a time period of T. The time required to travel from x = a to x = \(\frac{a}{2}\) is

  1. \(\frac{T}{6}\)
  2. \(\frac{T}{2}\)
  3. \(\frac{T}{3}\)
  4. \(\frac{T}{4}\)

Answer: 1. \(\frac{T}{6}\)

In an SHM, when we start counting time from the extreme position, the displacement is y = a cos at.

For \(y=\frac{a}{2}=a \cos \frac{2 \pi t}{T}\)

⇒ \(\frac{2 \pi t}{T}=\frac{\pi}{3}\)

⇒ \(t=\frac{T}{6}\)

Question 8. The angular velocity and the amplitude of a particle executing an SHM are co and A respectively. At a displacement of x from the mean position, the KE is K and the PE is U. Then, the ratio of K to U is equal to

  1. \(\frac{A^2-\omega^2 x^2}{\omega^2 x^2}\)
  2. \(\frac{x^2 \omega^2}{A^2-\omega^2 x^2}\)
  3. \(\frac{A^2-x^2}{x^2}\)
  4. \(\frac{x^2}{A^2-x^2}\)

Answer: 3. \(\frac{A^2-x^2}{x^2}\)

KE = \(K=\frac{1}{2} m v^2=\frac{1}{2} m \omega^2\left(A^2-x^2\right)\)

and \(\mathrm{PE}=U=\frac{1}{2} m \omega^2 x^2\)

∴ \(\frac{K}{U}=\frac{A^2-x^2}{x^2}\)

Question 9. A particle is executing an SHM along a straight line. Its velocities at the distances x1 and x2 from the mean position are v1 and v2 respectively. Its time period is

  1. \(2 \pi \sqrt{\frac{x_1^2+x_2^2}{v_1^2+v_2^2}}\)
  2. \(2 \pi \sqrt{\frac{x_2^2-x_1^2}{v_1^2-v_2^2}}\)
  3. \(2 \pi \sqrt{\frac{v_1{ }^2+v_2{ }^2}{x_1{ }^2+x_2{ }^2}}\)
  4. \(2 \pi \sqrt{\frac{v_1{ }^2-v_2{ }^2}{x_1^2-x_2{ }^2}}\)

Answer: 2. \(2 \pi \sqrt{\frac{x_2^2-x_1^2}{v_1^2-v_2^2}}\)

⇒ \(\text { At } x=x_1, v_1^2=\omega^2\left(A^2-x_1^2\right)\)…(1)

⇒ \(\text { At } x=x_2, v_2^2=\omega^2\left(A^2-x_2{ }^2\right)\)…(2)

Subtracting (2) from (1), we get

⇒ \(v_1^2-v_2^2=\omega^2\left(x_2^2-x_1^2\right)\)

⇒ \(\omega^2=\left(\frac{2 \pi}{T}\right)^2=\frac{v_1^2-v_2^2}{x_2^2-x_1^2}\)

∴ time period = \(T =2 \pi \sqrt{\frac{x_2^2-x_1^2}{v_1^2-v_2^2}}\)

Question 10. A particle executing an SHM with an amplitude of 5 m has a maximum speed of 31.4 cm s-1. The frequency of its oscillations is

  1. 3 Hz
  2. 2 Hz
  3. 4 Hz
  4. 1 Hz

Answer: 4. 1 Hz

Maximum velocity = \(v_{\max }=A \omega=A\left(\frac{2 \pi}{T}\right)=A(2 \pi f)\)

∴ frequency = \(f=\frac{v_{\max }}{A \cdot 2 \pi}\)

=\(\frac{31.4 \mathrm{~cm} \mathrm{~s}^{-1}}{(5 \mathrm{~cm}) \cdot 2(3.14)}\)

=\(\frac{10 \pi}{10 \pi} \mathrm{s}^{-1}\)

= 1Hz.

Question 11. A particle executes a linear SHM with an amplitude of 3 cm. When the particle is at 2 cm from the mean position, the numerical magnitude of its velocity is equal to that of its acceleration. Then, the time period is

  1. \(\frac{\sqrt{5}}{\pi} \mathrm{s}\)
  2. \(\frac{\sqrt{5}}{2 \pi} \mathrm{s}\)
  3. \(\frac{4 \pi}{\sqrt{5}} \mathrm{~s}\)
  4. \(\frac{2 \pi}{\sqrt{3}} \mathrm{~s}\)

Answer: 3. \(\frac{4 \pi}{\sqrt{5}} \mathrm{~s}\)

Given that amplitude = A = 3 cm.

At x = 2 cm, v = a

⇒ \(\omega \sqrt{A^2-x^2}=\omega^2 x .\)

Substituting the values,

⇒ \(\omega \sqrt{(3 \mathrm{~cm})^2-(2 \mathrm{~cm})^2}=\omega^2(2 \mathrm{~cm})\)

⇒ \(\sqrt{5} \mathrm{~cm}=\frac{2 \pi}{T}(2 \mathrm{~cm})\)

∴ time period = \(T=\frac{4 \pi}{\sqrt{5}} \mathrm{~s}\)

Question 12. When two displacements represented by the equations y1 = asinωt and y2 = bcos ωt are superposed, the motion is

  1. Not simple harmonic
  2. Simple harmonic with an amplitude of \(\frac{a}{b}\)
  3. Simple harmonic with an amplitude of \(\sqrt{a^2+b^2}\)
  4. Simple harmonic with an amplitude of \(\frac{(a+b)}{2}\)

Answer: 3. Simple harmonic with an amplitude of \(\sqrt{a^2+b^2}\)

When the two SHMs are superposed, the resultant displacement is

y = y1 + y2 = a sin ωt + b cos ωt

\(\sqrt{a^2+b^2}\left(\frac{a}{\sqrt{a^2+b^2}} \sin \omega t+\frac{b}{\sqrt{a^2+b^2}} \cos \omega t\right)\)

⇒ \(\sqrt{a^2+b^2}(\sin \omega t \cos \phi+\cos \omega t \sin \phi)\)

As shown in the adjoining figure,

⇒ \(y=\sqrt{a^2+b^2} \sin (\omega t+\phi)\)

This represents an SHM with an amplitude of

A = \(A=\sqrt{a^2+b^2}\)

Simple Harmonic Motion Multiple Choice Question And Answers Q 12

Question 13. Two simple harmonic motions of angular frequencies 100 rad s-1 and 1000 rad s-1 have the same displacement amplitude. The ratio of their maximum accelerations is

  1. 1:10
  2. 1: 10²
  3. 1: 10³
  4. 1: 104

Answer: 2. 1: 10²

The maximum acceleration is given by \(|a|_{\max }=\omega^2 A.\)

⇒ \(\frac{\left|a_1\right|_{\max }}{\left|a_2\right|_{\max }}=\left(\frac{\omega_1}{\omega_2}\right)^2\)

=\(\left(\frac{100 \mathrm{rad} \mathrm{s}^{-1}}{1000 \mathrm{rad} \mathrm{s}^{-1}}\right)^2=\frac{1}{100}\)

Hence, the required ratio is 1: 10²

Question 14. Which of the following equations of motion represents an SHM, where c, D, and a are constants?

  1. Acceleration = -cx + Dx²
  2. Acceleration = -c(x + a)
  3. Acceleration = c(x + a)
  4. Acceleration = cx

Answer: 2. Acceleration = -c(x + a)

In an SHM, the restoring force (F) is directly proportional to the negative of the displacement from the mean position.

Thus, F = -kx.

Then, acceleration = \(\frac{F}{m}=-\frac{k x}{m}=-\omega^2 x\)

This condition for an SHM is also satisfied for acceleration = -c(x + a), whereflisa constant.

Question 15. A particle executes an SHM. When the displacements from the mean position are 4 cm and 5 cm, foe corresponding velocities of the foe particle are 10 cm s-1 and 8 cm s-1 respectively. The period of oscillation of the particle is

  1. 2πs
  2. \(\frac{\pi}{2} \mathrm{~s}\)
  3. πs
  4. \(\frac{3 \pi}{2} s\)

Answer: 3. πs

Given that x1 = 4 cm, v1 = 10 m s-1, x2 = 5 cm and v2 = 8 cm s-1.

∴ time period = \(T=2 \pi \sqrt{\frac{x_2^2-x_1^2}{v_1^2-v_2^2}}\)

Substituting the values, we have

⇒ \(T=2 \pi \sqrt{\frac{5^2-4^2}{10^2-8^2}} \mathrm{~s}\)

=\(2 \pi \sqrt{\frac{9}{36}} \mathrm{~s}=\pi \mathrm{s}\)

Question 16. A simple harmonic oscillator at a given time has a displacement of 0.02 m and an acceleration equal to 2.0 m s-2. The angular frequency of the oscillator is

  1. 100 rad s-1
  2. 1 rad s-1
  3. 10 rads-1
  4. 2 rad s-1

Answer: 3. 10 rad s-1

⇒ \(|a|=\omega^2 x\)

⇒ \(\omega=\sqrt{\frac{|a|}{x}}\)

=\(\sqrt{\frac{2.0 \mathrm{~m} \mathrm{~s}^{-2}}{0.02 \mathrm{~m}}}\)

= 10 rad s-1

Question 17. If the maximum speed of a particle in an SHM (x = Asin ωt) is given by V0, what is its average speed from t = 0 to t = \(\frac{T}{4}\)?

  1. \(\frac{\pi v_0}{2}\)
  2. \(\frac{2 v_0}{\pi}\)
  3. \(\frac{\pi v_0}{4}\)
  4. \(\frac{v_0}{\sqrt{2}}\)

Answer: 2. \(\frac{2 v_0}{\pi}\)

The instantaneous speed is given by v = Aω cos ωt = v0 cos ωt.

∴ the average speed in the interval from t = 0 to t = \(\frac{T}{4}\) is

⇒ \(v_{\mathrm{av}}=\frac{1}{T / 4} \int v_0 \cos \omega t d t=\frac{4 v_0}{T}\left[\frac{\sin \omega t}{\omega}\right]_0^{T / 4}\)

=\(\frac{4 v_0}{T \omega}=\frac{2 v_0}{\pi}\)

Question 18. The time period of a simple pendulum kept in an artificial satellite orbiting the Earth is

  1. πs
  2. \(\frac{1}{\pi} \mathrm{s}\)s
  3. Zero
  4. Infinity

Answer: 4. Infinity

Any object inside an artificial satellite orbiting the earth is in a state of weightlessness, so geff = 0.

time period = T = \(2 \pi \sqrt{\frac{l}{g_{\text {eff }}}}\)

=\(2 \pi \sqrt{\frac{l}{0}}=\infty\)

Question 19. The period of oscillations of a spring-block system in an orbiting satellite is (take m = mass and k = spring constant)

  1. Zero
  2. Infinity
  3. \(2 \pi \sqrt{\frac{m}{k}}\)
  4. \(\sqrt{\frac{m g}{k}}\)

Answer: 3. \(2 \pi \sqrt{\frac{m}{k}}\)

The time period of oscillations of a spring-block system is given by \(T=2 \pi \sqrt{\frac{m}{k}}\) which is independent of the acceleration due to gravity (g).

In an orbiting satellite, even if the effective value of g is zero (condition of weightlessness), T will have the same value.

Question 20. The time period of a simple pendulum in a stationary lift is T. If the lift starts accelerating upwards with an acceleration of \(\frac{g}{3}\) the time period of the pendulum will be

  1. √3T
  2. \(\frac{T}{3}\)
  3. \(\frac{\sqrt{3} T}{2}\)
  4. \(\frac{T}{\sqrt{3}}\)

Answer: 3. \(\frac{\sqrt{3} T}{2}\)

The effective value of g in a lift accelerating upwards at a is g’ = g + a.

Here, a = \({g}{3}\) Hence,

⇒ \(T=2 \pi \sqrt{\frac{l}{g}}\)

and \(T^{\prime}=2 \pi \sqrt{\frac{l}{g+\frac{g}{3}}}\)

= \(\frac{\sqrt{3}}{2}\left(2 \pi \sqrt{\frac{l}{g}}\right)\)

= \(\frac{\sqrt{3} T}{2}\)

Question 21. Let an SHM be represented by the equation y = Asin 4πt + Bcos 4πt, where y, A, and B are in meters. The displacement amplitude of oscillations is

  1. A + B
  2. A-B
  3. \(\sqrt{A^2+B^2}\)
  4. \(\sqrt{A^2-B^2}\)

Answer: 3. \(\sqrt{A^2+B^2}\)

For an SHM, the displacement at a time t may be given by

x = a sin ωt + b cos ωt,

for which the displacement amplitude is \(\sqrt{a^2+b^2}\)

Here, y = A sin ωt + B cos ωt, where ω = 4π

∴ amplitude = \(\sqrt{A^2+B^2}\)

Question 22. A horizontal platform is executing an SHM in the vertical direction ’ with a frequency of f. A block of mass m is placed on the platform. What is the maximum amplitude of the platform so that the block is not detached from it?

  1. \(\frac{g}{4 \pi^2 f^2}\)
  2. \(\frac{g}{2 \pi^2 f^2}\)
  3. \(\frac{m g}{2 \pi^2 f^2}\)
  4. \(\frac{m g}{4 \pi^2 f^2}\)

Answer: 1. \(\frac{g}{4 \pi^2 f^2}\)

During the upward motion of the platform, the acceleration (a = -ω² x) acts downwards. At the uppermost extreme point, when the platform starts descending, acceleration changes its sign, so the weight (mg) and the force mamax = mω²A act in opposite directions. For the block not to detach from the platform, mω²A ≤ mg.

∴ maximum amplitude = \(A_{\max }=\frac{g}{\omega^2}=\frac{g}{4 \pi^2 f^2}\)

Question 23. A block is placed on a rough horizontal platform. The platform is executing an SHM of time period T in the horizontal plane. The coefficient of static friction between the block and the platform is mmm The maximum amplitude of the platform for which the block does not slip on the surface of the platform is

  1. \(\frac{\mu g T}{2 \pi}\)
  2. \(\frac{\mu g T^2}{2 \pi^2}\)
  3. \(\frac{\mu g T^2}{4 \pi^2}\)
  4. μgT²

Answer: 3. \(\frac{\mu g T^2}{2 \pi^2}\)

The force of limiting friction is fmax = μN = mμg.

For the block not to slip at the extreme position, fmax > mω²A.

For the maximum amplitude,

⇒ \(m \omega^2 A_{\max }=f_{\max }=\mu m g\)

⇒ \(A_{\max }=\frac{\mu g}{\omega^2}=\frac{\mu g T^2}{4 \pi^2}\)

Question 24. A tunnel is drilled along the diameter of the earth and a stone is dropped into the tunnel. Then,

  1. The stone will reach the center of the earth and stop there
  2. The stone will reach the other end of the earth and stop there
  3. The stone will execute an SHM about the center of the earth
  4. The stone will reach the other side of the tunnel and escape the gravitational pull of the earth

Answer: 3. The stone will execute an SHM about the center of the earth

The speed of the stone goes on increasing and gets its maximum value at the center of the earth. Due to inertia, it moves further away from the centre, and the speed goes on decreasing to become zero at the other end. Thus, the stone undergoes an SHM with the centre as the origin, the radius R as the amplitude, and 2R as the path length.

Question 25. In the preceding problem, if R is the radius of the earth and g is the acceleration due to gravity at the earth’s surface, the period of oscillations of the stone will be

  1. \(\frac{R}{g}\)
  2. \(2 \sqrt{R g}\)
  3. \(2 \pi \sqrt{\frac{R}{g}}\)
  4. \(\frac{1}{2 \pi} \sqrt{\frac{R}{g}}\)

Answer: 3. \(2 \pi \sqrt{\frac{R}{g}}\)

Let M = mass of the earth and R = radius of the earth. When the stone during its fall be at P, where OP = x, the force due to gravity is

⇒ \(F=\frac{G(\text { mass of the inner core }) \cdot m}{O P^2}\)

⇒ \(\frac{G m}{x^2} \cdot \frac{4}{3} \pi x^3 \cdot \frac{M}{\frac{4}{3} \pi R^3}\) [∵ \(\text { density }=\frac{M}{\frac{4}{3} \pi R^3}\)]

⇒ \(\frac{G M m x}{R^3}\)

Simple Harmonic Motion Multiple Choice Question And Answers Q 25

∴ acceleration = \(a=\frac{F}{m}=-\frac{G M x}{R^3}\) [∵ f and x are in opposite directions]

⇒ \(-\frac{g x}{R}\)

Thus, a ∝ -x.

Hence, the motion is an SHM for which

⇒ \(\omega^2=\frac{g}{R}\)

⇒ \(\omega=\frac{2 \pi}{T}=\sqrt{\frac{g}{R}}\)

⇒ \(T=2 \pi \sqrt{\frac{R}{g}}\)

Question 26. The time period of a simple pendulum of infinite length is

  1. Infinity
  2. \(2 \pi \sqrt{\frac{R}{g}}\)
  3. \(2 \pi \sqrt{\frac{2 R}{g}}\)
  4. \(2 \pi \sqrt{\frac{R}{2 g}}\)

Answer: 2. \(2 \pi \sqrt{\frac{R}{g}}\) Let the thread of infinite length be attached to the pendulum bob of mass m. When the bob is displaced from O to A, the weight (mg) acts radially along AC, whose component along AO \(\left(m g \sin \theta=m g \frac{O A}{A C}=\frac{m g}{R} \cdot x\right)\) provides the restoring force for the SHM.

Thus, the acceleration is

⇒ \(a=\frac{F}{m}=-\frac{g}{R} \cdot x\)….(1)

Note that the minus sign is due to the force along AO and the displacement along OA. Further, threads have been shown parallelly as die threads are of infinite lengths.

Comparing (1) with the acceleration in an SHM (a = -ω²x), we have

⇒ \(\omega^2=\frac{g}{R} \Rightarrow T=2 \pi \sqrt{\frac{R}{g}}\)

Simple Harmonic Motion Multiple Choice Question And Answers Q 26

Question 27. A cubical block of wood (density = pw) of length I and mass M floats in a liquid,(density = pliq). It is slightly pressed vertically and then released. It will execute an SHM. The period of oscillation is

  1. \(2 \pi \sqrt{\frac{L \rho_{\text {liq }}}{\rho_w g}}\)
  2. \(2 \pi \sqrt{\frac{M}{\rho_{\mathrm{w}} g}}\)
  3. \(2 \pi \sqrt{\frac{L \rho_{\mathrm{w}}}{\rho_{\mathrm{liq}} g}}\)
  4. \(2 \pi \sqrt{\frac{L}{g}}\)

Answer: 3. \(2 \pi \sqrt{\frac{L \rho_{\mathrm{w}}}{\rho_{\mathrm{liq}} g}}\)

Let the wooden block be displaced vertically down by x so that the force of buoyancy, or upthrust, \(\left(L^2 x\right) \rho_{\mathrm{liq}} g\) acts upwards.

∴ \(F=M a=-\left(L^2 \rho_{\text {liq }} g\right) x\)

⇒ \(a=-\left(\frac{L^2 \rho_{\mathrm{liq}} g}{M}\right) x=-\left(\frac{\rho_{\mathrm{liq}} g}{L \rho_{\mathrm{w}}}\right) x\)

[∵ M = L³pw]

Since a ∝ -X, the motion is an SHM, whose time period is

⇒ \(T=2 \pi \sqrt{\frac{L \rho_{\mathrm{w}}}{\rho_{\mathrm{liq} g} g}}\)

Question 28. A particle executing an SHM has its KE = K0 COS²ωt. The maximum values of the potential energy and the total energy are respectively

  1. 0 and 2K0
  2. \(\frac{K_0}{2} \text { and } K_0\)
  3. K0 and K0
  4. K0 and 2K0

Answer: 3. K0 and K0

KE = K0 COS² ωt, so the maximum KE is K0.

Since the total energy is conserved,

KEmax = PEmax = total energy = K0.

∴ PEmax = K0 and total energy = K0.

Question 29. The potential energy of a simple harmonic oscillator when the particle is halfway to its end point is (where E is the total energy)

  1. \(\frac{E}{2}\)
  2. \(\frac{E}{4}\)
  3. \(\frac{2E}{3}\)
  4. \(\frac{E}{8}\)

Answer: 2. \(\frac{E}{4}\)

At a distance x from the mean position, PE = \(\frac{1}{2} m \omega^2 x^2\)

For \(x=\frac{A}{2}, \mathrm{PE}=\frac{1}{2} m \omega^2 \frac{A^2}{4}\)

=\(\frac{1}{4}\left(\frac{1}{2} m \omega^2 A^2\right)\)

=\(\frac{\text { total energy }}{4}\)

=\(\frac{E}{4}\)

Question 30. In an SHM, when the displacement is half the displacement amplitude, what fraction of the total energy is kinetic?

  1. \(\frac{1}{2}\)
  2. \(\frac{1}{4}\)
  3. \(\frac{3}{4}\)
  4. \(\frac{2}{3}\)

Answer: 3. \(\frac{3}{4}\)

The KE in an SHM is \(\frac{1}{2} m \omega^2\left(A^2-x^2\right)\)

AT \(x=\frac{A}{2}, \mathrm{KE}=\frac{1}{2} m \omega^2\left(A^2-\frac{A^2}{4}\right)\)

=\(\frac{3}{4}\left(\frac{1}{2} m \omega^2 A^2\right)\)

⇒ \(\frac{3}{4}(\text { total energy })\)

=\(\frac{3 E}{4}\)

∴ \(\frac{\mathrm{KE}}{\text { total energy }}=\frac{\frac{3 E}{4}}{E}\)

=\(\frac{3}{4}\)

Question 31. A simple pendulum is displaced to an angle from its equilibrium position and then released. While passing through the equilibrium position, its velocity v will be

  1. \(\sqrt{2 g l}\)
  2. \(\sqrt{2 g l \cos \theta}\)
  3. \(\sqrt{2 g l \sin \theta}\)
  4. \(\sqrt{2 g l(1-\cos \theta)}\)

Answer: 4. \(\sqrt{2 g l(1-\cos \theta)}\)

When released from the angular position 0, the loss in PE is

mg(CA) = mg(OA-OC)

= mg(l-l cosθ)

= mgl(1-cosθ).

This loss in PE appears as KE = \(\frac{1}{2}\) mv2.

∴ \(\frac{1}{2}\)mv²= mgl(1-cos θ)

⇒ \(v=\sqrt{2 g l(1-\cos \theta)}\)

Simple Harmonic Motion Multiple Choice Question And Answers Q 31

Question 32. The displacement of a particle (executing an SHM) between the positions of the maximum PE and the maximum KE are (A being the displacement amplitude)

  1. \(\pm \frac{A}{2}\)
  2. ±A
  3. ±2A
  4. ±1

Answer: 2. ±2A

In an SHM, the maximum KE is obtained at x = 0, and the position for the maximum PE is x = ±A.

Hence, the required displacement is ±A.

Question 33. In an SHM, the restoring force is F = -kx, where k is the force constant, x is the displacement from the mean position and a is the amplitude. Then, the total mechanical energy depends upon

  1. k, a and m
  2. k, x and m
  3. k and a
  4. k and x

Answer: 3. k and a

Since F = -kx, the potential energy is

⇒ \(U=-W\)

⇒ \(=-\int d W\)

⇒ \(=-\int F d x\)

⇒ \(=-\int_0^x(-k x) d x\)

⇒ \(=\frac{1}{2} k x^2\)

∴ total mechanical energy = E = maximum PE

⇒ \(\frac{1}{2} k x^2\)

=\(\frac{1}{2} k a^2\) [∵ x = a]

Thus, the total energy depends on k and a.

Question 34. A force of 6.4 N stretches a vertical spring by 0.1 m. The mass that must be suspended from the spring s6 that it oscillates with a period of (π/4) s is

  1. \(\frac{\pi}{4} \mathrm{~kg}\)
  2. 1 kg
  3. \(\frac{1}{\pi} \mathrm{kg}\)
  4. 10 kg

Answer: 2. 1 kg

Since F = kx, the force constant is

⇒ \(k=\frac{F}{x}=\frac{6.4 \mathrm{~N}}{0.1 \mathrm{~m}}=64 \mathrm{~N} \mathrm{~m}^{-1}\)

The time period of the spring-block system is given by

⇒ \(T=2 \pi \sqrt{\frac{m}{k}} \Rightarrow \frac{\pi}{4} s=2 \pi \sqrt{\frac{m}{64}}=\frac{2 \pi \sqrt{m}}{8}\)

m = 1 kg.

Question 35. The periodic time of oscillations of a spring-block system of mass M is T. If the mass is increased to 2M, the time of oscillations T will be equal to

  1. 2T
  2. √2T
  3. \(\frac{T}{\sqrt{2}}\)
  4. T

Answer: 2. √2T

For a spring-block system,

⇒ \(T=2 \pi \sqrt{\frac{M}{k}}\)

and \(T^{\prime}=2 \pi \sqrt{\frac{2 M}{k}}\)

= \(\sqrt{2}\left(2 \pi \sqrt{\frac{M}{k}}\right)\)

= \(\sqrt{2} T\)

Question 36. A block of mass m is attached to the lower end of a spring whose upper end is fixed to a rigid support. The spring has a negligible mass. When the block is slightly pulled down and released it oscillates with a time period of 3 s. When the mass is increased by 1 kg, the time period of oscillations increases to 5 s. The value of mis

  1. \(\frac{3}{4}\) kg
  2. \(\frac{4}{3}\) kg
  3. \(\frac{16}{9}\) kg
  4. \(\frac{9}{16}\) kg

Answer: 4. \(\frac{9}{16}\) kg

With mass = m, T = \(2 \pi \sqrt{\frac{m}{k}}\)

= 3s.

With mass = m + 1 kg,

⇒ \(T^{\prime}=2 \pi \sqrt{\frac{m+1 \mathrm{~kg}}{k}}\)

= 5s.

∴ \(\frac{T^{\prime}}{T}=\frac{5 \mathrm{~s}}{3 \mathrm{~s}}\)

=\(\sqrt{\frac{m+1 \mathrm{~kg}}{m}}\)

⇒ \(m=\frac{9}{16} \mathrm{~kg}\)

Question 37. A block of mass fn is suspended vertically from a light spring which oscillates with a frequency f. With what frequency will the system oscillate if another block of mass 4m is suspended from the same spring?

  1. \(\frac{f}{4}\)
  2. 4f
  3. \(\frac{f}{2}\)
  4. 2f

Answer: 3.

Time period = \(T=2 \pi \sqrt{\frac{m}{k}}, \text { so frequency }=f=\frac{1}{T}\)

=\(\frac{1}{2 \pi} \sqrt{\frac{k}{m}}\)

When the mass is changed from m to 4m, the frequency becomes

⇒ \(f^{\prime}=\frac{1}{2 \pi} \sqrt{\frac{k}{4 m}}\)

=\(\frac{1}{2}\left(\frac{1}{2 \pi} \sqrt{\frac{k}{m}}\right)\)

=\(\frac{f}{2}\)

Question 38. A mass m is suspended successively by two springs of force constants and separately. The time periods of oscillations in die two cases are T1 and T2 respectively. If the same mass is suspended by connecting the two springs in parallel (as shown in the figure), the time period of oscillations is T. The correct relation is

Simple Harmonic Motion Multiple Choice Question And Answers The time periods of oscillation Q 38

  1. T2 = T1² + T2²
  2. \(\frac{1}{T}=\frac{1}{T_1}+\frac{1}{T_2}\)
  3. T = T1 + T2
  4. \(T^{-2}=T_1^{-2}+T_2^{-2}\)

Answer: 4. \(T^{-2}=T_1^{-2}+T_2^{-2}\)

With the first spring, \(T_1=2 \pi \sqrt{\frac{m}{k_1}}\)

With the second spring, \(T_2=2 \pi \sqrt{\frac{m}{k_2}}\)

With the two springs in parallel, the equivalent spring constant is k = k1 + k2, so the time period is

⇒ \(T=2 \pi \sqrt{\frac{m}{k_1+k_2}}\)

∴ \(\frac{1}{T_1^2}=\frac{1}{4 \pi^2} \cdot \frac{k_1}{m} \text { and } \frac{1}{T_2^2}=\frac{1}{4 \pi^2} \cdot \frac{k_2}{m}\)

∴ \(\frac{1}{T^2}=\frac{1}{4 \pi^2} \cdot \frac{k_1+k_2}{m}=\frac{1}{T_1^2}+\frac{1}{T_2^2}\)

⇒ \(T^{-2}=T_1^{-2}+T_2^{-2}\)

Question 39. A block having mass m = 2 kg is placed on a flat pan attached to a vertical spring fixed on the ground, as shown in the figure. The combined mass of the pan and the spring is negligible. When pressed slightly and released, the bloc executes an SHM. The spring constant is k = 200 N m-1. What should be the minimum amplitude of the motion so that the block gets detached from the pan? (Take g = 10 m s-2.)

Simple Harmonic Motion Multiple Choice Question And Answers The combined mass of the pan Q 39

  1. 8.0 cm
  2. 10.0 cm
  3. 4.0 cm
  4. Any value less than 12 cm

Answer: 2. 10.0 cm

The pan (as a frame) while moving upwards gets decelerated, i.e., its acceleration (a = -ω²x) is directed downwards. So, the pseudo force is

⇒ \(\vec{F}_{\mathrm{ps}}=(\text { mass of the block })\left(-\vec{a}_{\mathrm{fr}}\right)\)

which is vertically upwards.

Maximum pseudoforce = mamax = mω²A,

where A is the displacement amplitude.

For no detachment, m ω²A < mg. So, the minimum amplitude is

⇒ \(A=\frac{g}{\omega^2}=\frac{g}{\frac{k}{m}}=\frac{m g}{k}\)

Substituting the values,

⇒ \(A=\frac{(2 \mathrm{~kg})\left(10 \mathrm{~m} \mathrm{~s}^{-2}\right)}{200 \mathrm{~N} \mathrm{~m}^{-1}}\)

= 0.1 m

= 10.0 cm.

Question 40. What is the expression for the time period (T) of the block-spring system executing an SHM smooth horizontal surface?

Simple Harmonic Motion Multiple Choice Question And Answers Q 40

  1. \(T=2 \pi \sqrt{\frac{m}{k_1+k_2}}\)
  2. \(T=2 \pi \sqrt{\frac{k_1+k_2}{m}}\)
  3. \(T=2 \pi \sqrt{\frac{k_1 k_2 m}{k_1+k_2}}\)
  4. \(T=2 \pi \sqrt{\frac{\left(k_1+k_2\right) m}{k_1 k_2}}\)

Answer: 1. \(T=2 \pi \sqrt{\frac{m}{k_1+k_2}}\)

Let the block be displaced on either side by x so that the first spring gets extended by x and the second one gets compressed by x as shown in the adjoining figure.

The net force towards the left is (k1 + k2)x.

⇒ \(|a|=\frac{F}{m}=\left(\frac{k_1+k_2}{m}\right) x\)

=\(\omega^2 x\)

⇒ \(T=\frac{2 \pi}{\omega}\)

=\(2 \pi \sqrt{\frac{m}{k_1+k_2}}\)

Question 41. In the adjoining figure, two blocks, each of mass m, are connected to the two ends of a light spring of force constant K. Thereisnofriction anywhere. If the blocks are displaced slightly in opposite directions and then released, they will execute an SHM with a frequency f, which is equal to

Simple Harmonic Motion Multiple Choice Question And Answers Q 41

  1. \(\frac{1}{2 \pi} \sqrt{\frac{k}{2 m}}\)
  2. \(\frac{1}{2 \pi} \sqrt{\frac{2 k}{m}}\)
  3. \(\frac{1}{2 \pi} \sqrt{\frac{m}{k}}\)
  4. \(\frac{1}{2 \pi} \sqrt{\frac{k}{m}}\)

Answer: 2. \(\frac{1}{2 \pi} \sqrt{\frac{2 k}{m}}\)

In the absence of external forces, the center of mass of the system (lying at the center of the spring) will remain fixed. For the force constant k of a half spring,

⇒ \(k^{\prime}\left(\frac{l}{2}\right)=k l \Rightarrow k^{\prime}=2 k\)

With the mass m and the spring constant k – 2k, the time period is,

⇒ \(T=2 \pi \sqrt{\frac{m}{2 k}}\)

and the frequency is,

⇒ \(f=\frac{1}{2 \pi} \sqrt{\frac{2 k}{m}}\)

Question 42. Two blocks A and B of respective masses m and 2m are attached to the two ends of a light spring of force constant k. If the blocks are pulled apart in opposite directions and released, the system of the blocks oscillates with a time period T. Find the expression for T. (Assume the friction to be negligible.)

Simple Harmonic Motion Multiple Choice Question And Answers Q 42

  1. \(2 \pi \sqrt{\frac{2 m}{k}}\)
  2. \(2 \pi \sqrt{\frac{m}{k}}\)
  3. \(2 \pi \sqrt{\frac{2 m}{3 k}}\)
  4. \(2 \pi \sqrt{\frac{m}{2 k}}\)

Answer: 3. \(2 \pi \sqrt{\frac{2 m}{3 k}}\)

Taking block A as the origin, the position of the centre of mass of the system is

⇒ \(x_{\mathrm{CM}}=\frac{m \cdot 0+2 m \cdot l}{m+2 m}=\frac{2}{3} l\)

This position remains fixed in the absence of external forces.

The equivalent force constant is

⇒ \(\left(\frac{2}{3} l\right) k^{\prime}=l \cdot k\)

⇒ \(k^{\prime}=\frac{3}{2} k\)

∴ time period = \(T=2 \pi \sqrt{\frac{m}{k^{\prime}}}\)

=\(2 \pi \sqrt{\frac{2 m}{3 k}}\)

Question 43. Two springs of force constants k1 are connected to a block of mass m placed on a frictionless surface, as shown in the figure. When the force constants of both the springs have the same value k, the frequency of oscillations of the block is

Simple Harmonic Motion Multiple Choice Question And Answers frictionless sutrface Q 43

  1. \(\frac{1}{2 \pi} \sqrt{\frac{k}{m}}\)
  2. \(\frac{1}{2 \pi} \sqrt{\frac{k}{2 m}}\)
  3. \(\frac{1}{2 \pi} \sqrt{\frac{2 k}{m}}\)
  4. \(\frac{1}{2 \pi} \sqrt{\frac{m}{k}}\)

Answer: 2. \(\frac{1}{2 \pi} \sqrt{\frac{k}{2 m}}\)

For die given system of springs connected in series, the equivalent spring constant V is given by

⇒ \(\frac{1}{k^{\prime}}=\frac{1}{k_1}+\frac{1}{k_2}=\frac{k_1+k_2}{k_1 k_2}\)

⇒ \(k^{\prime}=\frac{k_1 k_2}{k_1+k_2}\)

∴ frequency = \(f=\frac{1}{2 \pi} \sqrt{\frac{k^{\prime}}{m}}=\frac{1}{2 \pi} \sqrt{\frac{k_1 k_2}{\left(k_1+k_2\right) m}}\)

When \(k_1=k_2=k, f=\frac{1}{2 \pi} \sqrt{\frac{k}{2 m}}\)

Question 44. In the preceding question, the frequency of oscillations of the block of mass m connected to a single spring horizontally is 4 Hz. When the spring is replaced by two identical springs, as shown in the figure, die effective frequency will be

  1. 4√2 Hz
  2. 1.5 Hz
  3. 1.31 Hz
  4. 2√2 Hz

Answer: 4. 2√2 Hz

The frequency of the spring-block system is \(f=\frac{1}{2 \pi} \sqrt{\frac{k}{m}}, \text { so } f \propto \sqrt{k}\)

With one single spring, f ∝ Vk.

With two identical springs in series,

⇒ \(f^{\prime} \propto \sqrt{\frac{k}{2}}\)

∴ \(\frac{f^{\prime}}{f}=\frac{\sqrt{\frac{k}{2}}}{\sqrt{k}}\)

⇒ \(\frac{1}{\sqrt{2}}\)

⇒ \(f^{\prime}=\frac{1}{\sqrt{2}} f\)

=\(\frac{1}{\sqrt{2}}(4 \mathrm{~Hz})\)

=\(2 \sqrt{2} \mathrm{~Hz}\)

Question 45. Four massless springs, whose force constants are 2k, 2k, k, and 2k, are attached to a block of mass M as shown in the figure. If the block is displaced in the horizontal direction to either 2k side released, the frequency of oscillations of the block is

Simple Harmonic Motion Multiple Choice Question And Answers massless springs Q 45

  1. \(\frac{1}{2 \pi} \sqrt{\frac{k}{7 M}}\)
  2. \(\frac{1}{2 \pi} \sqrt{\frac{k}{4 M}}\)
  3. \(\frac{1}{2 \pi} \sqrt{\frac{4 k}{M}}\)
  4. \(\frac{1}{2 \pi} \sqrt{\frac{7 k}{M}}\)

Answer: 3. \(\frac{1}{2 \pi} \sqrt{\frac{4 k}{M}}\)

The equivalent force constant to the left of the block is k (for the series combination) and that to the right is 2k + k = 3k. These taken together constitute a parallel combination for which the force constant is

keq = k + 3k = 4k and thus frequency = \(f=\frac{1}{2 \pi} \sqrt{\frac{k_{\text {eq }}}{M}}=\frac{1}{2 \pi} \sqrt{\frac{4 k}{M}}\)

Question 46. A particle executes an SHM between the extreme positions x = x1 and x = x2, the equilibrium position being O. Which of the following graphs correctly represents the variation of potential energy with a position?

Simple Harmonic Motion Multiple Choice Question And Answers variation of potential energy Q 46

Answer: 4.

Simple Harmonic Motion Multiple Choice Question And Answers variation of potential energy Q 46.

In an SHM, potential energy \(U=\frac{1}{2} k x^2=\frac{1}{2} m \omega^2 x^2\)

This is positive for both negative and positive values of x, and zero at the mean position (x = 0). This is given in the option (d).

Question 47. Which of the following graphs correctly represents the variation of acceleration with time (t) for a particle executing a simple harmonic motion for which the displacement is x = Acos ωt?

Simple Harmonic Motion Multiple Choice Question And Answersharmonic motion Q 47.

Answer: 3.

Simple Harmonic Motion Multiple Choice Question And Answersharmonic motion Q 47

The given expression for displacement in the SHM is x = Acos ωt, represented in the adjoining graph 1. On differentiation, we get acceleration

⇒ \(a=\frac{d^2 x}{d t^2}=-\omega^2 A \cos \omega t\)

= -a0 cos ωt.

This is just the mirror image of Graph 1 about the central line (x = 0), as shown in Graph 2.

Simple Harmonic Motion Multiple Choice Question And Answers Q 47

Question 48. The variation of the acceleration of a particle in an SHM with its displacement (x) is shown by the graph

Simple Harmonic Motion Multiple Choice Question And Answers the variation of the acceleratio Q 48

Answer: 3.

Simple Harmonic Motion Multiple Choice Question And Answers the variation of the acceleratio Q 48.

The acceleration in an SHM is given by a ∝ -x

=> a = -kx.

The a-x graph is a straight line with a negative slope and passing through the origin, as shown in the option (3)

Question 49. The displacement of a particle from the mean position is given by y = asin 4π(2t + 0.4). The time period of oscillations is

  1. 0.25 s
  2. 0.2s
  3. 1.0 s
  4. 2.0 s

Answer: 1. 0.25 s

In the given equation,

y = asin4 π(2t + 0.4)

= a sin(8πt +1.6π) = a sin(ωt + 0).

Thus, to = \(\frac{2 \pi}{T}\) = 8πt

T = \(\frac{1}{4}\) s

= 0.25 s.

Question 50. A particle executes an SHM such that its acceleration is given by a = -bx, where x is its displacement from the mean position. The time period of oscillations is

  1. 2π√b
  2. \(\frac{2 \pi}{\sqrt{b}}\)
  3. \(\frac{2 \pi}{b}\)
  4. \(\sqrt{\frac{2 \pi}{b}}\)

Answer: 2. \(\frac{2 \pi}{\sqrt{b}}\)

For an SHM, acceleration-a = -ω²x.

Comparing this with the given equation a = -bx, we get

⇒ \(\omega^2=b \Rightarrow \omega=\frac{2 \pi}{T}=\sqrt{b}\)

⇒ \(T=\frac{2 \pi}{\sqrt{b}}\)

Question 51. A particle executes an SHM with a frequency f. The frequency with which its KE oscillates is

  1. \(\frac{f}{2}\)
  2. f
  3. 2f
  4. 4f

Answer: 3. 2f

For an SHM, x = asin ωt

v = aω cos ωt

and KE = E = \(\frac{1}{2}\) mv²

From (1), the frequency of oscillations is to, while. the frequency of variation of KE is 2

This is also clear in the given graph. The x-t graph completes one cycle in the time period T, whereas KE completes two complete oscillations (one from O to \(\frac{T}{2}\) and the other from \({T}{2}\) to T.

Simple Harmonic Motion Multiple Choice Question And Answers Q 51

Question 52. The displacement of a particle executing an SHM is expressed by the equation y = A0 + A sin ωt + B cos ωt. Then, the amplitude of its oscillations is

  1. \(A_0+\sqrt{A^2+B^2}\)
  2. \(\sqrt{A^2+B^2}\)
  3. \(\sqrt{A_0^2+(A+B)^2}\)
  4. A + B

Answer: 2. \(\sqrt{A^2+B^2}\)

The given equation represents an SHM with a displaced origin of displacement amplitude \(\), as shown below.

y= A0+ A sin ωt + Bcos ωt

⇒ \(A_0+\sqrt{A^2+B^2}\left(\frac{A}{\sqrt{A^2+B^2}} \sin \omega t+\frac{B}{\sqrt{A^2+B^2}} \cos \omega t\right)\)

⇒ \(A_0+\sqrt{A^2+B^2}(\sin \omega t \cos \phi+\cos \omega t \sin \phi)\)

⇒ \(A_0+\sqrt{A^2+B^2} \sin (\omega t+\phi)\)

where \(\phi=\tan ^{-1}\left(\frac{B}{A}\right)\) from the triangle.

The amplitude \(\sqrt{A^2+B^2}\) is always associated with the sinusoidal function.

Simple Harmonic Motion Multiple Choice Question And Answers Q 52

Question 53. The average velocity of a particle executing an SHM in one r complete oscillation is

  1. \(\frac{A \omega}{2}\)
  2. \(\frac{A \omega^2}{2}\)
  3. Zero

Answer: 4. Zero

An SHM is expressed by y = Asin ωt and its velocity is given by

⇒ \(v=\frac{d y}{d t}=A \omega \cos \omega t\)

We know that the average value of any sinusoidal function (sine, cosine or any of their combinations) in one complete cycle (t = 0 to t = T) is always zero.

So, v = Aω cos ωt being sinusoidal has a zero average velocity

Question 54. The radius of a circle, the period of revolutions, the initial position, and the sense of revolutions are indicated in the adjoining figure. The projection of the rotating radius vector \(\overrightarrow{\mathrm{OP}}\) on the y-axis is given by

Simple Harmonic Motion Multiple Choice Question And Answers the radious of a circle Q 54

  1. y(t) = -3cos 2nt, where y is in metres
  2. \(y(t)=4 \sin \frac{\pi t}{2}\), where y is in metres
  3. \(y(t)=3 \cos \frac{3 \pi t}{2}\), where y is in metres
  4. \(y(t)=3 \cos \frac{\pi t}{2}\), where y is in metres

Answer: 4. \(y(t)=3 \cos \frac{\pi t}{2}\), where y is in metres

The projection of a uniform circular motion on its diameter executes an SHM with an amplitude of A = radius of the circle. Here, at t = 0, a particle is at the extreme position. So, the instantaneous position (y) will be a cosine function.

Further, T = 4 s.

So,

⇒ \(\omega=\frac{2 \pi}{T}=\frac{\pi}{2} \mathrm{rad} \mathrm{s}^{-1}\)

Hence, \(y=A \cos \omega t=3 \cos \frac{\pi t}{2}\)

Simple Harmonic Motion Multiple Choice Question And Answers uniform circle motion Q 54

Question 55. In a damped oscillation, the graph between velocity (v) and position (x)will be

Simple Harmonic Motion Multiple Choice Question And Answers graph between velocity and position Q 55

Answer: 3.

Simple Harmonic Motion Multiple Choice Question And Answers graph between velocity and position Q 55.

In a damped SHM, the velocity changes periodically between the extreme positive and negative values at the origin with the zero values at the two extremes. Due to damping, the velocity amplitude goes on decreasing. This variation corresponds to the option (3).

Question 56. For the system given in the adjoining figure, M = 1 kg, R = 2 cm, m = 250 g, and k = 100 N m-1. The angular frequency of the spring-block system will be

Simple Harmonic Motion Multiple Choice Question And Answers frequency of the spring block system Q 56

  1. \(\frac{20}{\sqrt{3}} \mathrm{rad} \mathrm{s}^{-1}\)
  2. 10√3 rad s-1
  3. \(\frac{10}{\sqrt{3}} \mathrm{rad} \mathrm{s}^{-1}\)
  4. 20√3 rad s-1

Answer: 1. \(\frac{20}{\sqrt{3}} \mathrm{rad} \mathrm{s}^{-1}\)

From the law of conservation of energy,

⇒ \(\frac{1}{2} m v^2+\frac{1}{2} I \omega^2+\frac{1}{2} k x^2=\text { constant }\)

Differentiating the above equation with respect to time,

⇒ \(\frac{1}{2} m \cdot 2 v \cdot \frac{d v}{d t}+\frac{1}{2} I \cdot 2 \omega \frac{d \omega}{d t}+\frac{1}{2} k \cdot 2 x \frac{d x}{d t}=0\)

⇒ \(m v a+I \omega \alpha+k x v=0 \Rightarrow m v a+\frac{1}{2} M R^2 \cdot \frac{v}{R} \cdot \frac{a}{R}+k x v=0\)

⇒ \(a\left(m+\frac{M}{2}\right)=-k x \Rightarrow a=-\left(\frac{k}{m+\frac{M}{2}}\right) x \Rightarrow a=-\omega^2 x\)

This represents an SHM with an angular frequency of

⇒ \(\omega=\sqrt{\frac{k}{m+\frac{M}{2}}}=\sqrt{\frac{100 \mathrm{Nm}^{-1}}{\frac{1}{4} \mathrm{~kg}+\frac{1}{2} \mathrm{~kg}}}\)

⇒ \(\sqrt{\frac{400}{3}} \mathrm{~s}^{-1}=\frac{20}{\sqrt{3}} \mathrm{rad} \mathrm{s}^{-1}\)

Question 57. The spring constant is given by k = 100 N m-1. The system executes an SHM with an amplitude of A. Find the maximum amplitude Amax so, that block A does not slip over B.

Simple Harmonic Motion Multiple Choice Question And Answers spring constant Q 57

  1. 2 cm
  2. 6 cm
  3. 8 cm
  4. 4 cm

Answer: 2. 6 cm

For no slipping between the blocks A and B, the limiting static friction is

⇒ \(f_{\max }=\mu \delta V \geq F_{\mathrm{ps}} \Rightarrow \mu m_{\mathrm{A}} g \geq m_{\mathrm{A}} a_{\mathrm{fr}}\)….(1)

For the combined system, (mA +mB)a = -kx.

∴ maximum acceleration = \(\frac{k A}{m_A+m_B}, \text { where } A=\text { amplitude }\)

∴ \(A \leq \frac{\mu g\left(m_{\mathrm{A}}+m_{\mathrm{B}}\right)}{k}\)

maximum amplitude = \(\frac{0.4 \times 10 \times 1.5}{100} \mathrm{~m}=6 \mathrm{~cm}\)

Question 58. A particle executes an SHM with an amplitude of 5 cm. When the particle is at 4 cm from the mean position, the numerical value of the magnitude of its velocity in SI units is equal to that of its acceleration. Its periodic time is

  1. \(\frac{8 \pi}{3} \mathrm{~s}\)
  2. \(\frac{3 \pi}{8} \mathrm{~s}\)
  3. \(\frac{7 \pi}{4} \mathrm{~s}\)
  4. \(\frac{4 \pi}{3} \mathrm{~s}\)

Answer: 1. \(\frac{8 \pi}{3} \mathrm{~s}\)

Given that amplitude = A = 5 cm, displacement-x-4 cm.

Hence, velocity = \(v=\omega \sqrt{A^2-x^2}\) and the magnitude of die acceleration is |a| = ω²x,

∵ \(\omega^2 x=\omega \sqrt{A^2-x^2}\)

∴ \(\frac{2 \pi}{T}=\frac{\sqrt{(5 \mathrm{~cm})^2-(4 \mathrm{~cm})^2}}{4 \mathrm{~cm}}\)

⇒ \(T=2 \pi \cdot \frac{4}{3} s=\frac{8 \pi}{3} s\)

Question 59. Two light identical springs, each of spring constant k, are attached horizontally at the two ends of a uniform horizontal rod AB of length l and mass m. The rod is pivoted at its centre O and can rotate freely in the horizontal plane. The other ends of the two springs are fixed to two rigid supports, as shown in the figure. The rod is now turned through a small angle and then released. The frequency of the resulting oscillations will be

Simple Harmonic Motion Multiple Choice Question And Answers uniform horizontal rod Q 59

  1. \(\frac{1}{2 \pi} \sqrt{\frac{k}{2 m}}\)
  2. \(\frac{1}{2 \pi} \sqrt{\frac{3 k}{2 m}}\)
  3. \(\frac{1}{2 \pi} \sqrt{\frac{6 k}{m}}\)
  4. \(\frac{1}{2 \pi} \sqrt{\frac{k}{m}}\)

Answer: 3. \(\frac{1}{2 \pi} \sqrt{\frac{6 k}{m}}\)

The compression in each spring is \(x=\frac{l}{2} \sin \theta \approx l\left(\frac{\theta}{2}\right) \text { for } \theta \rightarrow 0\)

The force on each end of the rod is

⇒ \(F=k x=\frac{k l \theta}{2}\)

The controlling torque Y during the rotational motion is

⇒ \(\tau=-k x\left(A^{\prime} C\right)=-\frac{k l \theta}{2}(l \cos \theta)\)

⇒ \(-\frac{k l^2 \theta}{2} \text { for } \cos \theta \rightarrow\)

But torque \(I \alpha=\left(\frac{m l^2}{12}\right)\)

⇒ \(\left(\frac{m l^2}{12}\right) \alpha=-\left(\frac{k l^2}{2}\right) \theta \Rightarrow \alpha=-\left(\frac{6 k}{m}\right) \theta\)

This represents an angular SHM with an angular frequency of

⇒ \(\omega=2 \pi f=\sqrt{\frac{6 k}{m}}\)

∴ \(f=\frac{1}{2 \pi} \sqrt{\frac{6 k}{m}}\)

Simple Harmonic Motion Multiple Choice Question And Answers Q 59

Question 60. An SHM is represented by y = 5(sin 3πt + √3 cos 3πt) cm. The amplitude and the time period of oscillations are respectively

  1. 5 cm and \(\frac{3}{2}\) s
  2. 5 cm and \(\frac{2}{3}\) s
  3. 10 cm and \(\frac{2}{3}\) s
  4. 10 cm and \(\frac{3}{2}\) s

Answer: 3. 10 cm and \(\frac{2}{3}\) s

The given equation is \(y=5 \sin (3 \pi t)+5 \sqrt{3} \cos (3 \pi t)\)

Comparing this equation with y = Asin

⇒\(a=\sqrt{A^2+B^2}=\sqrt{5^2+(5 \sqrt{3})^2} \mathrm{~cm}\)

= 10 cm.

∴ angular frequency = \(\omega=\frac{2 \pi}{T}=3 \pi \Rightarrow T=\frac{2}{3}\)

Question 61. A pendulum executes an SHM with its maximum kinetic energy Kv If the length of the pendulum is doubled and it performs an SHM with the same amplitude as in the first case, the maximum kinetic energy is K2. Then,

  1. K2 = 2K1
  2. K2 = K1
  3. \(K_2=\frac{K_1}{2}\)
  4. \(K_2=\frac{K_1}{4}\)

Answer: 3. \(K_2=\frac{K_1}{2}\)

The maximum KE in the given SHM is

⇒ \(K_1=\frac{1}{2} m \omega_1^2 A^2\)

When the length of the pendulum is doubled keeping its amplitude (A) unchanged, the maximum KE becomes

⇒ \(K_2=\frac{1}{2} m \omega_2^2 A^2\)

∴ \(\frac{K_1}{K_2}=\left(\frac{\omega_1}{\omega_2}\right)^2=\left(\frac{\frac{1}{2 \pi} \sqrt{\frac{g}{l}}}{\frac{1}{2 \pi} \sqrt{\frac{g}{2 l}}}\right)^2=2\)

⇒ \(K_2=\frac{K_1}{2}\)

Question 62. A spring-block system having a block of mass m executes an SHM with an amplitude of A and a frequency of l. While crossing the mean position, the mass is reduced to m/2. The resulting amplitude changes to kA, where k is equal to

  1. \(\frac{1}{2}\)
  2. \(\frac{1}{\sqrt{2}}\)
  3. √2
  4. 1

Answer: 3. √2

At the mean position, the velocity is maximum (= Aw) but the acceleration is zero, the net force is zero, and hence the momentum is conserved.

∴ \(m A \omega=\frac{m}{2} \cdot A^{\prime} \omega^{\prime}\)

⇒ \(A \sqrt{\frac{k}{m}}=\frac{A^{\prime}}{2} \sqrt{\frac{\frac{k}{m}}{2}}\)

⇒ \(A=\frac{A^{\prime}}{\sqrt{2}}\)

⇒ \(A^{\prime}=\sqrt{2} A\)

∴ k = √2.

Question 63. An object of mass m is suspended at the end of a light wire of length L and of the area of cross-section A. The Young modulus of the material of the wire is Y. If the mass is pulled down slightly and then released, its frequency of oscillations in the vertical direction is

  1. \(f=\frac{1}{2 \pi} \sqrt{\frac{Y L}{m A}}\)
  2. \(f=\frac{1}{2 \pi} \sqrt{\frac{m L}{Y A}}\)
  3. \(f=\frac{1}{2 \pi} \sqrt{\frac{Y A}{m L}}\)
  4. \(f=\frac{1}{2 \pi} \sqrt{\frac{m A}{Y L}}\)

Answer: 3. \(f=\frac{1}{2 \pi} \sqrt{\frac{Y A}{m L}}\)

For the extension x, let the restoring force be F.

∴ \(Y=\frac{\frac{F}{A}}{\frac{x}{L}}\)

⇒ \(F=-\left(\frac{Y A}{L}\right) x\)

∴ acceleration = \(a=\frac{F}{m}\)

=\(-\left(\frac{Y A}{m L}\right) x\)

=\(-\omega^2 x\)

This represents an SHM with

⇒ \(\omega=\sqrt{\frac{Y A}{m L}}\)

or, \(f=\frac{\omega}{2 \pi}=\frac{1}{2 \pi} \sqrt{\frac{Y A}{m L}}\)

Question 64. A spring of force constant is suspended from a support, and a block of mass m is attached to its lower end. The block is released when the spring is relaxed. The motion is described by y(t)= y0sin²ωf, where y is measured from the free end of the released spring. Then, die angular frequency (ω) is

  1. \(\frac{1}{2} \sqrt{\frac{g}{y_0}}\)
  2. \(\sqrt{\frac{g}{y_0}}\)
  3. \(\sqrt{\frac{g}{2 y_0}}\)
  4. \(\sqrt{\frac{2 g}{y_0}}\)

Answer: 3. \(\sqrt{\frac{g}{2 y_0}}\)

Given that \(y(t)=y_0 \sin ^2 \omega t=\frac{y_0}{2}(1-\cos 2 \omega t)\)

Hence, the elongation in the spring when in equilibrium (before the block is released) is \(\frac{y_0}{2}\), which is produced by the weight mg of the object.

Now, \(m g=k\left(\frac{y_0}{2}\right) \text { and } 2 \omega=\sqrt{\frac{k}{m}}=\sqrt{\frac{2 g}{y_0}}\) [∵ \(\text { in an SHM, } \omega^2=\frac{k}{m}\)]

∴ \(\omega=\frac{1}{2} \sqrt{\frac{2 g}{y_0}}=\sqrt{\frac{g}{2 y_0}}\)